You are on page 1of 459

yomna92009@yahoo.

com - MRCS Part A - My account

Reference ranges End and review

Question 1 of 398 Question stats Score: 100%

1
A 34.9%
A 34 year old male is being examined in the pre-operative assessment clinic. A B 11.6%
murmur is identified in the 5th intercostal space just next to the sternum. From
C 4.7%
where is it most likely to have originated?
D 2.3%

E 46.5%
A. Mitral valve
46.5% of users answered this
B. Aortic valve question correctly
C. Pulmonary valve
D. Right ventricular aneurysm
E. Tricuspid valve

Next question

Theme from September 2012 Exam

Heart sounds

Sites of auscultation
Valve Site

Pulmonary valve Left second intercostal space, at the upper sternal border

Aortic valve Right second intercostal space, at the upper sternal border

Mitral valve Left fifth intercostal space, just medial to mid clavicular line

Tricuspid valve Left fifth intercostal space, at the lower left sternal border

The diagram below demonstrates where the various cardiac valves are best
heard.

Image sourced from Wikipedia

Rate question: Next question

Com m ent on this question


yomna92009@yahoo.com - MRCS Part A - My account

Reference ranges End and review

Question 2 of 398 Question stats Score: 100%

1
A 10.8%
Which of the following are not generally supplied by the right coronary artery? 2
B 52.7%
C 15%

A. The sino atrial node D 13.4%

B. The circumflex artery E 8.1%

C. The atrioventricular node 52.7% of users answered this


question correctly
D. Most of the right ventricle
E. The right atrium

Next question

The circumflex artery is generally a branch of the left coronary artery.

Heart anatomy

The walls of each cardiac chamber comprise:

Epicardium
Myocardium
Endocardium

Cardiac muscle is attached to the cardiac fibrous skeleton.

Relations
The heart and roots of the great vessels within the pericardial sac are related
anteriorly to the sternum, medial ends of the 3rd to 5th ribs on the left and their
associated costal cartilages. The heart and pericardial sac are situated obliquely
two thirds to the left and one third to the right of the median plane.

The pulmonary valve lies at the level of the left third costal cartilage.
The mitral valve lies at the level of the fourth costal cartilage.

Coronary sinus
This lies in the posterior part of the coronary groove and receives blood from the
cardiac veins. The great cardiac vein lies at its left and the middle and small
cardiac veins lie on its right. The smallest cardiac veins drain into the atria
directly.

Aortic sinus
Right coronary artery arises from the right aortic sinus, the left is derived from the
left aortic sinus and no vessel emerges from the posterior sinus.

Right and left ventricles

Structure Left Ventricle

A-V Valve Mitral (double leaflet)

Walls Twice as thick as right

Trabeculae carnae Much thicker and more numerous

Right coronary artery


The RCA supplies:

Right atrium
Diaphragmatic part of the left ventricle
Usually the posterior third of the interventricular septum
The sino atrial node (60% cases)
The atrio ventricular node (80% cases)
Left coronary artery
The LCA supplies:

Left atrium
Most of left ventricle
Part of the right ventricle
Anterior two thirds of the inter ventricular septum
The sino atrial node (remaining 40% cases)

Innervation of the heart


Autonomic nerve fibres from the superficial and deep cardiac plexus. These lie
anterior to the bifurcation of the trachea, posterior to the ascending aorta and
superior to the bifurcation of the pulmonary trunk. The parasympathetic supply to
the heart is from presynaptic fibres of the vagus nerves.

Valves of the heart


Mitral valve Aortic valve Pulmonary valve Tricuspid valve

2 cusps 3 cusps 3 cusps 3 cusps

First heart sound Second heart Second heart First heart sound
sound sound

1 anterior cusp 1 anterior cusp 2 anterior cusps 2 anterior cusps

Attached to chordae No chordae No chordae Attached to chordae


tendinae tendinae

Rate question: Next question

Com m ent on this question

All contents of this site are © 2012 E-Medical Revision Ltd Terms and Conditions Privacy policy
yomna92009@yahoo.com - MRCS Part A - My account

Reference ranges End and review

Question 3 of 398 Question stats Score: 100%

1
A 7.9%
A 44 year old man has a malignant melanoma and is undergoing a block 2
B 7.6%
dissection of the groin. The femoral triangle is being explored for intra operative 3
C 53.3%
bleeding. Which of the following forms the medial border of the femoral triangle?
D 14.5%

E 16.7%
A. Femoral artery
53.3% of users answered this
B. Biceps femoris question correctly
C. Adductor longus
D. Sartorius
E. Adductor magnus

Next question

Vastus medialis forms the lateral border of the adductor canal. The sartorius
muscles forms the roof of the adductor canal.

Adductor longus forms the medial boundary of the femoral triangle (see below).

Femoral triangle anatomy

Boundaries
Superiorly Inguinal ligament

Laterally Sartorius

Medially Adductor longus

Floor Iliopsoas, adductor longus and pectineus

Roof Fascia lata and Superficial fascia


Superficial inguinal lymph nodes (palpable below the inguinal ligament)
Great saphenous vein

Image sourced from Wikipedia


Contents

Femoral vein (medial to lateral)


Femoral artery-pulse palpated at the mid inguinal point
Femoral nerve
Deep and superficial inguinal lymph nodes
Lateral cutaneous nerve
Great saphenous vein
Femoral branch of the genitofemoral nerve

Rate question: Next question

Com m ent on this question

All contents of this site are © 2012 E-Medical Revision Ltd Terms and Conditions Privacy policy
yomna92009@yahoo.com - MRCS Part A - My account

Reference ranges End and review

Question 4 of 398 Question stats Score: 75%

1
A 17.3%
The foramen marking the termination of the adductor canal is located in which of 2
B 49.9%
the following? 3
C 7.3%
D 12.7% 4
A. Adductor longus E 12.8%

B. Adductor magnus
49.9% of users answered this
C. Adductor brevis question correctly

D. Sartorius
E. Semimembranosus

Next question

The foramen marking the distal limit of the adductor canal is contained within
adductor magnus. The vessel passes through this region to enter the popliteal
fossa.

Adductor canal

Also called Hunter's or subsartorial canal

Immediately distal to the apex of the femoral triangle, lying in the middle
third of the thigh. Canal terminates at the adductor hiatus.

Borders Contents

Laterally Vastus medialis muscle Saphenous nerve

Posteriorly Adductor longus, adductor magnus Superficial femoral artery

Roof Sartorius Superficial femoral vein

In the image below the sartorius muscle is removed to expose the canal contents
yomna92009@yahoo.com - MRCS Part A - My account

Reference ranges End and review

Question 5 of 398 Question stats Score: 80%

1
A 50.6%
Which of the following is the first vessel to branch from the external carotid 2
B 19.6%
artery? 3
C 9.8%
D 10.9% 4
A. Superior thyroid artery E 9.1% 5
B. Inferior thyroid artery
50.6% of users answered this
C. Lingual artery question correctly

D. Facial artery
E. Occipital artery

Next question

The first branch of the external carotid artery is the superior thyroid artery. The
inferior thyroid artery is derived from the thyrocervical trunk. The other branches
are illustrated below.

External carotid artery

The external carotid artery (ECA) is a branch of the common carotid artery.

Path

Upper border of thyroid cartilage to pass in between the angle of the


mandible and mastoid process
Initially anteromedial to the ICA, then becomes lateral
Lower ECA covered by sternomastoid, passed by hypoglossal nerve/lingual
& facial vein
Then deep to digastric and stylohyoid, eventually passes into the parotid
gland where it divides into the superficial temporal and maxillary branches

Image sourced from Wikipedia

Rate question: Next question


yomna92009@yahoo.com - MRCS Part A - My account

Reference ranges End and review

Question 1 of 400 Question stats Score: 100%

1
A 41.8%
A 19 year old female is admitted with suspected meningitis. The House Officer is B 6.4%
due to perform a lumbar puncture. What is the most likely structure first
C 8.2%
encountered when the needle is inserted?
D 0.9%

E 42.7%
A. Ligamentum flavum
42.7% of users answered this
B. Denticulate ligament question correctly
C. Dural sheath
D. Pia Mater
E. Supraspinous ligament

Next question

Theme from September 2012 exam

Lumbar puncture

Lumbar punctures are performed to obtain cerebrospinal fluid. In adults, the


procedure is best performed at the level of L3/L4 or L4/5 interspace. These
regions are below the terminal of the spinal cord at L1.

During the procedure the needle passes through:

The supraspinous ligament which connects the tips of spinous processes


and the interspinous ligaments between adjacent borders of spinous
processes
Then the needle passes through the ligamentum flavum, which may cause
a give as it is penetrated
A second give represents penetration of the needle through the dura mater
into the subarachnoid space. Clear CSF should be obtained at this point

References
Boon et al Lumbar Puncture: Anatomical Review of a Clinical Skill. Clinical
Anatomy 17:544553 (2004)

Rate question: Next question

Com m ent on this question

All contents of this site are © 2012 E-Medical Revision Ltd Terms and Conditions Privacy policy
yomna92009@yahoo.com - MRCS Part A - My account

Reference ranges End and review

Question 2 of 400 Question stats Score: 50%

1
A 29.2%
A 44 year old lady is undergoing an abdominal hysterectomy and the ureter is 2
B 5.3%
identified during the ligation of the uterine artery. At which site does it insert into
C 3.5%
the bladder?
D 23.9%

E 38.1%
A. Posterior
23.9% of users answered this
B. Apex question correctly
C. Anterior
D. Base
E. Superior aspect of the lateral side

Next question

Theme from September 2012 Exam


The ureters enter the bladder at the upper lateral aspect of the base of the
bladder. They are about 5cm apart from each other in the empty bladder.
Internally this aspect is contained within the bladder trigone.

Ureter

25-35 cm long
Muscular tube lined by transitional epithelium
Surrounded by thick muscular coat. Becomes 3 muscular layers as it
crosses the bony pelvis
Retroperitoneal structure overlying transverse processes L2-L5
Lies anterior to bifurcation of iliac vessels
Blood supply is segmental; renal artery, aortic branches, gonadal
branches, common iliac and internal iliac
Lies beneath the uterine artery

Rate question: Next question

Com m ent on this question

All contents of this site are © 2012 E-Medical Revision Ltd Terms and Conditions Privacy policy
yomna92009@yahoo.com - MRCS Part A - My account

Reference ranges End and review

Question 1 of 398 Question stats Score: 100%

1
A 3.6%
Which muscle initiates abduction of the shoulder? B 5.1%
C 62%

A. Infraspinatus D 24.1%

B. Latissimus dorsi E 5.1%

C. Supraspinatus 62% of users answered this


question correctly
D. Deltoid
E. Teres major

Next question

Theme from September 2012 exam


The intermediate portion of the deltoid muscle is the chief abductor of the
humerus. However, it can only do this after the movement has been initiated by
supraspinatus. Damage to the tendon of supraspinatus is a common form of
rotator cuff disease.

Shoulder joint

Shallow synovial ball and socket type of joint.


It is an inherently unstable joint, but is capable to a wide range of
movement.
Stability is provided by muscles of the rotator cuff that pass from the
scapula to insert in the greater tuberosity (all except sub scapularis-lesser
tuberosity).

Glenoid labrum

Fibrocartilaginous rim attached to the free edge of the glenoid cavity


Tendon of the long head of biceps arises from within the joint from the
supraglenoid tubercle, and is fused at this point to the labrum.
The long head of triceps attaches to the infraglenoid tubercle

Fibrous capsule

Attaches to the scapula external to the glenoid labrum and to the labrum
itself (postero-superiorly)
Attaches to the humerus at the level of the anatomical neck superiorly and
the surgical neck inferiorly
Anteriorly the capsule is in contact with the tendon of subscapularis,
superiorly with the supraspinatus tendon, and posteriorly with the tendons
of infraspinatus and teres minor. All these blend with the capsule towards
their insertion.
Two defects in the fibrous capsule; superiorly for the tendon of biceps.
Anteriorly there is a defect beneath the subscapularis tendon.
The inferior extension of the capsule is closely related to the axillary nerve
at the surgical neck and this nerve is at risk in anteroinferior dislocations. It
also means that proximally sited osteomyelitis may progress to septic
arthritis.

Movements and muscles


Flexion Anterior part of deltoid
Pectoralis major
Biceps
Coracobrachialis

Extension Posterior deltoid


Teres major
Latissimus dorsi
Adduction Pectoralis major
Latissimus dorsi
Teres major
Coracobrachialis

Abduction Mid deltoid


Supraspinatus

Medial rotation Subscapularis


Anterior deltoid
Teres major
Latissimus dorsi

Lateral rotation Posterior deltoid


Infraspinatus
Teres minor

Important anatomical relations


Anteriorly Brachial plexus
Axillary artery and vein

Posterior Suprascapular nerve


Suprascapular vessels

Inferior Axillary nerve


Circumflex humeral vessels

Rate question: Next question

Com m ent on this question

All contents of this site are © 2012 E-Medical Revision Ltd Terms and Conditions Privacy policy
yomna92009@yahoo.com - MRCS Part A - My account

Reference ranges End and review

Question 2 of 398 Question stats Score: 50%

1
A 23.3%
What is the most important structure involved in supporting the uterus? 2
B 34.2%
C 14.4%

A. Round ligament D 14.4%

B. Broad ligament E 13.7%

C. Uterosacral ligaments 13.7% of users answered this


question correctly
D. Cardinal ligaments
E. Central perineal tendon

Next question

Theme from September 2012 Exam


The central perineal tendon provides the main structural support to the uterus.
Damage to this structure is commonly associated with the development of pelvic
organ prolapse, even when other structures are intact.

Uterus

The non pregnant uterus resides entirely within the pelvis. The peritoneum
invests the uterus and the structure is contained within the peritoneal cavity. The
blood supply to the uterine body is via the uterine artery (branch of the internal
iliac). The uterine artery passes from the inferior aspect of the uterus (lateral to
the cervix) and runs alongside the uterus. It frequently anastomoses with the
ovarian artery superiorly. Inferolaterally the ureter is a close relation and ureteric
injuries are a recognised complication when pathology brings these structures
into close proximity.

The supports of the uterus include the central perineal tendon (the most
important). The lateral cervical, round and uterosacral ligaments are
condensations of the endopelvic fascia and provide additional structural support.

Topography of the uterus

Image sourced from Wikipedia

Rate question: Next question

Com m ent on this question

All contents of this site are © 2012 E-Medical Revision Ltd Terms and Conditions Privacy policy
yomna92009@yahoo.com - MRCS Part A - My account

Reference ranges End and review

Question 3 of 398 Question stats Score: 33.3%

1
A 10.8%
Which of the following forms the medial wall of the femoral canal? 2
B 33.7%
C 8.7% 3

A. Pectineal ligament D 41.3%

B. Adductor longus E 5.5%

C. Sartorius 41.3% of users answered this


question correctly
D. Lacunar ligament
E. Inguinal ligament

Next question

Theme from September 2011 Exam


Theme from September 2012 Exam

Femoral canal

The femoral canal lies at the medial aspect of the femoral sheath. The femoral
sheath is a fascial tunnel containing both the femoral artery laterally and femoral
vein medially. The canal lies medial to the vein.

Borders of the femoral canal


Laterally Femoral vein

Medially Lacunar ligament

Anteriorly Inguinal ligament

Posteriorly Pectineal ligament

Image showing dissection of femoral canal


Image sourced from Wikipedia

Contents

Lymphatic vessels
Cloquet's lymph node

Physiological significance
Allows the femoral vein to expand to allow for increased venous return to the
lower limbs.

Pathological significance
As a potential space, it is the site of femoral hernias. The relatively tight neck
places these at high risk of strangulation.

Rate question: Next question

Com m ent on this question

All contents of this site are © 2012 E-Medical Revision Ltd Terms and Conditions Privacy policy
yomna92009@yahoo.com - MRCS Part A - My account

Reference ranges End and review

Question 4 of 398 Question stats Score: 50%

1
A 3.8%
What is the anatomical level of the transpyloric plane? 2
B 14.4%
C 62.5% 3

A. T11 D 3.8% 4

B. T12 E 15.4%

C. L1 62.5% of users answered this


question correctly
D. L4
E. T10

Next question

Theme from September 2012 Exam

Transpyloric plane

Transpyloric plane
Level of the body of L1

Pylorus stomach
Left kidney hilum (L1- left one!)
Right hilum of the kidney (1.5cm lower than the left)
Fundus of the gallbladder
Neck of pancreas
Duodenojejunal flexure
Superior mesenteric artery
Portal vein
Left and right colic flexure
Root of the transverse mesocolon
2nd part of the duodenum
Upper part of conus medullaris
Spleen

Rate question: Next question

Com m ent on this question

All contents of this site are © 2012 E-Medical Revision Ltd Terms and Conditions Privacy policy
yomna92009@yahoo.com - MRCS Part A - My account

Reference ranges End and review

Question 5 of 398 Question stats Score: 60%

1
A 5.8%
A 21 year old man is stabbed in the antecubital fossa. A decision is made to 2
B 5.8%
surgically explore the wound. At operation the surgeon dissects down onto the 3
C 7%
brachial artery. A nerve is identified medially, which nerve is it likely to be?
D 18.6% 4
E 62.8% 5
A. Radial
62.8% of users answered this
B. Recurrent branch of median question correctly
C. Anterior interosseous
D. Ulnar
E. Median

Next question

Theme from September 2012 Exam

Median nerve

The median nerve is formed by the union of a lateral and medial root respectively
from the lateral (C5,6,7) and medial (C8 and T1) cords of the brachial plexus; the
medial root passes anterior to the third part of the axillary artery. The nerve
descends lateral to the brachial artery, crosses to its medial side (usually passing
anterior to the artery). It passes deep to the bicipital aponeurosis and the median
cubital vein at the elbow.
It passes between the two heads of the pronator teres muscle, and runs on the
deep surface of flexor digitorum superficialis (within its fascial sheath).
Near the wrist it becomes superficial between the tendons of flexor digitorum
superficialis and flexor carpi radialis, deep to palmaris longus tendon. It passes
deep to the flexor retinaculum to enter the palm, but lies anterior to the long flexor
tendons within the carpal tunnel.

Branches
Region Branch

Upper No branches, although the nerve commonly communicates with the


arm musculocutaneous nerve

Forearm Pronator teres


Flexor carpi radialis
Palmaris longus
Flexor digitorum superficialis
Flexor pollicis longus
Flexor digitorum profundus (only the radial half)

Distal Palmar cutaneous branch


forearm

Hand Motor supply (LOAF)


(Motor)
Lateral 2 lumbricals
Opponens pollicis
Abductor pollicis brevis
Flexor pollicis brevis

Hand Over thumb and lateral 2 ½ fingers


(Sensory) On the palmar aspect this projects proximally, on the dorsal aspect only
the distal regions are innervated with the radial nerve providing the more
proximal cutaneous innervation.

Patterns of damage
Damage at wrist

e.g. carpal tunnel syndrome


paralysis and wasting of thenar eminence muscles and opponens pollicis
(ape hand deformity)
sensory loss to palmar aspect of lateral (radial) 2 ½ fingers

Damage at elbow, as above plus:

unable to pronate forearm


weak wrist flexion
ulnar deviation of wrist

Anterior interosseous nerve (branch of median nerve)

leaves just below the elbow


results in loss of pronation of forearm and weakness of long flexors of
thumb and index finger

Topography of the median nerve

Image sourced from Wikipedia

Rate question: Next question

Com m ent on this question

All contents of this site are © 2012 E-Medical Revision Ltd Terms and Conditions Privacy policy
yomna92009@yahoo.com - MRCS Part A - My account

Reference ranges End and review

Question 6 of 398 Question stats Score: 50%

1
A 12.8%
A 5 year old boy presents with recurrent headaches. As part of his assessment 2
B 7.8%
he undergoes an MRI scan of his brain. This demonstrates enlargement of the 3
C 18.1%
lateral and third ventricles. Where is the most likely site of obstruction?
D 56.1% 4
E 5.2% 5
A. Foramen of Luschka
56.1% of users answered this 6
B. Foramen of Magendie question correctly
C. Foramen of Munro
D. Aqueduct of Sylvius
E. None of the above

Next question

Theme based on September 2011 Exam


Theme based on April 2012 Exam
Theme from September 2012 Exam
The CSF flows from the 3rd to the 4th ventricle via the Aqueduct of Sylvius.

Cerebrospinal fluid

The CSF fills the space between the dura mater and surface of the brain. The
total volume of CSF in the brain is approximately 150ml. Approximately 500 ml is
produced by the ependymal cells in the choroid plexus (70%), or blood vessels
(30%). It is reabsorbed via the arachnoid granulations which project into the
venous sinuses.

Circulation
1. Lateral ventricles (via foramen Munro)
2. 3rd ventricle
3. Cerebral aqueduct (aqueduct Sylvius)
4. 4th ventricle (via foramina of Magendie and Luschka)
5. Subarachnoid space
6. Reabsorbed into venous system via arachnoid granulations in superior sagittal
sinus

Composition

Glucose: 50-80mg/dl
Protein: 15-40 mg/dl
Red blood cells: Nil
White blood cells: 0-3 cells/ mm3

Rate question: Next question

Com m ent on this question

All contents of this site are © 2012 E-Medical Revision Ltd Terms and Conditions Privacy policy
yomna92009@yahoo.com - MRCS Part A - My account

Reference ranges End and review

Question 7 of 398 Question stats Score: 42.9%

1
A 49.3%
A 43 year old lady presents to casualty with diplopia. On examination the right eye 2
B 2.7%
cannot look downwards and medially. Which of the nerves listed below is most 3
C 4%
likely to be responsible?
D 10.7% 4
E 33.3% 5
A. Trochlear
33.3% of users answered this 6
B. Trigeminal nerve question correctly
7
C. Facial
D. Abducens
E. Oculomotor

Next question

Theme from September 2012 Exam


In complete third nerve palsies the eye cannot be moved upward, downwards or
inwards. At rest there is external strabismus due to the activity of the lateral rectus
muscle. The unopposed activity of superior rectus may result in downwards
deflection.

Cranial nerves

Cranial nerve lesions


Olfactory nerve May be injured in basal skull fractures or involved in frontal lobe tumour
extension. Loss of olfactory nerve function in relation to major CNS
pathology is seldom an isolated event and thus it is poor localiser of
CNS pathology.

Optic nerve Problems with visual acuity may result from intra ocular disorders.
Problems with the blood supply such as amaurosis fugax may produce
temporary visual distortion. More important surgically is the pupillary
response to light. The pupillary size may be altered in a number of
disorders. Nerves involved in the resizing of the pupil connect to the
pretectal nucleus of the high midbrain, bypassing the lateral geniculate
nucleus and the primary visual cortex. From the pretectal nucleus
neurones pass to the Edinger - Westphal nucleus, motor axons from
here pass along with the oculomotor nerve. They synapse with ciliary
ganglion neurones; the parasympathetic axons from this then innervate
the iris and produce miosis. The miotic pupil is seen in disorders such
a Horner's syndrome or opiate overdose.
Mydriasis is the dilatation of the pupil in response to disease, trauma,

drugs (or the dark!). It is pathological when light fails to induce miosis.
The radial muscle is innervated by the sympathetic nervous system.
Because the parasympathetic fibres travel with the oculomotor nerve
they will be damaged by lesions affecting this nerve (e.g. cranial
trauma).
The response to light shone in one eye is usually a constriction of both
pupils. This indicates intact direct and consensual light reflexes. When
the optic nerve has an afferent defect the light shining on the affected
eye will produce a diminished pupillary response in both eyes.
Whereas light shone on the unaffected eye will produce a normal
pupillary response in both eyes. This is referred to as the Marcus Gunn
pupil and is seen in conditions such as optic neuritis. In a total CN II
lesion shining the light in the affected eye will produce no response.

Oculomotor nerve The pupillary effects are described above. In addition it supplies all
ocular muscles apart from lateral rectus and superior oblique. Thus the
affected eye will be deviated inferolaterally. Levator palpebrae
superioris may also be impaired resulting in impaired ability to close
the eye.

Trochlear nerve The eye will not be able to look down.

Trigeminal nerve Largest cranial nerve. Exits the brainstem at the pons. Branches are
ophthalmic, maxillary and mandibular. Only the mandibular branch has
both sensory and motor fibres. Branches converge to form the
trigeminal ganglion (located in Meckels cave). It supplies the muscles
of mastication and also tensor veli palatine, mylohyoid, anterior belly of
digastric and tensor tympani. The detailed descriptions of the various
sensory functions are described in other areas of the website. The
corneal reflex is important and is elicited by applying a small tip of
cotton wool to the cornea, a reflex blink should occur if it is intact. It is
mediated by: the naso ciliary branch of the ophthalmic branch of the
mediated by: the naso ciliary branch of the ophthalmic branch of the
trigeminal (sensory component) and the facial nerve producing the
motor response. Lesions of the afferent arc will produce bilateral
absent blink and lesions of the efferent arc will result in a unilateral
absent blink.

Abducens nerve The affected eye will have a deficit of abduction. This cranial nerve exits
the brainstem between the pons and medulla. It thus has a relatively
long intra cranial course which renders it susceptible to damage in
raised intra cranial pressure.

Facial nerve Emerges from brainstem between pons and medulla. It controls
muscles of facial expression and taste from the anterior 2/3 of the
tongue. The nerve passes into the petrous temporal bone and into the
internal auditory meatus. It then passes through the facial canal and
exits at the stylomastoid foramen. It passes through the parotid gland
and divides at this point. It does not innervate the parotid gland. Its
divisions are considered in other parts of the website. Its motor fibres
innervate orbicularis oculi to produce the efferent arm of the corneal
reflex. In surgical practice it may be injured during parotid gland surgery
or invaded by malignancies of the gland and a lower motor neurone on
the ipsilateral side will result.

Vestibulo- Exits from the pons and then passes through the internal auditory
cochlear nerve meatus. It is implicated in sensorineural hearing loss. Individuals with
sensorineural hearing loss will localise the sound in webers test to the
normal ear. Rinnes test will be reduced on the affected side but should
still work. These two tests will distinguish sensorineural hearing loss
from conductive deafness. In the latter condition webers test will
localise to the affected ear and Rinnes test will be impaired on the
affected side. Surgical lesions affecting this nerve include CNS
tumours and basal skull fractures. It may also be damaged by the
administration of ototoxic drugs (of which gentamicin is the most
commonly used in surgical practice).

Glossopharyngeal Exits the pons just above the vagus. Receives sensory fibres from
nerve posterior 1/3 tongue, tonsils, pharynx and middle ear (otalgia may occur
following tonsillectomy). It receives visceral afferents from the carotid
bodies. It supplies parasympathetic fibres to the parotid gland via the
otic ganglion and motor function to stylopharyngeaus muscle. The
sensory function of the nerve is tested using the gag reflex.

Vagus nerve Leaves the medulla between the olivary nucleus and the inferior
cerebellar peduncle. Passes through the jugular foramen and into the
carotid sheath. Details of the functions of the vagus nerve are covered
in the website under relevant organ sub headings.

Accessory nerve Exists from the caudal aspect of the brainstem (multiple branches)
supplies trapezius and sternocleidomastoid muscles. The distal
portion of this nerve is most prone to injury during surgical procedures.

Hypoglossal Emerges from the medulla at the preolivary sulcus, passes through the
nerve hypoglossal canal. It lies on the carotid sheath and passes deep to the
posterior belly of digastric to supply muscles of the tongue (except
palatoglossus). Its location of the carotid sheath makes it vulnerable
during carotid endarterectomy surgery and damage will produce
ipsilateral defect in muscle function.

Rate question: Next question

Com m ent on this question

All contents of this site are © 2012 E-Medical Revision Ltd Terms and Conditions Privacy policy
yomna92009@yahoo.com - MRCS Part A - My account

Reference ranges End and review

Question 8 of 398 Question stats Score: 50%

1
A 26.8%
A motorcyclist is injured in a road traffic accident and is not wearing a helmet. He 2
B 7%
suffers a severe closed head injury and develops raised intracranial pressure. 3
C 7.8%
The first cranial nerve to be affected by this process is likely to be:
D 50.8% 4
E 7.5% 5
A. Oculomotor
50.8% of users answered this 6
B. Hypoglossal question correctly
7
C. Motor branch of the trigeminal
8
D. Abducens
E. Trochlear

Next question

The abducens nerve (CN VI) has the longest intra cranial course and is thus the
most susceptible to raised intra cranial pressure. It also passes over the petrous
temporal bone and 6th nerve palsies are also seen in mastoiditis.

Cranial nerves

Cranial nerve lesions


Olfactory nerve May be injured in basal skull fractures or involved in frontal lobe tumour
extension. Loss of olfactory nerve function in relation to major CNS
pathology is seldom an isolated event and thus it is poor localiser of
CNS pathology.

Optic nerve Problems with visual acuity may result from intra ocular disorders.
Problems with the blood supply such as amaurosis fugax may produce
temporary visual distortion. More important surgically is the pupillary
response to light. The pupillary size may be altered in a number of
disorders. Nerves involved in the resizing of the pupil connect to the
pretectal nucleus of the high midbrain, bypassing the lateral geniculate
nucleus and the primary visual cortex. From the pretectal nucleus
neurones pass to the Edinger - Westphal nucleus, motor axons from
here pass along with the oculomotor nerve. They synapse with ciliary
ganglion neurones; the parasympathetic axons from this then innervate
the iris and produce miosis. The miotic pupil is seen in disorders such
a Horner's syndrome or opiate overdose.
Mydriasis is the dilatation of the pupil in response to disease, trauma,
drugs (or the dark!). It is pathological when light fails to induce miosis.
The radial muscle is innervated by the sympathetic nervous system.
Because the parasympathetic fibres travel with the oculomotor nerve

they will be damaged by lesions affecting this nerve (e.g. cranial


trauma).
The response to light shone in one eye is usually a constriction of both
pupils. This indicates intact direct and consensual light reflexes. When
the optic nerve has an afferent defect the light shining on the affected
eye will produce a diminished pupillary response in both eyes.
Whereas light shone on the unaffected eye will produce a normal
pupillary response in both eyes. This is referred to as the Marcus Gunn
pupil and is seen in conditions such as optic neuritis. In a total CN II
lesion shining the light in the affected eye will produce no response.

Oculomotor nerve The pupillary effects are described above. In addition it supplies all
ocular muscles apart from lateral rectus and superior oblique. Thus the
affected eye will be deviated inferolaterally. Levator palpebrae
superioris may also be impaired resulting in impaired ability to close
the eye.

Trochlear nerve The eye will not be able to look down.

Trigeminal nerve Largest cranial nerve. Exits the brainstem at the pons. Branches are
ophthalmic, maxillary and mandibular. Only the mandibular branch has
both sensory and motor fibres. Branches converge to form the
trigeminal ganglion (located in Meckels cave). It supplies the muscles
of mastication and also tensor veli palatine, mylohyoid, anterior belly of
digastric and tensor tympani. The detailed descriptions of the various
sensory functions are described in other areas of the website. The
corneal reflex is important and is elicited by applying a small tip of
cotton wool to the cornea, a reflex blink should occur if it is intact. It is
mediated by: the naso ciliary branch of the ophthalmic branch of the
trigeminal (sensory component) and the facial nerve producing the
motor response. Lesions of the afferent arc will produce bilateral
absent blink and lesions of the efferent arc will result in a unilateral
absent blink and lesions of the efferent arc will result in a unilateral
absent blink.

Abducens nerve The affected eye will have a deficit of abduction. This cranial nerve exits
the brainstem between the pons and medulla. It thus has a relatively
long intra cranial course which renders it susceptible to damage in
raised intra cranial pressure.

Facial nerve Emerges from brainstem between pons and medulla. It controls
muscles of facial expression and taste from the anterior 2/3 of the
tongue. The nerve passes into the petrous temporal bone and into the
internal auditory meatus. It then passes through the facial canal and
exits at the stylomastoid foramen. It passes through the parotid gland
and divides at this point. It does not innervate the parotid gland. Its
divisions are considered in other parts of the website. Its motor fibres
innervate orbicularis oculi to produce the efferent arm of the corneal
reflex. In surgical practice it may be injured during parotid gland surgery
or invaded by malignancies of the gland and a lower motor neurone on
the ipsilateral side will result.

Vestibulo- Exits from the pons and then passes through the internal auditory
cochlear nerve meatus. It is implicated in sensorineural hearing loss. Individuals with
sensorineural hearing loss will localise the sound in webers test to the
normal ear. Rinnes test will be reduced on the affected side but should
still work. These two tests will distinguish sensorineural hearing loss
from conductive deafness. In the latter condition webers test will
localise to the affected ear and Rinnes test will be impaired on the
affected side. Surgical lesions affecting this nerve include CNS
tumours and basal skull fractures. It may also be damaged by the
administration of ototoxic drugs (of which gentamicin is the most
commonly used in surgical practice).

Exits the pons just above the vagus. Receives sensory fibres from
Glossopharyngeal
posterior 1/3 tongue, tonsils, pharynx and middle ear (otalgia may occur
nerve
following tonsillectomy). It receives visceral afferents from the carotid
bodies. It supplies parasympathetic fibres to the parotid gland via the
otic ganglion and motor function to stylopharyngeaus muscle. The
sensory function of the nerve is tested using the gag reflex.

Vagus nerve Leaves the medulla between the olivary nucleus and the inferior
cerebellar peduncle. Passes through the jugular foramen and into the
carotid sheath. Details of the functions of the vagus nerve are covered
in the website under relevant organ sub headings.

Accessory nerve Exists from the caudal aspect of the brainstem (multiple branches)
supplies trapezius and sternocleidomastoid muscles. The distal
portion of this nerve is most prone to injury during surgical procedures.

Hypoglossal Emerges from the medulla at the preolivary sulcus, passes through the
nerve hypoglossal canal. It lies on the carotid sheath and passes deep to the
posterior belly of digastric to supply muscles of the tongue (except
palatoglossus). Its location of the carotid sheath makes it vulnerable
during carotid endarterectomy surgery and damage will produce
ipsilateral defect in muscle function.

Rate question: Next question

Com m ent on this question

All contents of this site are © 2012 E-Medical Revision Ltd Terms and Conditions Privacy policy
yomna92009@yahoo.com - MRCS Part A - My account

Reference ranges End and review

Question 9 of 398 Question stats Score: 55.6%

1
A 8.3%
A 32 year old man is undergoing a splenectomy. Division of which of the following 2
B 49.1%
will be necessary during the procedure? 3
C 15.9%
D 18% 4
A. Left crus of diaphragm E 8.8% 5
B. Short gastric vessels 6
49.1% of users answered this
C. Gerotas fascia question correctly
7
D. Splenic flexure of colon
8
E. Marginal artery
9

Next question

During a splenectomy the short gastric vessels which lie within the gastrosplenic
ligament will need to be divided. The splenic flexure of the colon may need to be
mobilised. However, it will almost never need to be divided, as this is watershed
area that would necessitate a formal colonic resection in the event of division.

Splenic anatomy

The spleen is the largest lymphoid organ in the body. It is an intraperitoneal


organ, the peritoneal attachments condense at the hilum where the vessels enter
the spleen. Its blood supply is from the splenic artery (derived from the coeliac
axis) and the splenic vein (which is joined by the IMV and unites with the SMV).

Embryology: derived from mesenchymal tissue


Shape: clenched fist
Position: below 9th-12th ribs
Weight: 75-150g

Relations

Superiorly- diaphragm
Anteriorly- gastric impression
Posteriorly- kidney
Inferiorly- colon
Hilum: tail of pancreas and splenic vessels
Forms apex of lesser sac (containing short gastric vessels)

Rate question: Next question

Com m ent on this question

All contents of this site are © 2012 E-Medical Revision Ltd Terms and Conditions Privacy policy
yomna92009@yahoo.com - MRCS Part A - My account

Reference ranges End and review

Question 10 of 398 Question stats Score: 50%

1
A 48.2%
A 24 year old motor cyclist is involved in a road traffic accident. He suffers a tibial 2
B 22.9%
fracture which is treated with an intra medullary nail. Post operatively he develops 3
C 13.8%
a compartment syndrome. Surgical decompression of the anterior compartment
D 7.9% 4
will relieve pressure on all of the following muscles except?
E 7.2% 5

48.2% of users answered this 6


A. Peroneus brevis
question correctly
7
B. Peroneus tertius
8
C. Extensor digitorum longus
D. Tibialis anterior 9

E. None of the above 10

Next question

The anterior compartment contains:


Tibialis anterior
Extensor digitorum longus
Peroneus tertius
Extensor hallucis longus
Anterior tibial artery
All the muscles are innervated by the deep peroneal nerve.

Lower limb- Muscular compartments

Anterior compartment
Muscle Nerve Action

Tibialis anterior Deep peroneal Dorsiflexes ankle joint, inverts foot


nerve

Extensor digitorum Deep peroneal Extends lateral four toes, dorsiflexes ankle
longus nerve joint

Peroneus tertius Deep peroneal Dorsiflexes ankle, everts foot


nerve

Extensor hallucis Deep peroneal Dorsiflexes ankle joint, extends big toe
longus nerve

Peroneal compartment
Muscle Nerve Action

Peroneus longus Superficial peroneal nerve Everts foot, assists in plantar flexion

Peroneus brevis Superficial peroneal nerve Plantar flexes the ankle joint

Superficial posterior compartment


Nerve Action

Gastrocnemius Tibial nerve Plantar flexes the foot, may also flex the knee

Soleus Tibial nerve Plantar flexor

Deep posterior compartment


Muscle Nerve Action

Flexor digitorum longus Tibial Flexes the lateral four toes

Flexor hallucis longus Tibial Flexes the great toe

Tibialis posterior Tibial Plantar flexor, inverts the foot

Rate question: Next question

Com m ent on this question


yomna92009@yahoo.com - MRCS Part A - My account

Reference ranges End and review

Question 11 of 398 Question stats Score: 54.5%

1
A 8.6%
A 43 year old lady underwent and attempted placement of a central line into the 2
B 13%
internal jugular vein. Unfortunately, the doctor damaged the carotid artery and 3
C 8.1%
this necessitated surgical exploration. As the surgeons incise the carotid sheath a
D 11.6% 4
nerve is identified lying between the internal jugular vein and the carotid artery.
E 58.6% 5
Which of the following is this nerve most likely to be?

58.6% of users answered this 6


question correctly
A. Glossopharyngeal nerve 7

B. Hypoglossal nerve 8

C. Superior laryngeal nerve 9

D. Recurrent laryngeal nerve 10

E. Vagus 11

Next question

The vagus lies in the carotid sheath. The hypoglossal nerve crosses the sheath,
but does not lie within it.

Common carotid artery

The right common carotid artery arises at the bifurcation of the brachiocephalic
trunk, the left common carotid arises from the arch of the aorta. Both terminate at
the level of the upper border of the thyroid cartilage (the lower border of the third
cervical vertebra) by dividing into the internal and external carotid arteries.

Left common carotid artery


This vessel arises immediately to the left and slightly behind the origin of the
brachiocephalic trunk. Its thoracic portion is 2.5- 3.5 cm in length and runs
superolaterally to the sternoclavicular joint.

In the thorax
The vessel is in contact, from below upwards, with the trachea, left recurrent
laryngeal nerve, left margin of the oesophagus. Anteriorly the left brachiocephalic
vein runs across the artery, and the cardiac branches from the left vagus
descend in front of it. These structures together with the thymus and the anterior
margins of the left lung and pleura separate the artery from the manubrium.

In the neck
The artery runs superiorly deep to sternocleidomastoid and then enters the
anterior triangle. At this point it lies within the carotid sheath with the vagus nerve
and the internal jugular vein. Posteriorly the sympathetic trunk lies between the
vessel and the prevertebral fascia. At the level of C7 the vertebral artery and
thoracic duct lie behind it. The anterior tubercle of C6 transverse process is
prominent and the artery can be compressed against this structure (it
corresponds to the level of the cricoid).
Anteriorly at C6 the omohyoid muscle passes superficial to the artery.
Within the carotid sheath the jugular vein lies lateral to the artery.

Right common carotid artery


The right common carotid arises from the brachiocephalic artery. The right
common carotid artery corresponds with the cervical portion of the left common
carotid, except that there is no thoracic duct on the right. The oesophagus is less
closely related to the right carotid than the left.

Summary points about the carotid anatomy

Path
Passes behind the sternoclavicular joint (12% patients above this level) to the
upper border of the thyroid cartilage, to divide into the external (ECA) and internal
carotid arteries (ICA).

Relations

Level of 6th cervical vertebra crossed by omohyoid


Then passes deep to the thyrohyoid, sternohyoid, sternomastoid muscles.
Passes behind the carotid tubercle (transverse process 6th cervical
vertebra)-NB compression here stops haemorrhage.
The inferior thyroid artery passes posterior to the common carotid artery.
Then : Left common carotid artery crossed by thoracic duct, Right common
carotid artery crossed by recurrent laryngeal nerve

Image sourced from Wikipedia

Rate question: Next question

Com m ent on this question

All contents of this site are © 2012 E-Medical Revision Ltd Terms and Conditions Privacy policy
yomna92009@yahoo.com - MRCS Part A - My account

Reference ranges End and review

Question 12 of 398 Question stats Score: 50%

1
A 7%
A patient has a chest drain insertion. There is fresh blood at the chest drain 2
B 36%
insertion area. Which vessel has been damaged? 3
C 5.5%
D 5% 4
A. Pericardiophrenic artery E 46.5% 5
B. Intercostal vein 6
46.5% of users answered this
C. Right ventricle question correctly
7
D. Vagus artery
8
E. Intercostal artery
9

10
Next question
11

Theme from 2009 Exam 12

Within the intercostal spaces there are thin, strong muscles, intercostal vessels,
nerves and lymphatics. There are 3 intercostal muscle layers corresponding to
the lateral abdominal wall; external, internal, innermost intercostals. At the mid
axillary line there are thin intracostals which is an extension of the internal
intercostal muscle. In each intercostal space lies the neurovascular bundle,
comprising, from superior to inferiorly; the posterior intercostal vein, artery and
nerve, lying protected in the subcostal groove of the rib above and situated
between the second and third layer of the intercostal muscles. These blood
vessels anastomose anteriorly with the anterior intercostal vessels, which arise
from the internal thoracic artery and vein.

Chest drains

There are a number of different indications for chest drain insertion. In general
terms large bore chest drains are preferred for trauma and haemothorax
drainage. Smaller diameter chest drains can be used for pneumothorax or pleural
effusion drainage.

Insertion can be performed either using anatomical guidance or through


ultrasound guidance. In the exam, the anatomical method is usually tested.

It is advised that chest drains are placed in the 'safe triangle'. The triangle is
located in the mid axillary line of the 5th intercostal space. It is bordered by:
Anterior edge latissimus dorsi, the lateral border of pectoralis major, a line
superior to the horizontal level of the nipple, and the apex below the axilla.

Another triangle is situated behind the scapula. It is bounded above by the


trapezius, below by the latissimus dorsi, and laterally by the vertebral border of
the scapula; the floor is partly formed by the rhomboid major. If the scapula is
drawn forward by folding the arms across the chest, and the trunk bent forward,
parts of the sixth and seventh ribs and the interspace between them become
subcutaneous and available for auscultation. The space is therefore known as
the triangle of auscultation.

References
Prof Harold Ellis. The applied anatomy of chest drains insertions. British Journal
of hospital medicine 2007; 68: 44-45

Laws D, Neville E, Duffy J. BTS guidelines for insertion of chest drains.


Thorax,May 2003; 58: ii53-ii59.

Rate question: Next question

Com m ent on this question


yomna92009@yahoo.com - MRCS Part A - My account

Reference ranges End and review

Question 13 of 398 Question stats Score: 53.8%

1
A 19.5%
Two teenagers are playing with an airgun when one accidentally shoots his friend 2
B 13.1%
in the abdomen. He is brought to the emergency department. On examination 3
C 13%
there is a bullet entry point immediately to the right of the rectus sheath at the
D 43% 4
level of the 1st lumbar vertebra. Which of the following structures is most likely to
E 11.4% 5
be injured by the bullet?

43% of users answered this 6


question correctly
A. Head of pancreas 7

B. Right ureter 8

C. Right adrenal gland 9

D. Fundus of the gallbladder 10

E. Gastric antrum 11

12
Next question
13

Theme from September 2011 Exam

The fundus of the gallbladder lies at this level and is the most superficially located
structure.

Levels

Transpyloric plane
Level of the body of L1

Pylorus stomach
Left kidney hilum (L1- left one!)
Right hilum of the kidney (1.5cm lower than the left)
Fundus of the gallbladder
Neck of pancreas
Duodenojejunal flexure
Superior mesenteric artery
Portal vein
Left and right colic flexure
Root of the transverse mesocolon
2nd part of the duodenum
Upper part of conus medullaris
Spleen

Can be identified by asking the supine patient to sit up without using their arms.
The plane is located where the lateral border of the rectus muscle crosses the
costal margin.

Anatomical planes
Subcostal plane Lowest margin of 10th costal cartilage

Intercristal plane Level of body L4 (highest point of iliac crest)

Intertubercular plane Level of body L5

Common level landmarks


Inferior mesenteric artery L3

Bifurcation of aorta into common iliac arteries L4

Formation of IVC L5 (union of common iliac veins)

Diaphragm apertures Vena cava T8


Oesophagus T10
Aortic hiatus T12
yomna92009@yahoo.com - MRCS Part A - My account

Reference ranges End and review

Question 14 of 398 Question stats Score: 50%

1
A 40.3%
Which of the following muscles inserts onto the lesser tuberostiy of the the 2
B 11.4%
humerus? 3
C 10.3%
D 25.1% 4
A. Subscapularis E 12.9% 5
B. Deltoid 6
40.3% of users answered this
C. Supraspinatus question correctly
7
D. Teres minor
8
E. Infraspinatus
9

10
Next question
11

With the exception of subscapularis which inserts into the lesser tuberosity, the 12
muscles of the rotator cuff insert into the greater tuberosity. 13

14
Shoulder joint

Shallow synovial ball and socket type of joint.


It is an inherently unstable joint, but is capable to a wide range of
movement.
Stability is provided by muscles of the rotator cuff that pass from the
scapula to insert in the greater tuberosity (all except sub scapularis-lesser
tuberosity).

Glenoid labrum

Fibrocartilaginous rim attached to the free edge of the glenoid cavity


Tendon of the long head of biceps arises from within the joint from the
supraglenoid tubercle, and is fused at this point to the labrum.
The long head of triceps attaches to the infraglenoid tubercle

Fibrous capsule

Attaches to the scapula external to the glenoid labrum and to the labrum
itself (postero-superiorly)
Attaches to the humerus at the level of the anatomical neck superiorly and
the surgical neck inferiorly
Anteriorly the capsule is in contact with the tendon of subscapularis,
superiorly with the supraspinatus tendon, and posteriorly with the tendons
of infraspinatus and teres minor. All these blend with the capsule towards
their insertion.
Two defects in the fibrous capsule; superiorly for the tendon of biceps.
Anteriorly there is a defect beneath the subscapularis tendon.
The inferior extension of the capsule is closely related to the axillary nerve
at the surgical neck and this nerve is at risk in anteroinferior dislocations. It
also means that proximally sited osteomyelitis may progress to septic
arthritis.

Movements and muscles


Flexion Anterior part of deltoid
Pectoralis major
Biceps
Coracobrachialis

Extension Posterior deltoid


Teres major
Latissimus dorsi

Adduction Pectoralis major


Latissimus dorsi
Teres major
Coracobrachialis

Abduction Mid deltoid


Supraspinatus

Medial rotation Subscapularis


Anterior deltoid
Teres major
Latissimus dorsi

Lateral rotation Posterior deltoid


Infraspinatus
Teres minor

Important anatomical relations


Anteriorly Brachial plexus
Axillary artery and vein

Posterior Suprascapular nerve


Suprascapular vessels

Inferior Axillary nerve


Circumflex humeral vessels

Rate question: Next question

Com m ent on this question

All contents of this site are © 2012 E-Medical Revision Ltd Terms and Conditions Privacy policy
yomna92009@yahoo.com - MRCS Part A - My account

Reference ranges End and review

Question 15 of 398 Question stats Score: 46.7%

1
A 9.1%
Which of the following nerves is not contained within the posterior triangle of the 2
B 27.9%
neck? 3
C 14.4%
D 37.7% 4
A. Accessory nerve E 11% 5
B. Phrenic nerve 6
37.7% of users answered this
C. Greater auricular nerve question correctly
7
D. Ansa cervicalis
8
E. Lesser occiptal nerve
9

10
Next question
11

Ansa cervicalis is a content of the anterior triangle of the neck. 12

13
Posterior triangle of the neck
14

15
Boundaries
Apex Sternocleidomastoid and the Trapezius muscles at the Occipital bone

Anterior Posterior border of the Sternocleidomastoid

Posterior Anterior border of the Trapezius

Base Middle third of the clavicle

Image sourced from Wikipedia

Contents

Nerves Accessory nerve


Phrenic nerve
Three trunks of the brachial plexus
Branches of the cervical plexus: Supraclavicular nerve, transverse cervical
nerve, great auricular nerve, lesser occipital nerve

Vessels External jugular vein


Subclavian artery

Muscles Inferior belly of omohyoid


Scalene

Lymph Supraclavicular
nodes Occipital

Rate question: Next question


yomna92009@yahoo.com - MRCS Part A - My account

Reference ranges End and review

Question 16 of 398 Question stats Score: 43.8%

1
A 48.6%
A 42 year old lady is reviewed in the outpatient clinic following a routine surgical 2
B 22.4%
procedure. She complains of diminished sensation at the lateral aspect of her 3
C 6.7%
foot. Which of the following nerves is likely to be affected?
D 6.3% 4
E 16.1% 5
A. Sural
48.6% of users answered this 6
B. Superficial peroneal question correctly
7
C. Deep peroneal
8
D. Medial plantar
9
E. Lateral Plantar
10

Next question 11

12
Theme from April 2012 Exam 13
The sural nerve supplies the lateral aspect of the foot. It runs alongside the short
14
saphenous vein and may be injured in short saphenous vein surgery.
15

Foot- Cutaneous sensation 16

Region Nerve

Lateral plantar Sural

Dorsum (not 1st web space) Superficial peroneal

1st Web space Deep peroneal

Extremities of toes Medial and lateral plantar nerves

Proximal plantar Tibial

Medial plantar Medial plantar nerve

Lateral plantar Lateral plantar nerve


Image sourced from Wikipedia

Rate question: Next question

Com m ent on this question

All contents of this site are © 2012 E-Medical Revision Ltd Terms and Conditions Privacy policy
yomna92009@yahoo.com - MRCS Part A - My account

Reference ranges End and review

Question 17 of 398 Question stats Score: 41.2%

1
A 8.3%
Which of the following anatomical planes separates the prostate from the rectum? 2
B 49.2%
C 12.7% 3

A. Sibsons fascia D 21.1% 4

B. Denonvilliers fascia E 8.7% 5

C. Levator ani muscle 49.2% of users answered this 6


question correctly
D. Waldeyers fascia 7

E. None of the above 8

9
Next question
10

11
The Denonvilliers fascia separates the rectum from the prostate. Waldeyers
fascia separates the rectum from the sacrum 12

13
Prostate gland
14

15
The prostate gland is approximately the shape and size of a walnut and is located
inferior to the bladder. It is separated from the rectum by Denonvilliers fascia and 16
its blood supply is derived from the internal iliac vessels. The internal sphincter 17
lies at the apex of the gland and may be damaged during prostatic surgery,
affected individuals may complain of retrograde ejaculation.

Summary of prostate gland


Arterial supply Inferior vesical artery (from internal iliac)

Venous Prostatic venous plexus (to paravertebral veins)


drainage

Lymphatic Internal iliac nodes


drainage

Innervation Inferior hypogastric plexus

Dimensions Transverse diameter (4cm)


AP diameter (2cm)
Height (3cm)

Lobes Posterior lobe: posterior to urethra


Median lobe: posterior to urethra, in between ejaculatory ducts

Lateral lobes x 2
Isthmus

Zones Peripheral zone: subcapsular portion of posterior prostate. Most


prostate cancers are here
Central zone
Transition zone
Stroma

Relations
Anterior Pubic symphysis
Prostatic venous plexus

Posterior Denonvilliers fascia


Rectum
Ejaculatory ducts

Lateral Venous plexus (lies on prostate)


Levator ani (immediately below the puboprostatic ligaments)
Image sourced from Wikipedia

Rate question: Next question

Com m ent on this question

All contents of this site are © 2012 E-Medical Revision Ltd Terms and Conditions Privacy policy
yomna92009@yahoo.com - MRCS Part A - My account

Reference ranges End and review

Question 18 of 398 Question stats Score: 38.9%

1
A 47.5%
A 56 year old lady is undergoing an adrenalectomy for Conns syndrome. During 2
B 23.3%
the operation the surgeon damages the middle adrenal artery and haemorrhage 3
C 11.5%
ensues. From which of the following structures does this vessel originate?
D 8.7% 4
E 9% 5
A. Aorta
47.5% of users answered this 6
B. Renal artery question correctly
7
C. Splenic artery
8
D. Coeliac axis
9
E. Superior mesenteric artery
10

Next question 11

12
The middle adrenal artery is usually a branch of the aorta, the lower adrenal 13
artery typically arises from the renal vessels.
14

Adrenal gland anatomy 15

16
Anatomy 17

18
Location Superomedially to the upper pole of each kidney

Relationships of Diaphragm-Posteriorly, Kidney-Inferiorly, Vena Cava-Medially, Hepato-


the right adrenal renal pouch and bare area of the liver-Anteriorly

Relationships of Crus of the diaphragm-Postero- medially, Pancreas and splenic


the left adrenal vessels-Inferiorly, Lesser sac and stomach-Anteriorly

Superior adrenal arteries- from inferior phrenic artery, Middle adrenal


Arterial supply arteries - from aorta, Inferior adrenal arteries -from renal arteries

Venous drainage Via one central vein directly into the IVC
of the right
adrenal

Venous drainage Via one central vein into the left renal vein
of the left adrenal

Rate question: Next question

Com m ent on this question

All contents of this site are © 2012 E-Medical Revision Ltd Terms and Conditions Privacy policy
9/21/12 www.emrcs.com/question/question.php?q=0

yomna92009@yahoo.com - MRCS Part A - My account

Reference ranges End and review

Question 1 of 380 Question stats Score: 100%

1
A 20.1%
A 73 year old lady suffers a fracture at the surgical neck of the humerus. The B 7.8%
decision is made to operate. There are difficulties in reducing the fracture and a
C 6.6%
vessel lying posterior to the surgical neck is injured. Which of the following is this
D 7.6%
vessel most likely to be?
E 57.9%

A. Axillary artery 57.9% of users answered this


question correctly
B. Brachial artery
C. Thoracoacromial artery
D. Transverse scapular artery
E. Posterior circumflex humeral artery

Next question

The circumflex humeral arteries lie at the surgical neck and is this scenario the
posterior circumflex is likely to be injured. The thoracoacromial and transverse
scapular arteries lie more superomedially. The posterior circumflex humeral artery
is a branch of the axillary artery.

Shoulder joint

Shallow synovial ball and socket type of joint.


It is an inherently unstable joint, but is capable to a wide range of
movement.
Stability is provided by muscles of the rotator cuff that pass from the
scapula to insert in the greater tuberosity (all except sub scapularis-lesser
tuberosity).

Glenoid labrum

Fibrocartilaginous rim attached to the free edge of the glenoid cavity


Tendon of the long head of biceps arises from within the joint from the
supraglenoid tubercle, and is fused at this point to the labrum.
The long head of triceps attaches to the infraglenoid tubercle

Fibrous capsule

Attaches to the scapula external to the glenoid labrum and to the labrum
itself (postero-superiorly)
Attaches to the humerus at the level of the anatomical neck superiorly and
the surgical neck inferiorly
Anteriorly the capsule is in contact with the tendon of subscapularis,
superiorly with the supraspinatus tendon, and posteriorly with the tendons
of infraspinatus and teres minor. All these blend with the capsule towards
their insertion.
Two defects in the fibrous capsule; superiorly for the tendon of biceps.
Anteriorly there is a defect beneath the subscapularis tendon.
The inferior extension of the capsule is closely related to the axillary nerve
at the surgical neck and this nerve is at risk in anteroinferior dislocations. It
also means that proximally sited osteomyelitis may progress to septic
arthritis.

Movements and muscles


Flexion Anterior part of deltoid
Pectoralis major
Biceps
Coracobrachialis

www.emrcs.com/question/question.php?q=0 1/2
9/21/12 www.emrcs.com/question/question.php?q=0
Extension Posterior deltoid
Teres major
Latissimus dorsi

Adduction Pectoralis major


Latissimus dorsi
Teres major
Coracobrachialis

Abduction Mid deltoid


Supraspinatus

Medial rotation Subscapularis


Anterior deltoid
Teres major
Latissimus dorsi

Lateral rotation Posterior deltoid


Infraspinatus
Teres minor

Important anatomical relations


Anteriorly Brachial plexus
Axillary artery and vein

Posterior Suprascapular nerve


Suprascapular vessels

Inferior Axillary nerve


Circumflex humeral vessels

Rate question: Next question

Com m ent on this question

All contents of this site are © 2012 E-Medical Revision Ltd Terms and Conditions Privacy policy

www.emrcs.com/question/question.php?q=0 2/2
9/21/12 www.emrcs.com/question/question.php?q=0

yomna92009@yahoo.com - MRCS Part A - My account

Reference ranges End and review

Question 2 of 380 Question stats Score: 50%

1
A 19.1%
Which of the structures listed below lies posterior to the carotid sheath at the level 2
B 11.2%
of the 6th cervical vertebra?
C 42.4%
D 16.8%

A. Hypoglossal nerve E 10.7%

B. Vagus nerve
42.4% of users answered this
C. Cervical sympathetic chain question correctly

D. Ansa cervicalis
E. Glossopharyngeal nerve

Next question

The carotid sheath is crossed anteriorly by the hypoglossal nerves and the ansa
cervicalis. The vagus lies within it. The cervical sympathetic chain lies posteriorly
between the sheath and the prevertebral fascia.

Common carotid artery

The right common carotid artery arises at the bifurcation of the brachiocephalic
trunk, the left common carotid arises from the arch of the aorta. Both terminate at
the level of the upper border of the thyroid cartilage (the lower border of the third
cervical vertebra) by dividing into the internal and external carotid arteries.

Left common carotid artery


This vessel arises immediately to the left and slightly behind the origin of the
brachiocephalic trunk. Its thoracic portion is 2.5- 3.5 cm in length and runs
superolaterally to the sternoclavicular joint.

In the thorax
The vessel is in contact, from below upwards, with the trachea, left recurrent
laryngeal nerve, left margin of the oesophagus. Anteriorly the left brachiocephalic
vein runs across the artery, and the cardiac branches from the left vagus
descend in front of it. These structures together with the thymus and the anterior
margins of the left lung and pleura separate the artery from the manubrium.

In the neck
The artery runs superiorly deep to sternocleidomastoid and then enters the
anterior triangle. At this point it lies within the carotid sheath with the vagus nerve
and the internal jugular vein. Posteriorly the sympathetic trunk lies between the
vessel and the prevertebral fascia. At the level of C7 the vertebral artery and
thoracic duct lie behind it. The anterior tubercle of C6 transverse process is
prominent and the artery can be compressed against this structure (it
corresponds to the level of the cricoid).
Anteriorly at C6 the omohyoid muscle passes superficial to the artery.
Within the carotid sheath the jugular vein lies lateral to the artery.

Right common carotid artery


The right common carotid arises from the brachiocephalic artery. The right
common carotid artery corresponds with the cervical portion of the left common
carotid, except that there is no thoracic duct on the right. The oesophagus is less
closely related to the right carotid than the left.

Summary points about the carotid anatomy

Path
Passes behind the sternoclavicular joint (12% patients above this level) to the
upper border of the thyroid cartilage, to divide into the external (ECA) and internal
carotid arteries (ICA).

www.emrcs.com/question/question.php?q=0 1/2
9/21/12 www.emrcs.com/question/question.php?q=0
Relations

Level of 6th cervical vertebra crossed by omohyoid


Then passes deep to the thyrohyoid, sternohyoid, sternomastoid muscles.
Passes behind the carotid tubercle (transverse process 6th cervical
vertebra)-NB compression here stops haemorrhage.
The inferior thyroid artery passes posterior to the common carotid artery.
Then : Left common carotid artery crossed by thoracic duct, Right common
carotid artery crossed by recurrent laryngeal nerve

Image sourced from Wikipedia

Rate question: Next question

Com m ent on this question

All contents of this site are © 2012 E-Medical Revision Ltd Terms and Conditions Privacy policy

www.emrcs.com/question/question.php?q=0 2/2
yomna92009@yahoo.com - MRCS Part A - My account

Reference ranges End and review

Question 3 of 380 Question stats Score: 33.3%

1
A 12.1%
A sprinter attends A&E with severe leg pain. He had forgotten to warm up and ran 2
B 9.6%
a 100m sprint race. Towards the end of the race he experienced pain in the 3
C 12.6%
posterior aspect of his thigh. The pain worsens, localising to the lateral aspect of
D 11.2%
the knee. The sprinter is unable to flex the knee. What structure has been
E 54.4%
injured?

54.4% of users answered this


question correctly
A. Anterior cruciate ligament
B. Posterior cruciate ligament
C. Semimembranosus tendon
D. Semiteninosus tendon
E. Biceps femoris tendon

Next question

Theme from 2009 Exam

The biceps femoris is commonly injured in sports that require explosive bending
of the knee as seen in sprinting, especially if the athlete has not warmed up first.
Avulsion most commonly occurs where the long head attaches to the ischial
tuberosity. Injuries to biceps femoris are more common than to the other
hamstrings.

Biceps femoris

The biceps femoris is one of the hamstring group of muscles located in the
posterior upper thigh. It has two heads.

Long head
Origin Ischial tuberosity

Insertion Fibular head

Action Knee flexion, lateral rotation tibia, extension hip

Innervation Tibial nerve (L5, S1, S2)

Arterial Profunda femoris artery, inferior gluteal artery, and the superior muscular
supply branches of popliteal artery

Image demonstrating the biceps femoris muscle, with the long head outlined
Image sourced from Wikipedia

Short head
Origin Lateral lip of linea aspera, lateral supracondylar ridge of femur

Insertion Fibular head

Action Knee flexion, lateral rotation tibia

Innervation Common peroneal nerve (L5, S1, S2)

Arterial Profunda femoris artery, inferior gluteal artery, and the superior muscular
supply branches of popliteal artery

Rate question: Next question

Com m ent on this question

All contents of this site are © 2012 E-Medical Revision Ltd Terms and Conditions Privacy policy
yomna92009@yahoo.com - MRCS Part A - My account

Reference ranges End and review

Question 1 of 380 Question stats Score: 100%

1
A 11.5%
A 24 year old man falls and sustains a fracture through his scaphoid bone. From B 13.4%
which of the following areas does the scaphoid derive the majority of its blood
C 9.9%
supply?
D 18.9%

E 46.4%
A. From its proximal medial border
46.4% of users answered this
B. From its proximal lateral border question correctly
C. From its proximal posterior surface
D. From the proximal end
E. From the distal end

Next question

Theme from April 2012 Exam


The blood supply to the scaphoid enters from a small non articular surface near
its distal end. Transverse fractures through the scaphoid therefore carry a risk of
non union.

Scaphoid bone

The scaphoid has a concave articular surface for the head of the capitate and at
the edge of this is a crescentic surface for the corresponding area on the lunate.
Proximally, it has a wide convex articular surface with the radius. It has a distally
sited tubercle that can be palpated. The remaining articular surface is to the
lateral side of the tubercle. It faces laterally and is associated with the trapezium
and trapezoid bones.

The narrow strip between the radial and trapezial surfaces and the tubercle gives
rise to the radial collateral carpal ligament. The tubercle receives part of the flexor
retinaculum. This area is the only part of the scaphoid that is available for the
entry of blood vessels. It is commonly fractured and avascular necrosis may
result.

Scaphoid bone

Image sourced from Wikipedia

Rate question: Next question

Com m ent on this question


yomna92009@yahoo.com - MRCS Part A - My account

Reference ranges End and review

Question 2 of 380 Question stats Score: 50%

1
A 37.5%
Which of the following statements relating to quadratus lumborum is false? 2
B 15.8%
C 16.6%

A. Causes flexion of the thoracic spine D 10.9%

B. Causes the rib cage to be pulled down E 19.2%

C. Innervated by anterior primary rami of T12 and L1-3 37.5% of users answered this
question correctly
D. Attached to the iliac crest
E. Inserts into the 12th rib

Next question

The rectus abdominis causes flexion of the thoracic spine.

Attached to the medial iliac crest and iliolumbar ligament.


Inserts: 12th rib
Action: pulls the rib cage inferiorly. Lateral flexion.
Nerve supply: anterior primary rami of T12 and L1-3

Abdominal wall

The 2 main muscles of the abdominal wall are the rectus abdominis (anterior)
and the quadratus lumborum (posterior).
The remaining abdominal wall consists of 3 muscular layers. Each muscle passes
from the lateral aspect of the quadratus lumborum posteriorly to the lateral margin
of the rectus sheath anteriorly. Each layer is muscular posterolaterally and
aponeurotic anteriorly.

Image sourced from Wikipedia

Muscles of abdominal wall


External Lies most superficially
oblique Originates from 5th to 12th ribs
Inserts into the anterior half of the outer aspect of the iliac crest, linea
alba and pubic tubercle
More medially and superiorly to the arcuate line, the aponeurotic layer
overlaps the rectus abdominis muscle
The lower border forms the inguinal ligament
The triangular expansion of the medial end of the inguinal ligament is
the lacunar ligament.

Internal Arises from the thoracolumbar fascia, the anterior 2/3 of the iliac crest
oblique and the lateral 2/3 of the inguinal ligament
The muscle sweeps upwards to insert into the cartilages of the lower
3 ribs
The lower fibres form an aponeurosis that runs from the tenth costal
cartilage to the body of the pubis
At its lowermost aspect it joins the fibres of the aponeurosis of
transversus abdominis to form the conjoint tendon.

Transversus Innermost muscle


abdominis Arises from the inner aspect of the costal cartilages of the lower 6 ribs
, from the anterior 2/3 of the iliac crest and lateral 1/3 of the inguinal
ligament
Its fibres run horizontally around the abdominal wall ending in an
aponeurosis. The upper part runs posterior to the rectus abdominis.
Lower down the fibres run anteriorly only.
The rectus abdominis lies medially running from the pubic crest and
symphysis to insert into the xiphoid process and 5th, 6th and 7th
costal cartilages. The muscles lies in a aponeurosis as described
above.
Nerve supply: anterior primary rami of T7-12

Surgical notes
During abdominal surgery it is usually necessary to divide either the muscles or
their aponeuroses. During a midline laparotomy it is desirable to divide the
aponeurosis. This will leave the rectus sheath intact above the arcuate line and
the muscles intact below it. Straying off the midline will often lead to damage to
the rectus muscles, particularly below the arcuate line where they may often be in
close proximity to each other.

Rate question: Next question

Com m ent on this question

All contents of this site are © 2012 E-Medical Revision Ltd Terms and Conditions Privacy policy
yomna92009@yahoo.com - MRCS Part A - My account

Reference ranges End and review

Question 3 of 380 Question stats Score: 33.3%

1
A 10%
A 24 year old man falls and lands astride a manhole cover. He suffers from a 2
B 45.5%
injury to the anterior bulbar urethra. Where will the extravasated urine tend to 3
C 23.4%
collect?
D 12.9%

E 8.2%
A. Lesser pelvis
45.5% of users answered this
B. Connective tissue of the scrotum question correctly
C. Deep perineal space
D. Ischiorectal fossa
E. Posterior abdominal wall

Next question

This portion of the urethra is contained between the perineal membrane an the
membranous layer of the superficial fascia. As these are densely adherent to the
ischiopubic rami, extravasated urine cannot pass posteriorly because the 2 layers
are continuous around the superficial transverse perineal muscles.

Lower genitourinary tract trauma

Most bladder injuries occur due to blunt trauma


85% associated with pelvic fractures
Easily overlooked during assessment in trauma
Up to 10% of male pelvic fractures are associated with urethral or bladder
injuries

Types of injury

Urethral injury Mainly in males


Blood at the meatus (50% cases)
There are 2 types:

i.Bulbar rupture
- most common
- straddle type injury e.g. bicycles
- triad signs: urinary retention, perineal haematoma, blood at
the meatus
ii. Membranous rupture
- can be extra or intraperitoneal
- commonly due to pelvic fracture
- Penile or perineal oedema/ hematoma
- PR: prostate displaced upwards (beware co-existing
retroperitoneal haematomas as they may make examination
difficult)

- Investigation: ascending urethrogram


- Management: suprapubic catheter (surgical placement, not
percutaneously)

External genitalia injuries Secondary to injuries caused by penetration, blunt


(i.e., the penis and the trauma, continence- or sexual pleasure-enhancing
scrotum) devices, and mutilation

Bladder injury rupture is intra or extraperitoneal


presents with haematuria or suprapubic pain
history of pelvic fracture and inability to void: always
suspect bladder or urethral injury
inability to retrieve all fluid used to irrigate the bladder
through a Foley catheter indicates bladder injury
investigation- IVU or cystogram
management: laparotomy if intraperitoneal,
conservative if extraperitoneal
yomna92009@yahoo.com - MRCS Part A - My account

Reference ranges End and review

Question 4 of 380 Question stats Score: 25%

1
A 9%
A 73 year old man presents with symptoms of mesenteric ischaemia. As part of 2
B 11.9%
his diagnostic work up a diagnostic angiogram is performed .The radiologist is 3
C 9%
attempting to cannulate the coeliac axis from the aorta. At which of the following
D 8.1% 4
vertebral levels does this is usually originate?
E 61.9%

A. T10 61.9% of users answered this


question correctly
B. L2
C. L3
D. T8
E. T12

Next question

Coeliac trunk branches:

Left Hand Side (LHS)

Left gastric
Hepatic
Splenic

The coeliac axis branches off the aorta at T12.

Coeliac axis

The coeliac axis has three main branches.

Left gastric
Hepatic: branches-Right Gastric, Gastroduodenal, Right Gastroepiploic,
Superior Pancreaticoduodenal, Cystic.
Splenic: branches- Pancreatic, Short Gastric, Left Gastroepiploic

It occasionally gives off one of the inferior phrenic arteries.

Image sourced from Wikipedia

Relations
Anteriorly Lesser omentum

Right Right coeliac ganglion and caudate process of liver

Left Left coeliac ganglion and gastric cardia

Inferiorly Upper border of pancreas and renal vein


yomna92009@yahoo.com - MRCS Part A - My account

Reference ranges End and review

Question 5 of 380 Question stats Score: 20%

1
A 13.1%
A 45 year old man presents with a lipoma located posterior to the posterior border 2
B 48.8%
of the sternocleidomastoid muscle, approximately 4cm superior to the middle third 3
C 12.6%
of the clavicle. During surgical excision of the lesion troublesome bleeding is
D 8.7% 4
encountered. Which of the following is the most likely source?
E 16.8% 5

A. Internal jugular vein 48.8% of users answered this


question correctly
B. External jugular vein
C. Common carotid artery
D. Vertebral artery
E. Second part of the subclavian artery

Next question

The external jugular vein runs obliquely in the superficial fascia of the posterior
triangle. It drains into the subclavian vein. During surgical exploration of this area
the external jugular vein may be injured and troublesome bleeding may result.
The internal jugular vein and carotid arteries are located in the anterior triangle.
The third, and not the second, part of the subclavian artery is also a content of
the posterior triangle

Posterior triangle of the neck

Boundaries
Apex Sternocleidomastoid and the Trapezius muscles at the Occipital bone

Anterior Posterior border of the Sternocleidomastoid

Posterior Anterior border of the Trapezius

Base Middle third of the clavicle

Image sourced from Wikipedia

Contents

Nerves Accessory nerve


Phrenic nerve
Three trunks of the brachial plexus
Branches of the cervical plexus: Supraclavicular nerve, transverse cervical
nerve, great auricular nerve, lesser occipital nerve

Vessels External jugular vein


Subclavian artery
yomna92009@yahoo.com - MRCS Part A - My account

Reference ranges End and review

0/3 Question 6-8 of 380 Question stats Score: 12.5%

Average score for registered users: 1

Theme: Levels of spinal injury 2


6 56.7%
3
7 57%
A. C2
8 72.7% 4
B. C3
5
C. C4
6-8 0/3
D. C5
E. C6
F. L1
G. L2
H. L3
I. L4
J. L5

Please select the most likely spinal level for the injury described. Each option may
be used once, more than once or not at all.

6. A 62 year old male complains of back pain. He has had a recent fall.
Walking causes pain of the left lower leg. On examination he is noted
to have reduced sensation over the knee.

You answered L5

The correct answer is L3

Sensation over the knee is equivalent to the L3 dermatome.

7. A 42 year old woman is found to have a burst fracture of the C5


vertebral body. After a few months where would the level of injury be?

You answered L5

The correct answer is C6

A C5 burst fracture usually injures the C6 spinal cord situated at the


C5 vertebrae and also the C4 spinal roots that exits the spinal
column between the C4 and C5 vertebra. Such an injury should
cause a loss of sensations in C4 dermatome and weak deltoids. Due
to oedema , the biceps (C5) may be initially weak but should recover.
The wrist extensors (C6), however, should remain weak and
sensation at and below C6 should be severely compromised. A
neurosurgeon would conclude that there is a burst fracture at C5
from the x-rays, an initial sensory level at C4 (the first abnormal
sensory dermatome) and the partial loss of deltoids and biceps would
imply a motor level at C4 (the highest abnormal muscle level). Over
time, as the patient recovers the C4 roots and the C5 spinal cord,
both the sensory level and motor level should end up at C6. Such
recovery is often attributed to 'root' recovery.

8. A 56 year old man suddenly develops severe back pain. His pain has
a radicular pattern. On examination he is unable to extend his great
toe.

You answered L4

The correct answer is L5

Extensor hallucis longus is derived from L5 and loss of EHL function


is a useful test to determine whether this level is involved.
Next question

Spinal disorders

Dorsal column lesion Loss vibration and proprioception


Tabes dorsalis, SACD

Spinothalamic tract Loss of pain, sensation and temperature


lesion

Central cord lesion Flaccid paralysis of the upper limbs

Osteomyelitis Normally progressive


Staph aureus in IVDU, normally cervical region affected
Fungal infections in immunocompromised
Thoracic region affected in TB

Infarction spinal cord Dorsal column signs (loss of proprioception and fine
discrimination

Cord compression UMN signs


Malignancy
Haematoma
Fracture

Brown-sequard Hemisection of the spinal cord


syndrome Ipsilateral paralysis
Ipsilateral loss of proprioception and fine discrimination
Contralateral loss of pain and temperature
Image sourced from Wikipedia
Image sourced from Wikipedia

Dermatomes

C2 to C4 The C2 dermatome covers the occiput and the top part of the
neck. C3 covers the lower part of the neck to the clavicle. C4 covers the
area just below the clavicle.
C5 to T1 Situated in the arms. C5 covers the lateral arm at and above the
elbow. C6 covers the forearm and the radial (thumb) side of the hand. C7 is
the middle finger, C8 is the lateral aspects of the hand, and T1 covers the
medial side of the forearm.
T2 to T12 The thoracic covers the axillary and chest region. T3 to T12
covers the chest and back to the hip girdle. The nipples are situated in the
middle of T4. T10 is situated at the umbilicus. T12 ends just above the hip
girdle.
L1 to L5 The cutaneous dermatome representing the hip girdle and groin
area is innervated by L1 spinal cord. L2 and 3 cover the front part of the
thighs. L4 and L5 cover medial and lateral aspects of the lower leg.
S1 to S5 S1 covers the heel and the middle back of the leg. S2 covers the
back of the thighs. S3 cover the medial side of the buttocks and S4-5
covers the perineal region. S5 is of course the lowest dermatome and
represents the skin immediately at and adjacent to the anus.

Myotomes

Upper limb
Elbow flexors/Biceps C5

Wrist extensors C6

Elbow extensors/Triceps C7

Long finger flexors C8

Small finger abductors T1

Lower limb
Hip flexors (psoas) L1 and L2

Knee extensors (quadriceps) L3

Ankle dorsiflexors (tibialis anterior) L4 and L5

Toe extensors (hallucis longus) L5

Ankle plantar flexors (gastrocnemius) S1

The anal sphincter is innervated by S2,3,4

Rate question: Next question

Com m ent on this question

All contents of this site are © 2012 E-Medical Revision Ltd Terms and Conditions Privacy policy
yomna92009@yahoo.com - MRCS Part A - My account

Reference ranges End and review

Question 9 of 380 Question stats Score: 11.1%

1
A 12.2%
The sciatic nerve lies deep to the following structures except: 2
B 25.5%
C 17.9% 3

A. Gluteus maximus D 27.3% 4

B. The femoral cutaneous nerve E 17.2% 5

C. Long head of biceps femoris 27.3% of users answered this 6-8 0/3
question correctly 9
D. Gluteus medius
E. Branch of the inferior gluteal artery

Next question

The gluteus medius does not extend around to the sciatic nerve.

Sciatic nerve

Origin Spinal nerves L4 - S3

Articular Branches Hip joint

Muscular branches in Semitendinosus


upper leg Semimembranosus
Biceps femoris
Part of adductor magnus

Cutaneous sensation Posterior aspect of thigh


Gluteal region
Entire lower leg (except the medial aspect)

Terminates At the upper part of the popliteal fossa by dividing into the tibial
and peroneal nerves

The nerve to the short head of the biceps femoris comes from the common
peroneal part of the sciatic and the other muscular branches arise from the
tibial portion.
The tibial nerve goes on to innervate all muscles of the foot except the
extensor digitorum brevis (which is innervated by the common peroneal
nerve).

Rate question: Next question

Com m ent on this question

All contents of this site are © 2012 E-Medical Revision Ltd Terms and Conditions Privacy policy
yomna92009@yahoo.com - MRCS Part A - My account

Reference ranges End and review

Question 10 of 380 Question stats Score: 10%

1
A 14.7%
Which of the following upper limb muscles is not innervated by the radial nerve? 2
B 48.2%
C 11.9% 3

A. Extensor carpi ulnaris D 11.7% 4

B. Abductor digit minimi E 13.5% 5

C. Anconeus 48.2% of users answered this 6-8 0/3


question correctly 9
D. Supinator
E. Brachioradialis 10

Next question

Mnemonic for radial nerve muscles: BEST

B rachioradialis
E xtensors
S upinator
T riceps

Abductor digiti minimi is innervated by the ulnar nerve.

Radial nerve

Continuation of posterior cord of the brachial plexus (root values C5 to T1)

Path

In the axilla: lies posterior to the axillary artery on subscapularis, latissimus


dorsi and teres major.
Enters the arm between the brachial artery and the long head of triceps
(medial to humerus).
Spirals around the posterior surface of the humerus in the groove for the
radial nerve.
At the distal third of the lateral border of the humerus it then pierces the
intermuscular septum and descends in front of the lateral epicondyle.
At the lateral epicondyle it lies deeply between brachialis and
brachioradialis where it then divides into a superficial and deep terminal
branch.
Deep branch crosses the supinator to become the posterior interosseous
nerve.

In the image below the relationships of the radial nerve can be appreciated
Image sourced from Wikipedia

Regions innervated

Motor (main Triceps


nerve) Anconeus
Brachioradialis
Extensor carpi radialis

Motor Extensor carpi ulnaris


(posterior Extensor digitorum
interosseous Extensor indicis
branch Extensor digiti minimi
Extensor pollicis longus and brevis
Abductor pollicis longus

Sensory The area of skin supplying the proximal phalanges on the dorsal
aspect of the hand is supplied by the radial nerve (this does not
apply to the little finger and part of the ring finger)

Muscular innervation and effect of denervation


Anatomical Muscle affected Effect of paralysis
location

Shoulder Long head of triceps Minor effects on shoulder stability in


abduction

Arm Triceps Loss of elbow extension

Forearm Supinator Weakening of supination of prone hand


Brachioradialis and elbow flexion in mid prone position
Extensor carpi
radialis longus and
brevis

The cutaneous sensation of the upper limb- illustrating the contribution of the
radial nerve

Image sourced from Wikipedia


yomna92009@yahoo.com - MRCS Part A - My account

Reference ranges End and review

Question 11 of 380 Question stats Score: 9.1%

1
A 11.4%
Which of the following forms the floor of the anatomical snuffbox? 2
B 4.8%
C 10% 3

A. Radial artery D 66.2% 4

B. Cephalic vein E 7.6% 5

C. Extensor pollicis brevis 66.2% of users answered this 6-8 0/3


question correctly 9
D. Scaphoid bone
E. Cutaneous branch of the radial nerve 10

11
Next question

The scaphoid bone forms the floor of the anatomical snuffbox. The cutaneous
branch of the radial nerve is much more superficially and proximally located.

Anatomical snuffbox

Posterior border Tendon of extensor pollicis longus

Anterior border Tendons of extensor pollicis brevis and abductor pollicis longus

Proximal border Styloid process of the radius

Distal border Apex of snuffbox triangle

Floor Trapezium and scaphoid

Content Radial artery

Image showing the anatomical snuffbox

Image sourced from Wikipedia

Rate question: Next question

Com m ent on this question

All contents of this site are © 2012 E-Medical Revision Ltd Terms and Conditions Privacy policy
yomna92009@yahoo.com - MRCS Part A - My account

Reference ranges End and review

Question 12 of 380 Question stats Score: 16.7%

1
A 21.7%
During a liver resection a surgeon performs a pringles manoeuvre to control 2
B 10.3%
bleeding. Which of the following structures will lie posterior to the epiploic foramen 3
C 6.4%
at this level?
D 9.3% 4
E 52.3% 5
A. Hepatic artery
52.3% of users answered this 6-8 0/3
B. Cystic duct question correctly 9
C. Greater omentum
10
D. Superior mesenteric artery
11
E. Inferior vena cava
12

Next question

Bleeding from liver trauma or a difficult cholecystectomy can be controlled


with a vascular clamp applied at the epiploic foramen.

The epiploic foramen has the following boundaries:


Anteriorly (in the free edge of the lesser omentum): Bile duct to the right, portal
vein behind and hepatic artery to the left.
PosteriorlyInferior vena cava
Inferiorly1st part of the duodenum
SuperiorlyCaudate process of the liver

Liver

Structure of the liver

Right lobe Supplied by right hepatic artery


Contains Couinard segments V to VIII (-/+Sg I)

Left lobe Supplied by the left hepatic artery


Contains Couinard segments II to IV (+/- Sg1)

Quadrate Part of the right lobe anatomically, functionally is part of the left
lobe Couinard segment IV
Porta hepatis lies behind
On the right lies the gallbladder fossa
On the left lies the fossa for the umbilical vein

Caudate lobe Supplied by both right and left hepatic arteries


Couinard segment I
Lies behind the plane of the porta hepatis
Anterior and lateral to the inferior vena cava
Bile from the caudate lobe drains into both right and left hepatic
ducts

Detailed knowledge of Couinard segments is not required for MRCS Part A

Between the liver lobules are portal canals which contain the portal triad:
Hepatic Artery, Portal Vein, tributary of Bile Duct.

Relations of the liver


Anterior Postero inferiorly

Diaphragm Oesophagus

Xiphoid process Stomach

Duodenum
Duodenum

Hepatic flexure of colon

Right kidney

Gallbladder

Inferior vena cava

Porta hepatis
Location Postero inferior surface, it joins nearly at right angles with the left sagittal fossa,
and separates the caudate lobe behind from the quadrate lobe in front

Transmits Common hepatic duct


Hepatic artery
Portal vein
Sympathetic and parasympathetic nerve fibres
Lymphatic drainage of the liver (and nodes)

Ligaments

Falciform 2 layer fold peritoneum from the umbilicus to anterior liver


ligament surface
Contains ligamentum teres (remnant umbilical vein)
On superior liver surface it splits into the coronary and left
triangular ligaments

Ligamentum Joins the left branch of the portal vein in the porta hepatis
teres

Ligamentum Remnant of ductus venosus


venosum

Arterial supply

Hepatic artery

Venous

Hepatic veins
Portal vein

Nervous supply

Sympathetic and parasympathetic trunks of coeliac plexus

Rate question: Next question

Com m ent on this question

All contents of this site are © 2012 E-Medical Revision Ltd Terms and Conditions Privacy policy
yomna92009@yahoo.com - MRCS Part A - My account

Reference ranges End and review

Question 13 of 380 Question stats Score: 15.4%

1
A 7.8%
A 72 year old lady is suspected of having a femoral hernia. At which of the 2
B 9.1%
following sites is it most likely to be identifiable clinically? 3
C 63.4%
D 12.3% 4
A. Mid inguinal point E 7.4% 5
B. Above and medial to the pubic tubercle 6-8 0/3
63.4% of users answered this
C. Below and lateral to the pubic tubercle question correctly 9

D. Mid point of the inguinal ligament 10

E. 3 cm superomedially to the superficial inguinal ring 11

12
Next question
13

Femoral hernias exit the femoral canal below and lateral to the pubic tubercle.
Femoral hernia occur mainly in women due to their difference in pelvic anatomy.
They are at high risk of strangulation and therefore should be repaired.

Femoral canal

The femoral canal lies at the medial aspect of the femoral sheath. The femoral
sheath is a fascial tunnel containing both the femoral artery laterally and femoral
vein medially. The canal lies medial to the vein.

Borders of the femoral canal


Laterally Femoral vein

Medially Lacunar ligament

Anteriorly Inguinal ligament

Posteriorly Pectineal ligament

Image showing dissection of femoral canal


yomna92009@yahoo.com - MRCS Part A - My account

Reference ranges End and review

Question 14 of 380 Question stats Score: 14.3%

1
A 17.7%
Which muscle is responsible for causing flexion of the distal interphalangeal joint 2
B 9.1%
of the ring finger? 3
C 7%
D 57.2% 4
A. Flexor digitorum superficialis E 9% 5
B. Lumbricals 6-8 0/3
57.2% of users answered this
C. Palmar interossei question correctly 9

D. Flexor digitorum profundus 10

E. Flexor digiti minimi brevis 11

12
Next question
13

14
Flexor digitorum superficialis and flexor digitorum profundus are responsible for
causing flexion. The superficialis tendons insert on the bases of the middle
phalanges; the profundus tendons insert on the bases of the distal phalanges.
Both tendons flex the wrist, MCP and PIP joints; however, only the profundus
tendons flex the DIP joints.

Hand

Anatomy of the hand

Bones 8 Carpal bones


5 Metacarpals
14 phalanges

Intrinsic 7 Interossei - Supplied by ulnar nerve


Muscles
3 palmar-adduct fingers
4 dorsal- abduct fingers

Intrinsic Lumbricals
muscles
Flex MCPJ and extend the IPJ.
Origin deep flexor tendon and insertion dorsal extensor hood
mechanism.
Innervation: 1st and 2nd- median nerve, 3rd and 4th- deep
branch of the ulnar nerve.

Thenar Abductor pollicis brevis


eminence Opponens pollicis
Flexor pollicis brevis

Hypothenar Opponens digiti minimi


eminence Flexor digiti minimi brevis
Abductor digiti minimi
yomna92009@yahoo.com - MRCS Part A - My account

Reference ranges End and review

Question 15 of 380 Question stats Score: 13.3%

1
A 9.3%
A 34 year old lady undergoes a thyroidectomy for Graves disease. Post 2
B 6.9%
operatively she develops a tense haematoma in the neck. In which of the following 3
C 68.4%
fascial planes will it be contained?
D 7.5% 4
E 7.9% 5
A. Gerotas fascia
68.4% of users answered this 6-8 0/3
B. Waldeyers fascia question correctly 9
C. Pretracheal fascia
10
D. Sibsons fascia
11
E. Clavipectoral fascia
12

13
Next question
14

The pretracheal fascia encloses the thyroid and is unyielding. Therefore tense 15
haematomas can develop.

Thyroid gland

Right and left lobes connected by isthmus


Surrounded by sheath from pretracheal layer of deep fascia
Apex: Lamina of thyroid cartilage
Base: 4th-5th tracheal ring
Pyramidal lobe: from isthmus
May be attached to foramen caecum at the base of the tongue

Relations

Anteromedially Sternothyroid
Superior belly of omohyoid
Sternohyoid
Anterior aspect of sternocleidomastoid

Posterolaterally Carotid sheath

Medially Larynx
Trachea
Pharynx
Oesophagus
Cricothyroid muscle
External laryngeal nerve (near superior thyroid artery)
Recurrent laryngeal nerve (near inferior thyroid artery)

Posterior Parathyroid glands


Anastomosis of superior and inferior thyroid arteries

Isthmus Anteriorly: Sternothyroids, sternohyoids, anterior jugular veins


Posteriorly: 2nd, 3rd, 4th tracheal rings (attached via Ligament of
Berry)

Blood Supply

Arterial Superior thyroid artery (1st branch of external carotid)


Inferior thyroid artery (from thyrocervical trunk)
Thyroidea ima (in 10% of population -from brachiocephalic artery or aorta)

Venous Superior and middle thyroid veins - into the IJV


Inferior thyroid vein - into the brachiocephalic veins
yomna92009@yahoo.com - MRCS Part A - My account

Reference ranges End and review

Question 16 of 380 Question stats Score: 12.5%

1
A 10.5%
A 32 year old lady complains of carpal tunnel syndrome. The carpal tunnel is 2
B 36.5%
explored surgically. Which of the following structures will lie in closest proximity to 3
C 16.9%
the hamate bone within the carpal tunnel?
D 25.1% 4
E 11% 5
A. The tendon of abductor pollicis longus
36.5% of users answered this 6-8 0/3
B. The tendons of flexor digitorum profundus question correctly 9
C. The tendons of flexor carpi radialis longus
10
D. Median nerve
11
E. Radial artery
12

13
Next question
14

The carpal tunnel contains nine flexor tendons: 15

16
Flexor digitorum profundus
Flexor digitorum superficialis
Flexor pollicis longus

The tendon of flexor digitorum profundus lies deepest in the tunnel and will thus
lie nearest to the hamate bone.

Carpal bones

Diagrammatic image of carpal bones

Image sourced from Wikipedia

Key to image
A Scaphoid

B Lunate

C Triquetrum

D Pisiform

E Trapezium
E Trapezium

F Trapezoid

G Capitate

H Hamate

1 Radius

2 Ulna

3 Metacarpals

No tendons attach to: Scaphoid, lunate, triquetrum (stabilised by ligaments)

Rate question: Next question

Com m ent on this question

All contents of this site are © 2012 E-Medical Revision Ltd Terms and Conditions Privacy policy
yomna92009@yahoo.com - MRCS Part A - My account

Reference ranges End and review

Question 17 of 380 Question stats Score: 11.8%

1
A 46.8%
A 45 year old man sustains a significant head injury and a craniotomy is 2
B 13.8%
performed. The sigmoid sinus is bleeding profusely, into which of the following 3
C 13.9%
structures does it drain?
D 16.7% 4
E 8.7% 5
A. Internal jugular vein
46.8% of users answered this 6-8 0/3
B. Straight sinus question correctly 9
C. Petrosal sinus
10
D. Inferior sagittal sinus
11
E. External jugular vein
12

13
Next question
14

The sigmoid sinus is joined by the inferior petrosal sinus to drain into the internal 15
jugular vein. 16

17
Cranial venous sinuses

The cranial venous sinuses are located within the dura mater. They have no
valves which is important in the potential for spreading sepsis. They eventually
drain into the internal jugular vein.

They are:

Superior sagittal sinus


Inferior sagittal sinus
Straight sinus
Transverse sinus
Sigmoid sinus
Confluence of sinuses
Occipital sinus
Cavernous sinus

Topography of cranial venous sinuses

Image sourced from Wikipedia

Rate question: Next question

Com m ent on this question


yomna92009@yahoo.com - MRCS Part A - My account

Reference ranges End and review

Question 18 of 380 Question stats Score: 11.1%

1
A 52.7%
In which of the following cranial bones does the foramen spinosum lie? 2
B 7.3%
C 21.1% 3

A. Sphenoid bone D 9.4% 4

B. Frontal bone E 9.6% 5

C. Temporal bone 52.7% of users answered this 6-8 0/3


question correctly 9
D. Occipital bone
E. Parietal bone 10

11
Next question 12

13
The foramen spinosum (which transmits the middle meningeal artery and vein)
14
lies in the sphenoid bone.
15

Foramina of the base of the skull 16

17
Foramen Location Contents
18
Foramen Sphenoid Otic ganglion
ovale bone V3 (Mandibular nerve:3rd branch of
trigeminal)
Accessory meningeal artery
Lesser petrosal nerve
Emissary veins

Foramen Sphenoid Middle meningeal artery


spinosum bone Meningeal branch of the Mandibular nerve

Foramen Sphenoid Maxillary nerve (V2)


rotundum bone

Foramen Sphenoid Base of the medial pterygoid plate.


lacerum bone Internal carotid artery
Nerve and artery of the pterygoid canal

Jugular Temporal Anterior: inferior petrosal sinus


foramen bone Intermediate: glossopharyngeal, vagus, and accessory nerves.
Posterior: sigmoid sinus (becoming the internal jugular vein) and
some meningeal branches from the occipital and ascending
pharyngeal arteries.

Foramen Occipital Anterior and posterior spinal arteries


magnum bone Vertebral arteries
Medulla oblongata

Stylomastoid Temporal Stylomastoid artery


foramen bone Facial nerve

Superior Sphenoid Oculomotor nerve (III)


orbital bone trochlear nerve (IV)
fissure lacrimal, frontal and nasociliary branches of ophthalmic nerve
(V1)
abducent nerve (VI)
Superior and inferior ophthalmic vein

Base of skull anatomical overview


yomna92009@yahoo.com - MRCS Part A - My account

Reference ranges End and review

Question 19 of 380 Question stats Score: 10.5%

1
A 11.1%
Which of the following is not considered a major branch of the descending 2
B 8.8%
thoracic aorta? 3
C 61%
D 10.6% 4
A. Bronchial artery E 8.5% 5
B. Mediastinal artery 6-8 0/3
61% of users answered this
C. Inferior thyroid artery question correctly 9

D. Posterior intercostal artery 10

E. Oesophageal artery 11

12
Next question
13

14
The inferior thyroid artery is usually derived from the thyrocervical trunk, a branch
of the subclavian artery. 15

16
Thoracic aorta
17

18
Origin T4
19
Terminates T12

Relations Anteriorly (from top to bottom)-root of the left lung, the pericardium, the
oesophagus, and the diaphragm
Posteriorly-vertebral column, azygos vein
Right- hemiazygos veins, thoracic duct
Left- left pleura and lung

Branches Lateral segmental branches: Posterior intercostal arteries


Lateral visceral: Bronchial arteries supply bronchial walls and lung
excluding the alveoli
Midline branches: Oesophageal arteries

Rate question: Next question

Com m ent on this question

All contents of this site are © 2012 E-Medical Revision Ltd Terms and Conditions Privacy policy
yomna92009@yahoo.com - MRCS Part A - My account

Reference ranges End and review

Question 20 of 380 Question stats Score: 10%

1
A 13.2%
An 18 year old lady with troublesome hyperhidrosis of the hands and arms is due 2
B 29.5%
to undergo a sympathectomy to treat the condition. Which of the following should 3
C 20.7%
the surgeons divide to most effectively treat her condition?
D 19.4% 4
E 17.2% 5
A. Sympathetic ganglia at T1, T2 and T3
29.5% of users answered this 6-8 0/3
B. Sympathetic ganglia at T2 and T3 question correctly 9
C. Sympathetic ganglia at T1 and T2
10
D. Stellate ganglion
11
E. Superior cervical ganglion
12

13
Next question
14

To treat hyperhidrosis the sympathetic ganglia at T2 and T3 should be divided. 15


Dividing the other structures listed would either carry a risk of Horners syndrome 16
or be ineffective.
17

Sympathetic nervous system- anatomy 18

19
The cell bodies of the pre-ganglionic efferent neurones lie in the lateral horn of
20
the grey matter of the spinal cord in the thoraco-lumbar regions.
The pre-ganglionic efferents leave the spinal cord at levels T1-L2. These pass to
the sympathetic chain.
Lateral branches of the sympathetic chain connect it to every spinal nerve. These
post ganglionic nerves will pass to structures that receive sympathetic innervation
at the periphery.

Sympathetic chains
These lie on the vertebral column and run from the base of the skull to the
coccyx.
Cervical Lie anterior to the transverse processes of the cervical vertebrae and posterior to
region the carotid sheath.

Thoracic Lie anterior to the neck of the upper ribs and and lateral sides of the lower
region thoracic vertebrae.They are covered by the parietal pleura

Lumbar Enter by passing posterior to the median arcuate ligament. Lie anteriorly to the
region vertebrae and medial to psoas major.

Sympathetic ganglia

Superior cervical ganglion lies anterior to C2 and C3.


Middle cervical ganglion (if present) C6
Stellate ganglion- anterior to transverse process of C7, lies posterior to the
subclavian artery, vertebral artery and cervical pleura.
Thoracic ganglia are segmentally arranged.
There are usually 4 lumbar ganglia.

Clinical importance

Interruption of the head and neck supply of the sympathetic nerves will
result in an ipsilateral Horners syndrome.
For treatment of hyperhidrosis the sympathetic denervation can be
achieved by removing the second and third thoracic ganglia with their rami.
Removal of T1 will cause a Horners syndrome and is therefore not
performed.
In patients with vascular disease of the lower limbs a lumbar sympathetomy
may be performed, either radiologically or (more rarely now) surgically. The
ganglia of L2 and below are disrupted. If L1 is removed then ejaculation
may be compromised (and little additional benefit conferred as the
preganglionic fibres do not arise below L2.
yomna92009@yahoo.com - MRCS Part A - My account

Reference ranges End and review

Question 21 of 380 Question stats Score: 9.5%

1
A 16.3%
A 44 year old lady is recovering following a transphenoidal hypophysectomy. 2
B 24.3%
Unfortunately there is a post operative haemorrhage. Which of the following 3
C 44%
features is most likely to occur initially?
D 7.5% 4
E 8% 5
A. Cavernous sinus thrombosis
44% of users answered this 6-8 0/3
B. Abducens nerve palsy question correctly 9
C. Bi-temporal homonymous hemianopia
10
D. Inferior homonymous hemianopia
11
E. Central retinal vein occlusion
12

13
Next question
14

The pituitary is covered by a sheath of dura and an expanding haematoma at this 15


site may compress the optic chiasm in the same manner as an expanding pituitary 16
tumour.
17

Pituitary Gland 18

19
The pituitary gland is located within the sella turcica within the sphenoid bone in
20
the middle cranial fossa. It is covered by a dural fold and weighs around 0.5g. It is
21
attached to the hypothalamus by the infundibulum. The anterior pituitary receives
hormonal stimuli from the hypothalamus by way of the hypothalamo-pituitary
portal system. It develops from a depression in the wall of the pharynx (Rathkes
pouch).

Anterior pituitary hormones

Growth hormone
Thyroid stimulating hormone
ACTH
Prolactin
LH and FSH
Melanocyte releasing hormone

Posterior pituitary hormones

Oxytocin
Anti diuretic hormone

Rate question: Next question

Com m ent on this question

All contents of this site are © 2012 E-Medical Revision Ltd Terms and Conditions Privacy policy
yomna92009@yahoo.com - MRCS Part A - My account

Reference ranges End and review

Question 22 of 380 Question stats Score: 13.6%

1
A 30.8%
During a right hemicolectomy the caecum is mobilised. As the bowel is retracted 2
B 16.4%
medially a vessel is injured, posterior to the colon. Which of the following is the 3
C 6.5%
most likely vessel?
D 15.8% 4
E 30.6% 5
A. Right colic artery
30.6% of users answered this 6-8 0/3
B. Inferior vena cava question correctly 9
C. Aorta
10
D. External iliac artery
11
E. Gonadal vessels
12

13
Next question
14

The gonadal vessels and ureter are important posterior relations that are at risk 15
during a right hemicolectomy. 16

17
Caecum
18

19
Location Proximal right colon below the ileocaecal valve
Intraperitoneal 20

21

Posterior relations Psoas 22


Iliacus
Femoral nerve
Genitofemoral nerve
Gonadal vessels

Anterior relations Greater omentum

Arterial supply Ileocolic artery

Lymphatic drainage Mesenteric nodes accompany the venous drainage

The caecum is the most distensible part of the colon and in complete large
bowel obstruction with a competent ileocaecal valve the most likely site of
eventual perforation.

Rate question: Next question

Com m ent on this question

All contents of this site are © 2012 E-Medical Revision Ltd Terms and Conditions Privacy policy
yomna92009@yahoo.com - MRCS Part A - My account

Reference ranges End and review

Question 23 of 380 Question stats Score: 17.4%

1
A 11.2%
A 53 year old man with a carcinoma of the lower third of the oesophagus is 2
B 22.8%
undergoing an oesophagogastrectomy. As the surgeons mobilise the lower part 3
C 22.5%
of the oesophagus, where are they most likely to encounter the thoracic duct?
D 14.2% 4
E 29.3% 5
A. Anterior to the oesophagus
29.3% of users answered this 6-8 0/3
B. On the left side of the oesophagus question correctly 9
C. On the right side of the oesophagus
10
D. Immediately anterior to the azygos vein
11
E. Posterior to the oesophagus
12

13
Next question
14

The thoracic duct lies posterior to the oesophagus and passes to the left at the 15
level of the Angle of Louis. It enters the thorax at T12 together with the aorta. 16

17
Thoracic duct
18

19
Continuation of the cisterna chyli in the abdomen.
20
Enters the thorax at T12
Lies posterior to the oesophagus for most of its intrathoracic course. 21
Passes to the left at T5.
22
Lymphatics draining the left side of the head and neck join the thoracic
23
duct prior to its insertion into the left brachiocephalic vein.
Lymphatics draining the right side of the head and neck drain via the
subclavian and jugular trunks into the right lymphatic duct and thence into
the mediastinal trunk and eventually the right brachiocephalic vein.
Its location in the thorax makes it prone to injury during oesophageal
surgery. Some surgeons administer cream to patients prior to
oesophagectomy so that it is easier to identify the cut ends of the duct.

Rate question: Next question

Com m ent on this question

All contents of this site are © 2012 E-Medical Revision Ltd Terms and Conditions Privacy policy
yomna92009@yahoo.com - MRCS Part A - My account

Reference ranges End and review

Question 24 of 380 Question stats Score: 16.7%

1
A 42.6%
Which of the following represents the root values of the sciatic nerve? 2
B 11%
C 17.1% 3

A. L4 to S3 D 13.8% 4

B. L1 to L4 E 15.6% 5

C. L3 to S1 42.6% of users answered this 6-8 0/3


question correctly 9
D. S1 to S4
E. L5 to S1 10

11
Next question 12

13
The sciatic nerve most commonly arises from L4 to S3.
14

Sciatic nerve 15

16
Origin Spinal nerves L4 - S3 17
Articular Branches Hip joint 18

Muscular branches in Semitendinosus 19


upper leg Semimembranosus
20
Biceps femoris
Part of adductor magnus 21

22
Cutaneous sensation Posterior aspect of thigh
23
Gluteal region
Entire lower leg (except the medial aspect) 24

Terminates At the upper part of the popliteal fossa by dividing into the tibial
and peroneal nerves

The nerve to the short head of the biceps femoris comes from the common
peroneal part of the sciatic and the other muscular branches arise from the
tibial portion.
The tibial nerve goes on to innervate all muscles of the foot except the
extensor digitorum brevis (which is innervated by the common peroneal
nerve).

Rate question: Next question

Com m ent on this question

All contents of this site are © 2012 E-Medical Revision Ltd Terms and Conditions Privacy policy
yomna92009@yahoo.com - MRCS Part A - My account

Reference ranges End and review

Question 25 of 380 Question stats Score: 16%

1
A 17.6%
The common peroneal nerve, or its branches, supply the following muscles 2
B 14.3%
except: 3
C 9.7%
D 48.3% 4
A. Peroneus longus E 10.2% 5
B. Tibialis anterior 6-8 0/3
48.3% of users answered this
C. Extensor hallucis longus question correctly 9

D. Flexor digitorum brevis 10

E. Extensor digitorum longus 11

12
Next question
13

14
Flexor digitorum is supplied by the tibial nerve.
15

Common peroneal nerve 16

17
Derived from the dorsal divisions of the sacral plexus (L4, L5, S1 and S2).
18

This nerve supplies the skin and fascia of the anterolateral surface of the leg and 19
the dorsum of the foot. It also innervates the muscles of the anterior and peroneal
20
compartments of the leg, extensor digitorum brevis as well as the knee, ankle and
21
foot joints.
22
It is laterally placed within the sciatic nerve. From the bifurcation of the sciatic 23
nerve it passes inferolaterally in the lateral and proximal part of the popliteal
24
fossa, under the cover of biceps femoris and its tendon. To reach the posterior
aspect of the fibular head. It ends by dividing into the deep and superficial 25
peroneal nerves at the point where it winds around the lateral surface of the neck
of the fibula in the body of peroneus longus, approximately 2cm distal to the apex
of the head of the fibula. It is palpable posterior to the head of the fibula.

Branches
In the thigh Nerve to the short head of biceps
Articular branch (knee)

In the popliteal fossa Lateral cutaneous nerve of the calf

Neck of fibula Superficial and deep peroneal nerves

Rate question: Next question

Com m ent on this question

All contents of this site are © 2012 E-Medical Revision Ltd Terms and Conditions Privacy policy
yomna92009@yahoo.com - MRCS Part A - My account

Reference ranges End and review

Question 26 of 380 Question stats Score: 15.4%

1
A 44.1%
An 83 year old lady presents with a femoral hernia and undergoes a femoral 2
B 16.3%
hernia repair. Which of the following forms the posterior wall of the femoral canal? 3
C 10.2%
D 20.8% 4
A. Pectineal ligament E 8.5% 5
B. Lacunar ligament 6-8 0/3
44.1% of users answered this
C. Inguinal ligament question correctly 9

D. Adductor longus 10

E. Sartorius 11

12
Next question
13

14

15

Femoral canal 16

17
The femoral canal lies at the medial aspect of the femoral sheath. The femoral
18
sheath is a fascial tunnel containing both the femoral artery laterally and femoral
vein medially. The canal lies medial to the vein. 19

20
Borders of the femoral canal
21
Laterally Femoral vein
22
Medially Lacunar ligament
23
Anteriorly Inguinal ligament
24
Posteriorly Pectineal ligament
25

Image showing dissection of femoral canal 26


yomna92009@yahoo.com - MRCS Part A - My account

Reference ranges End and review

Question 27 of 380 Question stats Score: 14.8%

1
A 9.9%
A 45 year man presents with hand weakness. He is given a piece of paper to hold 2
B 45.6%
between his thumb and index finger. When the paper is pulled, the patient has 3
C 19.8%
difficulty maintaining a grip. Grip pressure is maintained by flexing the thumb at
D 12.1% 4
the interphalangeal joint. What is the most likely nerve lesion?
E 12.6% 5

45.6% of users answered this 6-8 0/3


A. Posterior interosseous nerve
question correctly 9
B. Deep branch of ulnar nerve
10
C. Anterior interosseous nerve
11
D. Superficial branch of the ulnar nerve
12
E. Radial nerve
13

Next question 14

15
Theme from January 2012 exam 16

17
This is a description of Froment's sign, which tests for ulnar nerve palsy. It mainly
tests for the function of adductor pollicis. This is supplied by the deep branch of 18
the ulnar nerve. Remember the anterior interosseous branch, which innervates 19
the flexor pollicis longus (hence causing flexion of the thumb IP joint), branches
20
off more proximally to the wrist.
21

Ulnar nerve 22

23
Origin
24
C8, T1
25

26
Supplies (no muscles in the upper arm)
27

Flexor carpi ulnaris


Flexor digitorum profundus
Flexor digiti minimi
Abductor digiti minimi
Opponens digiti minimi
Adductor pollicis
Interossei muscle
Third and fourth lumbricals
Palmaris brevis

Path

Posteromedial aspect of ulna to flexor compartment of forearm, then along


the ulnar. Passes beneath the flexor carpi ulnaris muscle, then superficially
through the flexor retinaculum into the palm of the hand.
Image sourced from Wikipedia

Branches
Branch Supplies

Articular branch Flexor carpi ulnaris


Medial half of the flexor digitorum profundus

Palmar cutaneous branch (Arises near Skin on the medial part of the palm
the middle of the forearm)

Dorsal cutaneous branch Dorsal surface of the medial part of the hand

Superficial branch Cutaneous fibres to the anterior surfaces of the


medial one and one-half digits

Deep branch Hypothenar muscles


All the interosseous muscles
Third and fourth lumbricals
Adductor pollicis
Medial head of the flexor pollicis brevis

Effects of injury

Damage at the wrist Wasting and paralysis of intrinsic hand muscles (claw hand)
Wasting and paralysis of hypothenar muscles
Loss of sensation medial 1 and half fingers

Damage at the elbow Radial deviation of the wrist


Clawing less in 3rd and 4th digits

Rate question: Next question

Com m ent on this question

All contents of this site are © 2012 E-Medical Revision Ltd Terms and Conditions Privacy policy
yomna92009@yahoo.com - MRCS Part A - My account

Reference ranges End and review

Question 28 of 380 Question stats Score: 14.3%

1
A 14.5%
Which of the following statements relating to the right phrenic nerve is false? 2
B 26.9%
C 17.2% 3

A. It lies deep to the prevertebral layer of deep cervical fascia D 16.3% 4

B. Crosses posterior to the 2nd part of the subclavian artery E 25% 5

C. It runs on the anterior surface of the scalene muscle 26.9% of users answered this 6-8 0/3
question correctly 9
D. On the right side it leaves the mediastinum via the vena cava hiatus
at a level of T8 10
E. The right phrenic nerve passes over the right atrium 11

12
Next question
13

14

15
Phrenic nerve 16

17
Origin
18

19
C3,4,5
20

21
Supplies
22

23
Diaphragm, sensation central diaphragm and pericardium
24

25
Path
26

27
The phrenic nerve passes with the internal jugular vein across scalenus
anterior. It passes deep to prevertebral fascia of deep cervical fascia. 28

Left: crosses anterior to the 1st part of the subclavian artery.


Right: Anterior to scalenus anterior and crosses anterior to the 2nd part of
the subclavian artery.
On both sides, the phrenic nerve runs posterior to the subclavian vein and
posterior to the internal thoracic artery as it enters the thorax.

Right phrenic nerve

In the superior mediastinum: anterior to right vagus and laterally to superior


vena cava
Middle mediastinum: right of pericardium
It passes over the right atrium to exit the diaphragm at T8

Left phrenic nerve

Passes lateral to the left subclavian artery, aortic arch and left ventricle
Passes anterior to the root of the lung
Pierces the diaphragm alone

Image showing the passage of the phrenic nerve in the neck


yomna92009@yahoo.com - MRCS Part A - My account

Reference ranges End and review

Question 29 of 380 Question stats Score: 13.8%

1
A 16.5%
Which of the following cranial foramina pairings are incorrect? 2
B 13.8%
C 19.6% 3

A. The foramen lacerum and internal carotid artery. D 38.6% 4

B. Foramen ovale and mandibular nerve. E 11.5% 5

C. Optic canal and ophthalmic artery. 38.6% of users answered this 6-8 0/3
question correctly 9
D. Optic canal and ophthalmic nerve.
E. Foramen rotundum and maxillary nerve. 10

11
Next question 12

13
Question derived from 2010 and 2011 exams
14

The optic canal transmits the optic nerve. The ophthalmic nerve traverses the 15
superior orbital fissure. 16

17
Foramina of the base of the skull
18

Foramen Location Contents 19

Foramen Sphenoid Otic ganglion 20


ovale bone V3 (Mandibular nerve:3rd branch of
21
trigeminal)
Accessory meningeal artery
22
Lesser petrosal nerve
Emissary veins 23

Foramen Sphenoid Middle meningeal artery 24


spinosum bone Meningeal branch of the Mandibular nerve
25
Foramen Sphenoid Maxillary nerve (V2)
rotundum bone 26

Foramen Sphenoid Base of the medial pterygoid plate. 27


lacerum bone Internal carotid artery
Nerve and artery of the pterygoid canal 28

Jugular Temporal Anterior: inferior petrosal sinus 29


foramen bone Intermediate: glossopharyngeal, vagus, and accessory nerves.
Posterior: sigmoid sinus (becoming the internal jugular vein) and
some meningeal branches from the occipital and ascending
pharyngeal arteries.

Foramen Occipital Anterior and posterior spinal arteries


magnum bone Vertebral arteries
Medulla oblongata

Stylomastoid Temporal Stylomastoid artery


foramen bone Facial nerve

Superior Sphenoid Oculomotor nerve (III)


orbital bone trochlear nerve (IV)
fissure lacrimal, frontal and nasociliary branches of ophthalmic nerve
(V1)
abducent nerve (VI)
Superior and inferior ophthalmic vein

Base of skull anatomical overview


yomna92009@yahoo.com - MRCS Part A - My account

Reference ranges End and review

Question 30 of 380 Question stats Score: 13.3%

1
A 21.1%
A 10 year old by falls out of a tree has suffers a supracondylar fracture. He 2
B 12.3%
complains of a painful elbow and forearm. There is an obvious loss of pincer 3
C 27.9%
movement involving the thumb and index finger with minimal loss of sensation.
D 5.5% 4
The most likely nerve injury is to the:
E 33.2% 5

27.9% of users answered this 6-8 0/3


A. Ulnar nerve
question correctly 9
B. Radial nerve
10
C. Anterior interosseous nerve
11
D. Axillary nerve damage
12
E. Median nerve damage above the elbow
13

Next question 14

15
The anterior interosseous nerve is a motor branch of the median nerve just below 16
the elbow. When damaged it classically causes:
17
Pain in the forearm 18
Loss of pincer movement of the thumb and index finger (innervates the
19
long flexor muscles of flexor pollicis longus & flexor digitorum profundus of
the index and middle finger) 20
Minimal loss of sensation due to lack of a cutaneous branch 21

22

23
Median nerve
24

25
The median nerve is formed by the union of a lateral and medial root respectively
from the lateral (C5,6,7) and medial (C8 and T1) cords of the brachial plexus; the 26
medial root passes anterior to the third part of the axillary artery. The nerve
27
descends lateral to the brachial artery, crosses to its medial side (usually passing
28
anterior to the artery). It passes deep to the bicipital aponeurosis and the median
cubital vein at the elbow. 29
It passes between the two heads of the pronator teres muscle, and runs on the
30
deep surface of flexor digitorum superficialis (within its fascial sheath).
Near the wrist it becomes superficial between the tendons of flexor digitorum
superficialis and flexor carpi radialis, deep to palmaris longus tendon. It passes
deep to the flexor retinaculum to enter the palm, but lies anterior to the long flexor
tendons within the carpal tunnel.

Branches
Region Branch

Upper No branches, although the nerve commonly communicates with the


arm musculocutaneous nerve

Forearm Pronator teres


Flexor carpi radialis
Palmaris longus
Flexor digitorum superficialis
Flexor pollicis longus
Flexor digitorum profundus (only the radial half)

Distal Palmar cutaneous branch


forearm

Hand Motor supply (LOAF)


(Motor)
Lateral 2 lumbricals
Opponens pollicis
Abductor pollicis brevis
Flexor pollicis brevis

Hand Over thumb and lateral 2 ½ fingers


(Sensory) On the palmar aspect this projects proximally, on the dorsal aspect only
the distal regions are innervated with the radial nerve providing the more
yomna92009@yahoo.com - MRCS Part A - My account

Reference ranges End and review

Question 1 of 350 Question stats Score: 0%

1
A 48.9%
A 66 year old man is undergoing a left nephro-ureterectomy. The surgeons B 9.3%
remove the ureter, which of the following is responsible for the blood supply to the
C 13.8%
proximal ureter?
D 18.2%

E 9.7%
A. Branches of the renal artery
48.9% of users answered this
B. External iliac artery question correctly
C. Internal iliac artery
D. Direct branches from the aorta
E. Common iliac artery

Next question

The proximal ureter is supplied by branches from the renal artery. For the other
feeding vessels - see below.

Ureter

25-35 cm long
Muscular tube lined by transitional epithelium
Surrounded by thick muscular coat. Becomes 3 muscular layers as it
crosses the bony pelvis
Retroperitoneal structure overlying transverse processes L2-L5
Lies anterior to bifurcation of iliac vessels
Blood supply is segmental; renal artery, aortic branches, gonadal
branches, common iliac and internal iliac
Lies beneath the uterine artery

Rate question: Next question

Com m ent on this question

All contents of this site are © 2012 E-Medical Revision Ltd Terms and Conditions Privacy policy
yomna92009@yahoo.com - MRCS Part A - My account

Reference ranges End and review

Question 2 of 350 Question stats Score: 50%

1
A 7.8%
Which of the following structures does not pass behind the lateral malleolus? 2
B 11.4%
C 10.5%

A. Peroneus brevis tendon D 6.8%

B. Sural nerve E 63.5%

C. Short saphenous vein 63.5% of users answered this


question correctly
D. Peroneus longus tendon
E. Tibialis anterior tendon

Next question

Tibialis anterior tendon passes at the medial malleolus.

Lateral malleolus

Structures posterior to the lateral malleolus and superficial to superior


peroneal retinaculum

Sural nerve
Short saphenous vein

Structures posterior to the lateral malleolus and deep to superior


peroneal retinaculum

Peroneus longus tendon


Peroneus brevis tendon

The calcaneofibular ligament is attached at the lateral malleolus

Rate question: Next question

Com m ent on this question

All contents of this site are © 2012 E-Medical Revision Ltd Terms and Conditions Privacy policy
yomna92009@yahoo.com - MRCS Part A - My account

Reference ranges End and review

Question 3 of 350 Question stats Score: 33.3%

1
A 50.8%
A 78 year old man presents with symptoms consistent with intermittent 2
B 14.8%
claudication. To assess the severity of his disease you decide to measure his 3
C 9.6%
ankle brachial pressure index. To do this you will identify the dorsalis pedis artery.
D 12.5%
Which of the following statements relating to this vessel is false?
E 12.2%

A. It originates from the peroneal artery 50.8% of users answered this


question correctly
B. It is crossed by the tendon of extensor hallucis brevis
C. Two veins are usually closely related to it
D. It passes under the inferior extensor retinaculum
E. The tendon of extensor hallucis longus lies medial to it.

Next question

The dorsalis pedis artery is a direct continuation of the anterior tibial artery.

Foot- anatomy

Arches of the foot


The foot is conventionally considered to have two arches.

The longitudinal arch is higher on the medial than on the lateral side. The
posterior part of the calcaneum forms a posterior pillar to support the arch.
The lateral part of this structure passes via the cuboid bone and the lateral
two metatarsal bones. The medial part of this structure is more important.
The head of the talus marks the summit of this arch, located between the
sustentaculum tali and the navicular bone. The anterior pillar of the medial
arch is composed of the navicular bone, the three cuneiforms and the
medial three metatarsal bones.
The transverse arch is situated on the anterior part of the tarsus and the
posterior part of the metatarsus. The cuneiforms and metatarsal bases
narrow inferiorly, which contributes to the shape of the arch.

Intertarsal joints
Sub talar joint Formed by the cylindrical facet on the lower surface of the body of
the talus and the posterior facet on the upper surface of the
calcaneus. The facet on the talus is concave anteroposteriorly, the
other is convex. The synovial cavity of this joint does not
communicate with any other joint.

Talocalcaneonavicular The anterior part of the socket is formed by the concave articular
joint surface of the navicular bone, posteriorly by the upper surface of
the sustentaculum tali. The talus sits within this socket

Calcaneocuboid joint Highest point in the lateral part of the longitudinal arch. The lower
aspect of this joint is reinforced by the long plantar and plantar
calcaneocuboid ligaments.

Transverse tarsal joint The talocalcaneonavicular joint and the calcaneocuboid joint
extend accross the tarsus in an irregular transverse plane,
between the talus and calcaneus behind and the navicular and
cuboid bones in front. This plane is termed the transverse tarsal
joint.

Cuneonavicular joint Formed between the convex anterior surface of the navicular bone
and the concave surface of the the posterior ends of the three
cuneiforms.

Intercuneiform joints Between the three cuneiform bones.

Cuneocuboid joint Between the circular facets on the lateral cuneiform bone and the
cuboid. This joint contributes to the tarsal part of the transverse
arch.

A detailed knowledge of the joints is not required for MRCS Part A. However, the
contribution they play to the overall structure of the foot should be appreciated
Ligaments of the ankle joint and foot

Image sourced from Wikipedia

Muscles of the foot


Muscle Origin Insertion Nerve Action
supply

Abductor Medial side of the calcaneus, Medial side of Medial Abducts the great toe
hallucis flexor retinaculum, plantar the base of plantar
aponeurosis the proximal nerve
phalanx

Flexor Medial process of the Via 4 tendons Medial Flexes all the joints of
digitorum calcaneus, plantar into the plantar the lateral 4 toes
brevis eponeurosis. middle nerve except for the
phalanges of interphalangeal joint.
the lateral 4
toes.

Abductor From the tubercle of the Together with Lateral Abducts the little toe
digit calcaneus and from the flexor digit plantar at the
minimi plantar aponeurosis minimi brevis nerve metatarsophalangeal
into the lateral joint
side of the
base of the
proximal
phalanx of the
little toe

Flexor From the medial side of the Into the Medial Flexes the
hallucis plantar surface of the cuboid proximal plantar metatarsophalangeal
brevis bone, from the adjacent part phalanx of the nerve joint of the great toe.
of the lateral cuneiform bone great toe, the
and from the tendon of tendon
tibialis posterior. contains a
sesamoid
bone

Adductor Arises from two heads. The Lateral side of Lateral Adducts the great toe
hallucis oblique head arises from the the base of plantar towards the second
sheath of the peroneus the proximal nerve toe. Helps maintain
longus tendon, and from the phalanx of the the transverse arch of
plantar surfaces of the bases great toe. the foot.
of the 2nd, 3rd and 4th
metatarsal bones. The
transverse head arises from
the plantar surface of the
lateral 4
metatarsophalangeal joints
and from the deep transverse
metatarsal ligament.

Extensor On the dorsal surface of the Via four thin Deep Extend the
digitorum foot from the upper surface of tendons peroneal metatarsophalangeal
brevis the calcaneus and its which run joint of the medial
associated fascia forward and four toes. It is unable
medially to be to extend the
inserted into interphalangeal joint
the medial without the
four toes. The assistance of the
lateral three lumbrical muscles.
tendons join
with hoods of
extensor
digitorum
longus.
Detailed knowledge of the foot muscles are not needed for the MRCS part A

Nerves in the foot

Lateral plantar nerve


Passes anterolaterally towards the base of the 5th metatarsal between flexor
digitorum brevis and flexor accessorius. On the medial aspect of the lateral
plantar artery. At the base of the 5th metatarsal it splits into superficial and deep
branches.

Medial plantar nerve


Passes forwards with the medial plantar artery under the cover of the flexor
retinaculum to the interval between abductor hallucis and flexor digitorum brevis
on the sole of the foot.

Plantar arteries
Arise under the cover of the flexor retinaculum, midway between the tip of the
medial malleolus and the most prominent part of the medial side of the heel.

Medial plantar artery. Passes forwards medial to medial plantar nerve in the
space between abductor hallucis and flexor digitorum brevis.Ends by
uniting with a branch of the 1st plantar metatarsal artery.
Lateral plantar artery. Runs obliquely across the sole of the foot. It lies
lateral to the lateral plantar nerve. At the base of the 5th metatarsal bone it
arches medially across the foot on the metatarsals

Dorsalis pedis artery


This vessel is a direct continuation of the anterior tibial artery. It commences on
the front of the ankle joint and runs to the proximal end of the first metatarsal
space. Here is gives off the arcuate artery and continues forwards as the first
dorsal metatarsal artery. It is accompanied by two veins throughout its length. It is
crossed by the extensor hallucis brevis

Rate question: Next question

Com m ent on this question

All contents of this site are © 2012 E-Medical Revision Ltd Terms and Conditions Privacy policy
yomna92009@yahoo.com - MRCS Part A - My account

Reference ranges End and review

Question 4 of 350 Question stats Score: 25%

1
A 12.5%
Which of the following is not a content of the anterior triangle of the neck? 2
B 12.9%
C 44.1% 3

A. Vagus nerve D 11.5% 4

B. Submandibular gland E 19%

C. Phrenic nerve 44.1% of users answered this


question correctly
D. Internal jugular vein
E. Hypoglossal nerve

Next question

The phrenic nerve is a content of the posterior triangle. The anterior triangle
contains the carotid sheath and its contents.

Anterior triangle of the neck

Boundaries

Anterior border of the Sternocleidomastoid


Lower border of mandible
Anterior midline

Sub triangles (divided by Digastric above and Omohyoid)

Muscular triangle: Neck strap muscles


Carotid triangle: Carotid sheath
Submandibular Triangle (digastric)

Contents of the anterior triangle


Digastric triangle Submandibular gland
Submandibular nodes
Facial vessels
Hypoglossal nerve

Muscular triangle Strap muscles


External jugular vein

Carotid triangle Carotid sheath (Common carotid, vagus and internal jugular vein)
Ansa cervicalis

Nerve supply to digastric muscle

Anterior: Mylohyoid nerve


Posterior: Facial nerve

Image sourced from Wikipedia

Rate question: Next question


yomna92009@yahoo.com - MRCS Part A - My account

Reference ranges End and review

Question 5 of 350 Question stats Score: 20%

1
A 20.4%
A 32 year old attends neurology clinic complaining of tingling in his hand. He has 2
B 45.8%
radial deviation of his wrist and there is mild clawing of his fingers, with the 3rd 3
C 8.6%
and 4th digits being relatively spared. What is the most likely lesion?
D 12.5% 4
E 12.6% 5
A. Ulnar nerve damage at the wrist
45.8% of users answered this
B. Ulnar nerve damage at the elbow question correctly
C. Radial nerve damage at the elbow
D. Median nerve damage at the wrist
E. Median nerve damage at the elbow

Next question

At the elbow the ulnar nerve lesion affects the flexor carpi ulnaris and flexor
digitorum profundus.

Ulnar nerve

Origin

C8, T1

Supplies (no muscles in the upper arm)

Flexor carpi ulnaris


Flexor digitorum profundus
Flexor digiti minimi
Abductor digiti minimi
Opponens digiti minimi
Adductor pollicis
Interossei muscle
Third and fourth lumbricals
Palmaris brevis

Path

Posteromedial aspect of ulna to flexor compartment of forearm, then along


the ulnar. Passes beneath the flexor carpi ulnaris muscle, then superficially
through the flexor retinaculum into the palm of the hand.
yomna92009@yahoo.com - MRCS Part A - My account

Reference ranges End and review

Question 6 of 350 Question stats Score: 16.7%

1
A 11.6%
A 22 year old man is undergoing an endotracheal intubation. Which of the 2
B 7.8%
following vertebral levels is consistent with the origin of the trachea? 3
C 46.5%
D 20.2% 4
A. C2 E 13.9% 5
B. T1 6
46.5% of users answered this
C. C6 question correctly

D. C4
E. C3

Next question

The trachea commences at C6. It terminates at the level of T5 (or T6 in tall


subjects in deep inspiration).

Trachea

Trachea
Location C6 vertebra to the upper border of T5 vertebra (bifurcation)

Arterial and venous supply Inferior thyroid arteries and the thyroid venous plexus.

Nerve Branches of vagus, sympathetic and the recurrent nerves

Relations in the neck

Anterior(Superior to Isthmus of the thyroid gland


inferior) Inferior thyroid veins
Arteria thyroidea ima (when that vessel exists)
Sternothyroid
Sternohyoid
Cervical fascia
Anastomosing branches between the anterior jugular
veins

Posterior Oesophagus.

Laterally Common carotid arteries


Right and left lobes of the thyroid gland
Inferior thyroid arteries
Recurrent laryngeal nerves

Relations in the thorax

Anterior

Manubrium sterni, the remains of the thymus, the aortic arch, left common
carotid arteries, and the deep cardiac plexus

Lateral

In the superior mediastinum, on the right side is the pleura and right vagus;
on its left side are the left recurrent nerve, the aortic arch, and the left
common carotid and subclavian arteries.

Rate question: Next question

Com m ent on this question


yomna92009@yahoo.com - MRCS Part A - My account

Reference ranges End and review

Question 7 of 350 Question stats Score: 28.6%

1
A 11.8%
A young child undergoes a difficult craniotomy for fulminant mastoiditis and 2
B 17%
associated abscess. During the procedure the trigeminal nerve is severely 3
C 14.6%
damaged within Meckels cave. Which of the following deficits is least likely to be
D 26.6% 4
present?
E 29.9% 5

29.9% of users answered this 6


A. Anaesthesia over the ipsilateral anterior aspect of the scalp
question correctly
7
B. Loss of the corneal reflex
C. Weakness of the ipsilateral masseter muscle
D. Anaesthesia of the anterior aspect of the tongue
E. Anaesthesia over the entire ipsilateral side of the face

Next question

The angle of the jaw is not innervated by sensory fibres of the trigeminal nerve
and is spared in this type of injury.

Trigeminal nerve

The trigeminal nerve is the main sensory nerve of the head. In addition to its
major sensory role, it also innervates the muscles of mastication.

Distribution of the trigeminal nerve

Sensory Scalp
Face
Oral cavity (and teeth)
Nose and sinuses
Dura mater

Motor Muscles of mastication


Mylohyoid
Anterior belly of digastric
Tensor tympani
Tensor palati

Autonomic connections (ganglia) Ciliary


Sphenopalatine
Otic
Submandibular

Path

Originates at the pons


Sensory root forms the large, crescentic trigeminal ganglion within Meckel's
cave, and contains the cell bodies of incoming sensory nerve fibres. Here
the 3 branches exit.
The motor root cell bodies are in the pons and the motor fibres are
distributed via the mandibular nerve. The motor root is not part of the
trigeminal ganglion.

Branches of the trigeminal nerve


Ophthalmic nerve Sensory only

Maxillary nerve Sensory only

Mandibular nerve Sensory and motor

Sensory
Ophthalmic Exits skull via the superior orbital fissure
Sensation of: scalp and forehead, the upper eyelid, the conjunctiva and cornea
Sensation of: scalp and forehead, the upper eyelid, the conjunctiva and cornea
of the eye, the nose (including the tip of the nose, except alae nasi), the nasal
mucosa, the frontal sinuses, and parts of the meninges (the dura and blood
vessels).

Maxillary Exit skull via the foramen rotundum


nerve Sensation: lower eyelid and cheek, the nares and upper lip, the upper teeth
and gums, the nasal mucosa, the palate and roof of the pharynx, the maxillary,
ethmoid and sphenoid sinuses, and parts of the meninges.

Mandibular Exit skull via the foramen ovale


nerve Sensation: lower lip, the lower teeth and gums, the chin and jaw (except the
angle of the jaw), parts of the external ear, and parts of the meninges.

Motor
Distributed via the mandibular nerve.
The following muscles of mastication are innervated:

Masseter
Temporalis
Medial pterygoid
Lateral pterygoid

Other muscles innervated include:

Tensor veli palatini


Mylohyoid
Anterior belly of digastric
Tensor tympani

Rate question: Next question

Com m ent on this question

All contents of this site are © 2012 E-Medical Revision Ltd Terms and Conditions Privacy policy
yomna92009@yahoo.com - MRCS Part A - My account

Reference ranges End and review

0/3 Question 8-10 of 350 Question stats Score: 20%

Average score for registered users: 1

Theme: Nerve lesions 2


8 42.6%
3
9 62.1%
A. Iliohypogastric nerve
10 83.9% 4
B. Ilioinguinal nerve
5
C. Lateral cutaneous nerve of the thigh
6
D. Femoral nerve
7
E. Saphenous nerve
8-10 0 / 3
F. Genitofemoral nerve

Please select the most likely nerve implicated in the situation described. Each
option may be used once, more than once or not at all.

8. A 42 year old woman complains of a burning pain of her anterior


thigh which worsens on walking. There is a positive tinel sign over
the inguinal ligament.

You answered Genitofemoral nerve

The correct answer is Lateral cutaneous nerve of the thigh

The lateral cutaneous nerve supplies sensation to the anterior and


lateral aspect of the thigh. Entrapment is commonly due to intra and
extra pelvic causes. Treatment involves local anaesthetic injections.

9. A 29 year old woman has had a Pfannenstiel incision. She has pain
over the inguinal ligament which radiates to the lower abdomen.
There is tenderness when the inguinal canal is compressed.

You answered Genitofemoral nerve

The correct answer is Ilioinguinal nerve

10. A 22 year man is shot in the groin. On examination he has weak hip
flexion, weak knee extension, and impaired quadriceps tendon
reflex, as well as sensory deficit in the anteromedial aspect of the
thigh.

You answered Genitofemoral nerve

The correct answer is Femoral nerve

This is a classical description of a femoral nerve injury.

Next question

Nerve lesions during surgery

A variety of different procedures carry the risk of iatrogenic nerve injury. These
are important not only from the patients perspective but also from a medicolegal
standpoint.

The following operations and their associated nerve lesions are listed here:

Posterior triangle lymph node biopsy and accessory nerve lesion.


Lloyd Davies stirrups and common peroneal nerve.
Thyroidectomy and laryngeal nerve.
Anterior resection of rectum and hypogastric autonomic nerves.
Axillary node clearance; long thoracic nerve, thoracodorsal nerve and
intercostobrachial nerve.
Inguinal hernia surgery and ilioinguinal nerve.
Varicose vein surgery- sural and saphenous nerves.
Posterior approach to the hip and sciatic nerve.
Carotid endarterectomy and hypoglossal nerve.

There are many more, with sound anatomical understanding of the commonly
performed procedures the incidence of nerve lesions can be minimised. They
commonly occur when surgeons operate in an unfamiliar tissue plane or by blind
placement of haemostats (not recommended).

Rate question: Next question

Com m ent on this question

All contents of this site are © 2012 E-Medical Revision Ltd Terms and Conditions Privacy policy
yomna92009@yahoo.com - MRCS Part A - My account

Reference ranges End and review

Question 11 of 350 Question stats Score: 18.2%

1
A 7.3%
Which of the following is not a branch of the external carotid artery? 2
B 13.1%
C 16.7% 3

A. Facial artery D 46.9% 4

B. Lingual artery E 16% 5

C. Superior thyroid artery 46.9% of users answered this 6


question correctly
D. Mandibular artery 7

E. Maxillary artery 8-10 0 / 3

11
Next question

External carotid artery branches mnemonic:

'Some Angry Lady Figured Out PMS'

Superior thyroid (superior laryngeal artery branch)


Ascending pharyngeal
Lingual
Facial (tonsillar and labial artery)
Occipital
Posterior auricular
Maxillary (inferior alveolar artery, middle meningeal artery)
Superficial temporal

External carotid artery

The external carotid artery (ECA) is a branch of the common carotid artery.

Path

Upper border of thyroid cartilage to pass in between the angle of the


mandible and mastoid process
Initially anteromedial to the ICA, then becomes lateral
Lower ECA covered by sternomastoid, passed by hypoglossal nerve/lingual
& facial vein
Then deep to digastric and stylohyoid, eventually passes into the parotid
gland where it divides into the superficial temporal and maxillary branches
yomna92009@yahoo.com - MRCS Part A - My account

Reference ranges End and review

Question 12 of 350 Question stats Score: 16.7%

1
A 17.2%
A 23 year old man is stabbed in the groin, several structures are injured and the 2
B 53.1%
adductor longus muscle has been lacerated. Which of the following nerves is 3
C 13.2%
responsible for the innervation of adductor longus?
D 8.3% 4
E 8.3% 5
A. Femoral nerve
53.1% of users answered this 6
B. Obturator nerve question correctly
7
C. Sciatic nerve
8-10 0 / 3
D. Common peroneal nerve
11
E. Ilioinguinal nerve
12

Next question

The adductors are innervated by the obturator nerve

Adductor longus

Origin Anterior body of pubis

Insertion Middle third of linea aspera

Action Adducts and flexes the thigh, medially rotate the hip

Innervation Anterior division of obturator nerve (L2, L3, L4)

The schematic image below demonstrates the relationship of the adductor


muscles

Image sourced from Wikipedia


yomna92009@yahoo.com - MRCS Part A - My account

Reference ranges End and review

Question 13 of 350 Question stats Score: 15.4%

1
A 19.9%
Which of the following statements relating to the basilar artery and its branches is 2
B 16.5%
false? 3
C 16.1%
D 35.4% 4
A. The superior cerebellar artery may be decompressed to treat E 12.1% 5
trigeminal neuralgia
35.4% of users answered this 6
B. Occlusion of the posterior cerebral artery causes contralateral loss question correctly
of the visual field 7

C. The oculomotor nerve lies between the superior cerebellar and 8-10 0 / 3

posterior cerebral arteries 11

D. The posterior inferior cerebellar artery is the largest of the 12


cerebellar arteries arising from the basilar artery
13
E. The labyrinthine branch is accompanied by the facial nerve

Next question

The posterior inferior cerebellar artery is the largest of the cerebellar arteries
arising from the vertebral artery.

Circle of Willis

May also be called the circulus arteriosus

Inferior surface of brain


Supplied by the internal carotid arteries and the vertebral arteries
If artery is occluded, collaterals may be able to compensate
Components include:

1. Anterior communicating arteries


2. Anterior cerebral arteries
3. Internal carotid arteries
4. Posterior communicating arteries
5. Posterior cerebral arteries and the termination of the basilar artery

Supply: Corpus striatum, internal capsule, diencephalon, midbrain


Image sourced from Wikipedia

Vertebral arteries

Enter the cranial cavity via foramen magnum


Lie in the subarachnoid space
Ascend on anterior surface of medulla oblongata
Unite to form the basilar artery at the base of the pons

Branches:

Posterior spinal artery


Anterior spinal artery
Posterior inferior cerebellar artery

Basilar artery
Branches:

Anterior inferior cerebellar artery


Labyrinthine artery
Pontine arteries
Superior cerebellar artery
Posterior cerebral artery

Internal carotid arteries


Branches:

Posterior communicating artery


Anterior cerebral artery
Middle cerebral artery
Anterior choroid artery

Rate question: Next question

Com m ent on this question

All contents of this site are © 2012 E-Medical Revision Ltd Terms and Conditions Privacy policy
yomna92009@yahoo.com - MRCS Part A - My account

Reference ranges End and review

Question 14 of 350 Question stats Score: 14.3%

1
A 8.9%
Which of the following muscles does not recieve any innervation from the sciatic 2
B 49.1%
nerve? 3
C 11.6%
D 8.9% 4
A. Semimembranosus E 21.5% 5
B. Quadriceps femoris 6
49.1% of users answered this
C. Biceps femoris question correctly
7
D. Semitendinosus 8-10 0 / 3
E. Adductor magnus 11

12
Next question
13

14
The sciatic nerve is traditionally viewed as being a nerve of the posterior
compartment. It is known to contribute to the innervation of adductor magnus
(although the main innervation to this muscle is from the obturator nerve). The
quadriceps femoris is nearly always innervated by the femoral nerve.

Sciatic nerve

Origin Spinal nerves L4 - S3

Articular Branches Hip joint

Muscular branches in Semitendinosus


upper leg Semimembranosus
Biceps femoris
Part of adductor magnus

Cutaneous sensation Posterior aspect of thigh


Gluteal region
Entire lower leg (except the medial aspect)

Terminates At the upper part of the popliteal fossa by dividing into the tibial
and peroneal nerves

The nerve to the short head of the biceps femoris comes from the common
peroneal part of the sciatic and the other muscular branches arise from the
tibial portion.
The tibial nerve goes on to innervate all muscles of the foot except the
extensor digitorum brevis (which is innervated by the common peroneal
nerve).

Rate question: Next question

Com m ent on this question

All contents of this site are © 2012 E-Medical Revision Ltd Terms and Conditions Privacy policy
yomna92009@yahoo.com - MRCS Part A - My account

Reference ranges End and review

Question 15 of 350 Question stats Score: 20%

1
A 9.2%
A 23 year old man is involved in a fight and is stabbed in his upper arm. The ulnar 2
B 11.2%
nerve is transected. Which of the following muscles will not demonstrate 3
C 15.9%
compromised function as a result?
D 10.1% 4
E 53.7% 5
A. Flexor carpi ulnaris
53.7% of users answered this 6
B. Medial half of flexor digitorum profundus question correctly
7
C. Palmaris brevis
8-10 0 / 3
D. Hypothenar muscles
11
E. Pronator teres
12

13
Next question
14

15
M edial lumbricals
A dductor pollicis
F lexor digitorum profundus/Flexor digiti minimi
I nterossei
A bductor digiti minimi and opponens

Innervates all intrinsic muscles of the hand (EXCEPT 2: thenar muscles & first
two lumbricals - supplied by median nerve)

Pronator teres is innervated by the median nerve. Palmaris brevis is innervated


by the ulnar nerve

Ulnar nerve

Origin

C8, T1

Supplies (no muscles in the upper arm)

Flexor carpi ulnaris


Flexor digitorum profundus
Flexor digiti minimi
Abductor digiti minimi
Opponens digiti minimi
Adductor pollicis
Interossei muscle
Third and fourth lumbricals
Palmaris brevis

Path

Posteromedial aspect of ulna to flexor compartment of forearm, then along


the ulnar. Passes beneath the flexor carpi ulnaris muscle, then superficially
through the flexor retinaculum into the palm of the hand.
yomna92009@yahoo.com - MRCS Part A - My account

Reference ranges End and review

Question 16 of 350 Question stats Score: 25%

1
A 10.7%
Which of the following overlies the cephalic vein? 2
B 17.5%
C 16.5% 3

A. Extensor retinaculum D 31.6% 4

B. Bicipital aponeurosis E 23.6% 5

C. Biceps muscle 23.6% of users answered this 6


question correctly
D. Antebrachial fascia 7

E. None of the above 8-10 0 / 3

11
Next question 12

13
The cephalic vein is superficially located in the upper limb and overlies most the
14
fascial planes. It pierces the coracoid membrane (continuation of the clavipectoral
fascia) to terminate in the axillary vein. It lies anterolaterally to biceps. 15

16
Cephalic vein

Path

Dorsal venous arch drains laterally into the cephalic vein


Crosses the anatomical snuffbox and travels laterally up the arm
At the antecubital fossa connected to the basilic vein by the median cubital
vein
Pierces deep fascia of deltopectoral groove to join axillary vein

Rate question: Next question

Com m ent on this question

All contents of this site are © 2012 E-Medical Revision Ltd Terms and Conditions Privacy policy
yomna92009@yahoo.com - MRCS Part A - My account

Reference ranges End and review

Question 17 of 350 Question stats Score: 23.5%

1
A 11.8%
Which of the following pairings are incorrect? 2
B 9.4%
C 53% 3

A. Aortic bifurcation and L4 D 9.6% 4

B. Transpyloric plane and L1 E 16.2% 5

C. Termination of dural sac and L4 53% of users answered this 6


question correctly
D. Oesophageal passage through diaphragm and T10 7

E. Transition between pharynx and oesophagus at C6 8-10 0 / 3

11
Next question 12

13

Vena cava T8 (eight letters) 14

Oesophagus T10 (ten letters) 15


Aortic hiatus T12 (twelve letters)
16

17
It terminates at S2, which is why it is safe to undertake an LP at L4/5 levels. The
spinal cord itself terminates at L1.

Levels

Transpyloric plane
Level of the body of L1

Pylorus stomach
Left kidney hilum (L1- left one!)
Right hilum of the kidney (1.5cm lower than the left)
Fundus of the gallbladder
Neck of pancreas
Duodenojejunal flexure
Superior mesenteric artery
Portal vein
Left and right colic flexure
Root of the transverse mesocolon
2nd part of the duodenum
Upper part of conus medullaris
Spleen

Can be identified by asking the supine patient to sit up without using their arms.
The plane is located where the lateral border of the rectus muscle crosses the
costal margin.

Anatomical planes
Subcostal plane Lowest margin of 10th costal cartilage

Intercristal plane Level of body L4 (highest point of iliac crest)

Intertubercular plane Level of body L5

Common level landmarks


Inferior mesenteric artery L3

Bifurcation of aorta into common iliac arteries L4

Formation of IVC L5 (union of common iliac veins)

Diaphragm apertures Vena cava T8


Oesophagus T10
Aortic hiatus T12
yomna92009@yahoo.com - MRCS Part A - My account

Reference ranges End and review

Question 18 of 350 Question stats Score: 22.2%

1
A 16.5%
A 22 year old man is involved in a fight. He sustains a laceration to the posterior 2
B 35.5%
aspect of his wrist. In the emergency department the wound is explored and the 3
C 16.4%
laceration is found to be transversely orientated and overlies the region of the
D 13% 4
extensor retinaculum, which is intact. Which of the following structures is least
E 18.6% 5
likely to be injured in this scenario?

35.5% of users answered this 6


question correctly
A. Dorsal cutaneous branch of the ulnar nerve 7

B. Tendon of extensor indicis 8-10 0 / 3

C. Basilic vein 11

D. Superficial branch of the radial nerve 12

E. Cephalic vein 13

14
Next question 15

16
The extensor retinaculum attaches to the radius proximal to the styloid, thereafter
17
it runs obliquely and distally to wind around the ulnar styloid (but does not attach
to it). The extensor tendons lie deep to the extensor retinaculum and would 18
therefore be less susceptible to injury than the superficial structures.

Extensor retinaculum

The extensor rentinaculum is a thickening of the deep fascia that stretches across
the back of the wrist and holds the long extensor tendons in position.
Its attachments are:

The pisiform and hook of hamate medially


The end of the radius laterally

Structures related to the extensor retinaculum

Structures superficial to the Basilic vein


retinaculum Dorsal cutaneous branch of the ulnar
nerve
Cephalic vein
Superficial branch of the radial nerve

Structures passing deep to the Extensor carpi ulnaris tendon


extensor retinaculum Extensor digiti minimi tendon
Extensor digitorum and extensor indicis
tendon
Extensor pollicis longus tendon
Extensor carpi radialis longus tendon
Abductor pollicis longus and extensor
pollicis brevis tendons

Beneath the extensor retinaculum fibrous septa form six compartments that
contain the extensor muscle tendons. Each compartment has its own synovial
sheath.

The radial artery


The radial artery passes between the lateral collateral ligament of the wrist joint
and the tendons of the abductor pollicis longus and extensor pollicis brevis.

Image illustrating the topography of tendons passing under the extensor


retinaculum
Image sourced from Wikipedia

Rate question: Next question

Com m ent on this question

All contents of this site are © 2012 E-Medical Revision Ltd Terms and Conditions Privacy policy
yomna92009@yahoo.com - MRCS Part A - My account

Reference ranges End and review

Question 19 of 350 Question stats Score: 21.1%

1
A 6.1%
Which of the following is not a content of the porta hepatis? 2
B 11.3%
C 48.2% 3

A. Portal vein D 16.4% 4

B. Hepatic artery E 18% 5

C. Cystic duct 48.2% of users answered this 6


question correctly
D. Hepatic lymph nodes 7

E. None of the above 8-10 0 / 3

11
Next question 12

13
The cystic duct lies outside the porta hepatis and is an important landmark in
14
laparoscopic cholecystectomy. The structures in the porta hepatis are:
15
Portal vein
16
Hepatic artery
Common hepatic duct 17

18
These structures divide immediately after or within the porta hepatis to supply the
functional left and right lobes of the liver. 19
The porta hepatis is also surrounded by lymph nodes, that may enlarge to
produce obstructive jaundice and parasympathetic nervous fibres that travel
along vessels to enter the liver.

Liver

Structure of the liver

Right lobe Supplied by right hepatic artery


Contains Couinard segments V to VIII (-/+Sg I)

Left lobe Supplied by the left hepatic artery


Contains Couinard segments II to IV (+/- Sg1)

Quadrate Part of the right lobe anatomically, functionally is part of the left
lobe Couinard segment IV
Porta hepatis lies behind

On the right lies the gallbladder fossa


On the left lies the fossa for the umbilical vein

Caudate lobe Supplied by both right and left hepatic arteries


Couinard segment I
Lies behind the plane of the porta hepatis
Anterior and lateral to the inferior vena cava
Bile from the caudate lobe drains into both right and left hepatic
ducts

Detailed knowledge of Couinard segments is not required for MRCS Part A

Between the liver lobules are portal canals which contain the portal triad:
Hepatic Artery, Portal Vein, tributary of Bile Duct.

Relations of the liver


Anterior Postero inferiorly

Diaphragm Oesophagus

Xiphoid process Stomach

Duodenum
yomna92009@yahoo.com - MRCS Part A - My account

Reference ranges End and review

Question 20 of 350 Question stats Score: 25%

1
A 14.5%
Which of the following structures is not closely related to the carotid sheath? 2
B 11.5%
C 12.5% 3

A. Sternothyroid muscle D 17.9% 4

B. Sternohyoid muscle E 43.7% 5

C. Hypoglossal nerve 43.7% of users answered this 6


question correctly
D. Superior belly of omohyoid muscle 7

E. Anterior belly of digastric muscle 8-10 0 / 3

11
Next question 12

13
At its lower end the carotid sheath is related to sternohyoid and sternothyroid.
14
Opposite the cricoid cartilage the sheath is crossed by the superior belly of
omohyoid. Above this level the sheath is covered by the sternocleidomastoid 15
muscle. Above the level of the hyoid the vessels pass deep to the posterior belly 16
of digastric and stylohyoid. Opposite the hyoid bone the sheath is crossed
17
obliquely by the hypoglossal nerve.
18

Common carotid artery 19

20
The right common carotid artery arises at the bifurcation of the brachiocephalic
trunk, the left common carotid arises from the arch of the aorta. Both terminate at
the level of the upper border of the thyroid cartilage (the lower border of the third
cervical vertebra) by dividing into the internal and external carotid arteries.

Left common carotid artery


This vessel arises immediately to the left and slightly behind the origin of the
brachiocephalic trunk. Its thoracic portion is 2.5- 3.5 cm in length and runs
superolaterally to the sternoclavicular joint.

In the thorax
The vessel is in contact, from below upwards, with the trachea, left recurrent
laryngeal nerve, left margin of the oesophagus. Anteriorly the left brachiocephalic
vein runs across the artery, and the cardiac branches from the left vagus
descend in front of it. These structures together with the thymus and the anterior
margins of the left lung and pleura separate the artery from the manubrium.

In the neck
The artery runs superiorly deep to sternocleidomastoid and then enters the
anterior triangle. At this point it lies within the carotid sheath with the vagus nerve
and the internal jugular vein. Posteriorly the sympathetic trunk lies between the
vessel and the prevertebral fascia. At the level of C7 the vertebral artery and
thoracic duct lie behind it. The anterior tubercle of C6 transverse process is
prominent and the artery can be compressed against this structure (it
corresponds to the level of the cricoid).
Anteriorly at C6 the omohyoid muscle passes superficial to the artery.
Within the carotid sheath the jugular vein lies lateral to the artery.

Right common carotid artery


The right common carotid arises from the brachiocephalic artery. The right
common carotid artery corresponds with the cervical portion of the left common
carotid, except that there is no thoracic duct on the right. The oesophagus is less
closely related to the right carotid than the left.

Summary points about the carotid anatomy

Path
Passes behind the sternoclavicular joint (12% patients above this level) to the
upper border of the thyroid cartilage, to divide into the external (ECA) and internal
carotid arteries (ICA).

Relations

Level of 6th cervical vertebra crossed by omohyoid


Then passes deep to the thyrohyoid, sternohyoid, sternomastoid muscles.
Passes behind the carotid tubercle (transverse process 6th cervical
vertebra)-NB compression here stops haemorrhage.
The inferior thyroid artery passes posterior to the common carotid artery.
Then : Left common carotid artery crossed by thoracic duct, Right common
carotid artery crossed by recurrent laryngeal nerve

Image sourced from Wikipedia

Rate question: Next question

Com m ent on this question

All contents of this site are © 2012 E-Medical Revision Ltd Terms and Conditions Privacy policy
yomna92009@yahoo.com - MRCS Part A - My account

Reference ranges End and review

Question 21 of 350 Question stats Score: 23.8%

1
A 8.3%
A 21 year old develops tonsillitis. He is in considerable pain. Which of the 2
B 10.5%
following nerves is responsible for the sensory innervation of the tonsillar fossa? 3
C 64.7%
D 8.1% 4
A. Facial nerve E 8.5% 5
B. Trigeminal nerve 6
64.7% of users answered this
C. Glossopharyngeal nerve question correctly
7
D. Hypoglossal nerve 8-10 0 / 3
E. Vagus 11

12
Next question
13

14
The glossopharyngeal nerve is the main sensory nerve for the tonsillar fossa. A
lesser contribution is made by the lesser palatine nerve. Because of this otalgia 15
may occur following tonsillectomy. 16

17
Tonsil
18

Anatomy 19

20

Each palatine tonsil has two surfaces, a medial surface which projects into 21
the pharynx and a lateral surface that is embedded in the wall of the
pharynx.
They are usually 25mm tall by 15mm wide, although this varies according to
age and may be almost completely atrophied in the elderly.
Their arterial supply is from the tonsillar artery, a branch of the facial
artery.
Its veins pierce the constrictor muscle to join the external palatine or facial
veins. The external palatine vein is immediately lateral to the tonsil, which
may result in haemorrhage during tonsillectomy.
Lymphatic drainage is the jugulodigastric node and the deep cervical
nodes.

Tonsillitis

Usually bacterial (50%)- group A Streptococcus. Remainder viral.


May be complicated by development of abscess (quinsy). This may distort
the uvula.

- Indications for tonsillectomy include recurrent acute tonsillitis, suspected


malignancy, enlargement causing sleep apnoea.
- Dissection tonsillectomy is the preferred technique with haemorrhage being the
commonest complication. Delayed otaligia may occur owing to irritation of the
glossopharyngeal nerve.

Rate question: Next question

Com m ent on this question

All contents of this site are © 2012 E-Medical Revision Ltd Terms and Conditions Privacy policy
yomna92009@yahoo.com - MRCS Part A - My account

Reference ranges End and review

Question 22 of 350 Question stats Score: 22.7%

1
A 12.6%
A man is stabbed in the abdomen during a fight. He is brought to the emergency 2
B 20.9%
department. On examination there is a laceration in the anterior abdominal wall 3
C 35.1%
immediately lateral to the left rectus abdominis muscle on a level with the upper
D 7.1% 4
border of the first lumbar vertebra. Which of the following structures is most likely
E 24.3% 5
to have been injured?

35.1% of users answered this 6


question correctly
A. Head of the pancreas 7

B. Gastric antrum 8-10 0 / 3

C. Spleen 11

D. Right lobe of the liver 12

E. Superior mesenteric artery 13

14
Next question 15

16
The spleen is the most likely target in this instance.
17

Levels 18

19
Transpyloric plane
20
Level of the body of L1
21

22
Pylorus stomach
Left kidney hilum (L1- left one!)
Right hilum of the kidney (1.5cm lower than the left)
Fundus of the gallbladder
Neck of pancreas
Duodenojejunal flexure
Superior mesenteric artery
Portal vein
Left and right colic flexure
Root of the transverse mesocolon
2nd part of the duodenum
Upper part of conus medullaris
Spleen

Can be identified by asking the supine patient to sit up without using their arms.
The plane is located where the lateral border of the rectus muscle crosses the
costal margin.

Anatomical planes
Subcostal plane Lowest margin of 10th costal cartilage

Intercristal plane Level of body L4 (highest point of iliac crest)

Intertubercular plane Level of body L5

Common level landmarks


Inferior mesenteric artery L3

Bifurcation of aorta into common iliac arteries L4

Formation of IVC L5 (union of common iliac veins)

Diaphragm apertures Vena cava T8


Oesophagus T10
Aortic hiatus T12

Rate question:
yomna92009@yahoo.com - MRCS Part A - My account

Reference ranges End and review

Question 23 of 350 Question stats Score: 21.7%

1
A 23.4%
A man has an incision sited than runs 8cm from the deltopectoral groove to the 2
B 25.3%
midline. Which of the following is not at risk of injury? 3
C 13.4%
D 8.4% 4
A. Cephalic vein E 29.5% 5
B. Shoulder joint capsule 6
25.3% of users answered this
C. Axillary artery question correctly
7
D. Pectoralis major 8-10 0 / 3
E. Trunk of the brachial plexus 11

12
Next question
13

14
Theme from April 2012 Exam
15
This region will typically lie medial to the joint capsule. The diagram below 16
illustrates the plane that this would transect and as it can be appreciated the
17
other structures are all at risk of injury.
18

19

20

21

22

23
Image sourced from Wikipedia

Pectoralis major muscle

Origin From the medial two thirds of the clavicle, manubrium and sternocostal
angle

Insertion Crest of the greater tubercle of the humerus

Nerve Lateral pectoral nerve


supply

Actions Adductor and medial rotator of the humerus

Rate question: Next question

Com m ent on this question

All contents of this site are © 2012 E-Medical Revision Ltd Terms and Conditions Privacy policy
yomna92009@yahoo.com - MRCS Part A - My account

Reference ranges End and review

Question 24 of 350 Question stats Score: 20.8%

1
A 13.4%
A surgeon is due to perform a laparotomy for perforated duodenal ulcer. An 2
B 5.9%
upper midline incision is to be performed. Which of the following structures is the 3
C 68.3%
incision most likely to divide?
D 4.8% 4
E 7.6% 5
A. Rectus abdominis muscle
68.3% of users answered this 6
B. External oblique muscle question correctly
7
C. Linea alba
8-10 0 / 3
D. Internal oblique muscle
11
E. None of the above
12

13
Next question
14

Theme from September 2011 Exam 15

16
Upper midline abdominal incisions will involve the division of the linea alba.
17
Division of muscles will not usually improve access in this approach and they
would not be routinely encountered during this incision. 18

19
Abdominal incisions
20

21
Theme in January 2012 exam
22

23
Midline Commonest approach to the abdomen
incision Structures divided: linea alba, transversalis fascia, extraperitoneal 24
fat, peritoneum (avoid falciform ligament above the umbilicus)
Bladder can be accessed via an extraperitoneal approach through
the space of Retzius

Paramedian Parallel to the midline (about 3-4cm)


incision Structures divided/retracted: anterior rectus sheath, rectus
(retracted), posterior rectus sheath, transversalis fascia,
extraperitoneal fat, peritoneum
Incision is closed in layers

Battle Similar location to paramedian but rectus displaced medially (and


thus denervated)
Now seldom used

Kocher's Incision under right subcostal margin e.g. Cholecystectomy (open)

Lanz Incision in right iliac fossa e.g. Appendicectomy

Gridiron Oblique incision centered over McBurneys point- usually appendicectomy


(less cosmetically acceptable than Lanz

Gable Rooftop incision

Pfannenstiel's Transverse supra pubic, primarily used to access pelvic organs

McEvedy's Groin incision e.g. Emergency repair strangulated femoral hernia

Rutherford Extraperitoneal approach to left or right lower quadrants. Gives excellent


Morrison access to iliac vessels and is the approach of choice for first time renal
transplantation.
Image sourced from Wikipedia

Rate question: Next question

Com m ent on this question

All contents of this site are © 2012 E-Medical Revision Ltd Terms and Conditions Privacy policy
yomna92009@yahoo.com - MRCS Part A - My account

Reference ranges End and review

Question 25 of 350 Question stats Score: 20%

1
A 53%
A 59 year old man is undergoing an extended right hemicolectomy for a 2
B 10.7%
carcinoma of the hepatic flexure of the colon. The surgeons divide the middle 3
C 13.8%
colonic vein close to its origin. Into which of the following structures does this
D 9.8% 4
vessel primarily drain?
E 12.7% 5

53% of users answered this 6


A. Superior mesenteric vein
question correctly
7
B. Portal vein
8-10 0 / 3
C. Inferior mesenteric vein
11
D. Inferior vena cava
12
E. Ileocolic vein
13

Next question 14

15
The middle colonic vein drains into the SMV, if avulsed during mobilisation then 16
dramatic haemorrhage can occur and be difficult to control.
17

Transverse colon 18

19

20
The right colon undergoes a sharp turn at the level of the hepatic flexure to
become the transverse colon. 21
At this point it also becomes intraperitoneal.
22
It is connected to the inferior border of the pancreas by the transverse
23
mesocolon.
The greater omentum is attached to the superior aspect of the transverse 24
colon from which it can easily be separated. The mesentery contains the
25
middle colic artery and vein. The greater omentum remains attached to the
transverse colon up to the splenic flexure. At this point the colon undergoes
another sharp turn.

Relations
Superior Liver and gall-bladder, the greater curvature of the stomach, and
the lower end of the spleen

Inferior Small intestine

Anterior Greater omentum

Posterior From right to left with the descending portion of the duodenum, the
head of the pancreas, convolutions of the jejunum and ileum, spleen

Rate question: Next question

Com m ent on this question

All contents of this site are © 2012 E-Medical Revision Ltd Terms and Conditions Privacy policy
yomna92009@yahoo.com - MRCS Part A - My account

Reference ranges End and review

0/3 Question 26-28 of 350 Question stats Score: 17.9%

Average score for registered users: 1

Theme: Nerve Injury 2


26 62.3%
3
27 56.2%
A. Median nerve
28 46.7% 4
B. Ulnar nerve
5
C. Radial nerve
6
D. Musculocutaneous nerve
7
E. Axillary nerve
8-10 0 / 3
F. Anterior interosseous nerve
11
G. Posterior interosseous nerve
12

13
For each scenario please select the most likely underlying nerve injury. Each
option may be used once, more than once or not at all. 14

15

16
26. A 19 year old student is admitted to A&E after falling off a wall. He is
17
unable to flex his index finger. An x-ray confirms a supracondylar
fracture. 18

19
You answered Posterior interosseous nerve
20

The correct answer is Median nerve 21

22
This median nerve is at risk during a supracondylar fracture.
23

27. A well toned weight lifter attends clinic reporting weakness of his left 24
arm. There is weakness of flexion and supination of the forearm.
25

26-28 0 / 3
You answered Posterior interosseous nerve

The correct answer is Musculocutaneous nerve

Mucocutaneous nerve compression due to entrapment of the nerve


between biceps and brachialis. Elbow flexion and supination of the
arm are affected. This is a rare isolated injury.

28. An 18 year old girl sustains an Holstein-Lewis fracture. Which nerve


is at risk?

You answered Posterior interosseous nerve

The correct answer is Radial nerve

Proximal lesions affect the triceps. Also paralysis of wrist extensors


and forearm supinators occur. Reduced sensation of dorsoradial
aspect of hand and dorsal 31/2 fingers. Holstein-Lewis fractures are
fractures of the distal humerus with radial nerve entrapment.

Next question

Brachial plexus

Origin Anterior rami of C5 to T1

Sections of the Roots, trunks, divisions, cords, branches


plexus Mnemonic:Real Teenagers Drink Cold Beer
Roots Located in the posterior triangle
Pass between scalenus anterior and medius

Trunks Located posterior to middle third of clavicle


Upper and middle trunks related superiorly to the subclavian
artery
Lower trunk passes over 1st rib posterior to the subclavian
artery

Divisions Apex of axilla

Cords Related to axillary artery

Diagram illustrating the branches of the brachial plexus

Image sourced from Wikipedia

Cutaneous sensation of the upper limb

Image sourced from Wikipedia

Rate question: Next question

Com m ent on this question


yomna92009@yahoo.com - MRCS Part A - My account

Reference ranges End and review

Question 29 of 350 Question stats Score: 20.7%

1
A 9.5%
A 35 year old farm labourer is injures the posterior aspect of his hand with a 2
B 20.8%
mechanical scythe. He severs some of his extensor tendons in this injury. How 3
C 17%
many tunnels lie in the extensor retinaculum that transmit the tendons of the
D 13% 4
extensor muscles?
E 39.8% 5

39.8% of users answered this 6


A. One
question correctly
7
B. Three
8-10 0 / 3
C. Four
11
D. Five
12
E. Six
13

Next question 14

15
There are six tunnels, each lined by its own synovial sheath. 16

17
Extensor retinaculum
18

The extensor rentinaculum is a thickening of the deep fascia that stretches across 19
the back of the wrist and holds the long extensor tendons in position.
20
Its attachments are:
21
The pisiform and hook of hamate medially
22
The end of the radius laterally
23

24
Structures related to the extensor retinaculum
25
Structures superficial to the Basilic vein 26-28 0 / 3
retinaculum Dorsal cutaneous branch of the ulnar
nerve 29
Cephalic vein
Superficial branch of the radial nerve

Structures passing deep to the Extensor carpi ulnaris tendon


extensor retinaculum Extensor digiti minimi tendon
Extensor digitorum and extensor indicis
tendon
Extensor pollicis longus tendon
Extensor carpi radialis longus tendon
Abductor pollicis longus and extensor
pollicis brevis tendons

Beneath the extensor retinaculum fibrous septa form six compartments that
contain the extensor muscle tendons. Each compartment has its own synovial
sheath.

The radial artery


The radial artery passes between the lateral collateral ligament of the wrist joint
and the tendons of the abductor pollicis longus and extensor pollicis brevis.

Image illustrating the topography of tendons passing under the extensor


retinaculum
yomna92009@yahoo.com - MRCS Part A - My account

Reference ranges End and review

Question 30 of 350 Question stats Score: 20%

1
A 12.9%
A 23 year old man is stabbed in the chest approximately 10cm below the right 2
B 43.3%
nipple. In the emergency department a abdominal ultrasound scan shows a large 3
C 11.6%
amount of intraperitoneal blood. Which of the following statements relating to the
D 15.1% 4
likely site of injury is untrue?
E 17.1% 5

43.3% of users answered this 6


A. Part of its posterior surface is devoid of peritoneum.
question correctly
7
B. The quadrate lobe is contained within the functional right lobe.
8-10 0 / 3
C. Its nerve supply is from the coeliac plexus.
11
D. The hepatic flexure of the colon lies posterio-inferiorly.
12
E. The right kidney is closely related posteriorly.
13

Next question 14

15
The right lobe of the liver is the most likely site of injury. Therefore the answer is 16
B as the quadrate lobe is functionally part of the left lobe of the liver. The liver is
17
largely covered in peritoneum. Posteriorly there is an area devoid of peritoneum
(the bare area of the liver). The right lobe of the liver has the largest bare area 18
(ans is larger thant the left lobe). 19

20
Liver
21

Structure of the liver 22

23
Right lobe Supplied by right hepatic artery
Contains Couinard segments V to VIII (-/+Sg I) 24

25
Left lobe Supplied by the left hepatic artery
26-28 0 / 3
Contains Couinard segments II to IV (+/- Sg1)
29

30
Quadrate Part of the right lobe anatomically, functionally is part of the left
lobe Couinard segment IV
Porta hepatis lies behind
On the right lies the gallbladder fossa
On the left lies the fossa for the umbilical vein

Caudate lobe Supplied by both right and left hepatic arteries


Couinard segment I
Lies behind the plane of the porta hepatis
Anterior and lateral to the inferior vena cava
Bile from the caudate lobe drains into both right and left hepatic
ducts

Detailed knowledge of Couinard segments is not required for MRCS Part A

Between the liver lobules are portal canals which contain the portal triad:
Hepatic Artery, Portal Vein, tributary of Bile Duct.

Relations of the liver


Anterior Postero inferiorly

Diaphragm Oesophagus

Xiphoid process Stomach

Duodenum

Hepatic flexure of colon

Right kidney

Gallbladder
yomna92009@yahoo.com - MRCS Part A - My account

Reference ranges End and review

Question 1 of 320 Question stats Score: 0%

1
A 31.9%
A 22 year old man is involved in a fight and sustains a skull fracture with an injury B 20.4%
to the middle meningeal artery. A craniotomy is performed, and with considerable
C 24.2%
difficulty the haemorrhage from the middle meningeal artery is controlled by
D 12.1%
ligating it close to its origin. What is the most likely sensory impairment that the
E 11.5%
patient may notice post operatively?

31.9% of users answered this


question correctly
A. Parasthesia of the ipsilateral external ear
B. Loss of taste sensation from the anterior two thirds of the tongue
C. Parasthesia overlying the angle of the jaw
D. Loss of sensation from the ipsilateral side of the tongue
E. Loss of taste from the posterior two thirds of the tongue

Next question

The auriculotemporal nerve is closely related to the middle meningeal artery and
may be damaged in this scenario. The nerve supplied sensation to the external
ear and outermost part of the tympanic membrane. The angle of the jaw is
innervated by C2,3 roots and would not be affected. The tongue is supplied by
the glossopharyngeal nerve.

Middle meningeal artery

Middle meningeal artery is typically the third branch of the first part of the
maxillary artery, one of the two terminal branches of the external carotid
artery. After branching off the maxillary artery in the infratemporal fossa, it
runs through the foramen spinosum to supply the dura mater (the
outermost meninges) .
The middle meningeal artery is the largest of the three (paired) arteries
which supply the meninges, the others being the anterior meningeal artery
and the posterior meningeal artery.
The middle meningeal artery runs beneath the pterion. It is vulnerable to
injury at this point, where the skull is thin. Rupture of the artery may give
rise to an extra dural hematoma.
In the dry cranium, the middle meningeal, which runs within the dura mater
surrounding the brain, makes a deep indention in the calvarium.
The middle meningeal artery is intimately associated with the
auriculotemporal nerve which wraps around the artery making the two
easily identifiable in the dissection of human cadavers and also easily
damaged in surgery.

Rate question: Next question

Com m ent on this question

All contents of this site are © 2012 E-Medical Revision Ltd Terms and Conditions Privacy policy
yomna92009@yahoo.com - MRCS Part A - My account

Reference ranges End and review

Question 2 of 320 Question stats Score: 0%

1
A 16.6%
A 72 year old man presents with haemoptysis and undergoes a bronchoscopy. 2
B 54%
The carina is noted to be widened. At which level does the trachea bifurcate?
C 6.8%
D 14.4%

A. T3 E 8.2%

B. T5
54% of users answered this
C. T7 question correctly

D. T2
E. T8

Next question

The trachea birfurcates at the level of the fifth thoracic vertebra. Or the sixth in
tall subjects.

Trachea

Trachea
Location C6 vertebra to the upper border of T5 vertebra (bifurcation)

Arterial and venous supply Inferior thyroid arteries and the thyroid venous plexus.

Nerve Branches of vagus, sympathetic and the recurrent nerves

Relations in the neck

Anterior(Superior to Isthmus of the thyroid gland


inferior) Inferior thyroid veins
Arteria thyroidea ima (when that vessel exists)
Sternothyroid
Sternohyoid
Cervical fascia
Anastomosing branches between the anterior jugular
veins

Posterior Oesophagus.

Laterally Common carotid arteries


Right and left lobes of the thyroid gland
Inferior thyroid arteries
Recurrent laryngeal nerves

Relations in the thorax

Anterior

Manubrium sterni, the remains of the thymus, the aortic arch, left common
carotid arteries, and the deep cardiac plexus

Lateral

In the superior mediastinum, on the right side is the pleura and right vagus;
on its left side are the left recurrent nerve, the aortic arch, and the left
common carotid and subclavian arteries.

Rate question: Next question

Com m ent on this question


yomna92009@yahoo.com - MRCS Part A - My account

Reference ranges End and review

Question 3 of 320 Question stats Score: 0%

1
A 6.7%
A 23 year old man is injured during a game of rugby. He suffers a fracture of the 2
B 22.7%
distal third of his clavicle, it is a compound fracture and there is evidence of 3
C 48.3%
arterial haemorrhage. Which of the following vessels is most likely to be
D 11.3%
encountered first during subsequent surgical exploration?
E 11%

A. Posterior circumflex humeral artery 48.3% of users answered this


question correctly
B. Axillary artery
C. Thoracoacromial artery
D. Sub scapular artery
E. Lateral thoracic artery

Next question

Similar theme in September 2011 Exam

The thoracoacromial artery arises from the second part of the axillary artery. It is
a short, wide trunk, which pierces the clavipectoral fascia, and ends, deep to
pectoralis major by dividing into four branches.

Thoracoacromial artery

The thoracoacromial artery (acromiothoracic artery; thoracic axis) is a short trunk,


which arises from the forepart of the axillary artery, its origin being generally
overlapped by the upper edge of the Pectoralis minor.

Projecting forward to the upper border of the Pectoralis minor, it pierces the
coracoclavicular fascia and divides into four branches: pectoral, acromial,
clavicular, and deltoid.

Branch Description

Pectoral Descends between the two Pectoral muscles, and is distributed to them and to
branch the breast, anastomosing with the intercostal branches of the internal thoracic
artery and with the lateral thoracic.

Acromial Runs laterally over the coracoid process and under the Deltoid, to which it gives
branch branches; it then pierces that muscle and ends on the acromion in an arterial

network formed by branches from the suprascapular, thoracoacromial, and


posterior humeral circumflex arteries.

Clavicular Runs upwards and medially to the sternoclavicular joint, supplying this
branch articulation, and the Subclavius

Deltoid Arising with the acromial, it crosses over the Pectoralis minor and passes in the
branch same groove as the cephalic vein, between the Pectoralis major and Deltoid,
and gives branches to both muscles.

Rate question: Next question

Com m ent on this question

All contents of this site are © 2012 E-Medical Revision Ltd Terms and Conditions Privacy policy
yomna92009@yahoo.com - MRCS Part A - My account

Reference ranges End and review

Question 4 of 320 Question stats Score: 25%

1
A 7%
The following are true of the femoral nerve except: 2
B 8.8%
C 8.3% 3

A. It is derived from L2, L3 and L4 nerve roots D 12.1% 4

B. It supplies sartorius E 63.8%

C. It supplies quadriceps femoris 63.8% of users answered this


question correctly
D. It gives cutaneous innervations via the saphenous nerve
E. It supplies adductor longus

Next question

Adductor longus is supplied by the obturator nerve.

Femoral nerve

Root values L2, 3, 4

Innervates Pectineus
Sartorius
Quadriceps femoris
Vastus lateralis/medialis/intermedius

Branches Medial cutaneous nerve of thigh


Saphenous nerve
Intermediate cutaneous nerve of thigh

Path
Penetrates psoas major and exits the pelvis by passing under the inguinal
ligament to enter the femoral triangle, lateral to the femoral artery and vein.

Image sourced from Wikipedia


Image sourced from Wikipedia

Mnemonic for femoral nerve supply

(don't) M I S V Q Scan for PE


M edial cutaneous nerve of the thigh
I ntermediate cutaneous nerve of the thigh
S aphenous nerve

V astus
Q uadriceps femoris
S artorius

PE ectineus

Rate question: Next question

Com m ent on this question

All contents of this site are © 2012 E-Medical Revision Ltd Terms and Conditions Privacy policy
yomna92009@yahoo.com - MRCS Part A - My account

Reference ranges End and review

Question 5 of 320 Question stats Score: 20%

1
A 53%
Where is the vomiting centre located? 2
B 8.4%
C 5% 3

A. Medulla oblongata D 19% 4

B. Substantia nigra E 14.6% 5

C. Antrum of stomach 53% of users answered this


question correctly
D. Pons
E. Midbrain

Next question

ABC's of Non- GI causes of vomiting

Acute renal failure


Brain (Increased ICP)
Cardiac (Inferior MI)
DKA
Ears (labyrinthitis)
Foreign substances (Tylenol, theo, etc)
Glaucoma
Hyperemesis Gravidarum
Infections (pyelonephritis, meningitis)

Vomiting

Reflex oral expulsion of gastric (and sometimes intestinal) contents - reverse


peristalsis and abdominal contraction

The vomiting centre is in part of the medulla oblongata and is triggered by


receptors in several locations:

Labyrinthine receptors of ear (motion sickness)


Overdistention receptors of duodenum and stomach
Trigger zone of CNS - many drugs (e.g., opiates) act here
Touch receptors in throat
Sensory innervation rich, both extrinsic and intrinsic

Rate question: Next question

Com m ent on this question

All contents of this site are © 2012 E-Medical Revision Ltd Terms and Conditions Privacy policy
yomna92009@yahoo.com - MRCS Part A - My account

Reference ranges End and review

Question 6 of 320 Question stats Score: 16.7%

1
A 23.5%
Which of the following nerves conveys sensory information from the laryngeal 2
B 44.5%
mucosa? 3
C 11.8%
D 12.5% 4
A. Glossopharyngeal E 7.7% 5
B. Laryngeal branches of the vagus 6
44.5% of users answered this
C. Ansa cervicalis question correctly

D. Laryngeal branches of the trigeminal


E. None of the above

Next question

The laryngeal branches of the vagus supply sensory information from the larynx.

Larynx

The larynx lies in the anterior part of the neck at the levels of C3 to C6 vertebral
bodies. The laryngeal skeleton consists of a number of cartilagenous segments.
Three of these are paired; arytenoid, corniculate and cuneiform. Three are
single; thyroid, cricoid and epiglottic. The cricoid cartilage forms a complete ring
(the only one to do so).
The laryngeal cavity extends from the laryngeal inlet to the level of the inferior
border of the cricoid cartilage.

Divisions of the laryngeal cavity


Laryngeal vestibule Superior to the vestibular folds

Laryngeal ventricle Lies between vestibular folds and superior to the vocal cords

Infraglottic cavity Extends from vocal cords to inferior border of the cricoid cartilage

The vocal folds (true vocal cords) control sound production. The apex of each
fold projects medially into the laryngeal cavity. Each vocal fold includes:

Vocal ligament
Vocalis muscle (most medial part of thyroarytenoid muscle)

The glottis is composed of the vocal folds, processes and rima glottidis. The rima
glottidis is the narrowest potential site within the larynx, as the vocal cords may be
completely opposed, forming a complete barrier.

Muscles of the larynx


Muscle Origin Insertion Innervation Action

Posterior Posterior aspect Muscular process of Recurrent Abducts vocal fold


cricoarytenoid of lamina of arytenoid Laryngeal
cricoid

Lateral Arch of cricoid Muscular process of Recurrent Adducts vocal fold


cricoarytenoid arytenoid laryngeal

Thyroarytenoid Posterior aspect Muscular process of Recurrent Relaxes vocal fold


of thyroid cartilage arytenoid laryngeal

Transverse Arytenoid cartilage Contralateral Recurrent Closure of


and oblique arytenoid laryngeal intercartilagenous
arytenoids part of the rima
glottidis

Vocalis Depression Vocal ligament and Recurrent Relaxes posterior


between lamina vocal process of laryngeal vocal ligament,
of thyroid cartilage arytenoid cartilage tenses anterior
part

Cricothyroid Anterolateral part Inferior margin and External Tenses vocal fold
of cricoid horn of thyroid laryngeal
cartilage
Blood supply
Arterial supply is via the laryngeal arteries, branches of the superior and inferior
thyroid arteries. The superior laryngeal artery is closely related to the internal
laryngeal nerve. The inferior laryngeal artery is related to the inferior laryngeal
nerve. Venous drainage is via superior and inferior laryngeal veins, the former
draining into the superior thyroid vein and the latter draining into the middle
thyroid vein, or thyroid venous plexus.

Lymphatic drainage
The vocal cords have no lymphatic drainage and this site acts as a lymphatic
watershed.
Supraglottic part Upper deep cervical nodes

Subglottic part Prelaryngeal and pretracheal nodes and inferior deep cervical nodes

The aryepiglottic fold and vestibular folds have a dense plexus of lymphatics
associated with them and malignancies at these sites have a greater propensity
for nodal metastasis.

Topography of the larynx

Image sourced from Wikipedia

Rate question: Next question

Com m ent on this question

All contents of this site are © 2012 E-Medical Revision Ltd Terms and Conditions Privacy policy
yomna92009@yahoo.com - MRCS Part A - My account

Reference ranges End and review

Question 7 of 320 Question stats Score: 14.3%

1
A 74.5%
Which of the following nerves passes through the greater sciatic foramen and 2
B 7%
innervates the perineum? 3
C 6.5%
D 6.7% 4
A. Pudendal E 5.3% 5
B. Sciatic 6
74.5% of users answered this
C. Superior gluteal question correctly
7
D. Inferior gluteal
E. Posterior cutaneous nerve of the thigh

Next question

3 divisions of the pudendal nerve:

Rectal nerve
Perineal nerve
Dorsal nerve of penis/ clitoris

All these pass through the greater sciatic foramen.

The pudendal nerve innervates the perineum. It passes between piriformis and
coccygeus medial to the sciatic nerve.

Gluteal region

Gluteal muscles

Gluteus maximus: inserts to gluteal tuberosity of the femur and iliotibial tract
Gluteus medius: attach to lateral greater trochanter
Gluteus minimis: attach to anterior greater trochanter
All extend and abduct the hip

Deep lateral hip rotators

Piriformis
Gemelli
Obturator internus
Quadratus femoris

Nerves

Superior gluteal nerve (L5, S1) Gluteus medius


Gluteus minimis
Tensor fascia lata

Inferior gluteal nerve Gluteus maximus

Damage to the superior gluteal nerve will result in the patient developing a
Trendelenberg gait. Affected patients are unable to abduct the thigh at the hip
joint. During the stance phase, the weakened abductor muscles allow the pelvis to
tilt down on the opposite side. To compensate, the trunk lurches to the weakened
side to attempt to maintain a level pelvis throughout the gait cycle. The pelvis
sags on the opposite side of the lesioned superior gluteal nerve.

Rate question: Next question

Com m ent on this question


All contents of this site are © 2012 E-Medical Revision Ltd Terms and Conditions Privacy policy
yomna92009@yahoo.com - MRCS Part A - My account

Reference ranges End and review

Question 8 of 320 Question stats Score: 12.5%

1
A 27.2%
Which of the following is true in relation to the sartorius muscle? 2
B 10.5%
C 7.9% 3

A. Innervated by the deep branch of the femoral nerve D 38.2% 4

B. Inserts at the fibula E 16.2% 5

C. It is the shortest muscle in the body 38.2% of users answered this 6


question correctly
D. Forms the Pes anserinus with Gracilis and semitendinous muscle 7

E. Causes extension of the knee 8

Next question

It is innervated by the superficial branch of the femoral nerve. It is a component of


the pes anserinus.

Sartorius

Longest strap muscle in the body


Most superficial muscle in the anterior compartment of the thigh

Origin Anterior superior iliac spine

Insertion Medial surface of the of the body of the tibia (upper part). It inserts anterior to
gracilis and semitendinosus

Nerve Femoral nerve (L2,3)


Supply

Action Flexor of the hip and knee, slight abducts the thigh and rotates it laterally
It assists with medial rotation of the tibia on the femur. For example it
would play a pivotal role in placing the right heel onto the left knee ( and
vice versa)

Important The middle third of this muscle, and its strong underlying fascia forms the roof
relations of the adductor canal , in which lie the femoral vessels, the saphenous nerve
and the nerve to vastus medialis.

Rate question: Next question

Com m ent on this question

All contents of this site are © 2012 E-Medical Revision Ltd Terms and Conditions Privacy policy
yomna92009@yahoo.com - MRCS Part A - My account

Reference ranges End and review

0/3 Question 9-11 of 320 Question stats Score: 9.1%

Average score for registered users: 1

Theme: Nerve lesions 2


9 76.8%
3
10 92.1%
A. Sciatic nerve
11 79.4% 4
B. Peroneal nerve
5
C. Tibial Nerve
6
D. Obturator nerve
7
E. Ilioinguinal nerve
8
F. Femoral nerve
9-11 0 / 3
G. None of the above

Please select the most likely nerve injury for the scenario given. Each option may
be used once, more than once or not at all

9. A 56 year old man undergoes a low anterior resection with legs in


the Lloyd-Davies position. Post operatively he complains of foot
drop.

You answered None of the above

The correct answer is Peroneal nerve

Positioning legs in Lloyd- Davies stirrups can carry the risk of


peroneal nerve neuropraxia if not done carefully.

10. A 23 year old man complains of severe groin pain several weeks
after a difficult inguinal hernia repair.

You answered None of the above

The correct answer is Ilioinguinal nerve

The ilioinguinal nerve may have been entrapped in the mesh


causing a neuroma.

11. A 72 year old man develops a foot drop after a revision total hip
replacement.

You answered None of the above

The correct answer is Sciatic nerve

This may be done by a number of approaches, in this scenario a


posterior approach is the most likely culprit.

Next question

Lower limb- Muscular compartments

Anterior compartment
Muscle Nerve Action

Tibialis anterior Deep peroneal Dorsiflexes ankle joint, inverts foot


nerve

Extensor digitorum Deep peroneal Extends lateral four toes, dorsiflexes ankle
longus nerve joint

Peroneus tertius Deep peroneal Dorsiflexes ankle, everts foot


Peroneus tertius Deep peroneal Dorsiflexes ankle, everts foot
nerve

Extensor hallucis Deep peroneal Dorsiflexes ankle joint, extends big toe
longus nerve

Peroneal compartment
Muscle Nerve Action

Peroneus longus Superficial peroneal nerve Everts foot, assists in plantar flexion

Peroneus brevis Superficial peroneal nerve Plantar flexes the ankle joint

Superficial posterior compartment


Nerve Action

Gastrocnemius Tibial nerve Plantar flexes the foot, may also flex the knee

Soleus Tibial nerve Plantar flexor

Deep posterior compartment


Muscle Nerve Action

Flexor digitorum longus Tibial Flexes the lateral four toes

Flexor hallucis longus Tibial Flexes the great toe

Tibialis posterior Tibial Plantar flexor, inverts the foot

Rate question: Next question

Com m ent on this question

All contents of this site are © 2012 E-Medical Revision Ltd Terms and Conditions Privacy policy
yomna92009@yahoo.com - MRCS Part A - My account

Reference ranges End and review

Question 12 of 320 Question stats Score: 8.3%

1
A 5.6%
A 68 year old man falls onto an outstretched hand. Following the accident he is 2
B 76%
examined in the emergency department. On palpating his anatomical snuffbox 3
C 7.6%
there is tenderness noted in the base. What is the most likely injury in this
D 6.1% 4
scenario?
E 4.7% 5

76% of users answered this 6


A. Rupture of the tendon of flexor pollicis
question correctly
7
B. Scaphoid fracture
8
C. Distal radius fracture
9-11 0 / 3
D. Rupture of flexor carpi ulnaris tendon
12
E. None of the above

Next question

A fall onto an outstretched hand is a common mechanism of injury for a scaphoid


fracture. This should be suspected clinically if there is tenderness in the base of
the anatomical snuffbox. A tendon rupture would not result in bony tenderness.

Scaphoid bone

The scaphoid has a concave articular surface for the head of the capitate and at
the edge of this is a crescentic surface for the corresponding area on the lunate.
Proximally, it has a wide convex articular surface with the radius. It has a distally
sited tubercle that can be palpated. The remaining articular surface is to the
lateral side of the tubercle. It faces laterally and is associated with the trapezium
and trapezoid bones.

The narrow strip between the radial and trapezial surfaces and the tubercle gives
rise to the radial collateral carpal ligament. The tubercle receives part of the flexor
retinaculum. This area is the only part of the scaphoid that is available for the
entry of blood vessels. It is commonly fractured and avascular necrosis may
result.

Scaphoid bone

Image sourced from Wikipedia

Rate question: Next question

Com m ent on this question


yomna92009@yahoo.com - MRCS Part A - My account

Reference ranges End and review

Question 13 of 320 Question stats Score: 15.4%

1
A 9.4%
A 25 year old man sustains a severe middle cranial fossa basal skull fracture. 2
B 28.5%
Once he has recovered it is noticed that he has impaired tear secretion. This is 3
C 21.3%
most likely to be the result of damage to which of the following?
D 13.1% 4
E 27.7% 5
A. Stellate ganglion
27.7% of users answered this 6
B. Ciliary ganglion question correctly
7
C. Otic ganglion
8
D. Trigeminal nerve
9-11 0 / 3
E. Greater petrosal nerve
12

13
Next question

The greater petrosal nerve may be injured and carries fibres for lacrimation (see
below).

Lacrimal system

Lacrimal gland
Consists of an orbital part and palpebral part. They are continuous
posterolaterally around the concave lateral edge of the levator palpebrae
superioris muscle.
The ducts of the lacrimal gland open into the superior fornix. Those from the
orbital part penetrate the aponeurosis of levator palpebrae superioris to join
those from the palpebral part. Therefore excision of the palpebral part is
functionally similar to excision of the entire gland.

Blood supply
Lacrimal branch of the opthalmic artery. Venous drainage is to the superior
opthalmic vein.

Innervation
The gland is innervated by the secretomotor paraympathetic fibres from the
pterygopalatine ganglion which in turn may reach the gland via the zygomatic or
lacrimal branches of the maxillary nerve or pass directly to the gland. The
preganglionic fibres travel to the ganglion in the greater petrosal nerve (a branch
of the facial nerve at the geniculate ganglion).

Nasolacrimal duct
Descends from the lacrimal sac to open anteriorly in the inferior meatus of the
nose.

Lacrimation reflex
Occurs in response to conjunctival irritation (or emotional events). The
conjunctiva will send signals via the opthalmic nerve. These then pass to the
superior salivary centre. The efferent signals pass via the greater petrosal nerve
(parasympathetic preganglionic fibres) and the deep petrosal nerve which carries
the post ganglionic sympathetic fibres. The parasympathetic fibres will relay in the
pterygopalatine ganglion, the sympathetic fibres do not synapse. They in turn will
relay to the lacrimal apparatus.

Rate question: Next question

Com m ent on this question

All contents of this site are © 2012 E-Medical Revision Ltd Terms and Conditions Privacy policy
yomna92009@yahoo.com - MRCS Part A - My account

Reference ranges End and review

Question 14 of 320 Question stats Score: 14.3%

1
A 21.7%
Which of the following structures passes through the quadrangular space near 2
B 12.8%
the humeral head? 3
C 45.4%
D 7.7% 4
A. Axillary artery E 12.4% 5
B. Radial nerve 6
45.4% of users answered this
C. Axillary nerve question correctly
7
D. Median nerve
8
E. Transverse scapular artery 9-11 0 / 3

12
Next question
13

14
The quadrangular space is bordered by the humerus laterally, subscapularis
superiorly, teres major inferiorly and the long head of triceps medially. It lies
lateral to the triangular space. It transmits the axillary nerve and posterior
circumflex humeral artery.

Image sourced from Wikipedia

Shoulder joint

Shallow synovial ball and socket type of joint.


It is an inherently unstable joint, but is capable to a wide range of
movement.
Stability is provided by muscles of the rotator cuff that pass from the
scapula to insert in the greater tuberosity (all except sub scapularis-lesser
tuberosity).

Glenoid labrum

Fibrocartilaginous rim attached to the free edge of the glenoid cavity


Tendon of the long head of biceps arises from within the joint from the
supraglenoid tubercle, and is fused at this point to the labrum.
The long head of triceps attaches to the infraglenoid tubercle

Fibrous capsule

Attaches to the scapula external to the glenoid labrum and to the labrum
itself (postero-superiorly)
Attaches to the humerus at the level of the anatomical neck superiorly and
the surgical neck inferiorly
Anteriorly the capsule is in contact with the tendon of subscapularis,
superiorly with the supraspinatus tendon, and posteriorly with the tendons
yomna92009@yahoo.com - MRCS Part A - My account

Reference ranges End and review

Question 15 of 320 Question stats Score: 13.3%

1
A 13%
Which of the following pairings of foramina and their contents is not correct? 2
B 11.2%
C 48.5% 3

A. Superior orbital fissure and the oculomotor nerve D 13.1% 4

B. Foramina rotundum and the maxillary nerve E 14.3% 5

C. Jugular foramen and the hypoglossal nerve 48.5% of users answered this 6
question correctly
D. Foramina spinosum and the middle meningeal artery 7

E. Foramina lacerum and the internal carotid artery 8

9-11 0 / 3
Next question 12

13
The hypoglossal nerve passes through the hypoglossal canal.
14

Foramina of the base of the skull 15

Foramen Location Contents

Foramen Sphenoid Otic ganglion


ovale bone V3 (Mandibular nerve:3rd branch of
trigeminal)
Accessory meningeal artery
Lesser petrosal nerve
Emissary veins

Foramen Sphenoid Middle meningeal artery


spinosum bone Meningeal branch of the Mandibular nerve

Foramen Sphenoid Maxillary nerve (V2)


rotundum bone

Foramen Sphenoid Base of the medial pterygoid plate.


lacerum bone Internal carotid artery
Nerve and artery of the pterygoid canal

Jugular Temporal Anterior: inferior petrosal sinus


foramen bone Intermediate: glossopharyngeal, vagus, and accessory nerves.
Posterior: sigmoid sinus (becoming the internal jugular vein) and
some meningeal branches from the occipital and ascending
pharyngeal arteries.

Foramen Occipital Anterior and posterior spinal arteries


magnum bone Vertebral arteries
Medulla oblongata

Stylomastoid Temporal Stylomastoid artery


foramen bone Facial nerve

Superior Sphenoid Oculomotor nerve (III)


orbital bone trochlear nerve (IV)
fissure lacrimal, frontal and nasociliary branches of ophthalmic nerve
(V1)
abducent nerve (VI)
Superior and inferior ophthalmic vein

Base of skull anatomical overview

Image sourced from Wikipedia


yomna92009@yahoo.com - MRCS Part A - My account

Reference ranges End and review

Question 16 of 320 Question stats Score: 12.5%

1
A 56.2%
A 55 year old man with carcinoma of the larynx is undergoing a difficult 2
B 9.5%
laryngectomy. The surgeons divide the thyrocervical trunk, from which of the 3
C 8.8%
following vessels does this structure most commonly originate?
D 10.9% 4
E 14.6% 5
A. Subclavian artery
56.2% of users answered this 6
B. Common carotid artery question correctly
7
C. Vertebral artery
8
D. External carotid artery
9-11 0 / 3
E. Internal carotid artery
12

13
Next question
14

The thyrocervical trunk is a branch of the subclavian artery. It arises from the first 15
part between the subclavian artery and the inner border of scalenus anterior. It 16
branches off the subclavian distal to the vertebral artery.

Root of the neck

Thoracic Outlet

Where the subclavian artery and vein and the brachial plexus exit the
thorax and enter the arm.
They pass over the 1st rib and under the clavicle.
The subclavian vein is the most anterior structure and is immediately
anterior to scalenus anterior and its attachment to the first rib.
Scalenus anterior has 2 parts, the subclavian artery leaves the thorax by
passing over the first rib and between these 2 portions of the muscle.
At the level of the first rib, the lower cervical nerve roots combine to form
the 3 trunks of the brachial plexus. The lowest trunk is formed by the union
of C8 and T1, and this trunk lies directly posterior to the artery and is in
contact with the superior surface of the first rib.

Thoracic outlet obstruction causes neurovascular compromise.

Rate question: Next question

Com m ent on this question

All contents of this site are © 2012 E-Medical Revision Ltd Terms and Conditions Privacy policy
yomna92009@yahoo.com - MRCS Part A - My account

Reference ranges End and review

Question 17 of 320 Question stats Score: 11.8%

1
A 19.4%
The following structures are closely related to the brachiocephalic artery except: 2
B 11.5%
C 25.5% 3

A. Trachea posteriorly D 30.4% 4

B. Right brachiocephalic vein E 13.2% 5

C. Inferior thyroid vein 30.4% of users answered this 6


question correctly
D. Right recurrent laryngeal nerve 7

E. None of the above 8

9-11 0 / 3
Next question 12

13
There is no brachiocephalic artery on the left, however the left brachiocephalic
14
vein lies anteriorly to the roots of all the 3 great arteries (including the
brachiocephalic artery). The right recurrent laryngeal nerve has no relation to the 15
brachiocephalic artery. 16

17
Brachiocephalic artery

The brachiocephalic artery is the largest branch of the aortic arch. From its aortic
origin it ascends superiorly, it initially lies anterior to the trachea and then on its
right hand side. It branches into the common carotid and right subclavian arteries
at the level of the sternoclavicular joint.

Path
Origin- apex of the midline of the aortic arch
Passes superiorly and posteriorly to the right
Divides into the right subclavian and right common carotid artery

Relations

Anterior Sternohyoid
Sternothyroid
Thymic remnants
Left brachiocephalic vein
Right inferior thyroid veins

Posterior Trachea
Right pleura

Right lateral Right brachiocephalic vein


Superior part of SVC

Left lateral Thymic remnants


Origin of left common carotid
Inferior thyroid veins
Trachea (higher level)

Branches
Normally none but may have the thyroidea ima artery
Image sourced from Wikipedia

Rate question: Next question

Com m ent on this question

All contents of this site are © 2012 E-Medical Revision Ltd Terms and Conditions Privacy policy
yomna92009@yahoo.com - MRCS Part A - My account

Reference ranges End and review

Question 18 of 320 Question stats Score: 11.1%

1
A 9.1%
Which of the following structures separates the ulnar artery from the median 2
B 26.9%
nerve? 3
C 17.5%
D 38.5% 4
A. Brachioradialis E 8% 5
B. Pronator teres 6
26.9% of users answered this
C. Tendon of biceps brachii question correctly
7
D. Flexor carpi ulnaris
8
E. Brachialis 9-11 0 / 3

12
Next question
13

14
It lies deep to pronator teres and this separates it from the median nerve.
15

Ulnar artery 16

17
Path
18
Starts: middle of antecubital fossa
Passes obliquely downward, reaching the ulnar side of the forearm at a
point about midway between the elbow and the wrist. It follows the ulnar
border to the wrist, crossing over the flexor retinaculum. It then divides into
the superficial and deep volar arches.

Relations
Deep to- Pronator teres, Flexor carpi radialis, Palmaris longus
Lies on- Brachialis and Flexor digitorum profundus
Superficial to the flexor retinaculum at the wrist

The median nerve is in relation with the medial side of the artery for about 2.5 cm.
And then crosses the vessel, being separated from it by the ulnar head of the
Pronator teres

The ulnar nerve lies medially to the lower two-thirds of the artery

Branch

Anterior interosseous artery


Image sourced from Wikipedia

Rate question: Next question

Com m ent on this question

All contents of this site are © 2012 E-Medical Revision Ltd Terms and Conditions Privacy policy
yomna92009@yahoo.com - MRCS Part A - My account

Reference ranges End and review

Question 19 of 320 Question stats Score: 10.5%

1
A 18.8%
Which muscle is supplied by the superficial peroneal nerve? 2
B 6.4%
C 5.9% 3

A. Peroneus tertius D 61.6% 4

B. Sartorius E 7.3% 5

C. Adductor magnus 61.6% of users answered this 6


question correctly
D. Peroneus brevis 7

E. Gracilis 8

9-11 0 / 3
Next question 12

13

14

Superficial peroneal nerve 15

16
Supplies
17

Lateral compartment of leg: peroneus longus, peroneus brevis (action: 18


eversion and plantar flexion)
19
Sensation over dorsum of the foot (except the first web space, which is
innervated by the deep peroneal nerve)

Path

Passes between peroneus longus and peroneus brevis along the length of
the proximal one third of the fibula
10-12 cm above the tip of the lateral malleolus, the superficial peroneal
nerve pierces the fascia
6-7 cm distal to the fibula, the superficial peroneal nerve bifurcates into
intermediate and medial dorsal cutaneous nerves

Rate question: Next question

Com m ent on this question

All contents of this site are © 2012 E-Medical Revision Ltd Terms and Conditions Privacy policy
yomna92009@yahoo.com - MRCS Part A - My account

Reference ranges End and review

Question 20 of 320 Question stats Score: 15%

1
A 8.2%
A 32 year old motorcyclist is involved in a road traffic accident. His humerus is 2
B 25%
fractured and severely displaced. At the time of surgical repair the surgeon notes 3
C 33.5%
that the radial nerve has been injured. Which of the following muscles is least
D 10.1% 4
likely to be affected by an injury at this site?
E 23.2% 5

23.2% of users answered this 6


A. Extensor carpi radialis brevis
question correctly
7
B. Brachioradialis
8
C. Abductor pollicis longus
9-11 0 / 3
D. Extensor pollicis brevis
12
E. None of the above
13

Next question 14

15

16
Muscles supplied by the radial nerve
17

BEST 18
Brachioradialis
19
Extensors
Supinator 20

Triceps

The radial nerve supplies the extensor muscles, abductor pollicis longus and
extensor pollicis brevis (the latter two being innervated by the posterior
interosseous branch of the radial nerve).

Radial nerve

Continuation of posterior cord of the brachial plexus (root values C5 to T1)

Path

In the axilla: lies posterior to the axillary artery on subscapularis, latissimus


dorsi and teres major.
Enters the arm between the brachial artery and the long head of triceps
(medial to humerus).
Spirals around the posterior surface of the humerus in the groove for the
radial nerve.
At the distal third of the lateral border of the humerus it then pierces the
intermuscular septum and descends in front of the lateral epicondyle.
At the lateral epicondyle it lies deeply between brachialis and
brachioradialis where it then divides into a superficial and deep terminal
branch.
Deep branch crosses the supinator to become the posterior interosseous
nerve.

In the image below the relationships of the radial nerve can be appreciated
yomna92009@yahoo.com - MRCS Part A - My account

Reference ranges End and review

Question 21 of 320 Question stats Score: 14.3%

1
A 12.3%
A man develops an infection in his external auditory meatus. The infection is 2
B 11.2%
extremely painful. Which of the following nerves conveys sensation from this 3
C 12.7%
region?
D 49.6% 4
E 14.2% 5
A. Occipital branch of the trigeminal nerve
49.6% of users answered this 6
B. Vestibulocochlear nerve question correctly
7
C. Facial nerve
8
D. Auriculotemporal nerve
9-11 0 / 3
E. Maxillary branch of the trigeminal nerve
12

13
Next question
14

15
Tensor tympania and stapedius are the only two muscles of the middle ear.
16
Contraction of tensor tympani will tend to dampen the vibrations produced by
loud sounds, it is innervated by a branch of the trigeminal nerve. The 17

stapedius dampens movements of the ossicles in response to loud sounds 18


and is innervated by a branch of the facial nerve.
19

20
The auriculotemporal nerve, which is derived from the mandibular branch of the
21
trigeminal nerve supplies this area.

Ear- anatomy

The ear is composed of three anatomically distinct regions.

External ear
Auricle is composed of elastic cartilage covered by skin. The lobule has no
cartilage and contains fat and fibrous tissue.

External auditory meatus is approximately 2.5cm long.


Lateral third of the external auditory meatus is cartilaginous and the medial two
thirds is bony.

The region is innervated by the greater auricular nerve. The auriculotemporal


branch of the trigeminal nerve supplies most the of external auditory meatus and
the lateral surface of the auricle.

Middle ear
Space between the tympanic membrane and cochlea. The aditus leads to the
mastoid air cells is the route through which middle ear infections may cause
mastoiditis. Anteriorly the eustacian tube connects the middle ear to the naso
pharynx.
The tympanic membrane consists of:

Outer layer of stratified squamous epithelium.


Middle layer of fibrous tissue.
Inner layer of mucous membrane continuous with the middle ear.

The tympanic membrane is approximately 1cm in diameter.


The chorda tympani nerve passes on the medial side of the pars flaccida.

The middle ear is innervated by the glossopharyngeal nerve and pain may
radiate to the middle ear following tonsillectomy.

Ossicles
Malleus attaches to the tympanic membrane (the Umbo).
Malleus articulates with the incus (synovial joint).
Incus attaches to stapes (another synovial joint).

Internal ear
Cochlea, semi circular canals and vestibule

Organ of corti is the sense organ of hearing and is located on the inside of the
cochlear duct on the basilar membrane.

Vestibule accommodates the utricule and the saccule. These structures contain
endolymph and are surrounded by perilymph within the vestibule.

The semicircular canals lie at various angles to the petrous temporal bone. All
share a common opening into the vestibule.

Rate question: Next question

Com m ent on this question

All contents of this site are © 2012 E-Medical Revision Ltd Terms and Conditions Privacy policy
yomna92009@yahoo.com - MRCS Part A - My account

Reference ranges End and review

Question 1 of 299 Question stats Score: 0%

1
A 48.5%
Which muscle is responsible for causing flexion of the interphalangeal joint of the B 24.4%
thumb?
C 8.9%
D 11.3%

A. Flexor pollicis longus E 6.9%

B. Flexor pollicis brevis


48.5% of users answered this
C. Flexor digitorum superficialis question correctly

D. Flexor digitorum profundus


E. Adductor pollicis

Next question

There are 8 muscles:


1. Two flexors (flexor pollicis brevis and flexor pollicis longus)
2. Two extensors (extensor pollicis brevis and longus)
3. Two abductors (abductor pollicis brevis and longus)
4. One adductor (adductor pollicis)
5. One muscle that opposes the thumb by rotating the CMC joint (opponens
pollicis).

Flexor and extensor longus insert on the distal phalanx moving both the MCP and
IP joints.

Hand

Anatomy of the hand

Bones 8 Carpal bones


5 Metacarpals
14 phalanges

Intrinsic 7 Interossei - Supplied by ulnar nerve


Muscles
3 palmar-adduct fingers
4 dorsal- abduct fingers

Intrinsic Lumbricals
muscles
Flex MCPJ and extend the IPJ.
Origin deep flexor tendon and insertion dorsal extensor hood
mechanism.
Innervation: 1st and 2nd- median nerve, 3rd and 4th- deep
branch of the ulnar nerve.

Thenar Abductor pollicis brevis


eminence Opponens pollicis
Flexor pollicis brevis

Hypothenar Opponens digiti minimi


eminence Flexor digiti minimi brevis
Abductor digiti minimi
yomna92009@yahoo.com - MRCS Part A - My account

Reference ranges End and review

Question 2 of 299 Question stats Score: 0%

1
A 31.7%
Which of the following structures separates the posterior cruciate ligament from 2
B 16.4%
the popliteal artery?
C 30.9%
D 10%

A. Oblique popliteal ligament E 10.8%

B. Transverse ligament
31.7% of users answered this
C. Popliteus tendon question correctly

D. Biceps femoris
E. Semitendinosus

Next question

The posterior cruciate ligament is separated from the popliteal vessels at its origin
by the oblique popliteal ligament. The transverse ligament is located anteriorly.

Knee joint

The knee joint is a synovial joint, the largest and most complicated. It consists of
two condylar joints between the femure and tibia and a sellar joint between the
patella and the femur. The tibiofemoral articular surfaces are incongruent,
however, this is improved by the presence of the menisci. The degree of
congruence is related to the anatomical position of the knee joint and is greatest
in full extension.

Knee joint compartments

Tibiofemoral Comprise of the patella/femur joint, lateral and medial


compartments (between femur condyles and tibia)
Synovial membrane and cruciate ligaments partially
separate the medial and lateral compartments

Patellofemoral Ligamentum patellae


Actions: provides joint stability in full extension

Fibrous capsule
The capsule of the knee joint is a complex, composite structure with contributions
from adjacent tendons.
Anterior The capsule does not pass proximal to the patella. It blends with
fibres the tendinous expansions of vastus medialis and lateralis

Posterior These fibres are vertical and run from the posterior surface of the
fibres femoral condyles to the posterior aspect of the tibial condyle

Medial Attach to the femoral and tibial condyles beyond their articular
fibres margins, blending with the tibial collateral ligament

Lateral Attach to the femur superior to popliteus, pass over its tendon to
fibres head of fibula and tibial condyle

Bursae

Anterior Subcutaneous prepatellar bursa; between patella and skin


Deep infrapatellar bursa; between tibia and patellar ligament
Subcutaneous infrapatellar bursa; between distal tibial
tuberosity and skin

Laterally Bursa between lateral head of gastrocnemius and joint


capsule
Bursa between fibular collateral ligament and tendon of
biceps femoris
Bursa between fibular collateral ligament and tendon of
popliteus
Medially Bursa between medial head of gastrocnemius and the fibrous
capsule
Bursa between tibial collateral ligament and tendons of
sartorius, gracilis and semitendinosus
Bursa between the tendon of semimembranosus and medial
tibial condyle and medial head of gastrocnemius

Posterior Highly variable and inconsistent

Ligaments
Medial collateral Medial epicondyle femur to medial tibial condyle: valgus
ligament stability

Lateral collateral Lateral epicondyle femur to fibula head: varus stability


ligament

Anterior cruciate Anterior tibia to lateral intercondylar notch femur: prevents


ligament tibia sliding anteriorly

Posterior Posterior tibia to medial intercondylar notch femur:


cruciate ligament prevents tibia sliding posteriorly

Patellar ligament Central band of the tendon of quadriceps femoris, extends


from patella to tibial tuberosity

Image sourced from Wikipedia

Image sourced from Wikipedia

Menisci
Medial and lateral menisci compensate for the incongruence of the femoral and
tibial condyles.
Composed of fibrous tissue.
Medial meniscus is attached to the tibial collateral ligament.
Lateral meniscus is attached to the loose fibres at the lateral edge of the joint and
is separate from the fibular collateral ligament. The lateral meniscus is crossed by
the popliteus tendon.

Nerve supply
The knee joint is supplied by the femoral, tibial and common peroneal divisions of
the sciatic and by a branch from the obturator nerve. Hip pathology pain may be
referred to the knee.

Blood supply
Genicular branches of the femoral artery, popliteal and anterior tibial arteries all
supply the knee joint.

Rate question: Next question

Com m ent on this question

All contents of this site are © 2012 E-Medical Revision Ltd Terms and Conditions Privacy policy
yomna92009@yahoo.com - MRCS Part A - My account

Reference ranges End and review

Question 3 of 299 Question stats Score: 0%

1
A 43.5%
Which structure is not at the level of the sternal angle? 2
B 12.8%
C 14.2% 3

A. Left brachiocephalic vein D 8.7%

B. Intervertebral discs T4-T5 E 20.7%

C. Start of aortic arch 43.5% of users answered this


question correctly
D. 2nd pair of costal cartilages
E. Bifurcation of the trachea into left and right bronchi

Next question

The left brachiocephalic vein lies posterior to the manubrium, at the level of its
upper border. The sternal angle refers to the transition between manubrium and
sternum and therefore will not include the left brachiocephalic vein.

Sternal angle

Anatomical structures at the level of the manubrium and upper sternum

Upper part of the manubrium Left brachiocephalic vein


Brachiocephalic artery
Left common carotid
Left subclavian artery

Lower part of the manubrium/ Costal cartilages of the 2nd ribs


manubrio-sternal angle Transition point between superior and
inferior mediastinum
Arch of the aorta
Tracheal bifurcation
Union of the azygos vein and superior
vena cava
The thoracic duct crosses to the midline

Rate question: Next question

Com m ent on this question

All contents of this site are © 2012 E-Medical Revision Ltd Terms and Conditions Privacy policy
yomna92009@yahoo.com - MRCS Part A - My account

Reference ranges End and review

Question 4 of 299 Question stats Score: 0%

1
A 53.9%
A 13 month old boy is brought to the paediatric clinic by his mother who is 2
B 20.2%
concerned that his testis are not palpable. On examination his testis are not 3
C 7%
palpable either in the scrotum or inguinal region and cannot be visualised on
D 10.5% 4
ultrasound either. Which of the following is the most appropriate next stage in
E 8.4%
management?

53.9% of users answered this


question correctly
A. Laparoscopy
B. Re-assess at 5 years of age
C. Re-assess at 13 years of age
D. Administration of testosterone
E. Administration of cyproterone acetate

Next question

Truly impalpable testis and inability to visualise on ultrasound are indications for a
laparoscopy. They may be associated with an intra-abdominal location. Whilst it is
reasonable to defer orchidopexy for retractile testis completely absent testes
should be investigated further.

Cryptorchidism

A congenital undescended testis is one that has failed to reach the bottom of the
scrotum by 3 months of age. At birth up to 5% of boys will have an undescended
testis, post natal descent occurs in most and by 3 months the incidence of
cryptorchidism falls to 1-2%. In the vast majority of cases the cause of the
maldescent is unknown. A proportion may be associated with other congenital
defects including:

Patent processus vaginalis


Abnormal epididymis
Cerebral palsy
Mental retardation
Wilms tumour
Abdominal wall defects (e.g. gastroschisis, prune belly syndrome)

Reasons for correction of cryptorchidism

Reduce risk of infertility


Allows the testes to be examined for testicular cancer
Avoid testicular torsion
Cosmetic appearance

Males with undescended testis are 40 times as likely to develop testicular cancer
(seminoma) as males without undescended testis
The location of the undescended testis affects the relative risk of testicular cancer
(50% intra-abdominal testes)

Treatment

Orchidopexy at 6- 18 months of age. The operation usually consists of


inguinal exploration, mobilisation of the testis and implantation into a dartos
pouch.
Intra-abdominal testis should be evaluated laparoscopically and mobilised.
Whether this is a single stage or two stage procedure depends upon the
exact location.
After the age of 2 years in untreated individuals the Sertoli cells will
degrade and those presenting late in teenage years may be better served
by orchidectomy than to try and salvage a non functioning testis with an
increased risk of malignancy.
yomna92009@yahoo.com - MRCS Part A - My account

Reference ranges End and review

Question 5 of 299 Question stats Score: 0%

1
A 6.1%
A 53 year old man is undergoing a left hemicolectomy for carcinoma of the 2
B 61.3%
descending colon. From which embryological structure is this region of the 3
C 17.4%
gastrointestinal tract derived?
D 8.6% 4
E 6.5% 5
A. Vitellino-intestinal duct
61.3% of users answered this
B. Hind gut question correctly
C. Mid gut
D. Fore gut
E. Woolffian duct

Next question

The left colon is embryologically part of the hind gut. Which accounts for its
separate blood supply via the IMA.

Colon anatomy

The colon is about 1.5m long although this can vary considerably.
Components:

Ascending colon
Transverse colon
Descending colon
Sigmoid colon

Arterial supply
Superior mesenteric artery and inferior mesenteric artery: linked by the marginal
artery.
Ascending colon: ileocolic and right colic arteries
Transverse colon: middle colic artery
Descending and sigmoid colon: left colic artery

Venous drainage
From regional veins (that accompany arteries) to superior and inferior mesenteric
vein

Lymphatic drainage
Initially along nodal chains that accompany supplying arteries, then para-aortic
nodes.

Embryology
Midgut- Second part of duodenum to 2/3 transverse colon
Hindgut- Distal 1/3 transverse colon to anus

Peritoneal location
The right and left colon are part intraperitoneal and part extraperitoneal. The
sigmoid and transverse colon are generally wholly intraperitoneal. This has
implications for the sequelae of perforations, which will tend to result in
generalised peritonitis in the wholly intra peritoneal segments.

Rate question: Next question

Com m ent on this question

All contents of this site are © 2012 E-Medical Revision Ltd Terms and Conditions Privacy policy
yomna92009@yahoo.com - MRCS Part A - My account

Reference ranges End and review

Question 6 of 299 Question stats Score: 0%

1
A 14%
What is the most useful test to clinically distinguish between an upper and lower 2
B 10.9%
motor neurone lesion of the facial nerve? 3
C 16.8%
D 49.7% 4
A. Blow cheeks out E 8.5% 5
B. Loss of chin reflex 6
49.7% of users answered this
C. Close eye question correctly

D. Raise eyebrow
E. Open mouth against resistance

Next question

Upper motor neurone lesions of the facial nerve- Paralysis of the lower half of
face.
Lower motor neurone lesion- Paralysis of the entire ipsilateral face.

Theme from April 2012 Exam

Upper Vs Lower motor neurone lesions - Facial nerve

The nucleus of the facial nerve is located in the caudal aspect of the ventrolateral
pontine tegmentum. Its axons exit the ventral pons medial to the spinal trigeminal
nucleus.

Any lesion occurring within or affecting the corticobulbar tract is known as an


upper motor neuron lesion. Any lesion affecting the individual branches
(temporal, zygomatic, buccal, mandibular and cervical) is known as a lower motor
neuron lesion.

Branches of the facial nerve leaving the facial motor nucleus (FMN) for the
muscles do so via both left and right posterior (dorsal) and anterior (ventral)
routes. In other words, this means lower motor neurons of the facial nerve can
leave either from the left anterior, left posterior, right anterior or right posterior
facial motor nucleus. The temporal branch travels out from the left and right
posterior components. The inferior four branches do so via the left and right
anterior components. The left and right branches supply their respective sides of
the face (ipsilateral innervation). Accordingly, the posterior components receive
motor input from both hemispheres of the cerebral cortex (bilaterally), whereas
the anterior components receive strictly contralateral input. This means that the
temporal branch of the facial nerve receives motor input from both hemispheres
of the cerebral cortex whereas the zygomatic, buccal, mandibular and cervical
branches receive information from only contralateral hemispheres.

Now, because the anterior FMN receives only contralateral cortical input whereas
the posterior receives that which is bilateral, a corticobulbar lesion (UMN lesion)
occurring in the left hemisphere would eliminate motor input to the right anterior
FMN component, thus removing signaling to the inferior four facial nerve
branches, thereby paralyzing the right mid- and lower-face. The posterior
component, however, although now only receiving input from the right
hemisphere, is still able to allow the temporal branch to sufficiently innervate the
entire forehead. This means that the forehead will not be paralyzed.

The same mechanism applies for an upper motor neuron lesion in the right
hemisphere. The left anterior FMN component no longer receives cortical motor
input due to its strict contralateral innervation, whereas the posterior component
is still sufficiently supplied by the left hemisphere. The result is paralysis of the left
mid- and lower-face with an unaffected forehead.
On the other hand, a lower motor neuron lesion is a bit different.

A lesion on either the left or right side would affect both the anterior and posterior
routes on that side because of their close physical proximity to one another. So, a
lesion on the left side would inhibit muscle innervation from both the left posterior
and anterior routes, thus paralyzing the whole left side of the face (Bells Palsy).
With this type of lesion, the bilateral and contalateral inputs of the posterior and
anterior routes, respectively, become irrelevant because the lesion is below the
level of the medulla and the facial motor nucleus. Whereas at a level above the
medulla a lesion occurring in one hemisphere would mean that the other
hemisphere could still sufficiently innervate the posterior facial motor nucleus, a
lesion affecting a lower motor neuron would eliminate innervation altogether
because the nerves no longer have a means to receive compensatory
contralateral input at a downstream decussation.

Rate question: Next question

Com m ent on this question

All contents of this site are © 2012 E-Medical Revision Ltd Terms and Conditions Privacy policy
yomna92009@yahoo.com - MRCS Part A - My account

Reference ranges End and review

Question 7 of 299 Question stats Score: 0%

1
A 37.6%
An 18 year old man is stabbed in the axilla during a fight. His axillary artery is 2
B 19.6%
lacerated and repaired. However, the surgeon neglects to repair an associated 3
C 16.7%
injury to the upper trunk of the brachial plexus. Which of the following muscles is
D 12.5% 4
least likely to demonstrate impaired function as a result?
E 13.6% 5

37.6% of users answered this 6


A. Palmar interossei
question correctly
7
B. Infraspinatus
C. Brachialis
D. Supinator brevis
E. None of the above

Next question

The palmar interossei are supplied by the ulnar nerve. Which lies inferiorly and is
therefore less likely to be injured.

Brachial plexus

Origin Anterior rami of C5 to T1

Sections of the Roots, trunks, divisions, cords, branches


plexus Mnemonic:Real Teenagers Drink Cold Beer

Roots Located in the posterior triangle


Pass between scalenus anterior and medius

Trunks Located posterior to middle third of clavicle


Upper and middle trunks related superiorly to the subclavian
artery
Lower trunk passes over 1st rib posterior to the subclavian
artery

Divisions Apex of axilla

Cords Related to axillary artery

Diagram illustrating the branches of the brachial plexus

Image sourced from Wikipedia

Cutaneous sensation of the upper limb


yomna92009@yahoo.com - MRCS Part A - My account

Reference ranges End and review

Question 8 of 299 Question stats Score: 0%

1
A 74.5%
A 23 year old man is involved in a fight, during the dispute he sustains a 2
B 6.4%
laceration to the posterior aspect of his right arm, approximately 2cm proximal to 3
C 7.2%
the olecranon process. On assessment in the emergency department he is
D 5.5% 4
unable to extend his elbow joint. Which of the following tendons is most likely to
E 6.3% 5
have been cut?

74.5% of users answered this 6


question correctly
A. Triceps 7

B. Pronator teres 8

C. Brachioradialis
D. Brachialis
E. Biceps

Next question

Theme from 2009 Exam

The triceps muscle extends the elbow joint. The other muscles listed all produce
flexion of the elbow joint.

Triceps

Origin Long head- infraglenoid tubercle of the scapula.


Lateral head- dorsal surface of the humerus, lateral and proximal to the
groove of the radial nerve
Medial head- posterior surface of the humerus on the inferomedial side
of the radial groove and both of the intermuscular septae

Insertion Olecranon process of the ulna. Here the olecranon bursa is between
the triceps tendon and olecranon.
Some fibres insert to the deep fascia of the forearm, posterior capsule
of the elbow (preventing the capsule from being trapped between
olecranon and olecranon fossa during extension)

Innervation Radial nerve

Blood Profunda brachii artery


supply

Action Elbow extension. The long head can adduct the humerus and and extend it
from a flexed position

Relations The radial nerve and profunda brachii vessels lie between the lateral and
medial heads

Rate question: Next question

Com m ent on this question

All contents of this site are © 2012 E-Medical Revision Ltd Terms and Conditions Privacy policy
yomna92009@yahoo.com - MRCS Part A - My account

Reference ranges End and review

Question 9 of 299 Question stats Score: 0%

1
A 9.6%
A 25 year old man undergoes an excision of a pelvic chondrosarcoma, during the 2
B 12.1%
operation the obturator nerve is sacrificed. Which of the following muscles is least 3
C 8.7%
likely to be affected as a result?
D 60.7% 4
E 8.8% 5
A. Adductor longus
60.7% of users answered this 6
B. Pectineus question correctly
7
C. Adductor magnus
8
D. Sartorius
9
E. Gracilis

Next question

Sartorius is supplied by the femoral nerve. In approximately 20% of the


population, pectineus is supplied by the accessory obturator nerve.

Obturator nerve

The obturator nerve arises from L2, L3 and L4 by branches from the ventral
divisions of each of these nerve roots. L3 forms the main contribution and the
second lumbar branch is occasionally absent. These branches unite in the
substance of psoas major, descending vertically in its posterior part to emerge
from its medial border at the lateral margin of the sacrum. It then crosses the
sacroiliac joint to enter the lesser pelvis, it descends on obturator internus to
enter the obturator groove. In the lesser pelvis the nerve lies lateral to the internal
iliac vessels and ureter, and is joined by the obturator vessels lateral to the ovary
or ductus deferens.

Supplies

Medial compartment of thigh


Muscles supplied: external obturator, adductor longus, adductor brevis,
adductor magnus (not the lower part-sciatic nerve), gracilis
The cutaneous branch is often absent. When present, it passes between
gracilis and adductor longus near the middle part of the thigh, and supplies
the skin and fascia of the distal two thirds of the medial aspect.

Obturator canal

Connects the pelvis and thigh: contains the obturator artery, vein, nerve
which divides into anterior and posterior branches.

Cadaveric cross section demonstrating relationships of the obturator nerve

Image sourced from Wikipedia


yomna92009@yahoo.com - MRCS Part A - My account

Reference ranges End and review

Question 10 of 299 Question stats Score: 0%

1
A 57.7%
You excitedly embark on your first laparoscopic cholecystectomy and during the 2
B 9.4%
operation the anatomy of Calots triangle is more hostile than anticipated. Whilst 3
C 8.2%
trying to apply a haemostatic clip you avulse the cystic artery. This is followed by
D 6.2% 4
brisk haemorrhage. From which source is this most likely to originate ?
E 18.5% 5

57.7% of users answered this 6


A. Right hepatic artery
question correctly
7
B. Portal vein
8
C. Gastroduodenal artery
D. Liver bed 9

E. Common hepatic artery 10

Next question

The cystic artery is a branch of the right hepatic artery. There are recognised
variations in the anatomy of the blood supply to the gallbladder. However, the
commonest situation is for the cystic artery to branch from the right hepatic artery.

Gallbladder

Fibromuscular sac with capacity of 50ml


Columnar epithelium

Relations of the gallbladder


Anterior Liver

Posterior Covered by peritoneum


Transverse colon
1st part of the duodenum

Laterally Right lobe of liver

Medially Quadrate lobe of liver

Arterial supply
Cystic artery (branch of Right hepatic artery)

Venous drainage
Cystic vein

Nerve supply
Sympathetic- mid thoracic spinal cord, Parasympathetic- anterior vagal trunk

Common bile duct

Origin Confluence of cystic and common hepatic ducts

Relations at Medially - Hepatic artery


origin Posteriorly- Portal vein

Relations Duodenum - anteriorly


distally Pancreas - medially and laterally
Right renal vein - posteriorly

Arterial supply Branches of hepatic artery and retroduodenal branches of


gastroduodenal artery

Calot's triangle
Medially Common hepatic duct

Inferiorly Cystic duct

Superiorly Inferior edge of liver

Contents Cystic artery

Rate question: Next question

Com m ent on this question

All contents of this site are © 2012 E-Medical Revision Ltd Terms and Conditions Privacy policy
yomna92009@yahoo.com - MRCS Part A - My account

Reference ranges End and review

Question 11 of 299 Question stats Score: 0%

1
A 17%
A 43 year old man suffers a pelvic fracture which is complicated by an injury to 2
B 6.4%
the junction of the membranous urethra to the bulbar urethra. In which of the 3
C 10.4%
following directions is the extravasated urine most likely to pass?
D 58% 4
E 8.1% 5
A. Posteriorly into extra peritoneal tissues
58% of users answered this 6
B. Laterally into the buttocks question correctly
7
C. Into the abdomen
8
D. Anteriorly into the connective tissues surrounding the scrotum
9
E. None of the above
10

Next question 11

The superficial perineal pouch is a compartment bounded superficially by the


superficial perineal fascia, deep by the perineal membrane (inferior fascia of the
urogenital diaphragm), and laterally by the ischiopubic ramus. It contains the
crura of the penis or clitoris, muscles, viscera, blood vessels, nerves, the proximal
part of the spongy urethra in males, and the greater vestibular glands in
females.
When urethral rupture occurs as in this case the urine will tend to pass anteriorly
because the fascial condensations will prevent lateral and posterior passage of
the urine.

Urogenital triangle

The urogenital triangle is formed by the:

Ischiopubic inferior rami


Ischial tuberosities

A fascial sheet is attached to the sides, forming the inferior fascia of the
urogenital diaphragm.

It transmits the urethra in males and both the urethra and vagina in females. The
membranous urethra lies deep this structure and is surrounded by the external
urethral sphincter.

Superficial to the urogenital diaphragm lies the superficial perineal pouch. In


males this contains:

Bulb of penis
Crura of the penis
Superficial transverse perineal muscle
Posterior scrotal arteries
Posterior scrotal nerves

In females the internal pudendal artery branches to become the posterior labial
arteries in the superficial perineal pouch.

Rate question: Next question

Com m ent on this question

All contents of this site are © 2012 E-Medical Revision Ltd Terms and Conditions Privacy policy
yomna92009@yahoo.com - MRCS Part A - My account

Reference ranges End and review

Question 12 of 299 Question stats Score: 0%

1
A 8%
Which of the following does not pass through the superior orbital fissure? 2
B 12.7%
C 49.5% 3

A. Oculomotor nerve D 18.9% 4

B. Abducens nerve E 10.9% 5

C. Ophthalmic artery 49.5% of users answered this 6


question correctly
D. Ophthalmic division of the trigeminal nerve 7

E. Ophthalmic veins 8

9
Next question
10

11
The ophthalmic artery, a branch of the internal carotid enters the orbit with the
optic nerve in the canal. 12

Foramina of the base of the skull

Foramen Location Contents

Foramen Sphenoid Otic ganglion


ovale bone V3 (Mandibular nerve:3rd branch of
trigeminal)
Accessory meningeal artery
Lesser petrosal nerve
Emissary veins

Foramen Sphenoid Middle meningeal artery


spinosum bone Meningeal branch of the Mandibular nerve

Foramen Sphenoid Maxillary nerve (V2)


rotundum bone

Foramen Sphenoid Base of the medial pterygoid plate.


lacerum bone Internal carotid artery
Nerve and artery of the pterygoid canal

Jugular Temporal Anterior: inferior petrosal sinus


foramen bone Intermediate: glossopharyngeal, vagus, and accessory nerves.
Posterior: sigmoid sinus (becoming the internal jugular vein) and
some meningeal branches from the occipital and ascending
pharyngeal arteries.

Foramen Occipital Anterior and posterior spinal arteries


magnum bone Vertebral arteries
Medulla oblongata

Stylomastoid Temporal Stylomastoid artery


foramen bone Facial nerve

Superior Sphenoid Oculomotor nerve (III)


orbital bone trochlear nerve (IV)
fissure lacrimal, frontal and nasociliary branches of ophthalmic nerve
(V1)
abducent nerve (VI)
Superior and inferior ophthalmic vein

Base of skull anatomical overview


yomna92009@yahoo.com - MRCS Part A - My account

Reference ranges End and review

Question 13 of 299 Question stats Score: 7.7%

1
A 14.6%
Which of the following muscles does not attach to the radius? 2
B 21.5%
C 8.3% 3

A. Pronator quadratus D 12% 4

B. Biceps E 43.6% 5

C. Brachioradialis 43.6% of users answered this 6


question correctly
D. Supinator 7

E. Brachialis 8

9
Next question
10

11
The brachialis muscle inserts into the ulna. The other muscles are all inserted
onto the radius. 12

13
Radius

Bone of the forearm extending from the lateral side of the elbow to the
thumb side of the wrist

Upper end

Articular cartilage- covers medial > lateral side


Articulates with radial notch of the ulna by the annular ligament
Muscle attachment- biceps brachii at the tuberosity

Shaft

Muscle attachment-

Upper third of the body Supinator, Flexor digitorum superficialis, Flexor pollicis
longus
Middle third of the body Pronator teres
Lower quarter of the body Pronator quadratus , tendon of supinator longus

Lower end

Quadrilateral
Anterior surface- capsule of wrist joint
Medial surface- head of ulna
Lateral surface- ends in the styloid process
Posterior surface: 3 grooves containing:

1. Tendons of extensor carpi radialis longus and brevis


2. Tendon of extensor pollicis longus
3. Tendon of extensor indicis
Image sourced from Wikipedia

Rate question: Next question

Com m ent on this question

All contents of this site are © 2012 E-Medical Revision Ltd Terms and Conditions Privacy policy
yomna92009@yahoo.com - MRCS Part A - My account

Reference ranges End and review

Question 14 of 299 Question stats Score: 7.1%

1
A 5%
Which nerve supplies the 1st web space of the foot? 2
B 16.3%
C 56.1% 3

A. Popliteal nerve D 12.9% 4

B. Superficial peroneal nerve E 9.8% 5

C. Deep peroneal nerve 56.1% of users answered this 6


question correctly
D. Tibial nerve 7

E. Saphenous nerve 8

9
Next question
10

11
The first web space is innervated by the deep peroneal nerve. See diagram
below: 12

13

14
Image sourced from Wikipedia

Deep peroneal nerve

Origin From the common peroneal nerve, at the lateral aspect of the fibula, deep
to peroneus longus

Nerve root L4, L5, S1, S2


values

Course and Pierces the anterior intermuscular septum to enter the anterior
relation compartment of the lower leg
Passes anteriorly down to the ankle joint, midway between the two
malleoli

Terminates In the dorsum of the foot

Muscles Tibialis anterior

innervated Extensor hallucis longus


Extensor digitorum longus
Peroneus tertius
Extensor digitorum brevis

Cutaneous Web space of the first and second toes


innervation

Actions Dorsiflexion of ankle joint


Extension of all toes (extensor hallucis longus and extensor
digitorum longus)
Eversion of the foot

After its bifurcation past the ankle joint, the lateral branch of the deep peroneal
nerve innervates the extensor digitorum brevis and the extensor hallucis brevis
The medial branch supplies the web space between the first and second digits.

Rate question: Next question

Com m ent on this question

All contents of this site are © 2012 E-Medical Revision Ltd Terms and Conditions Privacy policy
yomna92009@yahoo.com - MRCS Part A - My account

Reference ranges End and review

Question 15 of 299 Question stats Score: 6.7%

1
A 13.8%
Which of the following statements relating to the gastroduodenal artery is untrue? 2
B 16.4%
C 14.1% 3

A. It runs posterior to the 1st part of the duodenum D 44.4% 4

B. It originates from the common hepatic artery E 11.4% 5

C. The bile duct is a close relation 44.4% of users answered this 6


question correctly
D. The portal vein is closely related anteriorly 7

E. It terminates as the gastro-epiploic and superior 8


pancreaticoduodenal artery
9

10
Next question
11

The portal vein is located posteriorly and then separated from the artery by the 12
pancreas. The anatomy of this artery is important as it is a site of bleeding in
13
posteriorly sited duodenal ulcers. At laparotomy for bleeding from this vessel, the
14
relation of the bile duct should be remembered less it be caught inadvertently in a
stitch. 15

Gastroduodenal artery

Supplies
Pylorus, proximal part of the duodenum, and indirectly to the pancreatic head (via
the anterior and posterior superior pancreaticoduodenal arteries)

Path
Most commonly arises from the common hepatic artery of the coeliac trunk
Terminates by bifurcating into the right gastroepiploic artery and the superior
pancreaticoduodenal artery

Image showing stomach reflected superiorly to illustrate the relationship of the


gastroduodenal artery to the first part of the duodenum

Image sourced from Wikipedia


yomna92009@yahoo.com - MRCS Part A - My account

Reference ranges End and review

Question 16 of 299 Question stats Score: 6.3%

1
A 8.4%
Which of the following is not an intrinsic muscle of the hand? 2
B 66.5%
C 8.3% 3

A. Opponens pollicis D 8.8% 4

B. Palmaris longus E 7.9% 5

C. Flexor pollicis brevis 66.5% of users answered this 6


question correctly
D. Flexor digiti minimi brevis 7

E. Opponens digiti minimi 8

9
Next question
10

11

Mnemonic for intrinsic hand muscles 12


'A OF A OF A'
13

A bductor pollicis brevis 14

O pponens pollicis 15
F lexor pollicis brevis
16
A dductor pollicis (thenar muscles)
O pponens digiti minimi
F lexor digiti minimi brevis
A bductor digiti minimi (hypothenar muscles)

Palmaris longus originates in the forearm.

Hand

Anatomy of the hand

Bones 8 Carpal bones


5 Metacarpals
14 phalanges

Intrinsic 7 Interossei - Supplied by ulnar nerve


Muscles
3 palmar-adduct fingers
4 dorsal- abduct fingers

Intrinsic Lumbricals
muscles
Flex MCPJ and extend the IPJ.
Origin deep flexor tendon and insertion dorsal extensor hood
mechanism.
Innervation: 1st and 2nd- median nerve, 3rd and 4th- deep
branch of the ulnar nerve.

Thenar Abductor pollicis brevis


eminence Opponens pollicis
Flexor pollicis brevis

Hypothenar Opponens digiti minimi


eminence Flexor digiti minimi brevis
Abductor digiti minimi
yomna92009@yahoo.com - MRCS Part A - My account

Reference ranges End and review

Question 17 of 299 Question stats Score: 5.9%

1
A 13.6%
A 23 year old man has a cannula inserted into his cephalic vein. Through which of 2
B 8.8%
the following structures does the cephalic vein pass? 3
C 7.6%
D 53.2% 4
A. Interosseous membrane E 16.8% 5
B. Triceps 6
53.2% of users answered this
C. Pectoralis major question correctly
7
D. Clavipectoral fascia
8
E. Tendon of biceps
9

10
Next question
11

12
The cephalic vein is a favored vessel for arteriovenous fistula formation and
13
should be preserved in patients with end stage renal failure
14

15
The cephalic vein penetrates the calvipectoral fascia (but not the pectoralis
major) prior to terminating in the axillary vein. 16

17
Cephalic vein

Path

Dorsal venous arch drains laterally into the cephalic vein


Crosses the anatomical snuffbox and travels laterally up the arm
At the antecubital fossa connected to the basilic vein by the median cubital
vein
Pierces deep fascia of deltopectoral groove to join axillary vein

Rate question: Next question

Com m ent on this question

All contents of this site are © 2012 E-Medical Revision Ltd Terms and Conditions Privacy policy
yomna92009@yahoo.com - MRCS Part A - My account

Reference ranges End and review

Question 18 of 299 Question stats Score: 5.6%

1
A 14.2%
A man with lung cancer and bone metastasis in the thoracic spinal vertebral 2
B 15.2%
bodies, sustains a pathological fracture at the level of T4. The fracture is unstable 3
C 35.1%
and the spinal cord is severely compressed at this level. Which of the following will
D 14.9% 4
not be present?
E 20.6% 5

35.1% of users answered this 6


A. Extensor plantar reflexes
question correctly
7
B. Spasticity of the lower limbs
8
C. Diminished patellar tendon reflex
D. Urinary incontinence 9

E. Sensory ataxia 10

11

Next question 12

13
A thoracic cord lesion causes spastic paraperesis, hyperrflexia and extensor
14
plantar responses (UMN lesion), incontinence, sensory loss below the lesion and
'sensory' ataxia. 15

16
Spinal cord
17

18
Located in a canal within the vertebral column that affords it structural
support.
Rostrally is continues to the medulla oblongata of the brain and caudally it
tapers at a level corresponding to the L1-2 interspace (in the adult), a
central structure, the filum terminale anchors the cord to the first coccygeal
vertebra.
The spinal cord is characterised by cervico-lumbar enlargements and
these, broadly speaking, are the sites which correspond to the brachial and
lumbar plexuses respectively.

There are some key points to note when considering the surgical anatomy of the
spinal cord:

* During foetal growth the spinal cord becomes shorter than the spinal canal,
hence the adult site of cord termination at the L1-2 level.

* Due to growth of the vertebral column the spine segmental levels may not
always correspond to bony landmarks as they do in the cervical spine.

* The spinal cord is incompletely divided into two symmetrical halves by a dorsal
median sulcus and ventral median fissure. Grey matter surrounds a central
canal that is continuous rostrally with the ventricular system of the CNS.

* The grey matter is sub divided cytoarchitecturally into Rexeds laminae.

* Afferent fibres entering through the dorsal roots usually terminate near their
point of entry but may travel for varying distances in Lissauers tract. In this way
they may establish synaptic connections over several levels

* At the tip of the dorsal horn are afferents associated with nociceptive stimuli.
The ventral horn contains neurones that innervate skeletal muscle.

The key point to remember when revising CNS anatomy is to keep a clinical
perspective in mind. So it is worth classifying the ways in which the spinal cord
may become injured. These include:

Trauma either direct or as a result of disc protrusion


Neoplasia either by direct invasion (rare) or as a result of pathological
vertebral fracture
Inflammatory diseases such as Rheumatoid disease, or OA (formation of
osteophytes compressing nerve roots etc.
Vascular either as a result of stroke (rare in cord) or as complication of
aortic dissection
Infection historically diseases such as TB, epidural abscesses.

The anatomy of the cord will, to an extent dictate the clinical presentation. Some
points/ conditions to remember:

Brown- Sequard syndrome-Hemisection of the cord producing ipsilateral


loss of proprioception and upper motor neurone signs, plus contralateral
loss of pain and temperature sensation. The explanation of this is that the
fibres decussate at different levels.
Lesions below L1 will tend to present with lower motor neurone signs

Rate question: Next question

Com m ent on this question

All contents of this site are © 2012 E-Medical Revision Ltd Terms and Conditions Privacy policy
yomna92009@yahoo.com - MRCS Part A - My account

Reference ranges End and review

Question 19 of 299 Question stats Score: 10.5%

1
A 26.3%
Through which of the following foramina does the genital branch of the 2
B 8%
genitofemoral nerve exit the abdominal cavity? 3
C 17.9%
D 15.1% 4
A. Superficial inguinal ring E 32.7% 5
B. Sciatic notch 6
32.7% of users answered this
C. Obturator foramen question correctly
7
D. Femoral canal
8
E. Deep inguinal ring
9

10
Next question
11

The genitofemoral nerve divides into two branches as it approaches the inguinal 12
ligament. The genital branch passes anterior to the external iliac artery through 13
the deep inguinal ring into the inguinal canal. It communicates with the ilioinguinal
14
nerve in the inguinal canal (though this is seldom of clinical significance).
15

Genitofemoral nerve 16

17
Supplies
18
- Small area of the upper medial thigh
19
Path
- Arises from the first and second lumbar nerves
- Passes obliquely through Psoas major, and emerges from its medial border
opposite the fibrocartilage between the third and fourth lumbar vertebrae.
- It then descends on the surface of Psoas major, under cover of the peritoneum
- Divides into genital and femoral branches.
- The genital branch passes through the inguinal canal, within the spermatic cord,
to supply the skin overlying the skin and fascia of the scrotum. The femoral
branch enters the thigh posterior to the inguinal ligament, lateral to the femoral
artery. It supplies an area of skin and fascia over the femoral triangle.

It may be injured during abdominal or pelvic surgery, or during inguinal


hernia repairs.

Rate question: Next question

Com m ent on this question

All contents of this site are © 2012 E-Medical Revision Ltd Terms and Conditions Privacy policy
yomna92009@yahoo.com - MRCS Part A - My account

Reference ranges End and review

Question 20 of 299 Question stats Score: 10%

1
A 49%
A 28 year old man lacerates the posterolateral aspect of his wrist with a knife in 2
B 19.7%
an attempted suicide. On arrival in the emergency department the wound is 3
C 11.6%
inspected and found to be located over the lateral aspect of the extensor
D 12.5% 4
retinaculum (which is intact). Which of the following structures is at greatest risk of
E 7.3% 5
injury?

49% of users answered this 6


question correctly
A. Superficial branch of the radial nerve 7

B. Radial artery 8

C. Dorsal branch of the ulnar nerve 9

D. Tendon of extensor carpi radialis brevis 10

E. Tendon of extensor digiti minimi 11

12
Next question
13

14
The superficial branch of the radial nerve passes superior to the extensor
retinaculum in the position of this laceration and is at greatest risk of injury. The 15
dorsal branch of the ulnar nerve and artery also pass superior to the extensor 16
retinaculum n but are located medially.
17

Extensor retinaculum 18

19
The extensor rentinaculum is a thickening of the deep fascia that stretches across 20
the back of the wrist and holds the long extensor tendons in position.
Its attachments are:

The pisiform and hook of hamate medially


The end of the radius laterally

Structures related to the extensor retinaculum

Structures superficial to the Basilic vein


retinaculum Dorsal cutaneous branch of the ulnar
nerve
Cephalic vein
Superficial branch of the radial nerve

Structures passing deep to the Extensor carpi ulnaris tendon


extensor retinaculum Extensor digiti minimi tendon
Extensor digitorum and extensor indicis
tendon
Extensor pollicis longus tendon
Extensor carpi radialis longus tendon
Abductor pollicis longus and extensor
pollicis brevis tendons

Beneath the extensor retinaculum fibrous septa form six compartments that
contain the extensor muscle tendons. Each compartment has its own synovial
sheath.

The radial artery


The radial artery passes between the lateral collateral ligament of the wrist joint
and the tendons of the abductor pollicis longus and extensor pollicis brevis.

Image illustrating the topography of tendons passing under the extensor


retinaculum
yomna92009@yahoo.com - MRCS Part A - My account

Reference ranges End and review

Question 1 of 279 Question stats Score: 100%

1
A 15.2%
A 43 year old man is reviewed in the clinic following a cardiac operation. A chest B 27.5%
x-ray is performed and a circular radio-opaque structure is noted medial to the
C 6.9%
4th interspace on the left. Which of the following procedures is the patient most
D 7.4%
likely to have undergone?
E 43%

A. Aortic valve replacement with metallic valve 43% of users answered this
question correctly
B. Tricuspid valve replacement with metallic valve
C. Tricuspid valve replacement with porcine valve
D. Pulmonary valve replacement with porcine valve
E. Mitral valve replacement with metallic valve

Next question

Theme from April 2012 Exam

Prosthetic heart valves on Chest X-rays

The aortic and mitral valves are most commonly replaced and when a metallic
valve is used, can be most readily identified on plain x-rays.
The presence of cardiac disease (such as cardiomegaly) may affect the figures
quoted here.

Aortic
Usually located medial to the 3rd interspace on the right.

Mitral
Usually located medial to the 4th interspace on the left.

Tricuspid
Usually located medial to the 5th interspace on the right.

Please note that these are the sites at which an artificial valve may be located
and are NOT the sites of auscultation.

Rate question: Next question

Com m ent on this question

All contents of this site are © 2012 E-Medical Revision Ltd Terms and Conditions Privacy policy
yomna92009@yahoo.com - MRCS Part A - My account

Reference ranges End and review

Question 2 of 279 Question stats Score: 50%

1
A 28.9%
A 63 year old lady is diagnosed as having an endometrial carcinoma arising from 2
B 44.9%
the uterine body. To which nodal region will the tumour initially metastasise?
C 12.1%
D 7.2%

A. Para aortic nodes E 7%

B. Iliac lymph nodes


44.9% of users answered this
C. Inguinal nodes question correctly

D. Pres sacral nodes


E. Mesorectal lymph nodes

Next question

Theme from 2011 exam

Tumours of the uterine body will tend to spread to the iliac nodes initially. When
the tumour is expanding to cross different nodal margins this is of considerable
clinical significance if nodal clearance is performed during a Wertheims type
hysterectomy.

Lymphatic drainage of the uterus and cervix

The uterine fundus has a lymphatic drainage that runs with the ovarian
vessels and may thus drain to the para-aortic nodes. Some drainage may
also pass along the round ligament to the inguinal nodes.
The body of the uterus drains through lymphatics contained within the
broad ligament to the iliac lymph nodes.
The cervix drains into three potential nodal stations; laterally through the
broad ligament to the external iliac nodes, along the lymphatics of the
uterosacral fold to the presacral nodes and posterolaterally along
lymphatics lying alongside the uterine vessels to the internal iliac nodes.

Rate question: Next question

Com m ent on this question

All contents of this site are © 2012 E-Medical Revision Ltd Terms and Conditions Privacy policy
yomna92009@yahoo.com - MRCS Part A - My account

Reference ranges End and review

Question 3 of 279 Question stats Score: 33.3%

1
A 12%
Transection of the radial nerve at the level of the axilla will result in all of the 2
B 31%
following except: 3
C 12.4%
D 18.5%

A. Loss of elbow extension. E 26.1%

B. Loss of extension of the interphalangeal joints.


31% of users answered this
C. Loss of metacarpophalangeal extension. question correctly

D. Loss of triceps reflex.


E. Loss of sensation overlying the first dorsal interosseous.

Next question

These may still extend by virtue of retained lumbrical muscle function.

Radial nerve

Continuation of posterior cord of the brachial plexus (root values C5 to T1)

Path

In the axilla: lies posterior to the axillary artery on subscapularis, latissimus


dorsi and teres major.
Enters the arm between the brachial artery and the long head of triceps
(medial to humerus).
Spirals around the posterior surface of the humerus in the groove for the
radial nerve.
At the distal third of the lateral border of the humerus it then pierces the
intermuscular septum and descends in front of the lateral epicondyle.
At the lateral epicondyle it lies deeply between brachialis and
brachioradialis where it then divides into a superficial and deep terminal
branch.
Deep branch crosses the supinator to become the posterior interosseous
nerve.

In the image below the relationships of the radial nerve can be appreciated

Image sourced from Wikipedia

Regions innervated

Motor (main Triceps


nerve) Anconeus
Brachioradialis
Extensor carpi radialis
yomna92009@yahoo.com - MRCS Part A - My account

Reference ranges End and review

Question 4 of 279 Question stats Score: 25%

1
A 16.7%
Which of the following structures is not located in the superficial perineal space in 2
B 35.6%
females? 3
C 9.1%
D 20.3% 4
A. Posterior labial arteries E 18.2%

B. Pudendal nerve
35.6% of users answered this
C. Superficial transverse perineal muscle question correctly

D. Greater vestibular glands


E. None of the above

Next question

The pudendal nerve is located in the deep perineal space and then branches to
innervate more superficial structures.

Urogenital triangle

The urogenital triangle is formed by the:

Ischiopubic inferior rami


Ischial tuberosities

A fascial sheet is attached to the sides, forming the inferior fascia of the
urogenital diaphragm.

It transmits the urethra in males and both the urethra and vagina in females. The
membranous urethra lies deep this structure and is surrounded by the external
urethral sphincter.

Superficial to the urogenital diaphragm lies the superficial perineal pouch. In


males this contains:

Bulb of penis
Crura of the penis
Superficial transverse perineal muscle
Posterior scrotal arteries
Posterior scrotal nerves

In females the internal pudendal artery branches to become the posterior labial
arteries in the superficial perineal pouch.

Rate question: Next question

Com m ent on this question

All contents of this site are © 2012 E-Medical Revision Ltd Terms and Conditions Privacy policy
yomna92009@yahoo.com - MRCS Part A - My account

Reference ranges End and review

Question 5 of 279 Question stats Score: 20%

1
A 53.1%
Which of the following is not a branch of the hepatic artery? 2
B 7.7%
C 13.1% 3

A. Pancreatic artery D 10.9% 4

B. Cystic artery E 15.3% 5

C. Right gastric artery 53.1% of users answered this


question correctly
D. Superior Pancreaticoduodenal artery
E. Right Gastroepiploic artery

Next question

The pancreatic artery is a branch of the splenic artery.

Coeliac axis

The coeliac axis has three main branches.

Left gastric
Hepatic: branches-Right Gastric, Gastroduodenal, Right Gastroepiploic,
Superior Pancreaticoduodenal, Cystic.
Splenic: branches- Pancreatic, Short Gastric, Left Gastroepiploic

It occasionally gives off one of the inferior phrenic arteries.

Image sourced from Wikipedia

Relations
Anteriorly Lesser omentum

Right Right coeliac ganglion and caudate process of liver

Left Left coeliac ganglion and gastric cardia

Inferiorly Upper border of pancreas and renal vein


yomna92009@yahoo.com - MRCS Part A - My account

Reference ranges End and review

Question 6 of 279 Question stats Score: 16.7%

1
A 12.1%
Which of the following structures does not pass behind the piriformis muscle in 2
B 17.3%
the greater sciatic foramen? 3
C 13.4%
D 46.6% 4
A. Sciatic nerve E 10.6% 5
B. Posterior cutaneous nerve of the thigh 6
46.6% of users answered this
C. Inferior gluteal artery question correctly

D. Obturator nerve
E. None of the above

Next question

The obturator nerve does not pass through the greater sciatic foramen.

Greater sciatic foramen

Contents

Nerves Sciatic Nerve


Superior and Inferior Gluteal Nerves
Internal Pudendal Nerve
Posterior Femoral Cutaneous Nerve
Nerve to Quadratus Femoris
Nerve to Obturator internus

Vessels Superior Gluteal Artery and vein


Inferior Gluteal Artery and vein
Internal Pudendal Artery and vein

Piriformis
The piriformis is a landmark for identifying structures passing out of the sciatic
notch

Above piriformis: Superior gluteal vessels


Below piriformis: Inferior gluteal vessels, sciatic nerve (10% pass through it,
<1% above it), posterior cutaneous nerve of the thigh

Greater sciatic foramen boundaries


Anterolaterally Greater sciatic notch of the ilium

Posteromedially Sacrotuberous ligament

Inferior Sacrospinous ligament and the ischial spine

Superior Anterior sacroiliac ligament

The greater sciatic foramen


Image sourced from Wikipedia

Structures passing between both foramina (Medial to lateral)

Pudendal nerve
Internal pudendal artery
Nerve to obturator internus

Contents of the lesser sciatic foramen

Tendon of the obturator internus


Pudendal nerve
Internal pudendal artery and vein
Nerve to the obturator internus

Rate question: Next question

Com m ent on this question

All contents of this site are © 2012 E-Medical Revision Ltd Terms and Conditions Privacy policy
yomna92009@yahoo.com - MRCS Part A - My account

Reference ranges End and review

Question 7 of 279 Question stats Score: 14.3%

1
A 9.9%
A 56 year old man is undergoing a nephrectomy. The surgeons divide the renal 2
B 59.7%
artery. At what level do these usually branch off the abdominal aorta? 3
C 13%
D 8.9% 4
A. T9 E 8.5% 5
B. L2 6
59.7% of users answered this
C. L3 question correctly
7
D. T10
E. L4

Next question

The renal arteries usually branch off the aorta on a level with L2.

Renal arteries

The right renal artery is longer than the left renal artery
The renal vein/artery/pelvis enter the kidney at the hilum

Relations

Right:

Anterior- IVC, right renal vein, the head of the pancreas, and the descending part
of the duodenum.

Left:

Anterior- left renal vein, the tail of the pancreas.

Branches

The renal arteries are direct branches off the aorta (upper border of L2)
In 30% there may be accessory arteries (mainly left side). Instead of
entering the kidney at the hilum, they usually pierce the upper or lower part
of the organ.
Before reaching the hilum of the kidney, each artery divides into four or five
segmental branches (renal vein anterior and ureter posterior); which then
divide within the sinus into lobar arteries supplying each pyramid and
cortex.
Each vessel gives off some small inferior suprarenal branches to the
suprarenal gland, the ureter, and the surrounding cellular tissue and
muscles.

Rate question: Next question

Com m ent on this question

All contents of this site are © 2012 E-Medical Revision Ltd Terms and Conditions Privacy policy
yomna92009@yahoo.com - MRCS Part A - My account

Reference ranges End and review

Question 8 of 279 Question stats Score: 12.5%

1
A 13.3%
A 23 year old man is shot in the chest during a robbery. The left lung is lacerated 2
B 9.6%
and is bleeding. An emergency thoracotomy is performed. The surgeons place a 3
C 11.1%
clamp over the hilum of the left lung. Which of the following structures lies most
D 45.7% 4
anteriorly at this level?
E 20.3% 5

45.7% of users answered this 6


A. Vagus nerve
question correctly
7
B. Oesophagus
8
C. Descending aorta
D. Phrenic nerve
E. Azygos vein

Next question

The phrenic nerve lies anteriorly at this point. The vagus passes anteriorly and
then arches backwards immediately superior to the root of the left bronchus,
giving off the recurrent laryngeal nerve as it does so.

Lung anatomy

The right lung is composed of 3 lobes divided by the oblique and transverse
fissures. The left lung has two lobes divided by the lingula.The apex of both lungs
is approximately 4cm superior to the sterno-costal joint of the first rib. Immediately
below this is a sulcus created by the subclavian artery.

Peripheral contact points of the lung

Base: diaphragm
Costal surface: corresponds to the cavity of the chest
Mediastinal surface: Contacts the mediastinal pleura. Has the cardiac
impression. Above and behind this concavity is a triangular depression
named the hilum, where the structures which form the root of the lung enter
and leave the viscus. These structures are invested by pleura, which,
below the hilus and behind the pericardial impression, forms the pulmonary
ligament

Right lung
Above the hilum is the azygos vein; Superior to this is the groove for the superior
vena cava and right innominate vein; behind this, and nearer the apex, is a furrow
for the innominate artery. Behind the hilus and the attachment of the pulmonary
ligament is a vertical groove for the oesophagus; In front and to the right of the
lower part of the oesophageal groove is a deep concavity for the extrapericardiac
portion of the inferior vena cava.

The root of the right lung lies behind the superior vena cava and the right atrium,
and below the azygos vein.

The right main bronchus is shorter, wider and more vertical than the left main
bronchus and therefore the route taken by most foreign bodies.

Left lung
Above the hilum is the furrow produced by the aortic arch, and then superiorly the
groove accommodating the left subclavian artery; Behind the hilum and
pulmonary ligament is a vertical groove produced by the descending aorta, and in
front of this, near the base of the lung, is the lower part of the oesophagus.

The root of the left lung passes under the aortic arch and in front of the
descending aorta.
Inferior borders of both lungs

6th rib in mid clavicular line


8th rib in mid axillary line
10th rib posteriorly

The pleura runs two ribs lower than the corresponding lung level.

Rate question: Next question

Com m ent on this question

All contents of this site are © 2012 E-Medical Revision Ltd Terms and Conditions Privacy policy
yomna92009@yahoo.com - MRCS Part A - My account

Reference ranges End and review

Question 9 of 279 Question stats Score: 11.1%

1
A 12.3%
A 22 year old man presents with appendicitis. At operation the appendix is 2
B 20.8%
retrocaecal and difficult to access. Division of which of the following anatomical 3
C 5.7%
structures should be undertaken?
D 53.9% 4
E 7.2% 5
A. Ileocolic artery
53.9% of users answered this 6
B. Mesentery of the caecum question correctly
7
C. Gonadal vessels
8
D. Lateral peritoneal attachments of the caecum
9
E. Right colic artery

Next question

The commonest appendiceal location is retrocaecal. Those struggling to find it at


operation should trace the tenia to the caecal pole where the appendix is located.
If it cannot be mobilised easily then division of the lateral caecal peritoneal
attachments (as for a right hemicolectomy) will allow caecal mobilisation and
facilitate the procedure.

Appendix

Location: Base of caecum.


Up to 10cm long.
Mainly lymphoid tissue (Hence mesenteric adenitis may mimic appendicitis).
Caecal taenia coli converge at base of appendix and form a longitudinal
muscle cover over the appendix. This convergence should facilitate its
identification at surgery if it is retrocaecal and difficult to find (which it can
be when people start doing appendicectomies!)
Arterial supply: Appendicular artery (branch of the ileocolic).
It is intra peritoneal.

McBurney's point

1/3 of the way along a line drawn from the Anterior Superior Iliac Spine to
the Umbilicus

6 Positions:

Retrocaecal 74%
Pelvic 21%
Postileal
Subcaecal
Paracaecal
Preileal

Rate question: Next question

Com m ent on this question

All contents of this site are © 2012 E-Medical Revision Ltd Terms and Conditions Privacy policy
yomna92009@yahoo.com - MRCS Part A - My account

Reference ranges End and review

Question 10 of 279 Question stats Score: 10%

1
A 6.5%
Which of the following muscles does not adduct the shoulder? 2
B 7.5%
C 15.3% 3

A. Teres major D 59.7% 4

B. Pectoralis major E 11% 5

C. Coracobrachialis 59.7% of users answered this 6


question correctly
D. Supraspinatus 7

E. Latissimus dorsi 8

9
Next question
10

Supraspinatus is an abductor of the shoulder.

Shoulder joint

Shallow synovial ball and socket type of joint.


It is an inherently unstable joint, but is capable to a wide range of
movement.
Stability is provided by muscles of the rotator cuff that pass from the
scapula to insert in the greater tuberosity (all except sub scapularis-lesser
tuberosity).

Glenoid labrum

Fibrocartilaginous rim attached to the free edge of the glenoid cavity


Tendon of the long head of biceps arises from within the joint from the
supraglenoid tubercle, and is fused at this point to the labrum.
The long head of triceps attaches to the infraglenoid tubercle

Fibrous capsule

Attaches to the scapula external to the glenoid labrum and to the labrum
itself (postero-superiorly)
Attaches to the humerus at the level of the anatomical neck superiorly and
the surgical neck inferiorly
Anteriorly the capsule is in contact with the tendon of subscapularis,
superiorly with the supraspinatus tendon, and posteriorly with the tendons
of infraspinatus and teres minor. All these blend with the capsule towards
their insertion.
Two defects in the fibrous capsule; superiorly for the tendon of biceps.
Anteriorly there is a defect beneath the subscapularis tendon.
The inferior extension of the capsule is closely related to the axillary nerve
at the surgical neck and this nerve is at risk in anteroinferior dislocations. It
also means that proximally sited osteomyelitis may progress to septic
arthritis.

Movements and muscles


Flexion Anterior part of deltoid
Pectoralis major
Biceps
Coracobrachialis

Extension Posterior deltoid


Teres major
Latissimus dorsi

Adduction Pectoralis major


Latissimus dorsi
Teres major
Coracobrachialis
yomna92009@yahoo.com - MRCS Part A - My account

Reference ranges End and review

Question 11 of 279 Question stats Score: 9.1%

1
A 13%
Which of the following muscles is innervated by the cervical branch of the facial 2
B 11.1%
nerve? 3
C 55.2%
D 10.9% 4
A. Masseter E 9.7% 5
B. Sternocleidomastoid 6
55.2% of users answered this
C. Platysma question correctly
7
D. Geniohyoid
8
E. Sternothyroid
9

10
Next question
11

The cervical branch of the facial nerve innervates platysma.

Facial nerve

The facial nerve is the main nerve supplying the structures of the second
embryonic branchial arch. It is predominantly an efferent nerve to the muscles of
facial expression, digastric muscle and also to many glandular structures. It
contains a few afferent fibres which originate in the cells of its genicular ganglion
and are concerned with taste.

Supply - 'face, ear, taste, tear'

Face: muscles of facial expression


Ear: nerve to stapedius
Taste: supplies anterior two-thirds of tongue
Tear: parasympathetic fibres to lacrimal glands, also salivary glands

Path
Subarachnoid path

Origin: motor- pons, sensory- nervus intermedius


Pass through the petrous temporal bone into the internal auditory meatus
with the vestibulocochlear nerve. Here they combine to become the facial
nerve.

Facial canal path

The canal passes superior to the vestibule of the inner ear


At the medial aspect of the middle ear, it becomes wider and contains the
geniculate ganglion.

- 3 branches:
1. greater petrosal nerve
2. nerve to stapedius
3. chorda tympani

Stylomastoid foramen

Passes through the stylomastoid foramen (tympanic cavity anterior and


mastoid antrum posteriorly)
Posterior auricular nerve and branch to Posterior belly of Digastric and
Stylohyoid muscle

Face
Enters parotid gland and divides into 5 branches:

Temporal branch
Zygomatic branch
Buccal branch
Marginal mandibular branch
Cervical branch

Rate question: Next question

Com m ent on this question

All contents of this site are © 2012 E-Medical Revision Ltd Terms and Conditions Privacy policy
yomna92009@yahoo.com - MRCS Part A - My account

Reference ranges End and review

Question 12 of 279 Question stats Score: 8.3%

1
A 11.8%
During a thyroidectomy the surgeons ligate the inferior thyroid artery. From which 2
B 59.7%
vessel does this structure usually originate? 3
C 8.7%
D 13.8% 4
A. External carotid artery E 6% 5
B. Thyrocervical trunk 6
59.7% of users answered this
C. Internal carotid artery question correctly
7
D. Subclavian artery
8
E. Vertebral artery
9

10
Next question
11

The inferior thyroid artery originates from the thyrocervical trunk. This is a branch 12
of the subclavian artery.

Thyroid gland

Right and left lobes connected by isthmus


Surrounded by sheath from pretracheal layer of deep fascia
Apex: Lamina of thyroid cartilage
Base: 4th-5th tracheal ring
Pyramidal lobe: from isthmus
May be attached to foramen caecum at the base of the tongue

Relations

Anteromedially Sternothyroid
Superior belly of omohyoid
Sternohyoid
Anterior aspect of sternocleidomastoid

Posterolaterally Carotid sheath

Medially Larynx
Trachea
Pharynx
Oesophagus
Cricothyroid muscle
External laryngeal nerve (near superior thyroid artery)
Recurrent laryngeal nerve (near inferior thyroid artery)

Posterior Parathyroid glands


Anastomosis of superior and inferior thyroid arteries

Isthmus Anteriorly: Sternothyroids, sternohyoids, anterior jugular veins


Posteriorly: 2nd, 3rd, 4th tracheal rings (attached via Ligament of
Berry)

Blood Supply

Arterial Superior thyroid artery (1st branch of external carotid)


Inferior thyroid artery (from thyrocervical trunk)
Thyroidea ima (in 10% of population -from brachiocephalic artery or aorta)

Venous Superior and middle thyroid veins - into the IJV


Inferior thyroid vein - into the brachiocephalic veins

Rate question: Next question


yomna92009@yahoo.com - MRCS Part A - My account

Reference ranges End and review

Question 13 of 279 Question stats Score: 7.7%

1
A 12.6%
A 56 year old man is left impotent following an abdomino-perineal excision of the 2
B 23.7%
colon and rectum. What is the most likely explanation? 3
C 5.1%
D 49.4% 4
A. Psychosexual issues related to an end colostomy E 9.2% 5
B. Damage to the sacral venous plexus during total mesorectal 6
49.4% of users answered this
excision question correctly
7
C. Damage to the left ureter during sigmoid mobilisation
8
D. Damage to the hypogastric plexus during mobilisation of the inferior
mesenteric artery 9

E. Damage to the internal iliac artery during total mesorectal excision 10

11
Next question
12

13
Autonomic nerve injury is the most common cause.

Nerve lesions during surgery

A variety of different procedures carry the risk of iatrogenic nerve injury. These
are important not only from the patients perspective but also from a medicolegal
standpoint.

The following operations and their associated nerve lesions are listed here:

Posterior triangle lymph node biopsy and accessory nerve lesion.


Lloyd Davies stirrups and common peroneal nerve.
Thyroidectomy and laryngeal nerve.
Anterior resection of rectum and hypogastric autonomic nerves.
Axillary node clearance; long thoracic nerve, thoracodorsal nerve and
intercostobrachial nerve.
Inguinal hernia surgery and ilioinguinal nerve.
Varicose vein surgery- sural and saphenous nerves.
Posterior approach to the hip and sciatic nerve.
Carotid endarterectomy and hypoglossal nerve.

There are many more, with sound anatomical understanding of the commonly
performed procedures the incidence of nerve lesions can be minimised. They
commonly occur when surgeons operate in an unfamiliar tissue plane or by blind
placement of haemostats (not recommended).

Rate question: Next question

Com m ent on this question

All contents of this site are © 2012 E-Medical Revision Ltd Terms and Conditions Privacy policy
yomna92009@yahoo.com - MRCS Part A - My account

Reference ranges End and review

Question 14 of 279 Question stats Score: 7.1%

1
A 20.4%
A 73 year old man is due to undergo a radical prostatectomy for carcinoma of the 2
B 53.7%
prostate gland. To which of the following lymph nodes will the tumour drain 3
C 11%
primarily?
D 8.8% 4
E 6.1% 5
A. Para aortic
53.7% of users answered this 6
B. Internal iliac question correctly
7
C. Superficial inguinal
8
D. Meso rectal
9
E. None of the above
10

Next question 11

12
The prostate lymphatic drainage is primarily to the internal iliac nodes and also 13
the sacral nodes. Although internal iliac is the first site.
14

Prostate gland

The prostate gland is approximately the shape and size of a walnut and is located
inferior to the bladder. It is separated from the rectum by Denonvilliers fascia and
its blood supply is derived from the internal iliac vessels. The internal sphincter
lies at the apex of the gland and may be damaged during prostatic surgery,
affected individuals may complain of retrograde ejaculation.

Summary of prostate gland


Arterial supply Inferior vesical artery (from internal iliac)

Venous Prostatic venous plexus (to paravertebral veins)


drainage

Lymphatic Internal iliac nodes


drainage

Innervation Inferior hypogastric plexus

Dimensions Transverse diameter (4cm)


AP diameter (2cm)
Height (3cm)

Lobes Posterior lobe: posterior to urethra


Median lobe: posterior to urethra, in between ejaculatory ducts
Lateral lobes x 2
Isthmus

Zones Peripheral zone: subcapsular portion of posterior prostate. Most


prostate cancers are here
Central zone
Transition zone
Stroma

Relations
Anterior Pubic symphysis
Prostatic venous plexus

Posterior Denonvilliers fascia


Rectum
Ejaculatory ducts

Lateral Venous plexus (lies on prostate)


Levator ani (immediately below the puboprostatic ligaments)
yomna92009@yahoo.com - MRCS Part A - My account

Reference ranges End and review

Question 15 of 279 Question stats Score: 13.3%

1
A 9%
Which of the following statements relating to the vertebral column is false? 2
B 22.1%
C 23.3% 3

A. There are 7 cervical vertebrae D 16.1% 4

B. The cervical and lumbar lordosis are secondary curves developing E 29.4% 5
after birth due to change in shape of the intervertebral discs
29.4% of users answered this 6
C. The lumbar vertebrae do not have a transverse process foramina question correctly
7
D. The lumbar vertebrae receive blood directly from the aorta
8
E. The spinous process is formed by the junction of the pedicles
9
posteriorly
10

Next question 11

12
The spinous process is formed by 2 laminae posteriorly.
13

14
Vertebral column
15

There are 7 cervical, 12 thoracic, 5 lumbar, and 5 sacral vertebrae.


The spinal cord segmental levels do not necessarily correspond to the
vertebral segments. For example, while the C1 cord is located at the C1
vertebra, the C8 cord is situated at the C7 vertebra. While the T1 cord is
situated at the T1 vertebra, the T12 cord is situated at the T8 vertebra.
The lumbar cord is situated between T9 and T11 vertebrae. The sacral
cord is situated between the T12 to L2 vertebrae.

Cervical vertebrae
The interface between the first and second vertebra is called the atlanto-axis
junction. The C3 cord contains the phrenic nucleus. The cervical cord innervates
the deltoids (C4), biceps (C4-5), wrist extensors (C6), triceps (C7), wrist extensors
(C8), and hand muscles (C8-T1).

Thoracic vertebrae
The thoracic vertebral segments are defined by those that have a rib. The spinal
roots form the intercostal nerves that run on the bottom side of the ribs and these
nerves control the intercostal muscles and associated dermatomes.

Lumbosacral vertebrae
Form the remainder of the segments below the vertebrae of the thorax. The
lumbosacral spinal cord, however, starts at about T9 and continues only to L2. It
contains most of the segments that innervate the hip and legs, as well as the
buttocks and anal regions.

Cauda Equina
The spinal cord ends at L2 vertebral level. The tip of the spinal cord is called the
conus. Below the conus, there is a spray of spinal roots that is called the cauda
equina. Injuries below L2 represent injuries to spinal roots rather than the spinal
cord proper.

Rate question: Next question

Com m ent on this question

All contents of this site are © 2012 E-Medical Revision Ltd Terms and Conditions Privacy policy
yomna92009@yahoo.com - MRCS Part A - My account

Reference ranges End and review

Question 16 of 279 Question stats Score: 12.5%

1
A 6.9%
The cephalic vein pierces the clavipectoral fascia to terminate in which of the 2
B 71%
following veins? 3
C 7.3%
D 6.2% 4
A. External jugular E 8.6% 5
B. Axillary 6
71% of users answered this
C. Internal jugular question correctly
7
D. Azygos
8
E. Brachial
9

10
Next question
11

12

13
Cephalic vein
14

15
Path
16
Dorsal venous arch drains laterally into the cephalic vein
Crosses the anatomical snuffbox and travels laterally up the arm
At the antecubital fossa connected to the basilic vein by the median cubital
vein
Pierces deep fascia of deltopectoral groove to join axillary vein

Rate question: Next question

Com m ent on this question

All contents of this site are © 2012 E-Medical Revision Ltd Terms and Conditions Privacy policy
yomna92009@yahoo.com - MRCS Part A - My account

Reference ranges End and review

Question 17 of 279 Question stats Score: 17.6%

1
A 9.2%
A 78 year old lady falls over in her nursing home and sustains a displaced 2
B 20.8%
intracapsular fracture of the femoral neck. A decision is made to perform a hemi 3
C 18.6%
arthroplasty through a lateral approach. Which of the following vessels will be
D 9.7% 4
divided to facilitate access?
E 41.7% 5

41.7% of users answered this 6


A. Saphenous vein
question correctly
7
B. Superior gluteal artery
8
C. Superficial circumflex iliac artery
D. Profunda femoris artery 9

E. Transverse branch of the lateral circumflex artery 10

11

Next question 12

13
During the Hardinge style lateral approach the transverse branch of the lateral
14
circumflex artery is divided to gain access. The vessels and its branches are
illustrated below: 15

16

17
Image sourced from Wikipedia

Hip joint

Head of femur articulates with acetabulum of the pelvis


Both covered by articular hyaline cartilage
The acetabulum forms at the union of the ilium, pubis, and ischium
The triradiate cartilage (Y-shaped growth plate) separates the pelvic bones
The acetabulum holds the femoral head by the acetabular labrum
Normal angle between femoral head and femoral shaft is 130o

Ligaments

Transverse ligament: joints anterior and posterior ends of the articular


cartilage
Head of femur ligament (ligamentum teres): acetabular notch to the fovea.
Contains arterial supply to head of femur in children.

Image sourced from Wikipedia

Image sourced from Wikipedia

Extracapsular ligaments

Iliofemoral ligament: inverted Y shape. Anterior iliac spine to the


trochanteric line
Pubofemoral ligament: acetabulum to lesser trochanter
Ischiofemoral ligament: posterior support. Ischium to greater trochanter.
Blood supply
Medial circumflex femoral and lateral circumflex femoral arteries (Branches of
profunda femoris)

2 anastomoses: Cruciate and the trochanteric anastomoses (provides most of the


blood to the head of the femur) Hence the need for hemiarthroplasty when there
is a displaced femoral head fracture. These anastomoses exist between the
femoral artery or profunda femoris and the gluteal vessels.

Rate question: Next question

Com m ent on this question

All contents of this site are © 2012 E-Medical Revision Ltd Terms and Conditions Privacy policy
yomna92009@yahoo.com - MRCS Part A - My account

Reference ranges End and review

Question 18 of 279 Question stats Score: 16.7%

1
A 13%
A 73 year old man undergoes a sub total oesophagectomy with anastomosis of 2
B 7.7%
the stomach to the cervical oesophagus. Which of the following vessels will be 3
C 24%
primarily responsible for the arterial supply to the oesophageal portion of the
D 44.9% 4
anastomosis?
E 10.5% 5

44.9% of users answered this 6


A. Superior thyroid artery
question correctly
7
B. Internal carotid artery
8
C. Direct branches from the thoracic aorta
D. Inferior thyroid artery 9

E. Subclavian artery 10

11

Next question 12

13
The cervical oesophagus is supplied by the inferior thyroid artery. The thoracic
14
oesophagus (removed in this case) is supplied by direct branches from the
thoracic aorta. 15

16
Oesophagus
17

18
25cm long
Starts at C6 vertebra, pierces diaphragm at T10 and ends at T11
Squamous epithelium

Constrictions of the oesophagus


Structure Distance from incisors

Cricoid cartilage 15cm

Arch of the Aorta 22.5cm

Left principal bronchus 27cm

Diaphragmatic hiatus 40cm

Relations

Anteriorly Trachea to T4
Recurrent laryngeal nerve
Left bronchus, Left atrium
Diaphragm

Posteriorly Thoracic duct to left at T5


Hemiazygos to right T8
Descending aorta
First 2 intercostal branches of aorta

Left Thoracic duct


Left subclavian artery

Right Azygos vein

Arterial, venous and lymphatic drainage of the oesophagus


Artery Vein Lymphatics Muscularis externa

Upper Inferior Inferior thyroid Deep Striated muscle


third thyroid cervical

Mid third Aortic Azygos branches Mediastinal Smooth & striated


branches muscle

Lower Left gastric Posterior mediastinal and Gastric Smooth muscle


third coeliac
Nerve supply

Upper half is supplied by recurrent laryngeal nerve


Lower half by oesophageal plexus (vagus)

Histology

Mucosa :Nonkeratinized stratified squamous epithelium


Submucosa: glandular tissue
Muscularis externa (muscularis): composition varies. See table
Adventitia

Rate question: Next question

Com m ent on this question

All contents of this site are © 2012 E-Medical Revision Ltd Terms and Conditions Privacy policy
yomna92009@yahoo.com - MRCS Part A - My account

Reference ranges End and review

Question 19 of 279 Question stats Score: 15.8%

1
A 20.7%
Which of the following structures is not closely related to the brachial artery? 2
B 8.4%
C 32.9% 3

A. Ulnar nerve D 27.8% 4

B. Median nerve E 10.2% 5

C. Cephalic vein 32.9% of users answered this 6


question correctly
D. Long head of triceps 7

E. Median cubital vein 8

9
Next question
10

11
The cephalic vein lies superficially and on the contralateral side of the arm to the
brachial artery. The relation of the ulnar nerves and others are demonstrated in 12
the image below: 13

14

15

16

17

18

19

Image sourced from Wikipedia

Brachial artery

The brachial artery begins at the lower border of teres major as a continuation of
the axillary artery. It terminates in the cubital fossa at the level of the neck of the
radius by dividing into the radial and ulnar arteries.

Relations
Posterior relations include the long head of triceps with the radial nerve and
profunda vessels intervening. Anteriorly it is overlapped by the medial border of
biceps.
It is crossed by the median nerve in the middle of the arm.
In the cubital fossa it is separated from the median cubital vein by the bicipital
aponeurosis.
The basilic vein is in contact at the most proximal aspect of the cubital fossa and
lies medially.

Rate question: Next question

Com m ent on this question

All contents of this site are © 2012 E-Medical Revision Ltd Terms and Conditions Privacy policy
yomna92009@yahoo.com - MRCS Part A - My account

Reference ranges End and review

Question 20 of 279 Question stats Score: 15%

1
A 9.4%
The following statements relating to the musculocutaneous nerve are true 2
B 8.7%
except? 3
C 61.3%
D 11.2% 4
A. It arises from the lateral cord of the brachial plexus E 9.3% 5
B. It provides cutaneous innervation to the lateral side of the forearm 6
61.3% of users answered this
C. If damaged then extension of the elbow joint will be impaired question correctly
7
D. It supplies the biceps muscle
8
E. It runs beneath biceps
9

10
Next question
11

It supplies biceps, brachialis and coracobrachialis so if damaged then elbow 12


flexion will be impaired. 13

14
Musculocutaneous nerve
15

16
Branch of lateral cord of brachial plexus
17

18
Path
19

20
It penetrates the Coracobrachialis muscle
Passes obliquely between the Biceps brachii and the Brachialis to the
lateral side of the arm
Above the elbow it pierces the deep fascia lateral to the tendon of the
Biceps brachii
Continues into the forearm as the lateral cutaneous nerve of the forearm

Innervates

Coracobrachialis
Biceps brachii
Brachialis

Rate question: Next question

Com m ent on this question

All contents of this site are © 2012 E-Medical Revision Ltd Terms and Conditions Privacy policy
yomna92009@yahoo.com - MRCS Part A - My account

Reference ranges End and review

Question 1 of 259 Question stats Score: 0%

1
A 62.6%
Which ligament keeps the head of the radius connected to the radial notch of the B 10.5%
ulna?
C 9.8%
D 7.5%

A. Annular (orbicular) ligament E 9.5%

B. Quadrate ligament
62.6% of users answered this
C. Radial collateral ligament of the elbow question correctly

D. Ulnar collateral ligament


E. Radial collateral ligament

Next question

The annular ligament connects the radial head to the radial notch of the ulna.
This is illustrated below:

Image sourced from Wikipedia

Radius

Bone of the forearm extending from the lateral side of the elbow to the
thumb side of the wrist

Upper end

Articular cartilage- covers medial > lateral side


Articulates with radial notch of the ulna by the annular ligament
Muscle attachment- biceps brachii at the tuberosity

Shaft

Muscle attachment-

Upper third of the body Supinator, Flexor digitorum superficialis, Flexor pollicis
longus
Middle third of the body Pronator teres
Lower quarter of the body Pronator quadratus , tendon of supinator longus
yomna92009@yahoo.com - MRCS Part A - My account

Reference ranges End and review

Question 2 of 259 Question stats Score: 0%

1
A 55.2%
Which of the following statements relating to the external carotid is false? 2
B 8.4%
C 10.4%

A. It ends by bifurcating into the superficial temporal and ascending D 10.4%


pharyngeal artery E 15.5%

B. Its first branch is the superior thyroid artery


55.2% of users answered this
C. The superior thyroid, lingual and facial arteries all arise from its question correctly
anterior surface
D. The ascending pharyngeal artery is a medial branch
E. Initially it lies anteromedial to the internal carotid

Next question

It terminates by dividing into the superficial temporal and maxillary branches. The
external carotid has eight branches, 3 from its anterior surface ; thyroid, lingual
and facial. The pharyngeal artery is a medial branch. The posterior auricular and
occipital are posterior branches.

External carotid artery

The external carotid artery (ECA) is a branch of the common carotid artery.

Path

Upper border of thyroid cartilage to pass in between the angle of the


mandible and mastoid process
Initially anteromedial to the ICA, then becomes lateral
Lower ECA covered by sternomastoid, passed by hypoglossal nerve/lingual
& facial vein
Then deep to digastric and stylohyoid, eventually passes into the parotid
gland where it divides into the superficial temporal and maxillary branches

Image sourced from Wikipedia

Rate question: Next question


yomna92009@yahoo.com - MRCS Part A - My account

Reference ranges End and review

Question 3 of 259 Question stats Score: 0%

1
A 53.5%
A 38 year old man presents to the clinic with shoulder weakness. On examination 2
B 5.8%
he has an inability to initiate shoulder abduction. Which of the nerves listed below 3
C 21.2%
is least likely to be functioning normally?
D 9%

E 10.5%
A. Suprascapular nerve
53.5% of users answered this
B. Medial pectoral nerve question correctly
C. Axillary nerve
D. Median nerve
E. Radial nerve

Next question

Theme from April 2012 Exam

Suprascapular nerve

The suprascapular nerve arises from the upper trunk of the brachial plexus. It lies
superior to the trunks of the brachial plexus and passes inferolaterally parallel to
them. It passes through the scapular notch, deep to trapezius. It innervates both
supraspinatus and infraspinatus and initiates abduction of the shoulder. If
damaged, patients may be able to abduct the shoulder by leaning over the
affected side and deltoid can then continue to abduct the shoulder.
yomna92009@yahoo.com - MRCS Part A - My account

Reference ranges End and review

Question 4 of 259 Question stats Score: 0%

1
A 13.3%
Which of the following statements relating to the Cavernous Sinus is false? 2
B 13.4%
C 10% 3

A. The pituitary gland lies medially D 49.7% 4

B. The internal carotid artery passes through it E 13.6%

C. The temporal lobe of the brain is a lateral relation 49.7% of users answered this
question correctly
D. The mandibular branch of the trigeminal and optic nerve lie on the
lateral wall
E. The ophthalmic veins drain into the anterior aspect of the sinus

Next question

The veins that drain into the sinus are important as sepsis can cause cavernous
sinus thrombosis. The maxillary branch of the trigeminal and not the mandibular
branches pass through the sinus

Cavernous sinus

Is a large collection of thin-walled veins creating a cavity


Bordered by the temporal bone of the skull and the sphenoid bone
Medially: pituitary fossa, sphenoid sinus
Laterally: temporal lobe

Contents

1. Lateral wall components, from top to bottom:

Oculomotor nerve
Trochlear nerve
Ophthalmic nerve
Maxillary nerve

2. Components within the sinus, from medial to lateral:

Internal carotid artery (and sympathetic plexus)


Abducens nerve

Blood supply

Ophthalmic vein, superficial cortical veins, basilar plexus of veins posteriorly.

Drains into the internal jugular vein via: the superior and inferior petrosal sinuses

Image sourced from Wikipedia


yomna92009@yahoo.com - MRCS Part A - My account

Reference ranges End and review

Question 5 of 259 Question stats Score: 0%

1
A 64.5%
Which of the following is not a branch of the subclavian artery? 2
B 11%
C 6.6% 3

A. Superior thyroid artery D 7.8% 4

B. Vertebral artery E 10.1% 5

C. Thyrocervical trunk 64.5% of users answered this


question correctly
D. Internal thoracic artery
E. Dorsal scapular artery

Next question

Mnemonic for the branches of the subclavian artery: VIT C & D

V ertebral artery
I nternal thoracic
T hyrocervical trunk

C ostalcervical trunk
D orsal scapular

Superior thyroid artery is a branch of the external carotid artery.

Subclavian artery

Path

The left subclavian comes directly off the arch of aorta


The right subclavian arises from the brachiocephalic artery (trunk) when it
bifurcates into the subclavian and the right common carotid artery.
From its origin, the subclavian artery travels laterally, passing between
anterior and middle scalene muscles, deep to scalenus anterior and
anterior to scalenus medius. As the subclavian artery crosses the lateral
border of the first rib, it becomes the axillary artery. At this point it is
superficial and within the subclavian triangle.

Image sourced from Wikipedia

Branches

Vertebral artery
Internal thoracic artery
Thyrocervical trunk
Costocervical trunk
Dorsal scapular artery
yomna92009@yahoo.com - MRCS Part A - My account

Reference ranges End and review

Question 6 of 259 Question stats Score: 0%

1
A 11.2%
Which muscle is not innervated by the trigeminal nerve? 2
B 19.9%
C 43.7% 3

A. Medial pterygoid D 12.1% 4

B. Mylohyoid E 13.1% 5

C. Stylohyoid 43.7% of users answered this 6


question correctly
D. Masseter
E. Temporalis

Next question

Stylohyoid is innervated by the facial nerve.

Trigeminal nerve

The trigeminal nerve is the main sensory nerve of the head. In addition to its
major sensory role, it also innervates the muscles of mastication.

Distribution of the trigeminal nerve

Sensory Scalp
Face
Oral cavity (and teeth)
Nose and sinuses
Dura mater

Motor Muscles of mastication


Mylohyoid
Anterior belly of digastric
Tensor tympani
Tensor palati

Autonomic connections (ganglia) Ciliary


Sphenopalatine
Otic
Submandibular

Path

Originates at the pons


Sensory root forms the large, crescentic trigeminal ganglion within Meckel's
cave, and contains the cell bodies of incoming sensory nerve fibres. Here
the 3 branches exit.
The motor root cell bodies are in the pons and the motor fibres are
distributed via the mandibular nerve. The motor root is not part of the
trigeminal ganglion.

Branches of the trigeminal nerve


Ophthalmic nerve Sensory only

Maxillary nerve Sensory only

Mandibular nerve Sensory and motor

Sensory
Ophthalmic Exits skull via the superior orbital fissure
Sensation of: scalp and forehead, the upper eyelid, the conjunctiva and cornea
of the eye, the nose (including the tip of the nose, except alae nasi), the nasal
mucosa, the frontal sinuses, and parts of the meninges (the dura and blood
vessels).

Maxillary Exit skull via the foramen rotundum


yomna92009@yahoo.com - MRCS Part A - My account

Reference ranges End and review

Question 7 of 259 Question stats Score: 0%

1
A 10.8%
A 73 year old man is undergoing a cardiac bypass operation. Which of the 2
B 60.5%
following structures is the largest tributary of the coronary sinus? 3
C 12.5%
D 7.7% 4
A. Thesbian veins E 8.5% 5
B. Great cardiac vein 6
60.5% of users answered this
C. Oblique vein question correctly
7
D. Small cardiac veins
E. None of the above

Next question

The great cardiac vein runs in the anterior interventricular groove, and is the
largest tributary of the coronary sinus. The Thesbian veins drain into the heart
directly.

Heart anatomy

The walls of each cardiac chamber comprise:

Epicardium
Myocardium
Endocardium

Cardiac muscle is attached to the cardiac fibrous skeleton.

Relations
The heart and roots of the great vessels within the pericardial sac are related
anteriorly to the sternum, medial ends of the 3rd to 5th ribs on the left and their
associated costal cartilages. The heart and pericardial sac are situated obliquely
two thirds to the left and one third to the right of the median plane.

The pulmonary valve lies at the level of the left third costal cartilage.
The mitral valve lies at the level of the fourth costal cartilage.

Coronary sinus
This lies in the posterior part of the coronary groove and receives blood from the
cardiac veins. The great cardiac vein lies at its left and the middle and small
cardiac veins lie on its right. The smallest cardiac veins drain into the atria
directly.

Aortic sinus
Right coronary artery arises from the right aortic sinus, the left is derived from the
left aortic sinus and no vessel emerges from the posterior sinus.

Right and left ventricles

Structure Left Ventricle

A-V Valve Mitral (double leaflet)

Walls Twice as thick as right

Trabeculae carnae Much thicker and more numerous

Right coronary artery


The RCA supplies:

Right atrium
Diaphragmatic part of the left ventricle
Usually the posterior third of the interventricular septum
The sino atrial node (60% cases)
The atrio ventricular node (80% cases)

Left coronary artery


The LCA supplies:

Left atrium
Most of left ventricle
Part of the right ventricle
Anterior two thirds of the inter ventricular septum
The sino atrial node (remaining 40% cases)

Innervation of the heart


Autonomic nerve fibres from the superficial and deep cardiac plexus. These lie
anterior to the bifurcation of the trachea, posterior to the ascending aorta and
superior to the bifurcation of the pulmonary trunk. The parasympathetic supply to
the heart is from presynaptic fibres of the vagus nerves.

Valves of the heart


Mitral valve Aortic valve Pulmonary valve Tricuspid valve

2 cusps 3 cusps 3 cusps 3 cusps

First heart sound Second heart Second heart First heart sound
sound sound

1 anterior cusp 1 anterior cusp 2 anterior cusps 2 anterior cusps

Attached to chordae No chordae No chordae Attached to chordae


tendinae tendinae

Rate question: Next question

Com m ent on this question

All contents of this site are © 2012 E-Medical Revision Ltd Terms and Conditions Privacy policy
yomna92009@yahoo.com - MRCS Part A - My account

Reference ranges End and review

Question 8 of 259 Question stats Score: 0%

1
A 8.9%
Which of the following vessels provides the greatest contribution to the arterial 2
B 11.4%
supply of the breast? 3
C 58.5%
D 13.1% 4
A. External mammary artery E 8.1% 5
B. Thoracoacromial artery 6
58.5% of users answered this
C. Internal mammary artery question correctly
7
D. Lateral thoracic artery
8
E. Subclavian artery

Next question

60% of the arterial supply to the breast is derived from the internal mammary
artery. The external mammary and lateral thoracic arteries also make a significant
(but lesser) contribution. This is of importance clinically in performing reduction
mammoplasty procedures.

Breast

The breast itself lies on a layer of pectoral fascia and the following muscles:
1. Pectoralis major
2. Serratus anterior
3. External oblique

Image showing the topography of the female breast

Image sourced from Wikipedia

Breast anatomy
Nerve supply Branches of intercostal nerves from T4-T6.

Arterial supply Internal mammary (thoracic) artery


External mammary artery (laterally)
External mammary artery (laterally)
Anterior intercostal arteries
Thoraco-acromial artery

Venous Superficial venous plexus to sub clavian, axillary and intercostal veins.
drainage

Lymphatic 70% Axillary nodes


drainage Internal mammary chain
Other lymphatic sites such as deep cervical and supraclavicular
fossa (later in disease)

Rate question: Next question

Com m ent on this question

All contents of this site are © 2012 E-Medical Revision Ltd Terms and Conditions Privacy policy
yomna92009@yahoo.com - MRCS Part A - My account

Reference ranges End and review

Question 9 of 259 Question stats Score: 0%

1
A 8.4%
Which of the following muscles is supplied by the external laryngeal nerve? 2
B 66.7%
C 7.3% 3

A. Transverse arytenoid D 11.4% 4

B. Cricothyroid E 6.1% 5

C. Thyro-arytenoid 66.7% of users answered this 6


question correctly
D. Posterior crico-arytenoid 7

E. Oblique arytenoid 8

9
Next question

The others are all supplied by the recurrent laryngeal nerve.

Larynx

The larynx lies in the anterior part of the neck at the levels of C3 to C6 vertebral
bodies. The laryngeal skeleton consists of a number of cartilagenous segments.
Three of these are paired; arytenoid, corniculate and cuneiform. Three are
single; thyroid, cricoid and epiglottic. The cricoid cartilage forms a complete ring
(the only one to do so).
The laryngeal cavity extends from the laryngeal inlet to the level of the inferior
border of the cricoid cartilage.

Divisions of the laryngeal cavity


Laryngeal vestibule Superior to the vestibular folds

Laryngeal ventricle Lies between vestibular folds and superior to the vocal cords

Infraglottic cavity Extends from vocal cords to inferior border of the cricoid cartilage

The vocal folds (true vocal cords) control sound production. The apex of each
fold projects medially into the laryngeal cavity. Each vocal fold includes:

Vocal ligament
Vocalis muscle (most medial part of thyroarytenoid muscle)

The glottis is composed of the vocal folds, processes and rima glottidis. The rima
glottidis is the narrowest potential site within the larynx, as the vocal cords may be
completely opposed, forming a complete barrier.

Muscles of the larynx


Muscle Origin Insertion Innervation Action

Posterior Posterior aspect Muscular process of Recurrent Abducts vocal fold


cricoarytenoid of lamina of arytenoid Laryngeal
cricoid

Lateral Arch of cricoid Muscular process of Recurrent Adducts vocal fold


cricoarytenoid arytenoid laryngeal

Thyroarytenoid Posterior aspect Muscular process of Recurrent Relaxes vocal fold


of thyroid cartilage arytenoid laryngeal

Transverse Arytenoid cartilage Contralateral Recurrent Closure of


and oblique arytenoid laryngeal intercartilagenous
arytenoids part of the rima
glottidis

Vocalis Depression Vocal ligament and Recurrent Relaxes posterior


between lamina vocal process of laryngeal vocal ligament,
of thyroid cartilage arytenoid cartilage tenses anterior
part

Cricothyroid Anterolateral part Inferior margin and External Tenses vocal fold
of cricoid horn of thyroid laryngeal
cartilage

Blood supply
yomna92009@yahoo.com - MRCS Part A - My account

Reference ranges End and review

Question 10 of 259 Question stats Score: 0%

1
A 65.9%
A 28 year old man has sustained a non salvageable testicular injury to his left 2
B 13.7%
testicle. The surgeon decides to perform an orchidectomy and divides the left 3
C 6.5%
testicular artery. From which of the following does this vessel originate?
D 7.5% 4
E 6.5% 5
A. Abdominal aorta
65.9% of users answered this 6
B. Internal iliac artery question correctly
7
C. Inferior epigastric artery
8
D. Inferior vesical artery
9
E. External iliac artery
10

Next question

The testicular artery is a branch of the abdominal aorta.

Scrotal and testicular anatomy

Spermatic cord
Formed by the vas deferens and is covered by the following structures:
Layer Origin

Internal spermatic fascia Transversalis fascia

Cremasteric fascia From the fascial coverings of internal oblique

External spermatic fascia External oblique aponeurosis

Contents of the cord


Vas deferens Transmits sperm and accessory gland secretions

Testicular artery Branch of abdominal aorta supplies testis and


epididymis

Artery of vas deferens Arises from inferior vesical artery

Cremasteric artery Arises from inferior epigastic artery

Pampiniform plexus Venous plexus, drains into right or left testicular vein

Sympathetic nerve fibres Lie on arteries, the parasympathetic fibres lie on the
vas

Genital branch of the genitofemoral Supplies cremaster


nerve

Lymphatic vessels Drain to lumbar and para-aortic nodes

Scrotum

Composed of skin and closely attached dartos fascia.


Arterial supply from the anterior and posterior scrotal arteries
Lymphatic drainage to the inguinal lymph nodes
Parietal layer of the tunica vaginalis is the innermost layer

Testes

The testes are surrounded by the tunica vaginalis (closed peritoneal sac).
The parietal layer of the tunica vaginalis adjacent to the internal spermatic
fascia.
The testicular arteries arise from the aorta immediately inferiorly to the
renal arteries.
The pampiniform plexus drains into the testicular veins, the left drains into
the left renal vein and the right into the inferior vena cava.
Lymphatic drainage is to the para-aortic nodes.
Rate question: Next question

Com m ent on this question

All contents of this site are © 2012 E-Medical Revision Ltd Terms and Conditions Privacy policy
yomna92009@yahoo.com - MRCS Part A - My account

Reference ranges End and review

Question 11 of 259 Question stats Score: 0%

1
A 10.3%
During a carotid endarterectomy the internal carotid artery is cross clamped. 2
B 10.2%
Assuming that no shunt is inserted, which of the following vessels will not have 3
C 12.4%
diminished or absent flow as a result?
D 59.3% 4
E 7.9% 5
A. Anterior cerebral artery
59.3% of users answered this 6
B. Ophthalmic artery question correctly
7
C. Middle cerebral artery
8
D. Maxillary artery
9
E. None of the above
10

Next question 11

The maxillary artery is a branch of the external carotid artery.

Internal carotid artery

The internal carotid artery is formed from the common carotid opposite the upper
border of the thyroid cartilage. It extends superiorly to enter the skull via the
carotid canal. From the carotid canal it then passes through the cavernous sinus,
above which it divides into the anterior and middle cerebral arteries.

Relations in the neck

Posterior Longus capitis


Pre-vertebral fascia
Sympathetic chain
Superior laryngeal nerve

Medially External carotid (near origin)


Wall of pharynx
Ascending pharyngeal artery

Laterally Internal jugular vein (moves posteriorly at entrance to skull)


Vagus nerve (most posterolaterally)

Anteriorly Sternocleidomastoid
Lingual and facial veins
Hypoglossal nerve

Relations in the carotid canal

Internal carotid plexus


Cochlea and middle ear cavity
Trigeminal ganglion (superiorly)
Leaves canal lies above the foramen lacerum

Path and relations in the cranial cavity


The artery bends sharply forwards in the cavernous sinus, the aducens nerve lies
close to its inferolateral aspect. The oculomotor, trochlear, opthalmic and, usually,
the maxillary nerves lie in the lateral wall of the sinus. Near the superior orbital
fissure it turns posteriorly and passes postero-medially to pierce the roof of the
cavernous sinus inferior to the optic nerve. It then passes between the optic and
oculomotor nerves to terminate below the anterior perforated substance by
dividing into the anterior and middle cerebral arteries.

Branches

Anterior and middle cerebral artery


Ophthalmic artery
Posterior communicating artery
Anterior choroid artery
Meningeal arteries
Hypophyseal arteries

Image demonstrating the internal carotid artery and its relationship to the external
carotid artery

Image sourced from Wikipedia

Rate question: Next question

Com m ent on this question

All contents of this site are © 2012 E-Medical Revision Ltd Terms and Conditions Privacy policy
yomna92009@yahoo.com - MRCS Part A - My account

Reference ranges End and review

Question 12 of 259 Question stats Score: 0%

1
A 10.8%
A 72 year old lady with osteoporosis falls and sustains an intracapsular femoral 2
B 7.1%
neck fracture. The fracture is completely displaced. Which of the following vessels 3
C 6.6%
is the main contributor to the arterial supply of the femoral head?
D 69.8% 4
E 5.6% 5
A. Deep external pudendal artery
69.8% of users answered this 6
B. Superficial femoral artery question correctly
7
C. External iliac artery
8
D. Circumflex femoral arteries
9
E. Superficial external pudendal artery
10

Next question 11

12
The vessels which form the anastomoses around the femoral head are derived
from the medial and lateral circumflex femoral arteries. These are usually derived
from the profunda femoris artery.

Hip joint

Head of femur articulates with acetabulum of the pelvis


Both covered by articular hyaline cartilage
The acetabulum forms at the union of the ilium, pubis, and ischium
The triradiate cartilage (Y-shaped growth plate) separates the pelvic bones
The acetabulum holds the femoral head by the acetabular labrum
Normal angle between femoral head and femoral shaft is 130o

Ligaments

Transverse ligament: joints anterior and posterior ends of the articular


cartilage
Head of femur ligament (ligamentum teres): acetabular notch to the fovea.
Contains arterial supply to head of femur in children.

Image sourced from Wikipedia


yomna92009@yahoo.com - MRCS Part A - My account

Reference ranges End and review

Question 13 of 259 Question stats Score: 0%

1
A 68.7%
A 21 year old man is hit with a hammer and sustains a depressed skull fracture at 2
B 6.8%
the vertex. Which of the following sinuses is at risk in this injury? 3
C 9.2%
D 8.1% 4
A. Superior sagittal sinus E 7.2% 5
B. Inferior petrosal sinus 6
68.7% of users answered this
C. Transverse sinus question correctly
7
D. Inferior sagittal sinus
8
E. Straight sinus
9

10
Next question
11

Theme in September 2011 Exam 12

13
The superior sagittal sinus is at greatest risk in this pattern of injury. This sinus
begins at the front of the crista galli and courses backwards along the falx cerebri.
It becomes continuous with the right transverse sinus near the internal occipital
protuberance.

Cranial venous sinuses

The cranial venous sinuses are located within the dura mater. They have no
valves which is important in the potential for spreading sepsis. They eventually
drain into the internal jugular vein.

They are:

Superior sagittal sinus


Inferior sagittal sinus
Straight sinus
Transverse sinus
Sigmoid sinus
Confluence of sinuses
Occipital sinus
Cavernous sinus

Topography of cranial venous sinuses

Image sourced from Wikipedia

Rate question: Next question

Com m ent on this question


yomna92009@yahoo.com - MRCS Part A - My account

Reference ranges End and review

Question 14 of 259 Question stats Score: 0%

1
A 11.5%
A 44 year old man is stabbed in the back and the left kidney is injured. A 2
B 8.3%
haematoma forms, which of the following fascial structures will contain the 3
C 9%
haematoma?
D 63.8% 4
E 7.3% 5
A. Waldeyers fascia
63.8% of users answered this 6
B. Sibsons fascia question correctly
7
C. Bucks fascia
8
D. Gerotas fascia
9
E. Denonvilliers fascia
10

Next question 11

12

13

14
Renal anatomy

Each kidney is about 11cm long, 5cm wide and 3cm thick. They are located in a
deep gutter alongside the projecting verterbral bodies, on the anterior surface of
psoas major. In most cases the left kidney lies approximately 1.5cm higher than
the right. The upper pole of both kidneys approximates with the 11th rib (beware
pneumothorax during nephrectomy). On the left hand side the hilum is located at
the L1 vertebral level and the right kidney at level L1-2. The lower border of the
kidneys is usually alongside L3.

The table below shows the anatomical relations of the kidneys:

Relations
Relations Right Kidney Left Kidney

Posterior Quadratus lumborum, diaphragm, Quadratus lumborum, diaphragm,


psoas major, transversus abdominis psoas major, transversus abdominis

Anterior Hepatic flexure of colon Stomach, Pancreatic tail

Superior Liver, adrenal gland Spleen, adrenal gland

Fascial covering
Each kidney and suprarenal gland is enclosed within a common and layer of
investing fascia that is derived from the transversalis fascia into anterior and
posterior layers (Gerotas fascia).

Renal structure
Kidneys are surrounded by an outer cortex and an inner medulla which usually
contains between 6 and 10 pyramidal structures. The papilla marks the innermost
apex of these. They terminate at the renal pelvis, into the ureter.
Lying in a hollow within the kidney is the renal sinus. This contains:
1. Branches of the renal artery
2. Tributaries of the renal vein
3. Major and minor calyces's
4. Fat

Structures at the renal hilum


The renal vein lies most anteriorly, then renal artery (it is an end artery) and the
ureter lies most posterior.

Rate question: Next question

Com m ent on this question


yomna92009@yahoo.com - MRCS Part A - My account

Reference ranges End and review

Question 15 of 259 Question stats Score: 0%

1
A 12.2%
Which of the following muscles is not innervated by the ansa cervicalis? 2
B 45.3%
C 13.1% 3

A. Sternohyoid D 18.4% 4

B. Mylohyoid E 11% 5

C. Omohyoid 45.3% of users answered this 6


question correctly
D. Sternothyroid 7

E. None of the above 8

9
Next question
10

11

Ansa cervicalis muscles: 12

13
GHost THought SOmeone Stupid Shot Irene
14

GenioHyoid 15
ThyroidHyoid
Superior Omohyoid
SternoThyroid
SternoHyoid
Inferior Omohyoid

Mylohyoid is innervated by the mylohyoid branch of the inferior alveolar nerve.

Ansa cervicalis

Superior Branch of C1 anterolateral to carotid sheath


root

Inferior Derived from C2 and C3 roots, passes posterolateral to the internal jugular
root vein (may lie either deep or superficial to it)

Innervation Sternohyoid
Sternothyroid
Omohyoid

The ansa cervicalis lies anterior to the carotid sheath. The nerve supply to the
inferior strap muscles enters at their inferior aspect. Therefore when dividing
these muscles to expose a large goitre, the muscles should be divided in their
upper half.
Image sourced from Wikipedia

Rate question: Next question

Com m ent on this question

All contents of this site are © 2012 E-Medical Revision Ltd Terms and Conditions Privacy policy
yomna92009@yahoo.com - MRCS Part A - My account

Reference ranges End and review

Question 16 of 259 Question stats Score: 0%

1
A 38.4%
A baby is found to have a Klumpke's palsy post delivery. Which of the following is 2
B 11.4%
most likely to be present? 3
C 17.6%
D 14.7% 4
A. Loss of flexors of the wrist E 17.9% 5
B. Weak elbow flexion 6
38.4% of users answered this
C. Pronation of the forearm question correctly
7
D. Adducted shoulder
8
E. Shoulder medially rotated
9

10
Next question
11

12
Features of Klumpkes Paralysis
13

Claw hand (MCP joints extended and IP joints flexed) 14


Loss of sensation over medial aspect of forearm and hand
15
Horner's syndrome
Loss of flexors of the wrist 16

A C8, T1 root lesion is called Klumpke's paralysis and is caused by delivery with
the arm extended.

Brachial plexus

Origin Anterior rami of C5 to T1

Sections of the Roots, trunks, divisions, cords, branches


plexus Mnemonic:Real Teenagers Drink Cold Beer

Roots Located in the posterior triangle


Pass between scalenus anterior and medius

Trunks Located posterior to middle third of clavicle


Upper and middle trunks related superiorly to the subclavian

artery
Lower trunk passes over 1st rib posterior to the subclavian
artery

Divisions Apex of axilla

Cords Related to axillary artery

Diagram illustrating the branches of the brachial plexus


yomna92009@yahoo.com - MRCS Part A - My account

Reference ranges End and review

Question 17 of 259 Question stats Score: 0%

1
A 24.3%
A 22 year old man undergoes a superficial parotidectomy for a pleomorphic 2
B 30.8%
adenoma. The operation does not proceed well and a diathermy malfunction 3
C 18.6%
results in division of the buccal branch of the facial nerve. Which of the following
D 12.4% 4
muscles will not demonstrate impaired function as a result?
E 14% 5

30.8% of users answered this 6


A. Zygomaticus minor
question correctly
7
B. Mentalis
8
C. Buccinator
D. Levator anguli oris 9

E. Risorius 10

11

Next question 12

13
Buccal branch supplies
14
Zygomaticus minor Elevates upper lip
15
Risorius Aids smile
16
Pulls corner of mouth backward and compresses cheek
Buccinator 17

Levator anguli oris Pulls angles of mouth upward and toward midline

Orbicularis Closes and tightens lips together

Nasalis Flares nostrils and compresses nostrils

Facial nerve

The facial nerve is the main nerve supplying the structures of the second
embryonic branchial arch. It is predominantly an efferent nerve to the muscles of
facial expression, digastric muscle and also to many glandular structures. It
contains a few afferent fibres which originate in the cells of its genicular ganglion
and are concerned with taste.

Supply - 'face, ear, taste, tear'

Face: muscles of facial expression


Ear: nerve to stapedius
Taste: supplies anterior two-thirds of tongue
Tear: parasympathetic fibres to lacrimal glands, also salivary glands

Path
Subarachnoid path

Origin: motor- pons, sensory- nervus intermedius


Pass through the petrous temporal bone into the internal auditory meatus
with the vestibulocochlear nerve. Here they combine to become the facial
nerve.

Facial canal path

The canal passes superior to the vestibule of the inner ear


At the medial aspect of the middle ear, it becomes wider and contains the
geniculate ganglion.

- 3 branches:
1. greater petrosal nerve
2. nerve to stapedius
3. chorda tympani
yomna92009@yahoo.com - MRCS Part A - My account

Reference ranges End and review

Question 18 of 259 Question stats Score: 0%

1
A 46.7%
At which of the following vertebral body levels does the common carotid artery 2
B 10.6%
typically bifurcate into the external and internal carotid arteries? 3
C 10.8%
D 23.3% 4
A. C4 E 8.6% 5
B. C2 6
46.7% of users answered this
C. C1 question correctly
7
D. C6
8
E. C7
9

10
Next question
11

It terminates at the upper border of the thyroid cartilege, Which is usually located 12
at C4. 13

14
Common carotid artery
15

The right common carotid artery arises at the bifurcation of the brachiocephalic 16
trunk, the left common carotid arises from the arch of the aorta. Both terminate at 17
the level of the upper border of the thyroid cartilage (the lower border of the third
18
cervical vertebra) by dividing into the internal and external carotid arteries.

Left common carotid artery


This vessel arises immediately to the left and slightly behind the origin of the
brachiocephalic trunk. Its thoracic portion is 2.5- 3.5 cm in length and runs
superolaterally to the sternoclavicular joint.

In the thorax
The vessel is in contact, from below upwards, with the trachea, left recurrent
laryngeal nerve, left margin of the oesophagus. Anteriorly the left brachiocephalic
vein runs across the artery, and the cardiac branches from the left vagus
descend in front of it. These structures together with the thymus and the anterior
margins of the left lung and pleura separate the artery from the manubrium.

In the neck
The artery runs superiorly deep to sternocleidomastoid and then enters the
anterior triangle. At this point it lies within the carotid sheath with the vagus nerve
and the internal jugular vein. Posteriorly the sympathetic trunk lies between the
vessel and the prevertebral fascia. At the level of C7 the vertebral artery and
thoracic duct lie behind it. The anterior tubercle of C6 transverse process is
prominent and the artery can be compressed against this structure (it
corresponds to the level of the cricoid).
Anteriorly at C6 the omohyoid muscle passes superficial to the artery.
Within the carotid sheath the jugular vein lies lateral to the artery.

Right common carotid artery


The right common carotid arises from the brachiocephalic artery. The right
common carotid artery corresponds with the cervical portion of the left common
carotid, except that there is no thoracic duct on the right. The oesophagus is less
closely related to the right carotid than the left.

Summary points about the carotid anatomy

Path
Passes behind the sternoclavicular joint (12% patients above this level) to the
upper border of the thyroid cartilage, to divide into the external (ECA) and internal
carotid arteries (ICA).

Relations
yomna92009@yahoo.com - MRCS Part A - My account

Reference ranges End and review

Question 19 of 259 Question stats Score: 0%

1
A 19.4%
A man is stabbed in the chest to the right of the manubriosternal angle. Which 2
B 14.1%
structure is least likely to be injured in this case? 3
C 11.9%
D 35.4% 4
A. Aortic arch E 19.2% 5
B. The trachea 6
35.4% of users answered this
C. Right phrenic nerve question correctly
7
D. Right recurrent laryngeal nerve
8
E. Brachiocephalic vein
9

10
Next question
11

The right recurrent laryngeal nerve branches off the right vagus more proximally 12
and arches posteriorly round the subclavian artery. So of the structures given it is 13
the least likely to be injured.
14

Mediastinum 15

16
Region between the pulmonary cavities. 17
It is covered by the mediastinal pleura. It does not contain the lungs.
18
It extends from the thoracic inlet superiorly to the diaphragm inferiorly.
19
Mediastinal regions

Superior mediastinum
Inferior mediastinum
Posterior mediastinum
Anterior mediastinum

Region Contents

Superior mediastinum Superior vena cava


Brachiocephalic veins
Arch of aorta
Thoracic duct
Trachea
Oesophagus

Thymus
Vagus nerve
Left recurrent laryngeal nerve
Phrenic nerve

Anterior mediastinum Thymic remnants


Lymph nodes
Fat

Middle mediastinum Pericardium


Heart
Aortic root
Arch of azygos vein
Main bronchi

Posterior mediastinum Oesophagus


Thoracic aorta
Azygos vein
Thoracic duct
Vagus nerve
Sympathetic nerve trunks
Splanchnic nerves
yomna92009@yahoo.com - MRCS Part A - My account

Reference ranges End and review

Question 20 of 259 Question stats Score: 0%

1
A 25.2%
An 18 year old man is stabbed in the neck and has to undergo repair of a 2
B 26.3%
laceration to the internal carotid artery. Post operatively he is noted to have a 3
C 13.9%
Horners syndrome. Which of the following will not be present?
D 6.3% 4
E 28.3% 5
A. Apparent endopthalmos
26.3% of users answered this 6
B. Loss of sweating on the entire ipsilateral side of the face question correctly
7
C. Constricted pupil
8
D. Mild ptosis
9
E. Normal sympathetic activity in the torso
10

Next question 11

12
The anhidrosis will be mild as this is a distal lesion and at worst only a very limited 13
area of the ipsilateral face will be anhidrotic.
14

Horners syndrome 15

16
Horners syndrome, clinical features: 17

Ptosis 18
Miosis 19
Endopthalmos
20
Anhydrosis

Primarily a disorder of the sympathetic nervous system. Extent of symptoms


depends upon the anatomical site of the lesion.

Proximal lesions occur along the hypothalamospinal tract

Distal lesions are usually post ganglionic e.g. at level of internal carotid artery or
beyond.

In contrast to a 3rd nerve palsy the ptosis is more mild and the pupil
constricted rather than dilated.

Rate question: Next question

Com m ent on this question

All contents of this site are © 2012 E-Medical Revision Ltd Terms and Conditions Privacy policy
yomna92009@yahoo.com - MRCS Part A - My account

Reference ranges End and review

Question 1 of 239 Question stats Score: 0%

1
A 55%
Which of the following types of epithelium lines the lumenal surface of the normal B 15.8%
oesophagus?
C 10.7%
D 11.6%

A. Non keratinised stratified squamous epithelium E 6.8%

B. Ciliated columnar epithelium


55% of users answered this
C. Keratinised stratified squamous epithelium question correctly

D. Non ciliated columnar epithelium


E. None of the above

Next question

The oesphagus is lined by non keratinised stratified squamous epithelium.


Changes to glandular type epithelium occur as part of metaplastic processes in
reflux.

Oesophagus

25cm long
Starts at C6 vertebra, pierces diaphragm at T10 and ends at T11
Squamous epithelium

Constrictions of the oesophagus


Structure Distance from incisors

Cricoid cartilage 15cm

Arch of the Aorta 22.5cm

Left principal bronchus 27cm

Diaphragmatic hiatus 40cm

Relations

Anteriorly Trachea to T4
Recurrent laryngeal nerve
Left bronchus, Left atrium

Diaphragm

Posteriorly Thoracic duct to left at T5


Hemiazygos to right T8
Descending aorta
First 2 intercostal branches of aorta

Left Thoracic duct


Left subclavian artery

Right Azygos vein

Arterial, venous and lymphatic drainage of the oesophagus


Artery Vein Lymphatics Muscularis externa

Upper Inferior Inferior thyroid Deep Striated muscle


third thyroid cervical

Mid third Aortic Azygos branches Mediastinal Smooth & striated


branches muscle

Lower Left gastric Posterior mediastinal and Gastric Smooth muscle


third coeliac
yomna92009@yahoo.com - MRCS Part A - My account

Reference ranges End and review

Question 2 of 239 Question stats Score: 0%

1
A 30.5%
A 42 year old male sustains a back injury resulting in the compression of the 2
B 5.1%
conus medullaris. Which of the dematomes below is most likely to be affected by
C 23.7%
this process?
D 33.9%

E 6.8%
A. S1
23.7% of users answered this
B. L1 question correctly
C. S3
D. L3
E. L5

Next question

Theme from the September 2012 Exam


The perineum is innervated by S3 and S4, S2 runs down the posterior aspect of
the leg and would also be affected.

Dermatomes

The common dermatomal levels and cutaneous nerves responsible for them is
illustrated below.
Image sourced from Wikipedia

Rate question: Next question

Com m ent on this question

All contents of this site are © 2012 E-Medical Revision Ltd Terms and Conditions Privacy policy
yomna92009@yahoo.com - MRCS Part A - My account

Reference ranges End and review

Question 3 of 239 Question stats Score: 0%

1
A 56.9%
A 23 year old man is stabbed in the neck, in the region between the omohyoid 2
B 18.8%
and digastric muscles, the injury is explored surgically. At operation a nerve injury 3
C 5.3%
is identified immediately superior to the lingual artery as is branches off the
D 12.4%
external carotid artery. Which of the following is the most likely result of this
E 6.6%
injury?

56.9% of users answered this


question correctly
A. Paralysis of the ipsilateral side of the tongue
B. Abduction of the ipsilateral vocal cord
C. Winging of the scapula
D. Paralysis of the ipsilateral hemi diaphragm
E. Inability to abduct the shoulder

Next question

The hypoglossal nerve runs anterior to the external carotid, above the lingual
arterial branch. If damaged then ipsilateral paralysis of the genioglossus,
hyoglossus and styloglossus muscles will occur. If the patient is asked to protrude
their tongue then it will tend to point to the affected side.

Anterior triangle of the neck

Boundaries

Anterior border of the Sternocleidomastoid


Lower border of mandible
Anterior midline

Sub triangles (divided by Digastric above and Omohyoid)

Muscular triangle: Neck strap muscles


Carotid triangle: Carotid sheath
Submandibular Triangle (digastric)

Contents of the anterior triangle


Digastric triangle Submandibular gland
Submandibular nodes
Facial vessels
Hypoglossal nerve

Muscular triangle Strap muscles


External jugular vein

Carotid triangle Carotid sheath (Common carotid, vagus and internal jugular vein)
Ansa cervicalis

Nerve supply to digastric muscle

Anterior: Mylohyoid nerve


Posterior: Facial nerve
yomna92009@yahoo.com - MRCS Part A - My account

Reference ranges End and review

Question 4 of 239 Question stats Score: 0%

1
A 11.5%
Which of the following structures is not directly related to the right adrenal gland? 2
B 22.3%
C 36.7% 3

A. Diaphragm posteriorly D 18.5% 4

B. Bare area of the liver anteriorly E 11%

C. Right renal vein 36.7% of users answered this


question correctly
D. Inferior vena cava
E. Hepato-renal pouch

Next question

The right renal vein is very short and lies more inferiorly.

Adrenal gland anatomy

Anatomy

Location Superomedially to the upper pole of each kidney

Relationships of Diaphragm-Posteriorly, Kidney-Inferiorly, Vena Cava-Medially, Hepato-


the right adrenal renal pouch and bare area of the liver-Anteriorly

Relationships of Crus of the diaphragm-Postero- medially, Pancreas and splenic


the left adrenal vessels-Inferiorly, Lesser sac and stomach-Anteriorly

Superior adrenal arteries- from inferior phrenic artery, Middle adrenal


Arterial supply arteries - from aorta, Inferior adrenal arteries -from renal arteries

Venous drainage Via one central vein directly into the IVC
of the right
adrenal

Venous drainage Via one central vein into the left renal vein
of the left adrenal

Rate question: Next question

Com m ent on this question

All contents of this site are © 2012 E-Medical Revision Ltd Terms and Conditions Privacy policy
yomna92009@yahoo.com - MRCS Part A - My account

Reference ranges End and review

Question 5 of 239 Question stats Score: 0%

1
A 40.4%
With respect to the basilic vein, which statement is false? 2
B 11.8%
C 8.8% 3

A. Its deep anatomical location makes it unsuitable for use as an D 12.5% 4


arteriovenous access site in fistula surgery E 26.5% 5
B. It originates from the dorsal venous network on the hand
40.4% of users answered this
C. It travels up the medial aspect of the forearm question correctly

D. Halfway between the shoulder and the elbow it lies deep to muscle
E. It joins the brachial vein to form the axillary vein

Next question

It is used in arteriovenous fistula surgery during a procedure known as a basilic


vein transposition.

Basilic vein

The basilic and cephalic veins both provide the main pathways of venous
drainage for the arm and hand. It is continuous with the palmar venous arch
distally and the axillary vein proximally.

Path

Originates on the medial side of the dorsal venous network of the hand,
and passes up the forearm and arm.
Most of its course is superficial.
Near the region anterior to the cubital fossa the vein joins the cephalic vein.
Midway up the humerus the basilic vein passes deep under the muscles.
At the lower border of the teres major muscle, the anterior and posterior
circumflex humeral veins feed into it.
Joins the brachial veins to form the axillary vein.

Rate question: Next question

Com m ent on this question

All contents of this site are © 2012 E-Medical Revision Ltd Terms and Conditions Privacy policy
yomna92009@yahoo.com - MRCS Part A - My account

Reference ranges End and review

Question 6 of 239 Question stats Score: 0%

1
A 44.8%
Mobilisation of the left lobe of the liver will facilitate surgical access to which of the 2
B 14.3%
following? 3
C 9.2%
D 8.9% 4
A. Abdominal oesophagus E 22.8% 5
B. Duodenum 6
44.8% of users answered this
C. Right colic flexure question correctly

D. Right kidney
E. Pylorus of stomach

Next question

The fundus of the stomach is a posterior relation. The pylorus lies more
inferolaterally. During a total gastrectomy division of the ligaments holding the left
lobe of the liver will facilitate access to the proximal stomach and abdominal
oesophagus. This manoeuvre is seldom beneficial during a distal gastrectomy.

Liver

Structure of the liver

Right lobe Supplied by right hepatic artery


Contains Couinard segments V to VIII (-/+Sg I)

Left lobe Supplied by the left hepatic artery


Contains Couinard segments II to IV (+/- Sg1)

Quadrate Part of the right lobe anatomically, functionally is part of the left
lobe Couinard segment IV
Porta hepatis lies behind
On the right lies the gallbladder fossa
On the left lies the fossa for the umbilical vein

Caudate lobe Supplied by both right and left hepatic arteries


Couinard segment I
Lies behind the plane of the porta hepatis
Anterior and lateral to the inferior vena cava

Bile from the caudate lobe drains into both right and left hepatic
ducts

Detailed knowledge of Couinard segments is not required for MRCS Part A

Between the liver lobules are portal canals which contain the portal triad:
Hepatic Artery, Portal Vein, tributary of Bile Duct.

Relations of the liver


Anterior Postero inferiorly

Diaphragm Oesophagus

Xiphoid process Stomach

Duodenum

Hepatic flexure of colon

Right kidney

Gallbladder

Inferior vena cava


yomna92009@yahoo.com - MRCS Part A - My account

Reference ranges End and review

Question 7 of 239 Question stats Score: 0%

1
A 8.5%
The following statements relating to the ankle joint are true except? 2
B 47.1%
C 7.4% 3

A. Three groups of ligaments provide mechanical stability D 17.4% 4

B. The sural nerve lies medial to the Achilles tendon at its point of E 19.6% 5
insertion
47.1% of users answered this 6
C. Eversion of the foot occurs at the sub talar joint question correctly
7
D. The flexor hallucis longus tendon is the most posterior structure at
the medial malleolus
E. The saphenous nerve crosses the ankle joint.

Next question

The sural nerve lies behind the distal fibula. Inversion and eversion are sub talar
movements. The structures passing behind the medial malleolus from anterior to
posterior include: tibialis posterior, flexor digitorum longus, posterior tibia vein,
posterior tibial artery, nerve, flexor hallucis longus.

Ankle joint

The ankle joint is a synovial joint composed of the tibia and fibula superiorly and
the talus inferiorly.

Ligaments of the ankle joint

Deltoid ligament (medially)


Lateral collateral ligament
Talofibular ligaments (both anteriorly and posteriorly)

The calcaneofibular ligament is separate from the fibrous capsule of the joint. The
two talofibular ligaments are fused with it.

The components of the syndesmosis are

Antero-inferior talofibular ligament


Postero-inferior talofibular ligament
Inferior transverse talofibular ligament
Interosseous ligament

Movements at the ankle joint

Plantar flexion (55 degrees)


Dorsiflexion (35 degrees)
Inversion and eversion movements occur at the level of the sub talar joint

Nerve supply
Branches of deep peroneal and tibial nerves.

References
Golano P et al. Anatomy of the ankle ligaments: a pictorial essay. Knee Surg
Sports Traumatol Arthrosc. 2010 May;18(5):557-69

Rate question: Next question

Com m ent on this question


yomna92009@yahoo.com - MRCS Part A - My account

Reference ranges End and review

Question 8 of 239 Question stats Score: 12.5%

1
A 11.7%
A 78 year old man is lifting a heavy object when a feels a pain in his forearm and 2
B 10.3%
is unable to continue. He has a swelling over his upper forearm. An MRI scan 3
C 7.3%
shows a small cuff of tendon still attached to the radial tuberosity consistent with a
D 23.7% 4
recent tear. Which of the following muscles has been injured?
E 47% 5

47% of users answered this 6


A. Pronator teres
question correctly
7
B. Supinator
8
C. Aconeus
D. Brachioradialis
E. Biceps brachii

Next question

Biceps inserts into the radial tuberosity. Distal injuries of this muscle are rare but
are reported and are clinically more important than more proximal ruptures.

Radius

Bone of the forearm extending from the lateral side of the elbow to the
thumb side of the wrist

Upper end

Articular cartilage- covers medial > lateral side


Articulates with radial notch of the ulna by the annular ligament
Muscle attachment- biceps brachii at the tuberosity

Shaft

Muscle attachment-

Upper third of the body Supinator, Flexor digitorum superficialis, Flexor pollicis
longus
Middle third of the body Pronator teres
Lower quarter of the body Pronator quadratus , tendon of supinator longus

Lower end

Quadrilateral
Anterior surface- capsule of wrist joint
Medial surface- head of ulna
Lateral surface- ends in the styloid process
Posterior surface: 3 grooves containing:

1. Tendons of extensor carpi radialis longus and brevis


2. Tendon of extensor pollicis longus
3. Tendon of extensor indicis
yomna92009@yahoo.com - MRCS Part A - My account

Reference ranges End and review

Question 9 of 239 Question stats Score: 11.1%

1
A 19.8%
The oesophagus is constricted at the following levels apart from: 2
B 14.2%
C 31.7% 3

A. Cricoid cartilage D 16.9% 4

B. Arch of the aorta E 17.4% 5

C. Lower oesophageal sphincter 31.7% of users answered this 6


question correctly
D. Left main stem bronchus 7

E. Diaphragmatic hiatus 8

9
Next question

The oesophagus is not constricted at the level of the lower oesophageal


sphincter.

Oesophagus

25cm long
Starts at C6 vertebra, pierces diaphragm at T10 and ends at T11
Squamous epithelium

Constrictions of the oesophagus


Structure Distance from incisors

Cricoid cartilage 15cm

Arch of the Aorta 22.5cm

Left principal bronchus 27cm

Diaphragmatic hiatus 40cm

Relations

Anteriorly Trachea to T4
Recurrent laryngeal nerve
Left bronchus, Left atrium
Diaphragm

Posteriorly Thoracic duct to left at T5


Hemiazygos to right T8
Descending aorta
First 2 intercostal branches of aorta

Left Thoracic duct


Left subclavian artery

Right Azygos vein

Arterial, venous and lymphatic drainage of the oesophagus


Artery Vein Lymphatics Muscularis externa

Upper Inferior Inferior thyroid Deep Striated muscle


third thyroid cervical

Mid third Aortic Azygos branches Mediastinal Smooth & striated


branches muscle

Lower Left gastric Posterior mediastinal and Gastric Smooth muscle


third coeliac

Nerve supply

Upper half is supplied by recurrent laryngeal nerve


yomna92009@yahoo.com - MRCS Part A - My account

Reference ranges End and review

Question 10 of 239 Question stats Score: 10%

1
A 9.7%
A 19 year old man is playing rugby when he suddenly notices a severe pain at the 2
B 10.3%
posterolateral aspect of his right thigh. Which of the following muscle groups is 3
C 48.3%
most likely to have been injured?
D 23% 4
E 8.8% 5
A. Semimembranosus
48.3% of users answered this 6
B. Semitendinosus question correctly
7
C. Long head of biceps femoris
8
D. Gastrocnemius
9
E. Soleus
10

Next question

Theme from April 2012 Exam


The biceps femoris is the laterally located hamstring muscle. The semitendinosus
and semimembranosus are located medially. Rupture of gastrocnemius and
soleus may occur but is less common.

Biceps femoris

The biceps femoris is one of the hamstring group of muscles located in the
posterior upper thigh. It has two heads.

Long head
Origin Ischial tuberosity

Insertion Fibular head

Action Knee flexion, lateral rotation tibia, extension hip

Innervation Tibial nerve (L5, S1, S2)

Arterial Profunda femoris artery, inferior gluteal artery, and the superior muscular
supply branches of popliteal artery

Image demonstrating the biceps femoris muscle, with the long head outlined
yomna92009@yahoo.com - MRCS Part A - My account

Reference ranges End and review

Question 11 of 239 Question stats Score: 18.2%

1
A 6.6%
Which of the following is a branch of the third part of the axillary artery? 2
B 7.8%
C 10.8% 3

A. Superior thoracic D 14.4% 4

B. Lateral thoracic E 60.4% 5

C. Dorsal scapular 60.4% of users answered this 6


question correctly
D. Thoracoacromial 7

E. Posterior circumflex humeral 8

9
Next question
10

11
The other branches include:

Subscapular
Anterior circumflex humeral

Axilla

Boundaries of the axilla


Medially Chest wall and Serratus anterior

Laterally Humeral head

Floor Subscapularis

Anterior aspect Lateral border of Pectoralis major

Fascia Clavipectoral fascia

Content:
Long thoracic Derived from C5-C7 and passes behind the brachial plexus to enter
nerve (of Bell) the axilla. It lies on the medial chest wall and supplies serratus
anterior. Its location puts it at risk during axillary surgery and damage
will lead to winging of the scapula.

Thoracodorsal Innervate and vascularise latissimus dorsi.


nerve and
thoracodorsal
trunk

Axillary vein Lies at the apex of the axilla, it is the continuation of the basilic vein.
Becomes the subclavian vein at the outer border of the first rib.

Intercostobrachial Traverse the axillary lymph nodes and are often divided during axillary
nerves surgery. They provide cutaneous sensation to the axillary skin.

Lymph nodes The axilla is the main site of lymphatic drainage for the breast.

Rate question: Next question

Com m ent on this question

All contents of this site are © 2012 E-Medical Revision Ltd Terms and Conditions Privacy policy
yomna92009@yahoo.com - MRCS Part A - My account

Reference ranges End and review

Question 12 of 239 Question stats Score: 16.7%

1
A 14.7%
Which of the following structures separates the intervertebral disks from the 2
B 37.9%
spinal cord? 3
C 5.9%
D 10.3% 4
A. Anterior longitudinal ligament E 31.2% 5
B. Posterior longitudinal ligament 6
37.9% of users answered this
C. Supraspinous ligament question correctly
7
D. Interspinous ligament
8
E. Ligamentum flavum
9

10
Next question
11

The posterior longitudinal ligament overlies the posterior aspect of the vertebral 12
bodies. It also overlies the posterior aspect of the intervertebral disks.

Intervertebral discs

Consist of an outer annulus fibrosus and an inner nucleus pulposus.


The anulus fibrosus consists of several layers of fibrocartilage.
The nucleus pulposus contains loose fibres suspended in a mucoprotein
gel with the consistency of jelly. The nucleus of the disc acts as a shock
absorber.
Pressure on the disc causes posterior protrusion of the nucleus pulposus.
Most commonly in the lumbrosacral and lower cervical areas.
The discs are separated by hyaline cartilage.
There is one disc between each pair of vertebrae, except for C1/2 and the
sacrococcygeal vertebrae.

Rate question: Next question

Com m ent on this question

All contents of this site are © 2012 E-Medical Revision Ltd Terms and Conditions Privacy policy
yomna92009@yahoo.com - MRCS Part A - My account

Reference ranges End and review

Question 13 of 239 Question stats Score: 15.4%

1
A 6%
At what level does the aorta bifurcate into the left and right common iliac arteries? 2
B 6.7%
C 8.1% 3

A. L1 D 70.6% 4

B. L2 E 8.6% 5

C. L3 70.6% of users answered this 6


question correctly
D. L4 7

E. L5 8

9
Next question
10

11
The aorta typically bifurcates at L4. This level is usually fairly constant and is
often tested in the exam. 12

13
Levels

Transpyloric plane
Level of the body of L1

Pylorus stomach
Left kidney hilum (L1- left one!)
Right hilum of the kidney (1.5cm lower than the left)
Fundus of the gallbladder
Neck of pancreas
Duodenojejunal flexure
Superior mesenteric artery
Portal vein
Left and right colic flexure
Root of the transverse mesocolon
2nd part of the duodenum
Upper part of conus medullaris
Spleen

Can be identified by asking the supine patient to sit up without using their arms.
The plane is located where the lateral border of the rectus muscle crosses the
costal margin.

Anatomical planes
Subcostal plane Lowest margin of 10th costal cartilage

Intercristal plane Level of body L4 (highest point of iliac crest)

Intertubercular plane Level of body L5

Common level landmarks


Inferior mesenteric artery L3

Bifurcation of aorta into common iliac arteries L4

Formation of IVC L5 (union of common iliac veins)

Diaphragm apertures Vena cava T8


Oesophagus T10
Aortic hiatus T12

Rate question: Next question

Com m ent on this question


yomna92009@yahoo.com - MRCS Part A - My account

Reference ranges End and review

Question 14 of 239 Question stats Score: 14.3%

1
A 53.3%
A 23 year old man is due to undergo a mitral valve repair for mitral regurgitation. 2
B 12.2%
Which of the following is a feature of the mitral valve? 3
C 8.4%
D 14.6% 4
A. Its closure is marked by the first heart sound E 11.4% 5
B. It has two anterior cusps 6
53.3% of users answered this
C. The chordae tendinae attach to the anterior cusps only question correctly
7
D. The chordae tendinae anchor the valve directly to the wall of the left
8
ventricle
9
E. It is best auscultated in the left third interspace
10

Next question 11

12
The mitral valve is best auscultated over the cardiac apex, where its closure
13
marks the first heart sound. It has only two cusps. These are attached to chordae
14
tendinae which themselves are linked to the wall of the ventricle by the papillary
muscles.

Heart anatomy

The walls of each cardiac chamber comprise:

Epicardium
Myocardium
Endocardium

Cardiac muscle is attached to the cardiac fibrous skeleton.

Relations
The heart and roots of the great vessels within the pericardial sac are related
anteriorly to the sternum, medial ends of the 3rd to 5th ribs on the left and their
associated costal cartilages. The heart and pericardial sac are situated obliquely
two thirds to the left and one third to the right of the median plane.

The pulmonary valve lies at the level of the left third costal cartilage.
The mitral valve lies at the level of the fourth costal cartilage.

Coronary sinus
This lies in the posterior part of the coronary groove and receives blood from the
cardiac veins. The great cardiac vein lies at its left and the middle and small
cardiac veins lie on its right. The smallest cardiac veins drain into the atria
directly.

Aortic sinus
Right coronary artery arises from the right aortic sinus, the left is derived from the
left aortic sinus and no vessel emerges from the posterior sinus.

Right and left ventricles

Structure Left Ventricle

A-V Valve Mitral (double leaflet)

Walls Twice as thick as right

Trabeculae carnae Much thicker and more numerous

Right coronary artery


The RCA supplies:

Right atrium
yomna92009@yahoo.com - MRCS Part A - My account

Reference ranges End and review

0/3 Question 15-17 of 239 Question stats Score: 11.8%

Average score for registered users: 1

Theme: Nerve lesions 2


15 72.7%
3
16 69.9%
A. Intercostobrachial
17 53.2% 4
B. Median
5
C. Axillary
6
D. Radial
7
E. Ulnar
8
F. Musculocutaneous
9
G. Brachial plexus upper cord
10
H. Brachial plexus lower cord
11

Please select the most likely nerve injury for the scenarios given. Each option 12

may be used once, more than once or not at all. 13

14

15-17 0 / 3
15. A 23 year old rugby player sustains a Smiths Fracture. On
examination opposition of the thumb is markedly weakened.

You answered Brachial plexus lower cord

The correct answer is Median

This high velocity injury can often produce significant angulation


and displacement. Both of these may impair the function of the
median nerve with loss of function of the muscles of the thenar
eminence

16. A 45 year old lady recovering from a mastectomy and axillary node
clearance notices that sensation in her armpit is impaired.

You answered Brachial plexus lower cord

The correct answer is Intercostobrachial

The intercostobrachial nerves are frequently injured during axillary


dissection. These nerves traverse the axilla and supply cutaneous
sensation.

17. An 8 year old boy falls onto an outstretched hand and sustains a
supracondylar fracture. In addition to a weak radial pulse the child is
noted to have loss of pronation of the affected hand.

You answered Brachial plexus lower cord

The correct answer is Median

This is a common injury in children. In this case the angulation and


displacement have resulted in median nerve injury.

Next question

Brachial plexus

Origin Anterior rami of C5 to T1

Sections of the Roots, trunks, divisions, cords, branches


plexus Mnemonic:Real Teenagers Drink Cold Beer
yomna92009@yahoo.com - MRCS Part A - My account

Reference ranges End and review

Question 18 of 239 Question stats Score: 11.1%

1
A 57.7%
A 23 year old lady with sialolithiasis of the submandibular gland is undergoing 2
B 10%
excision of the gland. Which of the following nerves is at risk as the duct is 3
C 12.5%
mobilised?
D 13.8% 4
E 5.9% 5
A. Lingual nerve
57.7% of users answered this 6
B. Buccal nerve question correctly
7
C. Facial nerve
8
D. Glossopharyngeal
9
E. Vagus
10

Next question 11

12
The lingual nerve wraps around Whartons duct. The lingual nerve provides 13
sensory supply to the anterior 2/3 of the tongue.
14

15-17 0 / 3
Submandibular gland
18

Relations of the submandibular gland


Superficial Platysma, deep fascia and mandible
Submandibular lymph nodes
Facial vein (facial artery near mandible)
Marginal mandibular nerve
Cervical branch of the facial nerve

Deep Facial artery (inferior to the mandible)


Mylohoid muscle
Sub mandibular duct
Hyoglossus muscle
Lingual nerve
Submandibular ganglion
Hypoglossal nerve

Submandibular duct (Wharton's duct)

Opens lateral to the lingual frenulum on the anterior floor of mouth.


5 cm length
Lingual nerve wraps around Wharton's duct. As the duct passes forwards it
crosses medial to the nerve to lie above it and then crosses back, lateral to
it, to reach a position below the nerve.

Innervation

Sympathetic innervation- Superior Cervical ganglion via the Lingual nerve


Parasympathetic innervation- Submandibular ganglion

Arterial supply
Branch of the Facial artery. The facial artery passes through the gland to groove
its deep surface. It then emerges onto the face by passing between the gland and
the mandible.

Venous drainage
Anterior Facial vein (lies deep to the Marginal Mandibular nerve)

Lymphatic drainage
Deep cervical and jugular chains of nodes

Rate question: Next question

Com m ent on this question


yomna92009@yahoo.com - MRCS Part A - My account

Reference ranges End and review

Question 19 of 239 Question stats Score: 10.5%

1
A 44.4%
Which of the following is true in connection with the phrenic nerves? 2
B 21.5%
C 9.6% 3

A. They both lie anterior to the hilum of the lungs D 12.3% 4

B. They are derived from spinal roots C 2,3,4 E 12.1% 5

C. They pierce the diaphragm at the level of T7 44.4% of users answered this 6
question correctly
D. They consist of motor fibres only 7

E. None of the above 8

9
Next question
10

11

C3,4,5 12
Keeps the diaphragm alive!
13

14
They both lie anterior to the hilum of the lung. The phrenic nerves have both
15-17 0 / 3
motor and sensory functions. For this reason sub diaphragmatic pathology may
18
cause referred pain to the shoulder.
19

Phrenic nerve

Origin

C3,4,5

Supplies

Diaphragm, sensation central diaphragm and pericardium

Path

The phrenic nerve passes with the internal jugular vein across scalenus
anterior. It passes deep to prevertebral fascia of deep cervical fascia.
Left: crosses anterior to the 1st part of the subclavian artery.
Right: Anterior to scalenus anterior and crosses anterior to the 2nd part of
the subclavian artery.
On both sides, the phrenic nerve runs posterior to the subclavian vein and
posterior to the internal thoracic artery as it enters the thorax.

Right phrenic nerve

In the superior mediastinum: anterior to right vagus and laterally to superior


vena cava
Middle mediastinum: right of pericardium
It passes over the right atrium to exit the diaphragm at T8

Left phrenic nerve

Passes lateral to the left subclavian artery, aortic arch and left ventricle
Passes anterior to the root of the lung
Pierces the diaphragm alone

Image showing the passage of the phrenic nerve in the neck


yomna92009@yahoo.com - MRCS Part A - My account

Reference ranges End and review

Question 20 of 239 Question stats Score: 10%

1
A 53.7%
A 32 year old man presents with an inguinal hernia and undergoes an open 2
B 9.5%
surgical repair. The surgeons decide to place a mesh on the posterior wall of the 3
C 16.2%
inguinal canal to complete the repair, which of the following structures will lie
D 8% 4
posterior to the mesh?
E 12.6% 5

53.7% of users answered this 6


A. Transversalis fascia
question correctly
7
B. External oblique
8
C. Rectus abdominis
D. Obturator nerve 9

E. None of the above 10

11

Next question 12

13

Inguinal canal walls: 'MALT: 2M, 2A, 2L, 2T': 14

15-17 0 / 3
Starting from superior, moving around in order to posterior: 18
Superior wall (roof): 2 Muscles:Internal oblique, transversus abdominis
19
Anterior wall: 2 Aponeuroses: Aponeurosis of external oblique, Aponeurosis
of internal oblique 20

Lower wall (floor): 2 Ligaments: Inguinal Ligament, Lacunar Ligament


Posterior wall: 2Ts: Transversalis fascia, Conjoint Tendon

This is actually quite a straightforward question. It is simply asking for the


structure that forms the posterior wall of the inguinal canal. This is composed of
the transversalis fascia, the conjoint tendon and more laterally the deep inguinal
ring.

Inguinal canal

Location

Above the inguinal ligament


The inguinal canal is 4cm long

Boundaries of the inguinal canal

Floor External oblique aponeurosis


Inguinal ligament
Lacunar ligament

Roof Internal oblique


Transversus abdominis

Anterior wall External oblique aponeurosis

Posterior wall Transversalis fascia


Conjoint tendon

Laterally Internal ring


Fibres of internal oblique

Medially External ring


Conjoint tendon

Contents
Males Spermatic cord and ilioinguinal As it passes through the canal the spermatic
nerve cord has 3 coverings:

External spermatic fascia from external


oblique aponeurosis
Cremasteric fascia
Internal spermatic fascia

Females Round ligament of uterus and


ilioinguinal nerve

Related anatomy of the inguinal region


The boundaries of Hesselbachs triangle are commonly tested and illustrated
below:

Image sourced from Wikipedia

The image below demonstrates the close relationship of the vessels to the lower
limb with the inguinal canal. A fact to be borne in mind when repairing hernial
defects in this region.

Image sourced from Wikipedia

Rate question: Next question

Com m ent on this question


yomna92009@yahoo.com - MRCS Part A - My account

Reference ranges End and review

Question 1 of 219 Question stats Score: 100%

1
A 10.1%
A 22 year old man is involved in a fight and is stabbed in the posterior aspect of B 6.7%
his right leg. The knife passes into the popliteal fossa. He sustains an injury to his
C 8.8%
tibial nerve. Which of the following muscles is least likely to be compromised as a
D 15.1%
result?
E 59.4%

A. Tibialis posterior 59.4% of users answered this


question correctly
B. Flexor hallucis longus
C. Flexor digitorum brevis
D. Soleus
E. Peroneus tertius

Next question

Peroneus tertius is innervated by the deep peroneal nerve.

Tibial nerve

Begins at the upper border of the popliteal fossa and is a branch of the sciatic
nerve.

Root values: L4, L5, S1, S2, S3

Muscles innervated

Popliteus
Gastrocnemius
Soleus
Plantaris
Tibialis posterior
Flexor hallucis longus
Flexor digitorum brevis

Terminates by dividing into the medial and lateral plantar nerves.

Rate question: Next question

Com m ent on this question

All contents of this site are © 2012 E-Medical Revision Ltd Terms and Conditions Privacy policy
yomna92009@yahoo.com - MRCS Part A - My account

Reference ranges End and review

Question 2 of 219 Question stats Score: 50%

1
A 22.1%
Which of the following overlies the outer muscular layer of the intrathoracic 2
B 10.2%
oesophagus?
C 16.9%
D 38.9%

A. Serosa E 11.9%

B. Meissners plexus
38.9% of users answered this
C. Auerbach's plexus question correctly

D. Loose connective tissue


E. None of the above

Next question

The oesophagus has no serosal covering and hence holds sutures poorly. The
Auerbach's and Meissner's nerve plexuses lie in between the longitudinal and
circular muscle layers and submucosally. The sub mucosal location of the
Meissner's nerve plexus facilitates its sensory role.

Oesophagus

25cm long
Starts at C6 vertebra, pierces diaphragm at T10 and ends at T11
Squamous epithelium

Constrictions of the oesophagus


Structure Distance from incisors

Cricoid cartilage 15cm

Arch of the Aorta 22.5cm

Left principal bronchus 27cm

Diaphragmatic hiatus 40cm

Relations

Anteriorly Trachea to T4
Recurrent laryngeal nerve
Left bronchus, Left atrium
Diaphragm

Posteriorly Thoracic duct to left at T5


Hemiazygos to right T8
Descending aorta
First 2 intercostal branches of aorta

Left Thoracic duct


Left subclavian artery

Right Azygos vein

Arterial, venous and lymphatic drainage of the oesophagus


Artery Vein Lymphatics Muscularis externa

Upper Inferior Inferior thyroid Deep Striated muscle


third thyroid cervical

Mid third Aortic Azygos branches Mediastinal Smooth & striated


branches muscle

Lower Left gastric Posterior mediastinal and Gastric Smooth muscle


third coeliac
yomna92009@yahoo.com - MRCS Part A - My account

Reference ranges End and review

Question 3 of 219 Question stats Score: 33.3%

1
A 21%
Which nerve lies medially on the thyroid gland, in the groove between the 2
B 8.7%
oesophagus and trachea? 3
C 55.9%
D 7%

A. Vagus nerve E 7.4%

B. External laryngeal nerve


55.9% of users answered this
C. Recurrent laryngeal nerve question correctly

D. Ansa cervicalis
E. Phrenic nerve

Next question

The recurrent laryngeal nerve may be injured at this site during ligation of the
inferior thyroid artery.

Thyroid gland

Right and left lobes connected by isthmus


Surrounded by sheath from pretracheal layer of deep fascia
Apex: Lamina of thyroid cartilage
Base: 4th-5th tracheal ring
Pyramidal lobe: from isthmus
May be attached to foramen caecum at the base of the tongue

Relations

Anteromedially Sternothyroid
Superior belly of omohyoid
Sternohyoid
Anterior aspect of sternocleidomastoid

Posterolaterally Carotid sheath

Medially Larynx
Trachea
Pharynx
Oesophagus
Cricothyroid muscle
External laryngeal nerve (near superior thyroid artery)
Recurrent laryngeal nerve (near inferior thyroid artery)

Posterior Parathyroid glands


Anastomosis of superior and inferior thyroid arteries

Isthmus Anteriorly: Sternothyroids, sternohyoids, anterior jugular veins


Posteriorly: 2nd, 3rd, 4th tracheal rings (attached via Ligament of
Berry)

Blood Supply

Arterial Superior thyroid artery (1st branch of external carotid)


Inferior thyroid artery (from thyrocervical trunk)
Thyroidea ima (in 10% of population -from brachiocephalic artery or aorta)

Venous Superior and middle thyroid veins - into the IJV


Inferior thyroid vein - into the brachiocephalic veins

Rate question: Next question


yomna92009@yahoo.com - MRCS Part A - My account

Reference ranges End and review

Question 4 of 219 Question stats Score: 25%

1
A 5.7%
At which of the following spinal levels does the oesophagus pass through the 2
B 5.4%
diaphragm into the abdominal cavity? 3
C 72%
D 8.4% 4
A. L2 E 8.5%

B. L1
72% of users answered this
C. T10 question correctly

D. T5
E. T12

Next question

The oesophagus passes into the abdomen at T10.

Oesophagus

25cm long
Starts at C6 vertebra, pierces diaphragm at T10 and ends at T11
Squamous epithelium

Constrictions of the oesophagus


Structure Distance from incisors

Cricoid cartilage 15cm

Arch of the Aorta 22.5cm

Left principal bronchus 27cm

Diaphragmatic hiatus 40cm

Relations

Anteriorly Trachea to T4
Recurrent laryngeal nerve
Left bronchus, Left atrium
Diaphragm

Posteriorly Thoracic duct to left at T5


Hemiazygos to right T8
Descending aorta
First 2 intercostal branches of aorta

Left Thoracic duct


Left subclavian artery

Right Azygos vein

Arterial, venous and lymphatic drainage of the oesophagus


Artery Vein Lymphatics Muscularis externa

Upper Inferior Inferior thyroid Deep Striated muscle


third thyroid cervical

Mid third Aortic Azygos branches Mediastinal Smooth & striated


branches muscle

Lower Left gastric Posterior mediastinal and Gastric Smooth muscle


third coeliac

Nerve supply

Upper half is supplied by recurrent laryngeal nerve


yomna92009@yahoo.com - MRCS Part A - My account

Reference ranges End and review

Question 5 of 219 Question stats Score: 20%

1
A 12.4%
Which of the following nerve roots contribute nerve fibres to the ansa cervicalis? 2
B 50.7%
C 14.9% 3

A. C1 only D 10.8% 4

B. C1, C2 and C3 E 11.3% 5

C. C2, C3 and C6 50.7% of users answered this


question correctly
D. C2, C4 and C5
E. C4, C5 and C6

Next question

Ansa cervicalis muscles:

GHost THought SOmeone Stupid Shot Irene

GenioHyoid
ThyroidHyoid
Superior Omohyoid
SternoThyroid
SternoHyoid
Inferior Omohyoid

The ansa cervicalis is composed of a superior and inferior root, derived from C1,
C2 and C3. The superior root arises where the nerve crosses the internal carotid
artery. It descends anterior to the carotid sheath in the anterior triangle. It is
joined in the region of the mid neck by the inferior root. The inferior root may pass
either superficially or deep to the internal jugular vein.

Ansa cervicalis

Superior Branch of C1 anterolateral to carotid sheath


root

Inferior Derived from C2 and C3 roots, passes posterolateral to the internal jugular
root vein (may lie either deep or superficial to it)

Innervation Sternohyoid
Sternothyroid
Omohyoid

The ansa cervicalis lies anterior to the carotid sheath. The nerve supply to the
inferior strap muscles enters at their inferior aspect. Therefore when dividing
these muscles to expose a large goitre, the muscles should be divided in their
upper half.
yomna92009@yahoo.com - MRCS Part A - My account

Reference ranges End and review

Question 6 of 219 Question stats Score: 16.7%

1
A 7%
At which of the following anatomical locations does the common peroneal nerve 2
B 58.8%
bifurcate into the superficial and deep peroneal nerves? 3
C 10%
D 13.9% 4
A. Immediately anterior to the linea aspera E 10.3% 5
B. At the lateral aspect of the neck of the fibula 6
58.8% of users answered this
C. Within the substance of tibialis anterior muscle question correctly

D. At the inferomedial aspect of the popliteal fossa


E. Under the medial head of gastrocnemius

Next question

The common peroneal nerve bifurcates at the neck of the fibula (where it is most
likely to be injured).

Common peroneal nerve

Derived from the dorsal divisions of the sacral plexus (L4, L5, S1 and S2).

This nerve supplies the skin and fascia of the anterolateral surface of the leg and
the dorsum of the foot. It also innervates the muscles of the anterior and peroneal
compartments of the leg, extensor digitorum brevis as well as the knee, ankle and
foot joints.

It is laterally placed within the sciatic nerve. From the bifurcation of the sciatic
nerve it passes inferolaterally in the lateral and proximal part of the popliteal
fossa, under the cover of biceps femoris and its tendon. To reach the posterior
aspect of the fibular head. It ends by dividing into the deep and superficial
peroneal nerves at the point where it winds around the lateral surface of the neck
of the fibula in the body of peroneus longus, approximately 2cm distal to the apex
of the head of the fibula. It is palpable posterior to the head of the fibula.

Branches
In the thigh Nerve to the short head of biceps
Articular branch (knee)

In the popliteal fossa Lateral cutaneous nerve of the calf

Neck of fibula Superficial and deep peroneal nerves

Rate question: Next question

Com m ent on this question

All contents of this site are © 2012 E-Medical Revision Ltd Terms and Conditions Privacy policy
yomna92009@yahoo.com - MRCS Part A - My account

Reference ranges End and review

Question 7 of 219 Question stats Score: 14.3%

1
A 10.1%
A 48 year old motor cyclist sustains a complex lower limb fracture in a motor 2
B 53.7%
accident. For a time the popliteal artery is occluded and eventually repaired. 3
C 14.7%
Subsequently he develops a compartment syndrome and the anterior and
D 10.2% 4
superficial posterior compartments of the lower leg are decompressed.
E 11.4% 5
Unfortunately, the operating surgeon neglects to decompress the deep posterior
compartment. Which of the following muscles is least likely to be affected as a 6
53.7% of users answered this
result? question correctly
7

A. Flexor digitorum longus


B. Plantaris
C. Tibialis posterior
D. Flexor hallucis longus
E. None of the above

Next question

Muscles of the deep posterior compartment:

Tibialis posterior
Flexor hallucis longus
Flexor digitorum longus
Popliteus

The plantaris muscle lies within the superficial posterior compartment of the lower
leg.

Lower limb- Muscular compartments

Anterior compartment
Muscle Nerve Action

Tibialis anterior Deep peroneal Dorsiflexes ankle joint, inverts foot


nerve

Extensor digitorum Deep peroneal Extends lateral four toes, dorsiflexes ankle
longus nerve joint

Peroneus tertius Deep peroneal Dorsiflexes ankle, everts foot


nerve

Extensor hallucis Deep peroneal Dorsiflexes ankle joint, extends big toe
longus nerve

Peroneal compartment
Muscle Nerve Action

Peroneus longus Superficial peroneal nerve Everts foot, assists in plantar flexion

Peroneus brevis Superficial peroneal nerve Plantar flexes the ankle joint

Superficial posterior compartment


Nerve Action

Gastrocnemius Tibial nerve Plantar flexes the foot, may also flex the knee

Soleus Tibial nerve Plantar flexor

Deep posterior compartment


Muscle Nerve Action

Flexor digitorum longus Tibial Flexes the lateral four toes

Flexor hallucis longus Tibial Flexes the great toe


yomna92009@yahoo.com - MRCS Part A - My account

Reference ranges End and review

Question 8 of 219 Question stats Score: 25%

1
A 14.2%
A 23 year old lady is undergoing a trendelenberg procedure for varicose veins. 2
B 18.4%
During the dissection of the saphenofemoral junction, which of the following is 3
C 20.3%
most liable to injury?
D 14.9% 4
E 32.2% 5
A. Superficial circumflex iliac artery
32.2% of users answered this 6
B. Superficial circumflex iliac vein question correctly
7
C. Femoral artery
8
D. Femoral nerve
E. Deep external pudendal artery

Next question

Theme from September 2011 exam

The deep external pudendal artery runs under the long saphenous vein close to
its origin and may be injured. It is at greatest risk of injury during the flush ligation
of the saphenofemoral junction. Provided an injury is identified and vessel ligated,
injury is seldom associated with any serious adverse sequelae.

Saphenous vein

Long saphenous vein

This vein may be harvested for triple or quadruple bypass surgery

Originates at the 1st digit where the dorsal vein merges with the dorsal
venous arch of the foot
Passes anterior to the medial malleolus and runs up the medial side of the
leg
At the knee, it runs over the posterior border of the medial epicondyle of
the femur bone
Then passes laterally to lie on the anterior surface of the thigh before
entering an opening in the fascia lata called the saphenous opening
It joins with the femoral vein in the region of the femoral triangle at the
saphenofemoral junction

Tributaries

Medial marginal
Superficial epigastric
Superficial iliac circumflex
Superficial external pudendal veins

Short saphenous vein

Originates at the 5th digit where the dorsal vein merges with the dorsal
venous arch of the foot, which attaches to the great saphenous vein.
It passes around the lateral aspect of the foot (inferior and posterior to the
lateral malleolus) and runs along the posterior aspect of the leg (with the
sural nerve)
Passes between the heads of the gastrocnemius muscle, and drains into
the popliteal vein, approximately at or above the level of the knee joint.
yomna92009@yahoo.com - MRCS Part A - My account

Reference ranges End and review

Question 9 of 219 Question stats Score: 22.2%

1
A 7.5%
Considering the pituitary gland, which of the following is false? 2
B 18.2%
C 16.5% 3

A. The anterior pituitary secretes thyroid stimulating hormone D 35% 4

B. The anterior pituitary develops from Rathkes pouch E 22.9% 5

C. Patients with craniopharyngioma may develop bi temporal 35% of users answered this 6
hemianopia question correctly
7
D. The pituitary is in direct contact with the optic chiasm
8
E. The posterior pituitary secretes oxytocin via a positive feedback
9
loop

Next question

Although the optic chiasm is closely related to the pituitary, and


craniopharyngiomas may compress this structure leading to bitemporal
hemianopia, it is separated from the chiasm itself by a dural fold.

Pituitary Gland

The pituitary gland is located within the sella turcica within the sphenoid bone in
the middle cranial fossa. It is covered by a dural fold and weighs around 0.5g. It is
attached to the hypothalamus by the infundibulum. The anterior pituitary receives
hormonal stimuli from the hypothalamus by way of the hypothalamo-pituitary
portal system. It develops from a depression in the wall of the pharynx (Rathkes
pouch).

Anterior pituitary hormones

Growth hormone
Thyroid stimulating hormone
ACTH
Prolactin
LH and FSH
Melanocyte releasing hormone

Posterior pituitary hormones

Oxytocin
Anti diuretic hormone

Rate question: Next question

Com m ent on this question

All contents of this site are © 2012 E-Medical Revision Ltd Terms and Conditions Privacy policy
yomna92009@yahoo.com - MRCS Part A - My account

Reference ranges End and review

Question 10 of 219 Question stats Score: 20%

1
A 12%
A 24 year old man is involved in a fight and his face is cut with a knife. The wound 2
B 41.7%
lies immediately anterior to the tragus of the ear and extends anteriorly. The 3
C 10.4%
wound is surgically explored and the laceration is found to be mainly superficial. It
D 26.8% 4
is extends slightly more deeply immediately inferior to the main trunk of the facial
E 9.1% 5
nerve. Bleeding is observed, from which of the following is it most likely to
originate? 6
41.7% of users answered this
question correctly
7
A. External carotid artery 8
B. Retromandibular vein 9
C. Occipital artery 10
D. Maxillary artery
E. Ascending pharyngeal artery

Next question

The retromandibular vein lies slightly more deeply than the facial nerve in the
parotid gland. It is formed from the maxillary and superficial temporal vein.

Retromandibular vein

Formed by a union of the maxillary vein and superficial temporal vein


It descends through the parotid gland and bifurcates within it
The anterior division passes forwards to join the facial vein, the posterior
division is one of the tributaries of the external jugular vein

Rate question: Next question

Com m ent on this question

All contents of this site are © 2012 E-Medical Revision Ltd Terms and Conditions Privacy policy
yomna92009@yahoo.com - MRCS Part A - My account

Reference ranges End and review

Question 11 of 219 Question stats Score: 18.2%

1
A 74.5%
A 52 year female post hysterectomy attends clinic. She reports pain and reduced 2
B 5.5%
sensation over the medial aspect of her thigh. Clinically thigh adduction is weak. 3
C 7.9%
What is the most likely nerve injury?
D 5.5% 4
E 6.6% 5
A. Obturator nerve
74.5% of users answered this 6
B. Sciatic nerve question correctly
7
C. Femoral nerve
8
D. L3 cord compression
9
E. Deep peroneal nerve
10

Next question 11

The obturator nerve supplies sensation to the medial aspect of the thigh and
causes adduction and internal rotation of the thigh.
Injury occurs during pelvic or abdominal surgery.
L3 cord compression is unlikely.

Obturator nerve

The obturator nerve arises from L2, L3 and L4 by branches from the ventral
divisions of each of these nerve roots. L3 forms the main contribution and the
second lumbar branch is occasionally absent. These branches unite in the
substance of psoas major, descending vertically in its posterior part to emerge
from its medial border at the lateral margin of the sacrum. It then crosses the
sacroiliac joint to enter the lesser pelvis, it descends on obturator internus to
enter the obturator groove. In the lesser pelvis the nerve lies lateral to the internal
iliac vessels and ureter, and is joined by the obturator vessels lateral to the ovary
or ductus deferens.

Supplies

Medial compartment of thigh


Muscles supplied: external obturator, adductor longus, adductor brevis,
adductor magnus (not the lower part-sciatic nerve), gracilis
The cutaneous branch is often absent. When present, it passes between
gracilis and adductor longus near the middle part of the thigh, and supplies
the skin and fascia of the distal two thirds of the medial aspect.

Obturator canal

Connects the pelvis and thigh: contains the obturator artery, vein, nerve
which divides into anterior and posterior branches.

Cadaveric cross section demonstrating relationships of the obturator nerve


yomna92009@yahoo.com - MRCS Part A - My account

Reference ranges End and review

Question 12 of 219 Question stats Score: 16.7%

1
A 20.4%
An ENT surgeon is performing a radical neck dissection. She wishes to fully 2
B 54.9%
expose the external carotid artery. To do so she inserts a self retaining retractor 3
C 7.7%
close to its origin. Which of the following structures lies posterolaterally to the
D 7.1% 4
external carotid at this point?
E 9.9% 5

54.9% of users answered this 6


A. Superior thyroid artery
question correctly
7
B. Internal carotid artery
8
C. Lingual artery
D. Facial artery 9

E. None of the above 10

11

Next question 12

The internal carotid artery lies posterolaterally to the external carotid artery at
their origin from the common carotid. The superior thyroid, lingual and facial
arteries all arise from its anterior surface.

External carotid artery

The external carotid artery (ECA) is a branch of the common carotid artery.

Path

Upper border of thyroid cartilage to pass in between the angle of the


mandible and mastoid process
Initially anteromedial to the ICA, then becomes lateral
Lower ECA covered by sternomastoid, passed by hypoglossal nerve/lingual
& facial vein
Then deep to digastric and stylohyoid, eventually passes into the parotid
gland where it divides into the superficial temporal and maxillary branches

Image sourced from Wikipedia

Rate question: Next question


yomna92009@yahoo.com - MRCS Part A - My account

Reference ranges End and review

Question 13 of 219 Question stats Score: 15.4%

1
A 12.7%
In froment's test which muscle function is tested? 2
B 17%
C 10.4% 3

A. Flexor pollicis longus D 40.5% 4

B. Adductor pollicis longus E 19.4% 5

C. Abductor pollicis brevis 40.5% of users answered this 6


question correctly
D. Adductor pollicis 7

E. Opponens pollicis 8

9
Next question
10

11

12

Nerve signs 13

Froment's sign

Assess for ulnar nerve palsy


Adductor pollicis muscle function tested
Hold a piece of paper between their thumb and index finger. The object is
then pulled away. If ulnar nerve palsy, unable to hold the paper and will flex
the flexor pollicis longus to compensate (flexion of thumb at interphalangeal
joint).

Phalen's test

Assess carpal tunnel syndrome


More sensitive than Tinel's sign
Hold wrist in maximum flexion and the test is positive if there is numbness in
the median nerve distribution.

Tinel's sign

Assess for carpal tunnel syndrome


Tap the median nerve at the wrist and the test is positive if there is
tingling/electric-like sensations over the distribution of the median nerve.

Rate question: Next question

Com m ent on this question

All contents of this site are © 2012 E-Medical Revision Ltd Terms and Conditions Privacy policy
yomna92009@yahoo.com - MRCS Part A - My account

Reference ranges End and review

Question 14 of 219 Question stats Score: 14.3%

1
A 16.3%
A 22 year old man is involved in a fight outside a nightclub. He is stabbed in the 2
B 55.4%
back, on the left side, approximately 3cm below the 12th rib in the mid scapular 3
C 10.5%
line. The structure most likely to be injured first as a result is the:
D 12.9% 4
E 5% 5
A. Spleen
55.4% of users answered this 6
B. Left kidney question correctly
7
C. Left adrenal gland
8
D. Left ureter
9
E. None of the above
10

Next question 11

12
The left kidney lies in this location and is the most likely structure to be injured. 13
The Spleen lies more superiorly, and the left adrenal and ureter are unlikely to be
14
injured in isolation.

Levels

Transpyloric plane
Level of the body of L1

Pylorus stomach
Left kidney hilum (L1- left one!)
Right hilum of the kidney (1.5cm lower than the left)
Fundus of the gallbladder
Neck of pancreas
Duodenojejunal flexure
Superior mesenteric artery
Portal vein
Left and right colic flexure
Root of the transverse mesocolon
2nd part of the duodenum
Upper part of conus medullaris
Spleen

Can be identified by asking the supine patient to sit up without using their arms.
The plane is located where the lateral border of the rectus muscle crosses the
costal margin.

Anatomical planes
Subcostal plane Lowest margin of 10th costal cartilage

Intercristal plane Level of body L4 (highest point of iliac crest)

Intertubercular plane Level of body L5

Common level landmarks


Inferior mesenteric artery L3

Bifurcation of aorta into common iliac arteries L4

Formation of IVC L5 (union of common iliac veins)

Diaphragm apertures Vena cava T8


Oesophagus T10
Aortic hiatus T12

Rate question:
yomna92009@yahoo.com - MRCS Part A - My account

Reference ranges End and review

0/3 Question 15-17 of 219 Question stats Score: 11.8%

Average score for registered users: 1

Theme: Cutaneous innervation 2


15 84.5%
3
16 55.6%
A. Ulnar nerve
17 82.5% 4
B. Fifth cervical spinal segment
5
C. Radial nerve
6
D. Musculocutaneous nerve
7
E. Median nerve
8
F. None of these
9

Please select the source of innervation for the region described. Each option may 10

be used once, more than once or not at all. 11

12

13
15. The skin on the palmar aspect of the thumb
14

15-17 0 / 3
You answered None of these

The correct answer is Median nerve

The median nerve supplies cutaneous sensation to this region.


See diagram below

16. The nail bed of the index finger

You answered None of these

The correct answer is Median nerve

17. The skin overlying the medial aspect of the palm

You answered None of these

The correct answer is Ulnar nerve

This area is innervated by the ulnar nerve.

Next question

Brachial plexus

Origin Anterior rami of C5 to T1

Sections of the Roots, trunks, divisions, cords, branches


plexus Mnemonic:Real Teenagers Drink Cold Beer

Roots Located in the posterior triangle


Pass between scalenus anterior and medius

Trunks Located posterior to middle third of clavicle


Upper and middle trunks related superiorly to the subclavian
artery
Lower trunk passes over 1st rib posterior to the subclavian
artery

Divisions Apex of axilla

Cords Related to axillary artery


yomna92009@yahoo.com - MRCS Part A - My account

Reference ranges End and review

Question 18 of 219 Question stats Score: 11.1%

1
A 11.7%
With regard to the sciatic nerve which statement is false? 2
B 51.4%
C 7.5% 3

A. It is derived from L4 to S3 D 11.7% 4

B. It contains nerve roots from the posterior division of the lumbosacral E 17.7% 5
plexus only
51.4% of users answered this 6
C. It divides to give the tibial nerve and common peroneal nerve question correctly
7
D. It provides cutaneous sensation to the posterior aspect of the thigh
8
E. It provides cutaneous sensation to the entire lower leg with the
9
exception of its medial aspect
10

Next question 11

12
It is derived from both anterior and posterior divisions of the lumbosacral plexus.
13
The sciatic nerve is the longest and widest nerve in the human body. It is
particularly susceptible to trauma in the posterior approach to the hip. 14

15-17 0 / 3

Sciatic nerve 18

Origin Spinal nerves L4 - S3

Articular Branches Hip joint

Muscular branches in Semitendinosus


upper leg Semimembranosus
Biceps femoris
Part of adductor magnus

Cutaneous sensation Posterior aspect of thigh


Gluteal region
Entire lower leg (except the medial aspect)

Terminates At the upper part of the popliteal fossa by dividing into the tibial
and peroneal nerves

The nerve to the short head of the biceps femoris comes from the common
peroneal part of the sciatic and the other muscular branches arise from the
tibial portion.
The tibial nerve goes on to innervate all muscles of the foot except the
extensor digitorum brevis (which is innervated by the common peroneal
nerve).

Rate question: Next question

Com m ent on this question

All contents of this site are © 2012 E-Medical Revision Ltd Terms and Conditions Privacy policy
yomna92009@yahoo.com - MRCS Part A - My account

Reference ranges End and review

Question 19 of 219 Question stats Score: 10.5%

1
A 24.5%
A 28 year old man has a pleomorphic adenoma and the decision is made to 2
B 10.3%
resect this surgically. Which of the following structures is least likely to be 3
C 20.8%
encountered during surgical resection of the parotid gland?
D 33.6% 4
E 10.9% 5
A. External carotid artery
33.6% of users answered this 6
B. Retromandibular vein question correctly
7
C. Auriculotemporal nerve
8
D. Mandibular nerve
9
E. Zygomatic branch of the facial nerve
10

Next question 11

12

13
Structures passing through the parotid gland
14
Facial nerve and branches 15-17 0 / 3
External carotid artery (and its branches; the maxillary and superficial
18
temporal)
Retromandibular vein 19

Auriculotemporal nerve

The mandibular nerve is well separated from the parotid gland.


The maxillary vein joins to the superficial temporal vein and they form the
retromandibular vein which then runs through the parotid gland.
The auriculotemporal nerve runs through the gland. Following a parotidectomy
this nerve may be damaged and during neuronal regrowth may then attach to
sweat glands in this region. This can then cause gustatory sweating (Freys
Syndrome).

Parotid gland

Anatomy of the parotid gland


Location Overlying the mandibular ramus; anterior and inferior to the ear.

Salivary duct Crosses the masseter, pierces the buccinator and drains adjacent to
the 2nd upper molar tooth (Stensen's duct).

Structures Facial nerve (Mnemonic: The Zebra Buggered My Cat; Temporal


passing through Zygomatic, Buccal, Mandibular, Cervical)
the gland External carotid artery
Retromandibular vein
Auriculotemporal nerve

Relations Anterior: masseter, medial pterygoid, superficial temporal and


maxillary artery, facial nerve, stylomandibular ligament
Posterior: posterior belly digastric muscle,
sternocleidomastoid, stylohyoid, internal carotid artery, mastoid
process, styloid process

Arterial supply Branches of external carotid artery

Venous drainage Retromandibular vein

Lymphatic Deep cervical nodes


drainage

Nerve innervation Parasympathetic-Secretomotor


Sympathetic-Superior cervical ganglion
Sensory- Greater auricular nerve

Parasympathetic stimulation produces a water rich, serous saliva. Sympathetic


stimulation leads to the production of a low volume, enzyme-rich saliva.

Rate question: Next question

Com m ent on this question

All contents of this site are © 2012 E-Medical Revision Ltd Terms and Conditions Privacy policy
yomna92009@yahoo.com - MRCS Part A - My account

Reference ranges End and review

Question 20 of 219 Question stats Score: 15%

1
A 7.2%
During a radical thyroidectomy for carcinoma the surgeons inadvertently damage 2
B 20.1%
the ansa cervicalis nerve. Which of the following will be affected as a result? 3
C 6.5%
D 10.2% 4
A. Sternohyoid muscle E 56% 5
B. Sternothyroid muscle 6
56% of users answered this
C. Thyrohyoid muscle question correctly
7
D. Omohyoid muscle
8
E. All of the above
9

10
Next question
11

The ansa cervicalis is derived from nerve roots C1, C2, C3 and is a nerve loop 12
that accompanies the internal jugular vein. All the infrahyoid strap muscles are 13
innervated by the ansa cervicalis.
14

15-17 0 / 3
Anterior triangle of the neck
18

Boundaries 19

Anterior border of the Sternocleidomastoid 20


Lower border of mandible
Anterior midline

Sub triangles (divided by Digastric above and Omohyoid)

Muscular triangle: Neck strap muscles


Carotid triangle: Carotid sheath
Submandibular Triangle (digastric)

Contents of the anterior triangle


Digastric triangle Submandibular gland
Submandibular nodes
Facial vessels
Hypoglossal nerve

Muscular triangle Strap muscles


External jugular vein

Carotid triangle Carotid sheath (Common carotid, vagus and internal jugular vein)
Ansa cervicalis

Nerve supply to digastric muscle

Anterior: Mylohyoid nerve


Posterior: Facial nerve

Image sourced from Wikipedia


yomna92009@yahoo.com - MRCS Part A - My account

Reference ranges End and review

Question 21 of 219 Question stats Score: 19%

1
A 12.4%
A 23 year old man is undergoing an hernia repair and the mesh is to be sutured 2
B 9.7%
to the inguinal ligament. From which of the following does the inguinal ligament 3
C 12.2%
arise?
D 7% 4
E 58.8% 5
A. Transversus abdominis fascia
58.8% of users answered this 6
B. Internal oblique question correctly
7
C. Rectus sheath
8
D. Rectus abdominis muscle
9
E. External oblique aponeurosis
10

Next question 11

12
The inguinal ligament is formed by the external oblique aponeurosis. It runs from 13
the pubic tubercle to the anterior superior iliac spine.
14

15-17 0 / 3
Abdominal wall
18

The 2 main muscles of the abdominal wall are the rectus abdominis (anterior) 19
and the quadratus lumborum (posterior).
20
The remaining abdominal wall consists of 3 muscular layers. Each muscle passes
21
from the lateral aspect of the quadratus lumborum posteriorly to the lateral margin
of the rectus sheath anteriorly. Each layer is muscular posterolaterally and
aponeurotic anteriorly.

Image sourced from Wikipedia

Muscles of abdominal wall

External Lies most superficially

oblique Originates from 5th to 12th ribs


Inserts into the anterior half of the outer aspect of the iliac crest, linea
alba and pubic tubercle
More medially and superiorly to the arcuate line, the aponeurotic layer
overlaps the rectus abdominis muscle
The lower border forms the inguinal ligament
The triangular expansion of the medial end of the inguinal ligament is
the lacunar ligament.

Internal Arises from the thoracolumbar fascia, the anterior 2/3 of the iliac crest
oblique and the lateral 2/3 of the inguinal ligament
The muscle sweeps upwards to insert into the cartilages of the lower
3 ribs
The lower fibres form an aponeurosis that runs from the tenth costal
cartilage to the body of the pubis
At its lowermost aspect it joins the fibres of the aponeurosis of
transversus abdominis to form the conjoint tendon.

Transversus Innermost muscle


abdominis Arises from the inner aspect of the costal cartilages of the lower 6 ribs
, from the anterior 2/3 of the iliac crest and lateral 1/3 of the inguinal
ligament
Its fibres run horizontally around the abdominal wall ending in an
aponeurosis. The upper part runs posterior to the rectus abdominis.
Lower down the fibres run anteriorly only.
The rectus abdominis lies medially running from the pubic crest and
symphysis to insert into the xiphoid process and 5th, 6th and 7th
yomna92009@yahoo.com - MRCS Part A - My account

Reference ranges End and review

Question 22 of 219 Question stats Score: 18.2%

1
A 58.7%
A 56 year old man is undergoing a carotid endarterectomy. The internal carotid 2
B 9.1%
artery is mobilised. How many branches does this vessel give off in the neck? 3
C 8.5%
D 13.9% 4
A. 0 E 9.9% 5
B. 1 6
58.7% of users answered this
C. 2 question correctly
7
D. 3
8
E. 6
9

10
Next question
11

The internal carotid does not have any branches in the neck. 12

13
Internal carotid artery
14

15-17 0 / 3
The internal carotid artery is formed from the common carotid opposite the upper
18
border of the thyroid cartilage. It extends superiorly to enter the skull via the
carotid canal. From the carotid canal it then passes through the cavernous sinus, 19
above which it divides into the anterior and middle cerebral arteries.
20

21
Relations in the neck
22
Posterior Longus capitis
Pre-vertebral fascia
Sympathetic chain
Superior laryngeal nerve

Medially External carotid (near origin)


Wall of pharynx
Ascending pharyngeal artery

Laterally Internal jugular vein (moves posteriorly at entrance to skull)


Vagus nerve (most posterolaterally)

Anteriorly Sternocleidomastoid
Lingual and facial veins
Hypoglossal nerve

Relations in the carotid canal

Internal carotid plexus


Cochlea and middle ear cavity
Trigeminal ganglion (superiorly)
Leaves canal lies above the foramen lacerum

Path and relations in the cranial cavity


The artery bends sharply forwards in the cavernous sinus, the aducens nerve lies
close to its inferolateral aspect. The oculomotor, trochlear, opthalmic and, usually,
the maxillary nerves lie in the lateral wall of the sinus. Near the superior orbital
fissure it turns posteriorly and passes postero-medially to pierce the roof of the
cavernous sinus inferior to the optic nerve. It then passes between the optic and
oculomotor nerves to terminate below the anterior perforated substance by
dividing into the anterior and middle cerebral arteries.

Branches

Anterior and middle cerebral artery


Ophthalmic artery
yomna92009@yahoo.com - MRCS Part A - My account

Reference ranges End and review

Question 23 of 219 Question stats Score: 17.4%

1
A 43.1%
Which of the following is a content of the adductor canal? 2
B 10.1%
C 18.2% 3

A. Saphenous nerve D 14.1% 4

B. Sural nerve E 14.5% 5

C. Femoral nerve 43.1% of users answered this 6


question correctly
D. Profunda branch of the femoral artery 7

E. Saphenous vein 8

9
Next question
10

11
It contains the saphenous nerve and the superficial branch of the femoral artery.
12

Adductor canal 13

14

Also called Hunter's or subsartorial canal 15-17 0 / 3

18

19
Immediately distal to the apex of the femoral triangle, lying in the middle
20
third of the thigh. Canal terminates at the adductor hiatus.
21

22
Borders Contents
23
Laterally Vastus medialis muscle Saphenous nerve

Posteriorly Adductor longus, adductor magnus Superficial femoral artery

Roof Sartorius Superficial femoral vein

In the image below the sartorius muscle is removed to expose the canal contents
yomna92009@yahoo.com - MRCS Part A - My account

Reference ranges End and review

Question 24 of 219 Question stats Score: 16.7%

1
A 11.3%
A 56 year old man is undergoing a high anterior resection. Which of the following 2
B 61.5%
structures is at greatest risk of injury in this procedure? 3
C 10.9%
D 9% 4
A. Superior mesenteric artery E 7.4% 5
B. Left ureter 6
61.5% of users answered this
C. External iliac vein question correctly
7
D. External iliac artery
8
E. Inferior vena cava
9

10
Next question
11

A careless surgeon may damage all of these structures. However, the structure at 12
greatest risk and most frequently encountered is the left ureter. 13

14
Colon anatomy
15-17 0 / 3

18
The colon is about 1.5m long although this can vary considerably.
Components: 19

20
Ascending colon
Transverse colon 21
Descending colon
22
Sigmoid colon
23

24
Arterial supply
Superior mesenteric artery and inferior mesenteric artery: linked by the marginal
artery.
Ascending colon: ileocolic and right colic arteries
Transverse colon: middle colic artery
Descending and sigmoid colon: left colic artery

Venous drainage
From regional veins (that accompany arteries) to superior and inferior mesenteric
vein

Lymphatic drainage
Initially along nodal chains that accompany supplying arteries, then para-aortic
nodes.

Embryology
Midgut- Second part of duodenum to 2/3 transverse colon
Hindgut- Distal 1/3 transverse colon to anus

Peritoneal location
The right and left colon are part intraperitoneal and part extraperitoneal. The
sigmoid and transverse colon are generally wholly intraperitoneal. This has
implications for the sequelae of perforations, which will tend to result in
generalised peritonitis in the wholly intra peritoneal segments.

Rate question: Next question

Com m ent on this question

All contents of this site are © 2012 E-Medical Revision Ltd Terms and Conditions Privacy policy
yomna92009@yahoo.com - MRCS Part A - My account

Reference ranges End and review

Question 25 of 219 Question stats Score: 16%

1
A 10.4%
From which of the following foramina does the opthalmic branch of the trigeminal 2
B 9.4%
nerve exit the skull? 3
C 9.7%
D 65.2% 4
A. Foramen ovale E 5.3% 5
B. Foramen rotundum 6
65.2% of users answered this
C. Foramen spinosum question correctly
7
D. Superior orbital fissure
8
E. Foramen magnum
9

10
Next question
11

12
Mnemonic:
13

Standing Room Only -Exit of branches of trigeminal nerve from the skull 14

15-17 0 / 3
V1 -Superior orbital fissure 18
V2 -foramen Rotundum
19
V3 -foramen Ovale
20

21
The opthalmic branch of the trigeminal nerve exits the skull through the superior
orbital fissure. 22

23
Trigeminal nerve
24

25
The trigeminal nerve is the main sensory nerve of the head. In addition to its
major sensory role, it also innervates the muscles of mastication.

Distribution of the trigeminal nerve

Sensory Scalp
Face
Oral cavity (and teeth)
Nose and sinuses
Dura mater

Motor Muscles of mastication


Mylohyoid
Anterior belly of digastric
Tensor tympani
Tensor palati

Autonomic connections (ganglia) Ciliary


Sphenopalatine
Otic
Submandibular

Path

Originates at the pons


Sensory root forms the large, crescentic trigeminal ganglion within Meckel's
cave, and contains the cell bodies of incoming sensory nerve fibres. Here
the 3 branches exit.
The motor root cell bodies are in the pons and the motor fibres are
distributed via the mandibular nerve. The motor root is not part of the
trigeminal ganglion.
yomna92009@yahoo.com - MRCS Part A - My account

Reference ranges End and review

Question 26 of 219 Question stats Score: 15.4%

1
A 11.9%
A 56 year old lady with metastatic breast cancer develops an oestolytic deposit in 2
B 46.2%
the proximal femur. One morning whilst getting out of bed she notices severe 3
C 15.9%
groin pain. X-rays show that the lesser trochanter has been avulsed. Which
D 11.2% 4
muscle is the most likely culprit?
E 14.8% 5

46.2% of users answered this 6


A. Vastus lateralis
question correctly
7
B. Psoas major
8
C. Piriformis
D. Gluteus maximus 9

E. Gluteus medius 10

11

Next question 12

13
The psoas major inserts into the lesser trochanter and contracts when raising the
14
trunk from the supine position. When oestolytic lesions are present in the femur
the lesser trochanter may be avulsed. 15-17 0 / 3

18

Psoas Muscle 19

20
Origin
21
The deep part originates from the transverse processes of the five lumbar
vertebrae, the superficial part originates from T12 and the first 4 lumbar 22
vertebrae. 23

24
Insertion
Lesser trochanter of the femur. 25

26
Innervation
Anterior rami of L1 to L3.

Action
Flexion and external rotation of the hip. Bilateral contraction can raise the trunk
from the supine position.

Rate question: Next question

Com m ent on this question

All contents of this site are © 2012 E-Medical Revision Ltd Terms and Conditions Privacy policy
yomna92009@yahoo.com - MRCS Part A - My account

Reference ranges End and review

Question 27 of 219 Question stats Score: 14.8%

1
A 13.8%
Which of the following nerves is responsible for the motor innervation of the 2
B 63.3%
sternocleidomastoid muscle? 3
C 8.8%
D 7.5% 4
A. Ansa cervicalis E 6.7% 5
B. Accessory nerve 6
63.3% of users answered this
C. Hypoglossal nerve question correctly
7
D. Facial nerve
8
E. Vagus nerve
9

10
Next question
11

The motor supply to the sternocleidomastoid is from the accessory nerve. The 12
ansa cervicalis supplies sensory information from the muscle. 13

14
Sternocleidomastoid
15-17 0 / 3

18
Anatomy
Origin Rounded tendon attached to upper manubrium sterni and muscular head 19
attached to medial third of the clavicle
20
Insertion Mastoid process of the temporal bone and lateral area of the superior nuchal
21
line of the occipital bone

Innervation Spinal part of accessory nerve and anterior rami of C2 and C3 (proprioception) 22

23
Action Both: extend the head at atlanto-occipital joint and flex the cervical
vertebral column. Accessory muscles of inspiration. 24
Single: lateral flexion of neck, rotates head so face looks upward to the
25
opposite side
26

27
Sternocleidomastoid divides the anterior and posterior triangles of the neck.

Rate question: Next question

Com m ent on this question

All contents of this site are © 2012 E-Medical Revision Ltd Terms and Conditions Privacy policy
yomna92009@yahoo.com - MRCS Part A - My account

Reference ranges End and review

Question 28 of 219 Question stats Score: 14.3%

1
A 20.1%
A 42 year old lady undergoes a difficult cholecystectomy and significant bleeding 2
B 6.1%
is occurring. The surgeons place a vascular clamp transversely across the 3
C 10.5%
anterior border of the epiploic foramen. Which of the following structures will be
D 53.6% 4
occluded in this manoeuvre?
E 9.7% 5

53.6% of users answered this 6


A. Cystic artery
question correctly
7
B. Cystic duct
8
C. Left gastric artery
D. Portal vein 9

E. None of the above 10

11

Next question 12

13
The portal vein, hepatic artery and common bile duct are occluded.
14

15-17 0 / 3
Epiploic Foramen
18

The epiploic foramen has the following boundaries: 19


Anteriorly (in the free edge of the Bile duct to the right, portal vein behind and hepatic 20
lesser omentum) artery to the left.
21
Posteriorly Inferior vena cava
22
Inferiorly 1st part of the duodenum
23
Superiorly Caudate process of the liver
24

During liver surgery bleeding may be controlled using a Pringles manoeuvre, this 25
involves placing a vascular clamp across the anterior aspect of the epiploic
26
foramen. Thereby occluding:
27
Common bile duct
28
Hepatic artery
Portal vein

Rate question: Next question

Com m ent on this question

All contents of this site are © 2012 E-Medical Revision Ltd Terms and Conditions Privacy policy
yomna92009@yahoo.com - MRCS Part A - My account

Reference ranges End and review

Question 29 of 219 Question stats Score: 17.2%

1
A 6.7%
A 34 year old man is injured by farm machinery and sustains a laceration at the 2
B 12%
superolateral aspect of the popliteal fossa. The medial aspect of biceps femoris is 3
C 10%
lacerated. Which of the following underlying structures is at greatest risk of injury?
D 12.8% 4
E 58.5% 5
A. Gracilis
58.5% of users answered this 6
B. Sural nerve question correctly
7
C. Nerve to semimembranosus
8
D. Popliteal artery
9
E. Common peroneal nerve
10

Next question 11

12
The common peroneal nerve lies under the medial aspect of biceps femoris and 13
is therefore at greatest risk of injury. The tibial nerve may also be damaged in
14
such an injury (but is not listed here). The sural nerve branches off more
inferiorly. 15-17 0 / 3

18

Common peroneal nerve 19

20
Derived from the dorsal divisions of the sacral plexus (L4, L5, S1 and S2).
21

This nerve supplies the skin and fascia of the anterolateral surface of the leg and 22
the dorsum of the foot. It also innervates the muscles of the anterior and peroneal 23
compartments of the leg, extensor digitorum brevis as well as the knee, ankle and
24
foot joints.
25
It is laterally placed within the sciatic nerve. From the bifurcation of the sciatic 26
nerve it passes inferolaterally in the lateral and proximal part of the popliteal
27
fossa, under the cover of biceps femoris and its tendon. To reach the posterior
aspect of the fibular head. It ends by dividing into the deep and superficial 28
peroneal nerves at the point where it winds around the lateral surface of the neck
29
of the fibula in the body of peroneus longus, approximately 2cm distal to the apex
of the head of the fibula. It is palpable posterior to the head of the fibula.

Branches
In the thigh Nerve to the short head of biceps
Articular branch (knee)

In the popliteal fossa Lateral cutaneous nerve of the calf

Neck of fibula Superficial and deep peroneal nerves

Rate question: Next question

Com m ent on this question

All contents of this site are © 2012 E-Medical Revision Ltd Terms and Conditions Privacy policy
yomna92009@yahoo.com - MRCS Part A - My account

Reference ranges End and review

Question 30 of 219 Question stats Score: 16.7%

1
A 5.6%
A 56 year old lady undergoes a Hartmans style resection of the sigmoid colon, 2
B 7.3%
with ligation of the vessels close to the colon. Which of the following vessels will 3
C 57%
be responsible to supplying the rectal stump directly?
D 20.7% 4
E 9.4% 5
A. Superior mesenteric artery
57% of users answered this 6
B. Middle colic artery question correctly
7
C. Superior rectal artery
8
D. Inferior mesenteric artery
9
E. External iliac artery
10

Next question 11

12
This question is addressing the blood supply to the rectum. Which is supplied by 13
the superior rectal artery. High ligation of the IMA may compromise this structure.
14
However, the question states that during the Hartmans procedure the vessels
were ligated close to the bowel. Implying that the superior rectal was preserved. 15-17 0 / 3

18

Rectum 19

20
The rectum is approximately 12 cm long. It is a capacitance organ. It has both
21
intra and extraperitoneal components. The transition between the sigmoid colon is
marked by the disappearance of the tenia coli.The extra peritoneal rectum is 22
surrounded by mesorectal fat that also contains lymph nodes. This mesorectal 23
fatty layer is removed surgically during rectal cancer surgery (Total Mesorectal
24
Excision). The fascial layers that surround the rectum are important clinical
landmarks, anteriorly lies the fascia of Denonvilliers. Posteriorly lies Waldeyers 25
fascia. 26

27
Extra peritoneal rectum
28
Posterior upper third
29
Posterior and lateral middle third
Whole lower third 30

Relations
Anteriorly (Males) Rectovesical pouch
Bladder
Prostate
Seminal vesicles

Anteriorly (Females) Recto-uterine pouch (Douglas)


Cervix
Vaginal wall

Posteriorly Sacrum
Coccyx
Middle sacral artery

Laterally Levator ani


Coccygeus

Arterial supply
Superior rectal artery

Venous drainage
Superior rectal vein

Lymphatic drainage

Mesorectal lymph nodes (superior to dentate line)


Internal iliac and then para-aortic nodes
Inguinal nodes (inferior to dentate line)
yomna92009@yahoo.com - MRCS Part A - My account

Reference ranges End and review

Question 1 of 189 Question stats Score: 0%

1
A 64.5%
A laceration to the upper lateral margin of the popliteal fossa may injure which of B 7.1%
the following nerves?
C 8.3%
D 6.7%

A. Common peroneal nerve E 13.4%

B. Sural nerve
64.5% of users answered this
C. Sciatic nerve question correctly

D. Saphenous nerve
E. Tibial nerve

Next question

The sural nerve exits at the lower latero-medial aspect of the fossa and is more at
risk in short saphenous vein surgery. The tibial nerve lies more medially and is
even less likely to be injured in this location.

Popliteal fossa

Boundaries of the popliteal fossa


Laterally Biceps femoris above, lateral head of gastrocnemius and plantaris below

Medially Semimembranosus and semitendinosus above, medial head of gastrocnemius


below

Floor Popliteal surface of the femur, posterior ligament of knee joint and popliteus
muscle

Roof Superficial and deep fascia

Image showing the popliteal fossa

Image sourced from Wikipedia

Contents

Popliteal artery and vein


Small saphenous vein
Common peroneal nerve
Tibial nerve
Posterior cutaneous nerve of the thigh
Genicular branch of the obturator nerve
Lymph nodes
yomna92009@yahoo.com - MRCS Part A - My account

Reference ranges End and review

Question 2 of 189 Question stats Score: 0%

1
A 7.6%
Which option is false in relation to the trigeminal nerve? 2
B 49%
C 7%

A. The nerve originates at the pons D 16.9%

B. The posterior scalp is supplied by the trigeminal nerve E 19.5%

C. The maxillary nerve exits via the foramen rotundum 49% of users answered this
question correctly
D. The maxillary nerve is purely sensory
E. The motor root is not in the trigeminal ganglion

Next question

The posterior scalp is supplied by C2-C3.

Trigeminal nerve

The trigeminal nerve is the main sensory nerve of the head. In addition to its
major sensory role, it also innervates the muscles of mastication.

Distribution of the trigeminal nerve

Sensory Scalp
Face
Oral cavity (and teeth)
Nose and sinuses
Dura mater

Motor Muscles of mastication


Mylohyoid
Anterior belly of digastric
Tensor tympani
Tensor palati

Autonomic connections (ganglia) Ciliary


Sphenopalatine
Otic
Submandibular

Path

Originates at the pons


Sensory root forms the large, crescentic trigeminal ganglion within Meckel's
cave, and contains the cell bodies of incoming sensory nerve fibres. Here
the 3 branches exit.
The motor root cell bodies are in the pons and the motor fibres are
distributed via the mandibular nerve. The motor root is not part of the
trigeminal ganglion.

Branches of the trigeminal nerve


Ophthalmic nerve Sensory only

Maxillary nerve Sensory only

Mandibular nerve Sensory and motor

Sensory
Ophthalmic Exits skull via the superior orbital fissure
Sensation of: scalp and forehead, the upper eyelid, the conjunctiva and cornea
of the eye, the nose (including the tip of the nose, except alae nasi), the nasal
mucosa, the frontal sinuses, and parts of the meninges (the dura and blood
vessels).

Maxillary Exit skull via the foramen rotundum


yomna92009@yahoo.com - MRCS Part A - My account

Reference ranges End and review

Question 3 of 189 Question stats Score: 0%

1
A 76.6%
A 45 year old man is undergoing a lymph node biopsy from the posterior triangle 2
B 5.6%
of his neck. Which of the following structures forms the posterior border of this 3
C 5.3%
structure?
D 5.2%

E 7.2%
A. Trapezius muscle
76.6% of users answered this
B. Diagastric muscle question correctly
C. External jugular vein
D. Omohyoid muscle
E. Sternocleidomastoid muscle

Next question

The borders are described below.

Posterior triangle of the neck

Boundaries
Apex Sternocleidomastoid and the Trapezius muscles at the Occipital bone

Anterior Posterior border of the Sternocleidomastoid

Posterior Anterior border of the Trapezius

Base Middle third of the clavicle

Image sourced from Wikipedia

Contents

Nerves Accessory nerve


Phrenic nerve
Three trunks of the brachial plexus
Branches of the cervical plexus: Supraclavicular nerve, transverse cervical
nerve, great auricular nerve, lesser occipital nerve
yomna92009@yahoo.com - MRCS Part A - My account

Reference ranges End and review

Question 4 of 189 Question stats Score: 0%

1
A 7.5%
On inspecting the caecum, which of the following structures is most likely to be 2
B 68%
identified at the point at which all the tenia coli converge? 3
C 6%
D 11.7% 4
A. Gonadal vessels E 6.8%

B. Appendix base
68% of users answered this
C. Appendix tip question correctly

D. Ileocaecal valve
E. Ileocolic artery

Next question

The tenia coli converge at the base of the appendix.

Caecum

Location Proximal right colon below the ileocaecal valve


Intraperitoneal

Posterior relations Psoas


Iliacus
Femoral nerve
Genitofemoral nerve
Gonadal vessels

Anterior relations Greater omentum

Arterial supply Ileocolic artery

Lymphatic drainage Mesenteric nodes accompany the venous drainage

The caecum is the most distensible part of the colon and in complete large
bowel obstruction with a competent ileocaecal valve the most likely site of
eventual perforation.

Rate question: Next question

Com m ent on this question

All contents of this site are © 2012 E-Medical Revision Ltd Terms and Conditions Privacy policy
yomna92009@yahoo.com - MRCS Part A - My account

Reference ranges End and review

Question 5 of 189 Question stats Score: 0%

1
A 8%
A 42 year old lady has had an axillary node clearance for breast malignancy. Post 2
B 6.6%
operatively she reports weakness of the shoulder. She is unable to push herself 3
C 6%
forwards from a wall with the right arm and the scapula is pushed out medially
D 72.1% 4
from the chest wall. What is the most likely nerve injury?
E 7.2% 5

A. C5, C6 72.1% of users answered this


question correctly
B. C8, T1
C. Axillary nerve
D. Long thoracic nerve
E. Spinal accessory nerve

Next question

Theme from January 2012 and 2009 Exam

The patient has a winged scapula caused by damage to the long thoracic nerve
(C5,6,7) during surgery. The long thoracic nerve innervates serratus anterior.
Serratus anterior causes pushing out of the scapula during a punch.

NB winging of the scapular laterally may indicate trapezius muscle weakness.


Innervated by the spinal accessory nerve.

Long thoracic nerve

Derived from ventral rami of C5, C6, and C7 (close to their emergence from
intervertebral foramina)
It runs downward and passes either anterior or posterior to the middle
scalene muscle
It reaches upper tip of serratus anterior muscle and descends on outer
surface of this muscle, giving branches into it
Winging of Scapula occurs in long thoracic nerve injury (most common) or
from spinal accessory nerve injury (which denervates the trapezius) or a
dorsal scapular nerve injury

Rate question: Next question

Com m ent on this question

All contents of this site are © 2012 E-Medical Revision Ltd Terms and Conditions Privacy policy
yomna92009@yahoo.com - MRCS Part A - My account

Reference ranges End and review

Question 6 of 189 Question stats Score: 16.7%

1
A 6%
A 36 year old male is admitted for elective surgery for a lymph node biopsy in the 2
B 11.1%
supraclavicular region. Post operatively the patient has difficulty shrugging his left 3
C 11.2%
shoulder. What nerve has been damaged?
D 5.7% 4
E 66% 5
A. Phrenic nerve
66% of users answered this 6
B. Axillary nerve question correctly
C. C5, C6 lesion
D. C8, T1 lesion
E. Accessory nerve

Next question

Theme from September 2011 Exam

The accessory nerve lies in the posterior triangle and may be injured in this
region. Apart from problems with shrugging the shoulder, he may also have
difficulty lifting his arm above his head.

Posterior triangle of the neck

Boundaries
Apex Sternocleidomastoid and the Trapezius muscles at the Occipital bone

Anterior Posterior border of the Sternocleidomastoid

Posterior Anterior border of the Trapezius

Base Middle third of the clavicle

Image sourced from Wikipedia

Contents

Nerves Accessory nerve


Phrenic nerve
Three trunks of the brachial plexus
Branches of the cervical plexus: Supraclavicular nerve, transverse cervical
nerve, great auricular nerve, lesser occipital nerve

Vessels External jugular vein


Subclavian artery
yomna92009@yahoo.com - MRCS Part A - My account

Reference ranges End and review

Question 7 of 189 Question stats Score: 14.3%

1
A 32%
A 78 year old man is undergoing a segmental resection of the superior lobe of the 2
B 30.4%
right lung for a non small cell lung cancer. As the surgeons divide the bronchus a 3
C 15.9%
vessel lying immediately inferiorly is bleeding. Which vessel is it likely to be?
D 11.1% 4
E 10.7% 5
A. Pulmonary vein
30.4% of users answered this 6
B. Pulmonary artery question correctly
7
C. Brachiocephalic vein
D. Superior vena cava
E. None of the above

Next question

The right superior lobe bronchus is the most superior of all the right bronchi. It
lies immediately above the pulmonary artery. The azygos vein is also closely
related.

Lung anatomy

The right lung is composed of 3 lobes divided by the oblique and transverse
fissures. The left lung has two lobes divided by the lingula.The apex of both lungs
is approximately 4cm superior to the sterno-costal joint of the first rib. Immediately
below this is a sulcus created by the subclavian artery.

Peripheral contact points of the lung

Base: diaphragm
Costal surface: corresponds to the cavity of the chest
Mediastinal surface: Contacts the mediastinal pleura. Has the cardiac
impression. Above and behind this concavity is a triangular depression
named the hilum, where the structures which form the root of the lung enter
and leave the viscus. These structures are invested by pleura, which,
below the hilus and behind the pericardial impression, forms the pulmonary
ligament

Right lung
Above the hilum is the azygos vein; Superior to this is the groove for the superior
vena cava and right innominate vein; behind this, and nearer the apex, is a furrow
for the innominate artery. Behind the hilus and the attachment of the pulmonary
ligament is a vertical groove for the oesophagus; In front and to the right of the
lower part of the oesophageal groove is a deep concavity for the extrapericardiac
portion of the inferior vena cava.

The root of the right lung lies behind the superior vena cava and the right atrium,
and below the azygos vein.

The right main bronchus is shorter, wider and more vertical than the left main
bronchus and therefore the route taken by most foreign bodies.

Left lung
Above the hilum is the furrow produced by the aortic arch, and then superiorly the
groove accommodating the left subclavian artery; Behind the hilum and
pulmonary ligament is a vertical groove produced by the descending aorta, and in
front of this, near the base of the lung, is the lower part of the oesophagus.

The root of the left lung passes under the aortic arch and in front of the
descending aorta.

Inferior borders of both lungs


yomna92009@yahoo.com - MRCS Part A - My account

Reference ranges End and review

Question 1 of 182 Question stats Score: 0%

1
A 72.2%
Which of the following muscles is supplied by the musculocutaneous nerve? B 7.2%
C 6.3%

A. Brachialis D 7.1%

B. Latissimus dorsi E 7.2%

C. Flexor carpi ulnaris 72.2% of users answered this


question correctly
D. Teres minor
E. Triceps

Next question

Mnemonic

Muscles innervated by the musculocutaneous nerve BBC:

Biceps brachii
Brachialis
Coracobrachialis

Musculocutaneous nerve

Branch of lateral cord of brachial plexus

Path

It penetrates the Coracobrachialis muscle


Passes obliquely between the Biceps brachii and the Brachialis to the
lateral side of the arm
Above the elbow it pierces the deep fascia lateral to the tendon of the
Biceps brachii
Continues into the forearm as the lateral cutaneous nerve of the forearm

Innervates

Coracobrachialis
Biceps brachii
Brachialis

Rate question: Next question

Com m ent on this question

All contents of this site are © 2012 E-Medical Revision Ltd Terms and Conditions Privacy policy
yomna92009@yahoo.com - MRCS Part A - My account

Reference ranges End and review

Question 2 of 182 Question stats Score: 0%

1
A 5.7%
Which of the following statements relating to the posterior cerebral artery is false? 2
B 14.8%
C 10.7%

A. It supplies the visual cortex D 57.8%

B. It is closely related to the 3rd cranial nerve E 11.1%

C. It is a branch of the basilar artery 57.8% of users answered this


question correctly
D. It is connected to the circle of Willis via the superior cerebellar artery
E. When occluded may result in contralateral loss of field of vision

Next question

The posterior cerebral arteries are formed by the bifurcation of the basilar artery
and is connected to the circle of Willis via the posterior communicating artery.

The posterior cerebral arteries supply the occipital lobe and part of the temporal
lobe.

Circle of Willis

May also be called the circulus arteriosus

Inferior surface of brain


Supplied by the internal carotid arteries and the vertebral arteries
If artery is occluded, collaterals may be able to compensate
Components include:

1. Anterior communicating arteries


2. Anterior cerebral arteries
3. Internal carotid arteries
4. Posterior communicating arteries
5. Posterior cerebral arteries and the termination of the basilar artery

Supply: Corpus striatum, internal capsule, diencephalon, midbrain


yomna92009@yahoo.com - MRCS Part A - My account

Reference ranges End and review

Question 3 of 182 Question stats Score: 0%

1
A 62.6%
How many valves lie between the superior vena cava and the right atrium? 2
B 12.4%
C 7.8% 3

A. None D 8.2%

B. One E 9%

C. Two 62.6% of users answered this


question correctly
D. Three
E. Four

Next question

There are no valves which is why it is relatively easy to insert a CVP line from the
internal jugular vein into the right atrium.

Superior vena cava

Drainage

Head and neck


Upper limbs
Thorax
Part of abdominal walls

Formation

Subclavian and internal jugular veins unite to form the right and left
brachiocephalic veins
These unite to form the SVC
Azygos vein joins the SVC before it enters the right atrium

Rate question: Next question

Com m ent on this question

All contents of this site are © 2012 E-Medical Revision Ltd Terms and Conditions Privacy policy
yomna92009@yahoo.com - MRCS Part A - My account

Reference ranges End and review

Question 4 of 182 Question stats Score: 0%

1
A 9.6%
An elderly lady falls and lands on her hip. On examination her hip is tender to 2
B 18.6%
palpation and x-rays are taken. There are concerns that she may have an 3
C 15.2%
intertrochanteric fracture. What is the normal angle between the femoral neck and
D 47.2% 4
the femoral shaft?
E 9.3%

A. 90o 47.2% of users answered this


question correctly
B. 105o

C. 80o

D. 130o

E. 180o

Next question

The normal angle between the femoral head and shaft is 130o. Changes to this
angle may occur as a result of disease or pathology and should be investigated.

Hip joint

Head of femur articulates with acetabulum of the pelvis


Both covered by articular hyaline cartilage
The acetabulum forms at the union of the ilium, pubis, and ischium
The triradiate cartilage (Y-shaped growth plate) separates the pelvic bones
The acetabulum holds the femoral head by the acetabular labrum
Normal angle between femoral head and femoral shaft is 130o

Ligaments

Transverse ligament: joints anterior and posterior ends of the articular


cartilage
Head of femur ligament (ligamentum teres): acetabular notch to the fovea.
Contains arterial supply to head of femur in children.

Image sourced from Wikipedia


yomna92009@yahoo.com - MRCS Part A - My account

Reference ranges End and review

Question 5 of 182 Question stats Score: 0%

1
A 6.7%
A 17 year old male presents to the clinic. He complains of difficulty using his left 2
B 5%
hand. It has been a persistent problem since he sustained a distal humerus 3
C 68.8%
fracture as a child. On examination there is diminished sensation overlying the
D 14.1% 4
hypothenar eminence and medial one and half fingers. What is the most likely
E 5.4% 5
nerve lesion?

68.8% of users answered this


question correctly
A. Anterior interosseous nerve
B. Posterior interosseous nerve
C. Ulnar nerve
D. Median nerve
E. Radial nerve

Next question

Theme from April 2012 Exam


This sensory deficit pattern is most consistent with ulnar nerve injury.
yomna92009@yahoo.com - MRCS Part A - My account

Reference ranges End and review

Question 6 of 182 Question stats Score: 0%

1
A 18%
An 18 year old athlete attends orthopaedic clinic reporting pain and swelling over 2
B 15.2%
the medial aspect of the knee joint. The pain occurs when climbing the stairs, but 3
C 28.8%
is not present when walking on flat ground. Clinically there is pain over the medial,
D 33% 4
proximal tibia and the McMurray test is negative. What is the most likely cause of
E 5.1% 5
this patient's symptoms?

33% of users answered this 6


question correctly
A. Anterior cruciate ligament tear
B. Prepatellar bursitis
C. Medial meniscus injury
D. Pes Anserinus Bursitis
E. Fracture of tibia

Next question

Pes anserinus: GOOSE'S FOOT

Combination of sartorius, gracilis and semitendinous tendons inserting into


the anteromedial proximal tibia.

Pes Anserinus Bursitis is common in sportsmen due to overuse injuries. The main
sign is of pain in the medial proximal tibia. As the McMurray test is negative,
medial meniscal injury is excluded.

Sartorius

Longest strap muscle in the body


Most superficial muscle in the anterior compartment of the thigh

Origin Anterior superior iliac spine

Insertion Medial surface of the of the body of the tibia (upper part). It inserts anterior to
gracilis and semitendinosus

Nerve Femoral nerve (L2,3)


Supply

Action Flexor of the hip and knee, slight abducts the thigh and rotates it laterally
It assists with medial rotation of the tibia on the femur. For example it
would play a pivotal role in placing the right heel onto the left knee ( and
vice versa)

Important The middle third of this muscle, and its strong underlying fascia forms the roof
relations of the adductor canal , in which lie the femoral vessels, the saphenous nerve
and the nerve to vastus medialis.

Rate question: Next question

Com m ent on this question

All contents of this site are © 2012 E-Medical Revision Ltd Terms and Conditions Privacy policy
yomna92009@yahoo.com - MRCS Part A - My account

Reference ranges End and review

Question 7 of 182 Question stats Score: 0%

1
A 7.7%
Which of the following structures lies most posteriorly at the porta hepatis? 2
B 11.8%
C 9.9% 3

A. Cystic artery D 58.9% 4

B. Common hepatic artery E 11.7% 5

C. Left hepatic artery 58.9% of users answered this 6


question correctly
D. Portal vein 7

E. Common bile duct

Next question

The portal vein is the most posterior structure at the porta hepatis.The common
bile duct is a continuation of the common hepatic duct and is formed by the union
of the common hepatic duct and the cystic duct.

Liver

Structure of the liver

Right lobe Supplied by right hepatic artery


Contains Couinard segments V to VIII (-/+Sg I)

Left lobe Supplied by the left hepatic artery


Contains Couinard segments II to IV (+/- Sg1)

Quadrate Part of the right lobe anatomically, functionally is part of the left
lobe Couinard segment IV
Porta hepatis lies behind
On the right lies the gallbladder fossa
On the left lies the fossa for the umbilical vein

Caudate lobe Supplied by both right and left hepatic arteries


Couinard segment I
Lies behind the plane of the porta hepatis
Anterior and lateral to the inferior vena cava
Bile from the caudate lobe drains into both right and left hepatic
ducts

Detailed knowledge of Couinard segments is not required for MRCS Part A

Between the liver lobules are portal canals which contain the portal triad:
Hepatic Artery, Portal Vein, tributary of Bile Duct.

Relations of the liver


Anterior Postero inferiorly

Diaphragm Oesophagus

Xiphoid process Stomach

Duodenum

Hepatic flexure of colon

Right kidney

Gallbladder

Inferior vena cava

Porta hepatis
Location Postero inferior surface, it joins nearly at right angles with the left sagittal fossa,
yomna92009@yahoo.com - MRCS Part A - My account

Reference ranges End and review

Question 8 of 182 Question stats Score: 0%

1
A 8.1%
A 76 year old man is undergoing an abdominal aortic aneurysm repair. The 2
B 6.2%
surgeons occlude the aorta with two clamps, the inferior clamp being placed at 3
C 68.9%
the point of aortic bifurcation. Which of the following vertebral bodies will lie
D 9.3% 4
posterior to the clamp at this level?
E 7.5% 5

68.9% of users answered this 6


A. L1
question correctly
7
B. T10
8
C. L4
D. L5
E. L2

Next question

The aorta bifurcates at L4. An important landmark that is tested frequently.

Abdominal aorta

Abdominal aortic topography


Origin T12

Termination L4

Posterior relations L1-L4 Vertebral bodies

Anterior relations Lesser omentum


Liver
Left renal vein
Inferior mesenteric vein
Third part of duodenum
Pancreas
Parietal peritoneum
Peritoneal cavity

Right lateral relations Right crus of the diaphragm


Cisterna chyli
Azygos vein
IVC (becomes posterior distally)

Left lateral relations 4th part of duodenum


Duodenal-jejunal flexure
Left sympathetic trunk

The abdominal aorta

Image sourced from Wikipedia


yomna92009@yahoo.com - MRCS Part A - My account

Reference ranges End and review

Question 9 of 182 Question stats Score: 0%

1
A 14.9%
Which of the following statements relating to the greater omentum is false? 2
B 52.1%
C 15.4% 3

A. It is less well developed in children under 5. D 9.7% 4

B. It has no relationship to the lesser sac. E 7.9% 5

C. It contains the gastroepiploic arteries. 52.1% of users answered this 6


question correctly
D. Has an attachment to the transverse colon. 7

E. It may be a site of metastatic disease in ovarian cancer. 8

9
Next question

It is connected with the lesser sac and the transverse colon. This plane is entered
when performing a colonic resection. It is a common site of metastasis in many
visceral malignancies.

Omentum

The omentum is divided into two parts which invest the stomach. Giving rise
to the greater and lesser omentum. The greater omentum is attached to
the inferolateral border of the stomach and houses the gastro-epiploic
arteries.
It is of variable size but is less well developed in children. This is important
as the omentum confers protection against visceral perforation (e.g.
Appendicitis).
Inferiorly between the omentum and transverse colon is one potential entry
point into the lesser sac.
Several malignant processes may involve the omentum of which ovarian
cancer is the most notable.

Rate question: Next question

Com m ent on this question

All contents of this site are © 2012 E-Medical Revision Ltd Terms and Conditions Privacy policy
yomna92009@yahoo.com - MRCS Part A - My account

Reference ranges End and review

Question 10 of 182 Question stats Score: 0%

1
A 15.1%
A 48 year old man with newly diagnosed hypertension is found to have a 2
B 33.6%
phaeochromocytoma of the left adrenal gland and is due to undergo a 3
C 24.4%
laparoscopic left adrenalectomy. Which of the following structures is not directly
D 12.2% 4
related to the left adrenal gland?
E 14.7% 5

24.4% of users answered this 6


A. Crus of the diaphragm
question correctly
7
B. Lesser curvature of the stomach
8
C. Splenic hilum
D. Pancreas 9

E. Splenic artery 10

Next question

The splenic hilum lies more laterally and is therefore not a direct relation of the
left adrenal gland.

Adrenal gland anatomy

Anatomy

Location Superomedially to the upper pole of each kidney

Relationships of Diaphragm-Posteriorly, Kidney-Inferiorly, Vena Cava-Medially, Hepato-


the right adrenal renal pouch and bare area of the liver-Anteriorly

Relationships of Crus of the diaphragm-Postero- medially, Pancreas and splenic


the left adrenal vessels-Inferiorly, Lesser sac and stomach-Anteriorly

Superior adrenal arteries- from inferior phrenic artery, Middle adrenal


Arterial supply arteries - from aorta, Inferior adrenal arteries -from renal arteries

Venous drainage Via one central vein directly into the IVC
of the right
adrenal

Venous drainage Via one central vein into the left renal vein
of the left adrenal

Rate question: Next question

Com m ent on this question

All contents of this site are © 2012 E-Medical Revision Ltd Terms and Conditions Privacy policy
yomna92009@yahoo.com - MRCS Part A - My account

Reference ranges End and review

Question 11 of 182 Question stats Score: 0%

1
A 17.2%
Which of the following nerves innervates the long head of the biceps femoris 2
B 32.7%
muscle? 3
C 12.2%
D 21.8% 4
A. Inferior gluteal nerve E 16.1% 5
B. Tibial nerve 6
32.7% of users answered this
C. Superior gluteal nerve question correctly
7
D. Common peroneal nerve
8
E. Obturator nerve
9

10
Next question
11

The short head of biceps femoris, which may occasionally be absent, is


innervated by the common peroneal component of the sciatic nerve. The long
head is innervated by the tibial nerve.

Biceps femoris

The biceps femoris is one of the hamstring group of muscles located in the
posterior upper thigh. It has two heads.

Long head
Origin Ischial tuberosity

Insertion Fibular head

Action Knee flexion, lateral rotation tibia, extension hip

Innervation Tibial nerve (L5, S1, S2)

Arterial Profunda femoris artery, inferior gluteal artery, and the superior muscular
supply branches of popliteal artery

Image demonstrating the biceps femoris muscle, with the long head outlined
yomna92009@yahoo.com - MRCS Part A - My account

Reference ranges End and review

Question 12 of 182 Question stats Score: 0%

1
A 11.9%
Which of the following is not closely related to the axillary artery? 2
B 47.9%
C 12.9% 3

A. Posterior cord of the brachial plexus D 7.7% 4

B. Scalenus anterior muscle E 19.6% 5

C. Pectoralis minor muscle 47.9% of users answered this 6


question correctly
D. Axillary vein 7

E. Lateral cord of the brachial plexus 8

9
Next question
10

11
The axillary artery is the continuation of the subclavian artery. It is surrounded by
the cords of the brachial plexus (from which they are named). The axillary vein 12
runs alongside the axillary artery throughout its length.

Axilla

Boundaries of the axilla


Medially Chest wall and Serratus anterior

Laterally Humeral head

Floor Subscapularis

Anterior aspect Lateral border of Pectoralis major

Fascia Clavipectoral fascia

Content:
Long thoracic Derived from C5-C7 and passes behind the brachial plexus to enter
nerve (of Bell) the axilla. It lies on the medial chest wall and supplies serratus
anterior. Its location puts it at risk during axillary surgery and damage
will lead to winging of the scapula.

Thoracodorsal Innervate and vascularise latissimus dorsi.


nerve and
thoracodorsal
trunk

Axillary vein Lies at the apex of the axilla, it is the continuation of the basilic vein.
Becomes the subclavian vein at the outer border of the first rib.

Intercostobrachial Traverse the axillary lymph nodes and are often divided during axillary
nerves surgery. They provide cutaneous sensation to the axillary skin.

Lymph nodes The axilla is the main site of lymphatic drainage for the breast.

Rate question: Next question

Com m ent on this question

All contents of this site are © 2012 E-Medical Revision Ltd Terms and Conditions Privacy policy
yomna92009@yahoo.com - MRCS Part A - My account

Reference ranges End and review

Question 13 of 182 Question stats Score: 0%

1
A 8.5%
A 28 year old man is shot in the right chest and develops a right haemothorax 2
B 46.3%
necessitating a thoracotomy. The surgeons decide to place a vascular clamp 3
C 12.4%
across the hilum of the right lung. Which of the following structures will lie most
D 19.1% 4
anteriorly at this point?
E 13.8% 5

46.3% of users answered this 6


A. Thoracic duct
question correctly
7
B. Phrenic nerve
8
C. Vagus nerve
D. Pulmonary artery 9

E. Pulmonary vein 10

11

Next question 12

13
The phrenic nerve lies anteriorly at the root of the right lung.

Lung anatomy

The right lung is composed of 3 lobes divided by the oblique and transverse
fissures. The left lung has two lobes divided by the lingula.The apex of both lungs
is approximately 4cm superior to the sterno-costal joint of the first rib. Immediately
below this is a sulcus created by the subclavian artery.

Peripheral contact points of the lung

Base: diaphragm
Costal surface: corresponds to the cavity of the chest
Mediastinal surface: Contacts the mediastinal pleura. Has the cardiac
impression. Above and behind this concavity is a triangular depression
named the hilum, where the structures which form the root of the lung enter
and leave the viscus. These structures are invested by pleura, which,
below the hilus and behind the pericardial impression, forms the pulmonary
ligament

Right lung
Above the hilum is the azygos vein; Superior to this is the groove for the superior
vena cava and right innominate vein; behind this, and nearer the apex, is a furrow
for the innominate artery. Behind the hilus and the attachment of the pulmonary
ligament is a vertical groove for the oesophagus; In front and to the right of the
lower part of the oesophageal groove is a deep concavity for the extrapericardiac
portion of the inferior vena cava.

The root of the right lung lies behind the superior vena cava and the right atrium,
and below the azygos vein.

The right main bronchus is shorter, wider and more vertical than the left main
bronchus and therefore the route taken by most foreign bodies.

Left lung
Above the hilum is the furrow produced by the aortic arch, and then superiorly the
groove accommodating the left subclavian artery; Behind the hilum and
pulmonary ligament is a vertical groove produced by the descending aorta, and in
front of this, near the base of the lung, is the lower part of the oesophagus.

The root of the left lung passes under the aortic arch and in front of the
descending aorta.

Inferior borders of both lungs


yomna92009@yahoo.com - MRCS Part A - My account

Reference ranges End and review

Question 14 of 182 Question stats Score: 0%

1
A 7.6%
An 18 year old boy is undergoing an appendicectomy for appendicitis. At which of 2
B 7.2%
the following locations is the appendix most likely to be found? 3
C 72%
D 7.5% 4
A. Pre ileal E 5.7% 5
B. Pelvic 6
72% of users answered this
C. Retrocaecal question correctly
7
D. Post ileal
8
E. None of the above
9

10
Next question
11

Most appendixes lie in the retrocaecal position. If a retrocaecal appendix is 12


difficult to remove then mobilisation of the right colon significantly improves 13
access.
14

Appendix

Location: Base of caecum.


Up to 10cm long.
Mainly lymphoid tissue (Hence mesenteric adenitis may mimic appendicitis).
Caecal taenia coli converge at base of appendix and form a longitudinal
muscle cover over the appendix. This convergence should facilitate its
identification at surgery if it is retrocaecal and difficult to find (which it can
be when people start doing appendicectomies!)
Arterial supply: Appendicular artery (branch of the ileocolic).
It is intra peritoneal.

McBurney's point

1/3 of the way along a line drawn from the Anterior Superior Iliac Spine to
the Umbilicus

6 Positions:

Retrocaecal 74%
Pelvic 21%
Postileal
Subcaecal
Paracaecal
Preileal

Rate question: Next question

Com m ent on this question

All contents of this site are © 2012 E-Medical Revision Ltd Terms and Conditions Privacy policy
yomna92009@yahoo.com - MRCS Part A - My account

Reference ranges End and review

Question 15 of 182 Question stats Score: 6.7%

1
A 15.4%
A 56 year old man is undergoing a pancreatectomy for carcinoma. During 2
B 13.9%
resection of the gland which of the following structures will the surgeon not 3
C 17.4%
encounter posterior to the pancreas itself?
D 11.5% 4
E 41.8% 5
A. Left crus of the diaphragm
41.8% of users answered this 6
B. Superior mesenteric vein question correctly
7
C. Common bile duct
8
D. Portal vein
9
E. Gastroduodenal artery
10

Next question 11

12
Theme from 2010 Exam 13
The gastroduodenal artery lies anterior to the pancreas.
14

Pancreas 15

The pancreas is a retroperitoneal organ and lies posterior to the stomach. It may
be accessed surgically by dividing the peritoneal reflection that connects the
greater omentum to the transverse colon. The pancreatic head sits in the
curvature of the duodenum. It's tail lies close to the hilum of the spleen, a site of
potential injury during splenectomy.

Relations
Posterior to the pancreas
Pancreatic head Inferior vena cava
Common bile duct
Right and left renal veins
Superior mesenteric vein and artery

Pancreatic neck Superior mesenteric vein, portal vein

Pancreatic body- Left renal vein


Crus of diaphragm
Psoas muscle
Adrenal gland
Kidney
Aorta

Pancreatic tail Left kidney

Anterior to the pancreas


Pancreatic head 1st part of the duodenum
Pylorus
Gastroduodenal artery
SMA and SMV(uncinate process)

Pancreatic body Stomach


Duodenojejunal flexure

Pancreatic tail Splenic hilum

Superior to the pancreas


Coeliac trunk and its branches common hepatic artery and splenic artery

Grooves of the head of the pancreas


2nd and 3rd part of the duodenum

Arterial supply

Head: pancreaticoduodenal artery


Rest: splenic artery
Venous drainage

Head: superior mesenteric vein


Body and tail: splenic vein

Ampulla of Vater

Merge of pancreatic duct and common bile duct


Is an important landmark, halfway along the second part of the duodenum,
that marks the anatomical transition from foregut to midgut (also the site of
transition between regions supplied by coeliac trunk and SMA).

Image sourced from Wikipedia

Rate question: Next question

Com m ent on this question

All contents of this site are © 2012 E-Medical Revision Ltd Terms and Conditions Privacy policy
yomna92009@yahoo.com - MRCS Part A - My account

Reference ranges End and review

Question 16 of 182 Question stats Score: 6.3%

1
A 8.7%
Which of the following bones is related to the cuboid at its distal articular surface? 2
B 42.1%
C 12.1% 3

A. All metatarsals D 21.3% 4

B. 5th metatarsal E 15.8% 5

C. Calcaneum 42.1% of users answered this 6


question correctly
D. Medial cuneiform 7

E. 3rd metatarsal 8

9
Next question
10

11
Theme from April 2012 Exam
The cuboid is located at the lateral aspect of the foot between the calcaneus 12
posteriorly and the 4th and 5th metatarsals distally. 13

14
Foot- anatomy
15

Arches of the foot 16


The foot is conventionally considered to have two arches.

The longitudinal arch is higher on the medial than on the lateral side. The
posterior part of the calcaneum forms a posterior pillar to support the arch.
The lateral part of this structure passes via the cuboid bone and the lateral
two metatarsal bones. The medial part of this structure is more important.
The head of the talus marks the summit of this arch, located between the
sustentaculum tali and the navicular bone. The anterior pillar of the medial
arch is composed of the navicular bone, the three cuneiforms and the
medial three metatarsal bones.
The transverse arch is situated on the anterior part of the tarsus and the
posterior part of the metatarsus. The cuneiforms and metatarsal bases
narrow inferiorly, which contributes to the shape of the arch.

Intertarsal joints
Sub talar joint Formed by the cylindrical facet on the lower surface of the body of
the talus and the posterior facet on the upper surface of the
calcaneus. The facet on the talus is concave anteroposteriorly, the

other is convex. The synovial cavity of this joint does not


communicate with any other joint.

Talocalcaneonavicular The anterior part of the socket is formed by the concave articular
joint surface of the navicular bone, posteriorly by the upper surface of
the sustentaculum tali. The talus sits within this socket

Calcaneocuboid joint Highest point in the lateral part of the longitudinal arch. The lower
aspect of this joint is reinforced by the long plantar and plantar
calcaneocuboid ligaments.

Transverse tarsal joint The talocalcaneonavicular joint and the calcaneocuboid joint
extend accross the tarsus in an irregular transverse plane,
between the talus and calcaneus behind and the navicular and
cuboid bones in front. This plane is termed the transverse tarsal
joint.

Cuneonavicular joint Formed between the convex anterior surface of the navicular bone
and the concave surface of the the posterior ends of the three
cuneiforms.

Intercuneiform joints Between the three cuneiform bones.

Cuneocuboid joint Between the circular facets on the lateral cuneiform bone and the
cuboid. This joint contributes to the tarsal part of the transverse
arch.

A detailed knowledge of the joints is not required for MRCS Part A. However, the
contribution they play to the overall structure of the foot should be appreciated

Ligaments of the ankle joint and foot


yomna92009@yahoo.com - MRCS Part A - My account

Reference ranges End and review

Question 17 of 182 Question stats Score: 5.9%

1
A 9.2%
Where does Stensens duct primarily open? 2
B 14.7%
C 59.7% 3

A. Immediately lateral to the foramen caecum D 7.3% 4

B. Floor of mouth E 9.1% 5

C. Opposite the second molar tooth 59.7% of users answered this 6


question correctly
D. Opposite the fifth molar tooth 7

E. Into the post nasal space 8

9
Next question
10

11
Stensens duct conveys secretions from the parotid gland and these enter the oral
cavity at the level of the second molar tooth. 12

13
Parotid gland
14

15
Anatomy of the parotid gland
Location Overlying the mandibular ramus; anterior and inferior to the ear. 16

Salivary duct Crosses the masseter, pierces the buccinator and drains adjacent to 17
the 2nd upper molar tooth (Stensen's duct).

Structures Facial nerve (Mnemonic: The Zebra Buggered My Cat; Temporal


passing through Zygomatic, Buccal, Mandibular, Cervical)
the gland External carotid artery
Retromandibular vein
Auriculotemporal nerve

Relations Anterior: masseter, medial pterygoid, superficial temporal and


maxillary artery, facial nerve, stylomandibular ligament
Posterior: posterior belly digastric muscle,
sternocleidomastoid, stylohyoid, internal carotid artery, mastoid
process, styloid process

Arterial supply Branches of external carotid artery

Venous drainage Retromandibular vein

Lymphatic Deep cervical nodes


drainage

Nerve innervation Parasympathetic-Secretomotor


Sympathetic-Superior cervical ganglion
Sensory- Greater auricular nerve

Parasympathetic stimulation produces a water rich, serous saliva. Sympathetic


stimulation leads to the production of a low volume, enzyme-rich saliva.

Rate question: Next question

Com m ent on this question

All contents of this site are © 2012 E-Medical Revision Ltd Terms and Conditions Privacy policy
yomna92009@yahoo.com - MRCS Part A - My account

Reference ranges End and review

Question 18 of 182 Question stats Score: 5.6%

1
A 46.7%
Which of the following nerves is responsible for the innervation of the posterior 2
B 16.9%
belly of the digastric muscle? 3
C 9.8%
D 16% 4
A. Facial nerve E 10.5% 5
B. Hypoglossal nerve 6
46.7% of users answered this
C. Trigeminal nerve question correctly
7
D. Ansa cervicalis
8
E. Mylohoid nerve
9

10
Next question
11

The posterior belly of digastric is innervated by the facial nerve and the anterior 12
belly by the mylohoid nerve. 13

14
Anterior triangle of the neck
15

Boundaries 16

Anterior border of the Sternocleidomastoid 17


Lower border of mandible
18
Anterior midline

Sub triangles (divided by Digastric above and Omohyoid)

Muscular triangle: Neck strap muscles


Carotid triangle: Carotid sheath
Submandibular Triangle (digastric)

Contents of the anterior triangle


Digastric triangle Submandibular gland
Submandibular nodes
Facial vessels
Hypoglossal nerve

Muscular triangle Strap muscles


External jugular vein

Carotid triangle Carotid sheath (Common carotid, vagus and internal jugular vein)
Ansa cervicalis

Nerve supply to digastric muscle

Anterior: Mylohyoid nerve


Posterior: Facial nerve

Image sourced from Wikipedia

Rate question: Next question


yomna92009@yahoo.com - MRCS Part A - My account

Reference ranges End and review

Question 19 of 182 Question stats Score: 5.3%

1
A 10.4%
Which of the following carpal bones is a sesamoid bone in the tendon of flexor 2
B 8%
carpi ulnaris? 3
C 65.9%
D 8.5% 4
A. Triquetrum E 7.1% 5
B. Lunate 6
65.9% of users answered this
C. Pisiform question correctly
7
D. Scaphoid
8
E. Capitate
9

10
Next question
11

This small bone has a single articular facet. It projects from the triquetral bone at 12
the ulnar aspect of the wrist where most regard it as a sesamoid bine lying within 13
the tendon of flexor carpi ulnaris.
14

Carpal bones 15

16
Diagrammatic image of carpal bones 17

18

19

Image sourced from Wikipedia

Key to image
A Scaphoid

B Lunate

C Triquetrum

D Pisiform

E Trapezium

F Trapezoid

G Capitate

H Hamate

1 Radius

2 Ulna

3 Metacarpals

No tendons attach to: Scaphoid, lunate, triquetrum (stabilised by ligaments)

Rate question: Next question

Com m ent on this question


yomna92009@yahoo.com - MRCS Part A - My account

Reference ranges End and review

Question 20 of 182 Question stats Score: 5%

1
A 70.7%
A 70 year old man falls and fractures his scaphoid bone. The fracture is displaced 2
B 6.4%
and the decision is made to insert a screw to fix the fracture. Which of the 3
C 8.8%
following structures lies directly medial to the scaphoid?
D 7.2% 4
E 6.9% 5
A. Lunate
70.7% of users answered this 6
B. Pisiform question correctly
7
C. Trapezoid
8
D. Trapezium
9
E. None of the above
10

Next question 11

12
The lunate lies medially in the anatomical plane. Fractures of the scaphoid that 13
are associated with high velocity injuries may cause associated lunate dislocation.
14

Scaphoid bone 15

16
The scaphoid has a concave articular surface for the head of the capitate and at 17
the edge of this is a crescentic surface for the corresponding area on the lunate.
18
Proximally, it has a wide convex articular surface with the radius. It has a distally
sited tubercle that can be palpated. The remaining articular surface is to the 19
lateral side of the tubercle. It faces laterally and is associated with the trapezium 20
and trapezoid bones.

The narrow strip between the radial and trapezial surfaces and the tubercle gives
rise to the radial collateral carpal ligament. The tubercle receives part of the flexor
retinaculum. This area is the only part of the scaphoid that is available for the
entry of blood vessels. It is commonly fractured and avascular necrosis may
result.

Scaphoid bone

Image sourced from Wikipedia

Rate question: Next question

Com m ent on this question


yomna92009@yahoo.com - MRCS Part A - My account

Reference ranges End and review

Question 21 of 182 Question stats Score: 4.8%

1
A 13.4%
A 55 year old man is admitted with a brisk haematemesis. He is taken to the 2
B 47.6%
endoscopy department and an upper GI endoscopy is performed by the 3
C 21.8%
gastroenterologist. He identifies an ulcer on the posterior duodenal wall and
D 8.3% 4
spends an eternity trying to control the bleeding with all the latest haemostatic
E 8.9% 5
techniques. He eventually asks the surgeons for help. A laparotomy and anterior
duodenotomy are performed, as the surgeon opens the duodenum a vessel is 6
47.6% of users answered this
spurting blood into the duodenal lumen. From which of the following does this question correctly
7
vessel arise?
8

9
A. Left gastric artery
10
B. Common hepatic artery
11
C. Right hepatic artery
12
D. Superior mesenteric artery
E. Splenic artery 13

14

Next question 15

16
The vessel will be the gastroduodenal artery, this arises from the common hepatic
17
artery.
18

Gastroduodenal artery 19

20
Supplies
21
Pylorus, proximal part of the duodenum, and indirectly to the pancreatic head (via
the anterior and posterior superior pancreaticoduodenal arteries)

Path
Most commonly arises from the common hepatic artery of the coeliac trunk
Terminates by bifurcating into the right gastroepiploic artery and the superior
pancreaticoduodenal artery

Image showing stomach reflected superiorly to illustrate the relationship of the


gastroduodenal artery to the first part of the duodenum

Image sourced from Wikipedia


yomna92009@yahoo.com - MRCS Part A - My account

Reference ranges End and review

Question 22 of 182 Question stats Score: 4.5%

1
A 20.2%
A 73 year old lady is hit by a car. She suffers a complex fracture of the distal 2
B 7.9%
aspect of her humerus with associated injury to the radial nerve. Which of the 3
C 6.1%
following movements will be most impaired as a result?
D 60.8% 4
E 5% 5
A. Elbow extension
60.8% of users answered this 6
B. Elbow flexion question correctly
7
C. Shoulder abduction
8
D. Wrist extension
9
E. None of the above
10

Next question 11

12
The triceps will not be affected so elbow extension will be preserved. Loss of wrist 13
extension will be the most obvious effect.
14

Radial nerve 15

16
Continuation of posterior cord of the brachial plexus (root values C5 to T1) 17

18
Path
19
In the axilla: lies posterior to the axillary artery on subscapularis, latissimus
20
dorsi and teres major.
Enters the arm between the brachial artery and the long head of triceps 21
(medial to humerus). 22
Spirals around the posterior surface of the humerus in the groove for the
radial nerve.
At the distal third of the lateral border of the humerus it then pierces the
intermuscular septum and descends in front of the lateral epicondyle.
At the lateral epicondyle it lies deeply between brachialis and
brachioradialis where it then divides into a superficial and deep terminal
branch.
Deep branch crosses the supinator to become the posterior interosseous
nerve.

In the image below the relationships of the radial nerve can be appreciated

Image sourced from Wikipedia

Regions innervated

Motor (main Triceps


yomna92009@yahoo.com - MRCS Part A - My account

Reference ranges End and review

Question 23 of 182 Question stats Score: 4.3%

1
A 10.8%
Which of the following is not a content of the rectus sheath? 2
B 7%
C 7.4% 3

A. Pyramidalis D 68.9% 4

B. Superior epigastric artery E 5.9% 5

C. Inferior epigastric vein 68.9% of users answered this 6


question correctly
D. Internal iliac artery 7

E. Rectus abdominis 8

9
Next question
10

11
The rectus sheath also contains:
superior epigastric vein 12
inferior epigastric artery 13

14
Abdominal wall
15

The 2 main muscles of the abdominal wall are the rectus abdominis (anterior) 16
and the quadratus lumborum (posterior). 17
The remaining abdominal wall consists of 3 muscular layers. Each muscle passes
18
from the lateral aspect of the quadratus lumborum posteriorly to the lateral margin
of the rectus sheath anteriorly. Each layer is muscular posterolaterally and 19
aponeurotic anteriorly. 20

21

22

23

Image sourced from Wikipedia

Muscles of abdominal wall

External Lies most superficially


oblique Originates from 5th to 12th ribs
Inserts into the anterior half of the outer aspect of the iliac crest, linea
alba and pubic tubercle
More medially and superiorly to the arcuate line, the aponeurotic layer
overlaps the rectus abdominis muscle
The lower border forms the inguinal ligament
The triangular expansion of the medial end of the inguinal ligament is
the lacunar ligament.

Internal Arises from the thoracolumbar fascia, the anterior 2/3 of the iliac crest
oblique and the lateral 2/3 of the inguinal ligament
The muscle sweeps upwards to insert into the cartilages of the lower
3 ribs
The lower fibres form an aponeurosis that runs from the tenth costal
cartilage to the body of the pubis
At its lowermost aspect it joins the fibres of the aponeurosis of
transversus abdominis to form the conjoint tendon.

Transversus Innermost muscle


abdominis Arises from the inner aspect of the costal cartilages of the lower 6 ribs
, from the anterior 2/3 of the iliac crest and lateral 1/3 of the inguinal
ligament
Its fibres run horizontally around the abdominal wall ending in an
aponeurosis. The upper part runs posterior to the rectus abdominis.
Lower down the fibres run anteriorly only.
The rectus abdominis lies medially running from the pubic crest and
symphysis to insert into the xiphoid process and 5th, 6th and 7th
costal cartilages. The muscles lies in a aponeurosis as described
above.
Nerve supply: anterior primary rami of T7-12
yomna92009@yahoo.com - MRCS Part A - My account

Reference ranges End and review

Question 24 of 182 Question stats Score: 4.2%

1
A 51.3%
Which of the following vessels does not drain directly into the inferior vena cava? 2
B 7.4%
C 11% 3

A. Superior mesenteric vein D 16.6% 4

B. Right common iliac E 13.7% 5

C. Right hepatic vein 51.3% of users answered this 6


question correctly
D. Left hepatic vein 7

E. Right testicular vein 8

9
Next question
10

11
The superior mesenteric vein drains into the portal vein. The right and left hepatic
veins drain into it directly, this can account for major bleeding in more extensive 12
liver shearing type injuries. 13

14
Inferior vena cava
15

Origin 16

17
L5
18

19
Path
20
Left and right common iliac veins merge to form the IVC.
21
Passes right of midline
Paired segmental lumbar veins drain into the IVC throughout its length 22
The right gonadal vein empties directly into the cava and the left gonadal 23
vein generally empties into the left renal vein.
24
The next major veins are the renal veins and the hepatic veins
Pierces the central tendon of diaphragm at T8
Right atrium

Image sourced from Wikipedia

Relations
Anteriorly Small bowel, first and third part of duodenum, head of pancreas, liver and bile
duct, right common iliac artery, right gonadal artery

Posteriorly Right renal artery, right psoas, right sympathetic chain, coeliac ganglion

Levels
Level Vein

T8 Hepatic vein, inferior phrenic vein, pierces diaphragm

L1 Suprarenal veins, renal vein

L2 Gonadal vein

L1-5 Lumbar veins

L5 Common iliac vein, formation of IVC

Rate question: Next question

Com m ent on this question

All contents of this site are © 2012 E-Medical Revision Ltd Terms and Conditions Privacy policy
yomna92009@yahoo.com - MRCS Part A - My account

Reference ranges End and review

Question 1 of 158 Question stats Score: 0%

1
A 7.9%
A 17 year old male has a suspected testicular torsion and the scrotum is to be B 19.6%
explored surgically. The surgeon incises the skin and then the dartos muscle.
C 21.3%
What is the next tissue layer that will be encountered during the dissection?
D 45.9%

E 5.3%
A. Visceral layer of the tunica vaginalis
45.9% of users answered this
B. Cremasteric fascia question correctly
C. Parietal layer of the tunica vaginalis
D. External spermatic fascia
E. Internal spermatic fascia

Next question

The layers that will be encountered are (in order):


1. Skin
2. Dartos fascia and muscle
3. External spermatic fascia
4. Cremasteric muscle and fascia
5. Internal spermatic fascia
6. Parietal layer of the tunica vaginalis

The layers of the spermatic cord and scrotum are a popular topic in the MRCS
exam.

Scrotal and testicular anatomy

Spermatic cord
Formed by the vas deferens and is covered by the following structures:
Layer Origin

Internal spermatic fascia Transversalis fascia

Cremasteric fascia From the fascial coverings of internal oblique

External spermatic fascia External oblique aponeurosis

Contents of the cord


Vas deferens Transmits sperm and accessory gland secretions

Testicular artery Branch of abdominal aorta supplies testis and


epididymis

Artery of vas deferens Arises from inferior vesical artery

Cremasteric artery Arises from inferior epigastic artery

Pampiniform plexus Venous plexus, drains into right or left testicular vein

Sympathetic nerve fibres Lie on arteries, the parasympathetic fibres lie on the
vas

Genital branch of the genitofemoral Supplies cremaster


nerve

Lymphatic vessels Drain to lumbar and para-aortic nodes

Scrotum

Composed of skin and closely attached dartos fascia.


Arterial supply from the anterior and posterior scrotal arteries
Lymphatic drainage to the inguinal lymph nodes
Parietal layer of the tunica vaginalis is the innermost layer

Testes

The testes are surrounded by the tunica vaginalis (closed peritoneal sac).
yomna92009@yahoo.com - MRCS Part A - My account

Reference ranges End and review

Question 2 of 158 Question stats Score: 0%

1
A 11.4%
A 25 year old man is stabbed in the groin and the area, which lies within the 2
B 8.5%
femoral triangle is explored. Which structure forms the lateral wall of the femoral
C 7.5%
triangle?
D 66.7%

E 5.9%
A. Adductor longus
66.7% of users answered this
B. Pectineus question correctly
C. Adductor magnus
D. Sartorius
E. Conjoint tendon

Next question

The sartorius forms the lateral wall of the femoral triangle (see below).

Femoral triangle anatomy

Boundaries
Superiorly Inguinal ligament

Laterally Sartorius

Medially Adductor longus

Floor Iliopsoas, adductor longus and pectineus

Roof Fascia lata and Superficial fascia


Superficial inguinal lymph nodes (palpable below the inguinal ligament)
Great saphenous vein

Image sourced from Wikipedia


Contents

Femoral vein (medial to lateral)


Femoral artery-pulse palpated at the mid inguinal point
Femoral nerve
Deep and superficial inguinal lymph nodes
Lateral cutaneous nerve
Great saphenous vein
Femoral branch of the genitofemoral nerve

Rate question: Next question

Com m ent on this question

All contents of this site are © 2012 E-Medical Revision Ltd Terms and Conditions Privacy policy
yomna92009@yahoo.com - MRCS Part A - My account

Reference ranges End and review

Question 3 of 158 Question stats Score: 0%

1
A 14%
An 18 year old man develops a severe spreading sepsis of the hand. The palm is 2
B 54.9%
explored surgically and the flexor digiti minimi brevis muscle is mobilised to 3
C 10.7%
facilitate drainage of the infection. Which of the following structures is not closely
D 11.9%
related to this muscle?
E 8.5%

A. The hook of hamate 54.9% of users answered this


question correctly
B. Median nerve
C. Superficial palmar arterial arch
D. Digital nerves arising from the ulnar nerve
E. None of the above

Next question

The flexor digiti minimi brevis originates from the Hamate, on its undersurface lie
the ulnar contribution to the superficial palmar arterial arch and digital nerves
derived from the ulnar nerve. The median nerve overlies the flexor tendons.

Hand

Anatomy of the hand

Bones 8 Carpal bones


5 Metacarpals
14 phalanges

Intrinsic 7 Interossei - Supplied by ulnar nerve


Muscles
3 palmar-adduct fingers
4 dorsal- abduct fingers

Intrinsic Lumbricals
muscles
Flex MCPJ and extend the IPJ.
Origin deep flexor tendon and insertion dorsal extensor hood
mechanism.
Innervation: 1st and 2nd- median nerve, 3rd and 4th- deep
branch of the ulnar nerve.

Thenar Abductor pollicis brevis


eminence Opponens pollicis
Flexor pollicis brevis

Hypothenar Opponens digiti minimi


eminence Flexor digiti minimi brevis
Abductor digiti minimi
yomna92009@yahoo.com - MRCS Part A - My account

Reference ranges End and review

Question 4 of 158 Question stats Score: 0%

1
A 25.2%
A 19 year old man undergoes an open inguinal hernia repair. The cord is 2
B 37.2%
mobilised and the deep inguinal ring identified. Which of the following structures 3
C 16.7%
forms its lateral wall?
D 15.9% 4
E 5%
A. External oblique aponeurosis
37.2% of users answered this
B. Transversalis fascia question correctly
C. Conjoint tendon
D. Inferior epigastric artery
E. Inferior epigastric vein

Next question

The transversalis fascia forms the superolateral edge of the deep inguinal ring.
The epigastric vessels form its inferomedial wall.

Inguinal canal

Location

Above the inguinal ligament


The inguinal canal is 4cm long

Boundaries of the inguinal canal

Floor External oblique aponeurosis


Inguinal ligament
Lacunar ligament

Roof Internal oblique


Transversus abdominis

Anterior wall External oblique aponeurosis

Posterior wall Transversalis fascia


Conjoint tendon

Laterally Internal ring


Fibres of internal oblique

Medially External ring


Conjoint tendon

Contents
Males Spermatic cord and ilioinguinal As it passes through the canal the spermatic
nerve cord has 3 coverings:

External spermatic fascia from external


oblique aponeurosis
Cremasteric fascia
Internal spermatic fascia

Females Round ligament of uterus and


ilioinguinal nerve

Related anatomy of the inguinal region


The boundaries of Hesselbachs triangle are commonly tested and illustrated
below:
yomna92009@yahoo.com - MRCS Part A - My account

Reference ranges End and review

Question 5 of 158 Question stats Score: 20%

1
A 8%
A 40-year-old man presents with pain in his lower back and 'sciatica' for the past 2
B 7.8%
three days. He describes bending down to pick up a washing machine when he 3
C 12.6%
felt 'something go'. He now has severe pain radiating from his back down the right
D 36.7% 4
leg. On examination he describes paraesthesia over the anterior aspect of the
E 34.9% 5
right knee and the medial aspect of his calf. Power is intact and the right knee
reflex is diminished. The femoral stretch test is positive on the right side. Which
34.9% of users answered this
nerve or nerve root is most likely to be affected? question correctly

A. Common peroneal nerve


B. Lateral cutaneous nerve of the thigh
C. L5
D. L3
E. L4

Next question

Prolapsed disc

A prolapsed lumbar disc usually produces clear dermatomal leg pain associated
with neurological deficits.

Features

Leg pain usually worse than back


Pain often worse when sitting

The table below demonstrates the expected features according to the level of
compression:

L3 nerve root Sensory loss over anterior thigh


compression Weak quadriceps
Reduced knee reflex
Positive femoral stretch test

L4 nerve root Sensory loss anterior aspect of knee


compression Weak quadriceps
Reduced knee reflex
Positive femoral stretch test

L5 nerve root Sensory loss dorsum of foot


compression Weakness in foot and big toe dorsiflexion
Reflexes intact
Positive sciatic nerve stretch test

S1 nerve root Sensory loss posterolateral aspect of leg and lateral aspect of
compression foot
Weakness in plantar flexion of foot
Reduced ankle reflex
Positive sciatic nerve stretch test

Management

Similar to that of other musculoskeletal lower back pain: analgesia,


physiotherapy, exercises
Persistent symptoms, muscular weakness, bladder or bowel dysfunction are
indications for urgent MRI scanning to delineate the disease extent to allow
surgical planning
Plain spinal x-rays have no useful role in establishing the extent of disk
disease
yomna92009@yahoo.com - MRCS Part A - My account

Reference ranges End and review

Question 6 of 158 Question stats Score: 16.7%

1
A 14.4%
A 22 year old man develops an infection in the pulp of his little finger. What is the 2
B 22.1%
most proximal site to which this infection may migrate? 3
C 13.4%
D 35% 4
A. The metacarpophalangeal joint E 15.1% 5
B. The distal interphalangeal joint 6
35% of users answered this
C. The proximal interphalangeal joint question correctly

D. Proximal to the flexor retinaculum


E. Immediately distal to the carpal tunnel

Next question

The 5th tendon sheath extends from the little finger to the proximal aspect of the
carpal tunnel. This carries a significant risk of allowing infections to migrate
proximally.

Hand

Anatomy of the hand

Bones 8 Carpal bones


5 Metacarpals
14 phalanges

Intrinsic 7 Interossei - Supplied by ulnar nerve


Muscles
3 palmar-adduct fingers
4 dorsal- abduct fingers

Intrinsic Lumbricals
muscles
Flex MCPJ and extend the IPJ.
Origin deep flexor tendon and insertion dorsal extensor hood
mechanism.
Innervation: 1st and 2nd- median nerve, 3rd and 4th- deep
branch of the ulnar nerve.

Thenar Abductor pollicis brevis

eminence Opponens pollicis


Flexor pollicis brevis

Hypothenar Opponens digiti minimi


eminence Flexor digiti minimi brevis
Abductor digiti minimi
yomna92009@yahoo.com - MRCS Part A - My account

Reference ranges End and review

Question 7 of 158 Question stats Score: 14.3%

1
A 14.5%
Which of the following muscles is not innervated by the deep branch of the ulnar 2
B 8.9%
nerve? 3
C 9%
D 58% 4
A. Adductor pollicis E 9.6% 5
B. Hypothenar muscles 6
58% of users answered this
C. All the interosseous muscles question correctly
7
D. Opponens pollicis
E. Third and fourth lumbricals

Next question

Ulnar nerve

Origin

C8, T1

Supplies (no muscles in the upper arm)

Flexor carpi ulnaris


Flexor digitorum profundus
Flexor digiti minimi
Abductor digiti minimi
Opponens digiti minimi
Adductor pollicis
Interossei muscle
Third and fourth lumbricals
Palmaris brevis

Path

Posteromedial aspect of ulna to flexor compartment of forearm, then along


the ulnar. Passes beneath the flexor carpi ulnaris muscle, then superficially
through the flexor retinaculum into the palm of the hand.
yomna92009@yahoo.com - MRCS Part A - My account

Reference ranges End and review

Question 8 of 158 Question stats Score: 12.5%

1
A 15.9%
Onto which of the following structures does the anterior cruciate ligament insert? 2
B 40.5%
C 17.8% 3

A. Posterolateral aspect of the lateral femoral condyle D 17.3% 4

B. Posteromedial aspect of the lateral femoral condyle E 8.6% 5

C. Posterolateral aspect of the medial femoral condyle 40.5% of users answered this 6
question correctly
D. Posteromedial aspect of the medial femoral condyle 7

E. None of the above 8

Next question

The anterior cruciate ligament is attached to the anterior intercondylar area of the
tibia. Is then passes posterolaterally to insert into the posteromedial aspect of the
lateral femoral condyle.

Knee joint

The knee joint is a synovial joint, the largest and most complicated. It consists of
two condylar joints between the femure and tibia and a sellar joint between the
patella and the femur. The tibiofemoral articular surfaces are incongruent,
however, this is improved by the presence of the menisci. The degree of
congruence is related to the anatomical position of the knee joint and is greatest
in full extension.

Knee joint compartments

Tibiofemoral Comprise of the patella/femur joint, lateral and medial


compartments (between femur condyles and tibia)
Synovial membrane and cruciate ligaments partially
separate the medial and lateral compartments

Patellofemoral Ligamentum patellae


Actions: provides joint stability in full extension

Fibrous capsule
The capsule of the knee joint is a complex, composite structure with contributions
from adjacent tendons.
Anterior The capsule does not pass proximal to the patella. It blends with
fibres the tendinous expansions of vastus medialis and lateralis

Posterior These fibres are vertical and run from the posterior surface of the
fibres femoral condyles to the posterior aspect of the tibial condyle

Medial Attach to the femoral and tibial condyles beyond their articular
fibres margins, blending with the tibial collateral ligament

Lateral Attach to the femur superior to popliteus, pass over its tendon to
fibres head of fibula and tibial condyle

Bursae

Anterior Subcutaneous prepatellar bursa; between patella and skin


Deep infrapatellar bursa; between tibia and patellar ligament
Subcutaneous infrapatellar bursa; between distal tibial
tuberosity and skin

Laterally Bursa between lateral head of gastrocnemius and joint


capsule
Bursa between fibular collateral ligament and tendon of
biceps femoris
Bursa between fibular collateral ligament and tendon of
popliteus
yomna92009@yahoo.com - MRCS Part A - My account

Reference ranges End and review

Question 9 of 158 Question stats Score: 11.1%

1
A 15.5%
During an arch aortogram the brachiocephalic artery is entered with an 2
B 5.9%
angiography catheter. The radiologist continues to advance the catheter. Into 3
C 59.9%
which of the following vessels is it likely to enter?
D 7.7% 4
E 11% 5
A. Left subclavian artery
59.9% of users answered this 6
B. Left axillary artery question correctly
7
C. Right subclavian artery
8
D. Right axillary artery
9
E. None of the above

Next question

The axillary artery is a branch of the subclavian artery and although


developmental anomalies may occur they are rare. The catheter may also enter
the right carotid. There is no brachiocephalic artery on the left side.

Brachiocephalic artery

The brachiocephalic artery is the largest branch of the aortic arch. From its aortic
origin it ascends superiorly, it initially lies anterior to the trachea and then on its
right hand side. It branches into the common carotid and right subclavian arteries
at the level of the sternoclavicular joint.

Path
Origin- apex of the midline of the aortic arch
Passes superiorly and posteriorly to the right
Divides into the right subclavian and right common carotid artery

Relations

Anterior Sternohyoid
Sternothyroid
Thymic remnants
Left brachiocephalic vein
Right inferior thyroid veins

Posterior Trachea
Right pleura

Right lateral Right brachiocephalic vein


Superior part of SVC

Left lateral Thymic remnants


Origin of left common carotid
Inferior thyroid veins
Trachea (higher level)

Branches
Normally none but may have the thyroidea ima artery
yomna92009@yahoo.com - MRCS Part A - My account

Reference ranges End and review

Question 10 of 158 Question stats Score: 10%

1
A 61.2%
Which of the following structures lie between the lateral and medial heads of the 2
B 9.8%
triceps muscle? 3
C 9.6%
D 11.2% 4
A. Radial nerve E 8.2% 5
B. Median nerve 6
61.2% of users answered this
C. Ulnar nerve question correctly
7
D. Axillary nerve
8
E. Medial cutaneous nerve of the forearm
9

10
Next question

The radial nerve runs in its groove on between the two heads. The ulnar nerve
lies anterior to the medial head. The axillary nerve passes through the
quadrangular space. This lies superior to lateral head of the triceps muscle and
thus the lateral border of the quadrangular space is the humerus. Therefore the
correct answer is the radial nerve.

Triceps

Origin Long head- infraglenoid tubercle of the scapula.


Lateral head- dorsal surface of the humerus, lateral and proximal to the
groove of the radial nerve
Medial head- posterior surface of the humerus on the inferomedial side
of the radial groove and both of the intermuscular septae

Insertion Olecranon process of the ulna. Here the olecranon bursa is between
the triceps tendon and olecranon.
Some fibres insert to the deep fascia of the forearm, posterior capsule
of the elbow (preventing the capsule from being trapped between
olecranon and olecranon fossa during extension)

Innervation Radial nerve

Blood Profunda brachii artery


supply

Action Elbow extension. The long head can adduct the humerus and and extend it
from a flexed position

Relations The radial nerve and profunda brachii vessels lie between the lateral and
medial heads

Rate question: Next question

Com m ent on this question

All contents of this site are © 2012 E-Medical Revision Ltd Terms and Conditions Privacy policy
yomna92009@yahoo.com - MRCS Part A - My account

Reference ranges End and review

Question 11 of 158 Question stats Score: 18.2%

1
A 16.3%
Into which of the following structures does the superior part of the fibrous capsule 2
B 6.1%
of the shoulder joint insert? 3
C 13.5%
D 21.3% 4
A. The surgical neck of the humerus E 42.9% 5
B. The body of the humerus 6
42.9% of users answered this
C. The bicipital groove question correctly
7
D. Immediately distal to the greater tuberosity
8
E. The anatomical neck of the humerus
9

10
Next question
11

The shoulder joint is a shallow joint, hence its great mobility. However, this comes
at the expense of stability. The fibrous capsule attaches to the anatomical neck
superiorly and the surgical neck inferiorly

Shoulder joint

Shallow synovial ball and socket type of joint.


It is an inherently unstable joint, but is capable to a wide range of
movement.
Stability is provided by muscles of the rotator cuff that pass from the
scapula to insert in the greater tuberosity (all except sub scapularis-lesser
tuberosity).

Glenoid labrum

Fibrocartilaginous rim attached to the free edge of the glenoid cavity


Tendon of the long head of biceps arises from within the joint from the
supraglenoid tubercle, and is fused at this point to the labrum.
The long head of triceps attaches to the infraglenoid tubercle

Fibrous capsule

Attaches to the scapula external to the glenoid labrum and to the labrum
itself (postero-superiorly)
Attaches to the humerus at the level of the anatomical neck superiorly and
the surgical neck inferiorly
Anteriorly the capsule is in contact with the tendon of subscapularis,
superiorly with the supraspinatus tendon, and posteriorly with the tendons
of infraspinatus and teres minor. All these blend with the capsule towards
their insertion.
Two defects in the fibrous capsule; superiorly for the tendon of biceps.
Anteriorly there is a defect beneath the subscapularis tendon.
The inferior extension of the capsule is closely related to the axillary nerve
at the surgical neck and this nerve is at risk in anteroinferior dislocations. It
also means that proximally sited osteomyelitis may progress to septic
arthritis.

Movements and muscles


Flexion Anterior part of deltoid
Pectoralis major
Biceps
Coracobrachialis

Extension Posterior deltoid


Teres major
Latissimus dorsi

Adduction Pectoralis major


yomna92009@yahoo.com - MRCS Part A - My account

Reference ranges End and review

Question 12 of 158 Question stats Score: 16.7%

1
A 7.5%
A 34 year old lady presents with symptoms of faecal incontinence. Ten years 2
B 6.5%
previously she gave birth to a child by normal vaginal delivery. Injury to which of 3
C 66.5%
the following nerves is most likely to account for this process?
D 11.8% 4
E 7.8% 5
A. Genitofemoral
66.5% of users answered this 6
B. Ilioinguinal question correctly
7
C. Pudendal
8
D. Hypogastric autonomic nerve
9
E. Obturator
10

Next question 11

12
Theme from April 2012 Exam
Damage to the pudendal nerve is classically associated with faecal incontinence
and it is for this reason that sacral neuromodulation is a popular treatment for the
condition. Injury to the hypogastric autonomic nerves is an aetiological factor in
the development of constipation.

Pudendal nerve

The pudendal nerve arises from nerve roots S2, S3 and S4 and exits the pelvis
through the greater sciatic foramen. It re-enters the pelvis through the lesser
sciatic foramen. It travels inferior to give innervation to the anal sphincters and
external urethral sphincter. It also provides cutaneous innervation to the region of
perineum surrounding the anus and posterior vulva.

Traction and compression of the pudendal nerve by the foetus in late pregnancy
may result in late onset pudendal neuropathy which may be part of the process
involved in the development of faecal incontinence.

Rate question: Next question

Com m ent on this question

All contents of this site are © 2012 E-Medical Revision Ltd Terms and Conditions Privacy policy
yomna92009@yahoo.com - MRCS Part A - My account

Reference ranges End and review

Question 13 of 158 Question stats Score: 15.4%

1
A 12.7%
During a difficult thyroidectomy haemorrhage is noted from the thyroidea ima 2
B 7.9%
vessel. From which structure does this vessel usually arise? 3
C 60.9%
D 6.1% 4
A. External carotid artery E 12.3% 5
B. Internal carotid artery 6
60.9% of users answered this
C. Brachiocephalic artery question correctly
7
D. Axillary artery
8
E. Superior thyroid artery
9

10
Next question
11

12
Rhyme isthmus location:
13

Rings 2,3,4 make the isthmus floor

This accessory vessel which usually lies at the inferior aspect of the gland is
derived either from the brachiocephalic artery or the arch of the aorta.

Thyroid gland

Right and left lobes connected by isthmus


Surrounded by sheath from pretracheal layer of deep fascia
Apex: Lamina of thyroid cartilage
Base: 4th-5th tracheal ring
Pyramidal lobe: from isthmus
May be attached to foramen caecum at the base of the tongue

Relations

Anteromedially Sternothyroid
Superior belly of omohyoid
Sternohyoid
Anterior aspect of sternocleidomastoid

Posterolaterally Carotid sheath

Medially Larynx
Trachea
Pharynx
Oesophagus
Cricothyroid muscle
External laryngeal nerve (near superior thyroid artery)
Recurrent laryngeal nerve (near inferior thyroid artery)

Posterior Parathyroid glands


Anastomosis of superior and inferior thyroid arteries

Isthmus Anteriorly: Sternothyroids, sternohyoids, anterior jugular veins


Posteriorly: 2nd, 3rd, 4th tracheal rings (attached via Ligament of
Berry)

Blood Supply

Arterial Superior thyroid artery (1st branch of external carotid)


Inferior thyroid artery (from thyrocervical trunk)
Thyroidea ima (in 10% of population -from brachiocephalic artery or aorta)

Venous Superior and middle thyroid veins - into the IJV


yomna92009@yahoo.com - MRCS Part A - My account

Reference ranges End and review

Question 14 of 158 Question stats Score: 14.3%

1
A 5.7%
A 56 year old man undergoes an abdomino-perineal excision of the rectum. He is 2
B 6.9%
assessed in the outpatient clinic post operatively. His wounds are well healed. 3
C 62.5%
However, he complains of impotence. Which of the following best explains this
D 10% 4
problem?
E 14.9% 5

62.5% of users answered this 6


A. Sciatic nerve injury
question correctly
7
B. Damage to the internal iliac artery
8
C. Damage to the hypogastric nerve plexus
D. Damage to the vas 9

E. Damage to the genitofemoral nerve 10

11

Next question 12

13
Theme from 2012 Exam
14
Autonomic supply to the penis is via the hypogastric plexus of nerves. These may
be damaged during mobilisation of the proximal rectum from the sacrum and
result in impotence post operatively. The addition of radiotherapy greatly
increases the risks of impotence following surgery.

Penile erection

Physiology of erection

Autonomic Sympathetic nerves originate from T11-L2 and parasympathetic nerves


from S2-4 join to form pelvic plexus.
Parasympathetic discharge causes erection, sympathetic discharge
causes ejaculation and detumescence.

Somatic Supplied by dorsal penile and pudendal nerves. Efferent signals are relayed
nerves from Onufs nucleus (S2-4) to innervate ischiocavernosus and
bulbocavernosus muscles.

Autonomic discharge to the penis will trigger the veno-occlusive mechanism which
triggers the flow of arterial blood into the penile sinusoidal spaces. As the inflow
increases the increased volume in this space will secondarily lead to compression
of the subtunical venous plexus with reduced venous return. During the
detumesence phase the arteriolar constriction will reduce arterial inflow and
thereby allow venous return to normalise.

Priapism
Prolonged unwanted erection, in the absence of sexual desire, lasting more than
4 hours.

Classification of priaprism
Low flow priaprism Due to veno-occlusion (high intracavernosal pressures).

Most common type


Often painful
Often low cavernosal flow
If present for >4 hours requires emergency treatment

High flow priaprism Due to unregulated arterial blood flow.

Usually presents as semi rigid painless erection

Recurrent Typically seen in sickle cell disease, most commonly of high flow
priaprism type.

Causes

Intracavernosal drug therapies (e.g. for erectile dysfunction>


Blood disorders such as leukaemia and sickle cell disease
Neurogenic disorders such as spinal cord transection
Trauma to penis resulting in arterio-venous malformations

Tests

Exclude sickle cell/ leukaemia


Consider blood sampling from cavernosa to determine whether high or low
flow (low flow is often hypoxic)

Management

Ice packs/ cold showers


If due to low flow then blood may be aspirated from copora or try
intracavernosal alpha adrenergic agonists.
Delayed therapy of low flow priaprism may result in erectile dysfunction.

Rate question: Next question

Com m ent on this question

All contents of this site are © 2012 E-Medical Revision Ltd Terms and Conditions Privacy policy
yomna92009@yahoo.com - MRCS Part A - My account

Reference ranges End and review

Question 15 of 158 Question stats Score: 13.3%

1
A 7.2%
Which of the following is not a branch of the descending abdominal aorta? 2
B 16.5%
C 7% 3

A. Inferior mesenteric artery D 62.4% 4

B. Inferior phrenic artery E 6.8% 5

C. Superior mesenteric artery 62.4% of users answered this 6


question correctly
D. Superior phrenic artery 7

E. Renal artery 8

9
Next question
10

11

Mnemonic for the Descending abdominal aorta branches from diaphragm to 12


iliacs:
13

'Prostitutes Cause Sagging Swollen Red Testicles [in men] Living In 14

Sin': 15

Phrenic [inferior]
Celiac
Superior mesenteric
Suprarenal [middle]
Renal
Testicular ['in men' only]
Lumbars
Inferior mesenteric
Sacral

The superior phrenic artery branches from the aorta in the thorax.

Abdominal aortic branches

Branches Level Paired Type

Inferior phrenic T12 (Upper border) Yes Parietal

Coeliac T12 No Visceral

Superior mesenteric L1 No Visceral

Middle suprarenal L1 Yes Visceral

Renal L1-L2 Yes Visceral

Gonadal L2 Yes Visceral

Lumbar L1-L4 Yes Parietal

Inferior mesenteric L3 No Visceral

Median sacral L4 No Parietal

Common iliac L4 Yes Terminal

Rate question: Next question

Com m ent on this question

All contents of this site are © 2012 E-Medical Revision Ltd Terms and Conditions Privacy policy
yomna92009@yahoo.com - MRCS Part A - My account

Reference ranges End and review

Question 16 of 158 Question stats Score: 12.5%

1
A 7.2%
A 40 year old lady presents with varicose veins, these are found to originate from 2
B 60.1%
the short saphenous vein. As the vein is mobilised close to its origin which of the 3
C 13.3%
following structures is at greatest risk of injury?
D 11.2% 4
E 8.3% 5
A. Sciatic nerve
60.1% of users answered this 6
B. Sural nerve question correctly
7
C. Common peroneal nerve
8
D. Tibial nerve
9
E. Popliteal artery
10

Next question 11

12
The sural nerve is closely related and damage to this structure is a major cause 13
of litigation. The other structures may all be injured but the risks are lower.
14

Popliteal fossa 15

16
Boundaries of the popliteal fossa
Laterally Biceps femoris above, lateral head of gastrocnemius and plantaris below

Medially Semimembranosus and semitendinosus above, medial head of gastrocnemius


below

Floor Popliteal surface of the femur, posterior ligament of knee joint and popliteus
muscle

Roof Superficial and deep fascia

Image showing the popliteal fossa

Image sourced from Wikipedia

Contents

Popliteal artery and vein


Small saphenous vein
Common peroneal nerve
Tibial nerve
Posterior cutaneous nerve of the thigh
Genicular branch of the obturator nerve
Lymph nodes
yomna92009@yahoo.com - MRCS Part A - My account

Reference ranges End and review

Question 17 of 158 Question stats Score: 11.8%

1
A 53.7%
A 23 year old man is admitted with a suspected ureteric colic. A KUB style x-ray is 2
B 10.3%
obtained. In which of the following locations is the stone most likely to be 3
C 17.5%
visualised?
D 7.5% 4
E 11.1% 5
A. The tips of the transverse processes between L2 and L5
53.7% of users answered this 6
B. The tips of transverse processes between T10-L1 question correctly
7
C. At the crest of the ilium
8
D. Over the S3 foramina
9
E. Over the sacrococcygeal joint
10

Next question 11

12
The ureter lies anterior to L2 to L5 and stones may be visualised at these points, 13
they may also be identified over the sacro-iliac joints.
14

Ureter 15

16

17
25-35 cm long
Muscular tube lined by transitional epithelium
Surrounded by thick muscular coat. Becomes 3 muscular layers as it
crosses the bony pelvis
Retroperitoneal structure overlying transverse processes L2-L5
Lies anterior to bifurcation of iliac vessels
Blood supply is segmental; renal artery, aortic branches, gonadal
branches, common iliac and internal iliac
Lies beneath the uterine artery

Rate question: Next question

Com m ent on this question

All contents of this site are © 2012 E-Medical Revision Ltd Terms and Conditions Privacy policy
yomna92009@yahoo.com - MRCS Part A - My account

Reference ranges End and review

Question 18 of 158 Question stats Score: 16.7%

1
A 20.5%
Damage to the posterior cord of the brachial plexus will not result in any of the 2
B 18.5%
following except: 3
C 11.8%
D 14.9% 4
A. Klumpkes palsy E 34.3% 5
B. Anaesthesia overlying the lateral aspect of the forearm 6
34.3% of users answered this
C. A warm sweaty hand on the affected side question correctly
7
D. Loss of flexion of the arm
8
E. Anaesthesia overlying the posterior surface of the arm
9

10
Next question
11

The radial nerve gives cutaneous branches which supply the forearm posteriorly 12
and the arm laterally. Division of the posterior cord will impair the upper level of 13
cutaneous sensation. However, the lateral cutaneous nerve of the forearm arises
14
from the musculocutaneous nerve and would be unaffected. Loss of sympathetic
function would not result in a sweaty hand. Klumpkes palsy occurs when the lower 15
roots are C8-T1 are damaged. 16

17
Brachial plexus
18

Origin Anterior rami of C5 to T1

Sections of the Roots, trunks, divisions, cords, branches


plexus Mnemonic:Real Teenagers Drink Cold Beer

Roots Located in the posterior triangle


Pass between scalenus anterior and medius

Trunks Located posterior to middle third of clavicle


Upper and middle trunks related superiorly to the subclavian
artery
Lower trunk passes over 1st rib posterior to the subclavian
artery

Divisions Apex of axilla

Cords Related to axillary artery

Diagram illustrating the branches of the brachial plexus

Image sourced from Wikipedia

Cutaneous sensation of the upper limb


yomna92009@yahoo.com - MRCS Part A - My account

Reference ranges End and review

Question 19 of 158 Question stats Score: 15.8%

1
A 9.7%
A 72 year old man with non reconstructible arterial disease is undergoing an 2
B 67.9%
above knee amputation. The posterior compartment muscles are divided. Which 3
C 6.3%
of the following muscles does not lie in the posterior compartment of the thigh?
D 7.5% 4
E 8.6% 5
A. Biceps femoris
67.9% of users answered this 6
B. Quadriceps femoris question correctly
7
C. Semitendinosus
8
D. Semimembranosus
9
E. None of the above
10

Next question 11

12
The quadriceps femoris lies in the anterior compartment. 13

14
Fascial compartments of the leg
15

Compartments of the thigh 16

17
Formed by 3 septae passing from the femur to the fascia lata.
18
Compartment Nerve Muscles Blood supply
19
Anterior Femoral Iliacus Femoral artery
compartment Tensor fasciae latae
Sartorius
Quadriceps femoris

Medial Obturator Adductor Profunda femoris artery and


compartment longus/magnus/brevis obturator artery
Gracilis
Obturator externus

Posterior Sciatic Semimembranosus Branches of Profunda


compartment Semitendinosus femoris artery
Biceps femoris

Compartments of the lower leg


Separated by the interosseous membrane (anterior and posterior compartments),
anterior fascial septum (separate anterior and lateral compartments) and
posterior fascial septum (separate lateral and posterior compartments)

Compartment Nerve Muscles Blood


supply

Anterior Deep Tibialis anterior Anterior


compartment peroneal Extensor digitorum longus tibial
nerve Extensor hallucis longus artery
Peroneus tertius

Posterior Tibial Muscles: deep and superficial Posterior


compartment compartments (separated by deep tibial
transverse fascia)
Deep: Flexor hallucis longus, Flexor
digitalis longus, Tibialis posterior,
Popliteus
Superficial: Gastrocnemius, Soleus,
Plantaris

Lateral Superficial Peroneus longus/brevis Anterior


compartment peroneal tibial
yomna92009@yahoo.com - MRCS Part A - My account

Reference ranges End and review

Question 20 of 158 Question stats Score: 15%

1
A 5.8%
A woman develops winging of the scapula following a Patey mastectomy. What is 2
B 5.6%
the most likely cause? 3
C 74.9%
D 6.8% 4
A. Division of pectoralis minor to access level 3 axillary nodes E 6.8% 5
B. Damage to the brachial plexus during axillary dissection 6
74.9% of users answered this
C. Damage to the long thoracic nerve during axillary dissection question correctly
7
D. Division of the thoracodorsal trunk during axillary dissection
8
E. Damage to the thoracodorsal trunk during axillary dissection
9

10
Next question
11

Theme from January 2012 exam 12


The serratus anterior muscle is supplied by the long thoracic nerve which runs 13
along the surface of serratus anterior and is liable to injury during nodal
14
dissection. Although pectoralis minor is divided during a Patey mastectomy (now
seldom performed) it is rare for this alone to produce winging of the scapula. 15

16
Long thoracic nerve
17

18
Derived from ventral rami of C5, C6, and C7 (close to their emergence from 19
intervertebral foramina)
20
It runs downward and passes either anterior or posterior to the middle
scalene muscle
It reaches upper tip of serratus anterior muscle and descends on outer
surface of this muscle, giving branches into it
Winging of Scapula occurs in long thoracic nerve injury (most common) or
from spinal accessory nerve injury (which denervates the trapezius) or a
dorsal scapular nerve injury

Rate question: Next question

Com m ent on this question

All contents of this site are © 2012 E-Medical Revision Ltd Terms and Conditions Privacy policy
yomna92009@yahoo.com - MRCS Part A - My account

Reference ranges End and review

Question 21 of 158 Question stats Score: 14.3%

1
A 10.9%
Which of the following structures is not closely related to the posterior tibial 2
B 12.6%
artery? 3
C 38.2%
D 14.8% 4
A. Soleus posteriorly E 23.5% 5
B. Tibial nerve medially 6
38.2% of users answered this
C. Deep peroneal nerve laterally question correctly
7
D. Flexor hallucis longus postero-inferiorly
8
E. Popliteus
9

10
Next question
11

The deep peroneal nerve lies in the anterior compartment. The tibial nerve lies 12
medially. At its termination it lies deep to the flexor retinaculum. 13

14
Posterior tibial artery
15

16
Larger terminal branch of the popliteal artery
17
Terminates by dividing into the medial and lateral plantar arteries
Accompanied by two veins throughout its length 18
Position of the artery corresponds to a line drawn from the lower angle of 19
the popliteal fossa, at the level of the neck of the fibula, to a point midway
20
between the medial malleolus and the most prominent part of the heel
21

Relations of the posterior tibial artery


Proximal to distal
Anteriorly Tibialis posterior
Flexor digitorum longus
Posterior surface of tibia and ankle joint

Posterior Tibial nerve 2.5 cm distal to its origin


Fascia overlying the deep muscular layer
Proximal part covered by gastrocnemius and soleus
Distal part covered by skin and fascia

Rate question: Next question

Com m ent on this question

All contents of this site are © 2012 E-Medical Revision Ltd Terms and Conditions Privacy policy
yomna92009@yahoo.com - MRCS Part A - My account

Reference ranges End and review

Question 22 of 158 Question stats Score: 13.6%

1
A 8.3%
Which of the following is not closely related to the capitate bone? 2
B 40.4%
C 31% 3

A. Lunate bone D 7.6% 4

B. Scaphoid bone E 12.7% 5

C. Ulnar nerve 31% of users answered this 6


question correctly
D. Hamate bone 7

E. Trapezoid bone 8

9
Next question
10

11
The ulnar nerve and artery lie adjacent to the pisiform bone. The capitate bone
articulates with the lunate, scaphoid, hamate and trapezoid bones, which are 12
therefore closely related to it. 13

14
Capitate bone
15

This is the largest of the carpal bones. It is centrally placed with a rounded head 16
set into the cavities of the lunate and scaphoid bones. Flatter articular surfaces 17
are present for the hamate medially and the trapezoid laterally. Distally the bone
18
articulates predominantly with the middle metacarpal.
19

Rate question: Next question 20

21
Com m ent on this question
22

All contents of this site are © 2012 E-Medical Revision Ltd Terms and Conditions Privacy policy
yomna92009@yahoo.com - MRCS Part A - My account

Reference ranges End and review

Question 23 of 158 Question stats Score: 13%

1
A 14.5%
An 82 year old lady with aortic stenosis is due to undergo an aortic valve 2
B 32.5%
replacement. Which of the following is not a feature of the aortic valve? 3
C 18.5%
D 20.4% 4
A. The left coronary artery arises superior to the left posterior cusp E 14.2% 5
B. The right coronary artery arises superior to the right posterior cusp 6
32.5% of users answered this
C. It has 1 anterior cusp question correctly
7
D. It has no chordae
8
E. It has 3 cusps in total
9

10
Next question
11

The right posterior valve has a sinus but the right coronary artery does not arise 12
from this. The coronary arteries arise superior to the anterior and left posterior 13
cusps.
14

Heart anatomy 15

16
The walls of each cardiac chamber comprise: 17

Epicardium 18
Myocardium 19
Endocardium
20

21
Cardiac muscle is attached to the cardiac fibrous skeleton.
22

Relations 23
The heart and roots of the great vessels within the pericardial sac are related
anteriorly to the sternum, medial ends of the 3rd to 5th ribs on the left and their
associated costal cartilages. The heart and pericardial sac are situated obliquely
two thirds to the left and one third to the right of the median plane.

The pulmonary valve lies at the level of the left third costal cartilage.
The mitral valve lies at the level of the fourth costal cartilage.

Coronary sinus
This lies in the posterior part of the coronary groove and receives blood from the
cardiac veins. The great cardiac vein lies at its left and the middle and small
cardiac veins lie on its right. The smallest cardiac veins drain into the atria
directly.

Aortic sinus
Right coronary artery arises from the right aortic sinus, the left is derived from the
left aortic sinus and no vessel emerges from the posterior sinus.

Right and left ventricles

Structure Left Ventricle

A-V Valve Mitral (double leaflet)

Walls Twice as thick as right

Trabeculae carnae Much thicker and more numerous

Right coronary artery


The RCA supplies:

Right atrium
Diaphragmatic part of the left ventricle
Usually the posterior third of the interventricular septum
yomna92009@yahoo.com - MRCS Part A - My account

Reference ranges End and review

Question 24 of 158 Question stats Score: 12.5%

1
A 14.2%
Which of the following statements relating to the tympanic membrane is false? 2
B 12%
C 39.1% 3

A. The umbo marks the point of attachment of the handle of the D 21.4% 4
malleus to the tympanic membrane E 13.3% 5
B. The lateral aspect of the tympanic membrane is lined by stratified
39.1% of users answered this 6
squamous epithelium
question correctly
7
C. The chorda tympani nerve runs medial to the pars tensa
8
D. The medial aspect of the tympanic membrane is lined by mucous
membrane 9

E. The tympanic membrane is approximately 1cm in diameter 10

11
Next question
12

13
The chorda tympani runs medially to the pars flaccida. The relationship is shown
from the medial aspect in the dissection below. 14

15

16

17

18

19

20

21

22

23

24

Image sourced from Wikipedia

Ear- anatomy

The ear is composed of three anatomically distinct regions.

External ear
Auricle is composed of elastic cartilage covered by skin. The lobule has no
cartilage and contains fat and fibrous tissue.

External auditory meatus is approximately 2.5cm long.


Lateral third of the external auditory meatus is cartilaginous and the medial two
thirds is bony.

The region is innervated by the greater auricular nerve. The auriculotemporal


branch of the trigeminal nerve supplies most the of external auditory meatus and
the lateral surface of the auricle.

Middle ear
Space between the tympanic membrane and cochlea. The aditus leads to the
mastoid air cells is the route through which middle ear infections may cause
mastoiditis. Anteriorly the eustacian tube connects the middle ear to the naso
pharynx.
The tympanic membrane consists of:
yomna92009@yahoo.com - MRCS Part A - My account

Reference ranges End and review

Question 25 of 158 Question stats Score: 12%

1
A 6.9%
An injury to the spinal accessory nerve will affect which of the following 2
B 9%
movements? 3
C 20.4%
D 49% 4
A. Lateral rotation of the arm E 14.6% 5
B. Adduction of the arm at the glenohumeral joint 6
49% of users answered this
C. Protraction of the scapula question correctly
7
D. Upward rotation of the scapula
8
E. Depression of the scapula
9

10
Next question
11

The spinal accessory nerve innervates trapezius. The entire muscle will retract 12
the scapula. However, its upper and lower fibres act together to upwardly rotate it. 13

14
Shoulder joint
15

16
Shallow synovial ball and socket type of joint.
17
It is an inherently unstable joint, but is capable to a wide range of
movement. 18
Stability is provided by muscles of the rotator cuff that pass from the 19
scapula to insert in the greater tuberosity (all except sub scapularis-lesser
20
tuberosity).
21

22
Glenoid labrum
23
Fibrocartilaginous rim attached to the free edge of the glenoid cavity
24
Tendon of the long head of biceps arises from within the joint from the
supraglenoid tubercle, and is fused at this point to the labrum. 25
The long head of triceps attaches to the infraglenoid tubercle

Fibrous capsule

Attaches to the scapula external to the glenoid labrum and to the labrum
itself (postero-superiorly)
Attaches to the humerus at the level of the anatomical neck superiorly and
the surgical neck inferiorly
Anteriorly the capsule is in contact with the tendon of subscapularis,
superiorly with the supraspinatus tendon, and posteriorly with the tendons
of infraspinatus and teres minor. All these blend with the capsule towards
their insertion.
Two defects in the fibrous capsule; superiorly for the tendon of biceps.
Anteriorly there is a defect beneath the subscapularis tendon.
The inferior extension of the capsule is closely related to the axillary nerve
at the surgical neck and this nerve is at risk in anteroinferior dislocations. It
also means that proximally sited osteomyelitis may progress to septic
arthritis.

Movements and muscles


Flexion Anterior part of deltoid
Pectoralis major
Biceps
Coracobrachialis

Extension Posterior deltoid


Teres major
Latissimus dorsi

Adduction Pectoralis major


Latissimus dorsi
yomna92009@yahoo.com - MRCS Part A - My account

Reference ranges End and review

Question 26 of 158 Question stats Score: 11.5%

1
A 13.1%
Which of the following is not contained within the middle mediastinum? 2
B 19.7%
C 46.5% 3

A. Main bronchi D 8.1% 4

B. Arch of the azygos vein E 12.7% 5

C. Thoracic duct 46.5% of users answered this 6


question correctly
D. Pericardium 7

E. Aortic root 8

9
Next question
10

11
The thoracic duct lies within the posterior and superior mediastinum.
12

Mediastinum 13

14
Region between the pulmonary cavities.
15
It is covered by the mediastinal pleura. It does not contain the lungs.
It extends from the thoracic inlet superiorly to the diaphragm inferiorly. 16

17
Mediastinal regions
18
Superior mediastinum 19
Inferior mediastinum
20
Posterior mediastinum
Anterior mediastinum 21

22

Region Contents 23

24
Superior mediastinum Superior vena cava
Brachiocephalic veins 25
Arch of aorta
26
Thoracic duct
Trachea
Oesophagus
Thymus
Vagus nerve
Left recurrent laryngeal nerve
Phrenic nerve

Anterior mediastinum Thymic remnants


Lymph nodes
Fat

Middle mediastinum Pericardium


Heart
Aortic root
Arch of azygos vein
Main bronchi

Posterior mediastinum Oesophagus


Thoracic aorta
Azygos vein
Thoracic duct
Vagus nerve
Sympathetic nerve trunks
Splanchnic nerves

Rate question: Next question

Com m ent on this question


yomna92009@yahoo.com - MRCS Part A - My account

Reference ranges End and review

Question 27 of 158 Question stats Score: 11.1%

1
A 7.4%
A 55 year old man is due to undergo a radical prostatectomy for carcinoma of the 2
B 5.1%
prostate gland. Which of the following vessels directly supplies the prostate? 3
C 20%
D 61.2% 4
A. External iliac artery E 6.3% 5
B. Common iliac artery 6
61.2% of users answered this
C. Internal iliac artery question correctly
7
D. Inferior vesical artery
8
E. None of the above
9

10
Next question
11

The arterial supply to the prostate gland is from the inferior vesical artery, it is a 12
branch of the prostatovesical artery. The prostatovesical artery usually arises 13
from the internal pudendal and inferior gluteal arterial branches of the internal
14
iliac artery.
15

Prostate gland 16

17
The prostate gland is approximately the shape and size of a walnut and is located
18
inferior to the bladder. It is separated from the rectum by Denonvilliers fascia and
its blood supply is derived from the internal iliac vessels. The internal sphincter 19
lies at the apex of the gland and may be damaged during prostatic surgery, 20
affected individuals may complain of retrograde ejaculation.
21

Summary of prostate gland 22

Arterial supply Inferior vesical artery (from internal iliac) 23


Venous Prostatic venous plexus (to paravertebral veins) 24
drainage
25
Lymphatic Internal iliac nodes
drainage 26

Innervation Inferior hypogastric plexus 27

Dimensions Transverse diameter (4cm)


AP diameter (2cm)
Height (3cm)

Lobes Posterior lobe: posterior to urethra


Median lobe: posterior to urethra, in between ejaculatory ducts
Lateral lobes x 2
Isthmus

Zones Peripheral zone: subcapsular portion of posterior prostate. Most


prostate cancers are here
Central zone
Transition zone
Stroma

Relations
Anterior Pubic symphysis
Prostatic venous plexus

Posterior Denonvilliers fascia


Rectum
Ejaculatory ducts

Lateral Venous plexus (lies on prostate)


Levator ani (immediately below the puboprostatic ligaments)
yomna92009@yahoo.com - MRCS Part A - My account

Reference ranges End and review

Question 28 of 158 Question stats Score: 10.7%

1
A 50%
Which of the tumour types listed below is found most frequently in a person with B 0% 2
aggressive fibromatosis? C 25% 3
D 0%
4
E 25%
A. Medullary thyroid cancer 5
B. Basal cell carcinoma of the skin 25% of users answered this
6
question correctly
C. Desmoid tumours 7
D. Dermoid tumours
8
E. Malignant melanoma
9

10
Next question
11

12

13
Aggressive fibromatosis
14

15
Aggressive fibromatosis is a disorder consisting of desmoid tumours, which
behave in a locally aggressive manner. Desmoid tumours may be identified in 16
both abdominal and extra-abdominal locations. Metastatic disease is rare. The 17
main risk factor (for abdominal desmoids) is having APC variant of familial
18
adenomatous polyposis coli. Most cases are sporadic.
Treatment is by surgical excision. 19

20

Rate question: Next question 21

Com m ent on this question 22

23

24

25

26

27

28

All contents of this site are © 2012 E-Medical Revision Ltd Terms and Conditions Privacy policy
yomna92009@yahoo.com - MRCS Part A - My account

Reference ranges End and review

Question 29 of 158 Question stats Score: 13.8%

1
A 15.3%
Which nerve directly innervates the sinoatrial node? 2
B 37.2%
C 19.9% 3

A. Superior cardiac nerve D 5.3% 4

B. Right vagus nerve E 22.3% 5

C. Left vagus nerve 22.3% of users answered this 6


question correctly
D. Inferior cardiac nerve 7

E. None of the above 8

9
Next question
10

11
Theme from September 2011 Exam
Theme from September 2012 Exam 12
No single one of the above nerves is responsible for direct cardiac innervation 13
(which those who have handled the heart surgically will appreciate).
14
The heart receives its nerves from the superficial and deep cardiac plexuses. The
cardiac plexuses send small branches to the heart along the major vessels, 15
continuing with the right and left coronary arteries. The vagal efferent fibres 16
emerge from the brainstem in the roots of the vagus and accessory nerves, and
17
run to ganglia in the cardiac plexuses and within the heart itself.
18
The background vagal discharge serves to limit heart rate, and loss of this 19
background vagal tone accounts for the higher resting heart rate seen following
20
cardiac transplant.
21

Sinoatrial node 22

23

Located in the wall of the right atrium in the upper part of the sulcus 24
terminalis from which it extends anteriorly over the opening of the superior 25
vena cava.
26
In most cases it is supplied by the right coronary artery.
It has a complicated nerve supply from the cardiac nerve plexus that takes 27
both sympathetic and parasympathetic fibres that run alongside the main 28
vessels.
29

Rate question: Next question

Com m ent on this question

All contents of this site are © 2012 E-Medical Revision Ltd Terms and Conditions Privacy policy
yomna92009@yahoo.com - MRCS Part A - My account

Reference ranges End and review

Question 1 of 129 Question stats Score: 0%

1
A 6.9%
A 30 year old man presents with back pain and the surgeon tests the ankle reflex. B 17.7%
Which of the following nerve roots are tested in this manoeuvre?
C 9.8%
D 59.4%

A. S3 and S4 E 6.3%

B. L4 and L5
59.4% of users answered this
C. L3 and L4 question correctly

D. S1 and S2
E. S4 only

Next question

Theme from April 2012 Exam

Ankle reflex

The ankle reflex is elicited by tapping the Achilles tendon with a tendon hammer. It
tests the S1 and S2 nerve roots. It is typically delayed in L5 and S1 disk
prolapses.

Rate question: Next question

Com m ent on this question

All contents of this site are © 2012 E-Medical Revision Ltd Terms and Conditions Privacy policy
yomna92009@yahoo.com - MRCS Part A - My account

Reference ranges End and review

Question 2 of 129 Question stats Score: 50%

1
A 16.1%
Which of the following structures is not closely related to the piriformis muscle? 2
B 12.1%
C 9.4%

A. Superior gluteal nerve D 9.8%

B. Sciatic nerve E 52.6%

C. Inferior gluteal artery 52.6% of users answered this


question correctly
D. Inferior gluteal nerve
E. Medial femoral circumflex artery

Next question

Nerve supply of lateral hip rotators

Piriformis: ventral rami S1, S2


Obturator internus: nerve to obturator internus
Superior gemellus: nerve to obturator internus
Inferior gemellus: nerve to quadratus femoris
Quadrator femoris: nerve to quadrator femoris

The piriformis muscle is an important anatomical landmark in the gluteal region.


The following structures are closely related:

Sciatic nerve
Inferior gluteal artery and nerve
Superior gluteal artery and nerve

The medial femoral circumflex artery runs deep to quadratus femoris.

Gluteal region

Gluteal muscles

Gluteus maximus: inserts to gluteal tuberosity of the femur and iliotibial tract
Gluteus medius: attach to lateral greater trochanter
Gluteus minimis: attach to anterior greater trochanter
All extend and abduct the hip

Deep lateral hip rotators

Piriformis
Gemelli
Obturator internus
Quadratus femoris

Nerves

Superior gluteal nerve (L5, S1) Gluteus medius


Gluteus minimis
Tensor fascia lata

Inferior gluteal nerve Gluteus maximus

Damage to the superior gluteal nerve will result in the patient developing a
Trendelenberg gait. Affected patients are unable to abduct the thigh at the hip
joint. During the stance phase, the weakened abductor muscles allow the pelvis to
tilt down on the opposite side. To compensate, the trunk lurches to the weakened
side to attempt to maintain a level pelvis throughout the gait cycle. The pelvis
sags on the opposite side of the lesioned superior gluteal nerve.
yomna92009@yahoo.com - MRCS Part A - My account

Reference ranges End and review

Question 3 of 129 Question stats Score: 33.3%

1
A 3%
An 18 year old male presents to casualty with a depressed skull fracture. This is 2
B 30.3%
managed surgically. Over the next few days he complains of double vision on 3
C 25.8%
walking down stairs and reading. On examination the left eye cannot look
D 37.9%
downwards and medially. Which of the nerves listed below is most likely to be
E 3%
responsible?

37.9% of users answered this


question correctly
A. Facial
B. Oculomotor
C. Abducens
D. Trochlear
E. Trigeminal nerve

Next question

Theme from September 2012 Exam


The trochlear nerve has a relatively long intracranial course and this makes it
vulnerable to injury in head trauma. Head trauma is the commonest cause of an
acute fourth nerve palsy. The affected eye typically drifts upwards compared to
the normal eye. A 4th nerve palsy is the commonest cause of a vertical diplopia.
Looking down and in is difficult and the patient notices diplopia on descending the
stairs.

Disorders of the oculomotor system

Nerve Path Nerve palsy


features

Oculomotor Large nucleus at the midbrain Ptosis


nerve Fibres pass through the red nucleus and the Eye down and out
pyramidal tract; through the cavernous sinus into Unable to move the
the orbit eye superiorly,
inferiorly, medially
Pupil fixed and
dilated

Trochlear Longest intracranial course Vertical diplopia


nerve Only nerve to exit the dorsal aspect of brainstem (diplopia on

Nucleus at midbrain, passes between the descending the


posterior cerebral and superior cerebellar stairs)
arteries, through the cavernous sinus into the Unable to look
orbit down and in

Abducens Nucleus lies in the mid pons Convergence of


nerve eyes in primary
position
Lateral diplopia
towards side of
lesion
Eye deviates
medially

Rate question: Next question

Com m ent on this question

All contents of this site are © 2012 E-Medical Revision Ltd Terms and Conditions Privacy policy
yomna92009@yahoo.com - MRCS Part A - My account

Reference ranges End and review

Question 4 of 129 Question stats Score: 25%

1
A 9.8%
A 77 year old man with symptoms of intermittent claudication is due to have his 2
B 4.8%
ankle brachial pressure indices measured. The vessel is impalpable. Which of the 3
C 62.1%
following tendinous structures lies medial to it, that may facilitate its identification?
D 5.9% 4
E 17.3%
A. Extensor digitorum longus tendon
62.1% of users answered this
B. Peroneus tertius tendon question correctly
C. Extensor hallucis longus tendon
D. Extensor digitorum brevis tendon
E. Flexor digitorum longus tendon

Next question

The extensor hallucis longus tendon lies medial to the dorsalis pedis artery.

Foot- anatomy

Arches of the foot


The foot is conventionally considered to have two arches.

The longitudinal arch is higher on the medial than on the lateral side. The
posterior part of the calcaneum forms a posterior pillar to support the arch.
The lateral part of this structure passes via the cuboid bone and the lateral
two metatarsal bones. The medial part of this structure is more important.
The head of the talus marks the summit of this arch, located between the
sustentaculum tali and the navicular bone. The anterior pillar of the medial
arch is composed of the navicular bone, the three cuneiforms and the
medial three metatarsal bones.
The transverse arch is situated on the anterior part of the tarsus and the
posterior part of the metatarsus. The cuneiforms and metatarsal bases
narrow inferiorly, which contributes to the shape of the arch.

Intertarsal joints
Sub talar joint Formed by the cylindrical facet on the lower surface of the body of
the talus and the posterior facet on the upper surface of the
calcaneus. The facet on the talus is concave anteroposteriorly, the

other is convex. The synovial cavity of this joint does not


communicate with any other joint.

Talocalcaneonavicular The anterior part of the socket is formed by the concave articular
joint surface of the navicular bone, posteriorly by the upper surface of
the sustentaculum tali. The talus sits within this socket

Calcaneocuboid joint Highest point in the lateral part of the longitudinal arch. The lower
aspect of this joint is reinforced by the long plantar and plantar
calcaneocuboid ligaments.

Transverse tarsal joint The talocalcaneonavicular joint and the calcaneocuboid joint
extend accross the tarsus in an irregular transverse plane,
between the talus and calcaneus behind and the navicular and
cuboid bones in front. This plane is termed the transverse tarsal
joint.

Cuneonavicular joint Formed between the convex anterior surface of the navicular bone
and the concave surface of the the posterior ends of the three
cuneiforms.

Intercuneiform joints Between the three cuneiform bones.

Cuneocuboid joint Between the circular facets on the lateral cuneiform bone and the
cuboid. This joint contributes to the tarsal part of the transverse
arch.

A detailed knowledge of the joints is not required for MRCS Part A. However, the
contribution they play to the overall structure of the foot should be appreciated

Ligaments of the ankle joint and foot


yomna92009@yahoo.com - MRCS Part A - My account

Reference ranges End and review

Question 5 of 129 Question stats Score: 20%

1
A 15.6%
A 23 year old man falls over whilst intoxicated and a shard of glass transects his 2
B 41%
median nerve at the proximal border of the flexor retinaculum. His tendons escape 3
C 12.1%
injury. Which of the following features will not be present?
D 19.8% 4
E 11.4% 5
A. Weakness of thumb abduction
41% of users answered this
B. Loss of sensation on the dorsal aspect of the thenar eminence question correctly
C. Loss of power of opponens pollicis
D. Adduction and lateral rotation of the thumb at rest
E. Loss of power of abductor pollicis brevis

Next question

The median nerve may be injured proximal to the flexor retinaculum. This will
result in loss of flexor pollicis brevis, opponens pollicis and the first and second
lumbricals. When the patient is asked to close the hand slowly there is a lag of the
index and middle fingers reflecting the impaired lumbrical muscle function. The
sensory changes are minor and do not extend to the dorsal aspect of the thenar
eminence.
Abductor pollicis longus will contribute to thumb abduction (and is innervated by
the posterior interosseous nerve) and therefore abduction will be weaker than
prior to the injury.

Median nerve

The median nerve is formed by the union of a lateral and medial root respectively
from the lateral (C5,6,7) and medial (C8 and T1) cords of the brachial plexus; the
medial root passes anterior to the third part of the axillary artery. The nerve
descends lateral to the brachial artery, crosses to its medial side (usually passing
anterior to the artery). It passes deep to the bicipital aponeurosis and the median
cubital vein at the elbow.
It passes between the two heads of the pronator teres muscle, and runs on the
deep surface of flexor digitorum superficialis (within its fascial sheath).
Near the wrist it becomes superficial between the tendons of flexor digitorum
superficialis and flexor carpi radialis, deep to palmaris longus tendon. It passes
deep to the flexor retinaculum to enter the palm, but lies anterior to the long flexor
tendons within the carpal tunnel.

Branches
Region Branch

Upper No branches, although the nerve commonly communicates with the


arm musculocutaneous nerve

Forearm Pronator teres


Flexor carpi radialis
Palmaris longus
Flexor digitorum superficialis
Flexor pollicis longus
Flexor digitorum profundus (only the radial half)

Distal Palmar cutaneous branch


forearm

Hand Motor supply (LOAF)


(Motor)
Lateral 2 lumbricals
Opponens pollicis
Abductor pollicis brevis
Flexor pollicis brevis

Hand Over thumb and lateral 2 ½ fingers


(Sensory) On the palmar aspect this projects proximally, on the dorsal aspect only
the distal regions are innervated with the radial nerve providing the more
proximal cutaneous innervation.
yomna92009@yahoo.com - MRCS Part A - My account

Reference ranges End and review

Question 6 of 129 Question stats Score: 16.7%

1
A 6.6%
The following muscles are supplied by the recurrent laryngeal nerve except: 2
B 8.6%
C 71.3% 3

A. Transverse arytenoid D 5.5% 4

B. Posterior crico-arytenoid E 8% 5

C. Cricothyroid 71.3% of users answered this 6


question correctly
D. Oblique arytenoid
E. Thyroarytenoid

Next question

Innervates: all intrinsic larynx muscles (excluding cricothyroid)

The external branch of the superior laryngeal nerve innervates the cricothyroid
muscle.

Recurrent laryngeal nerve

Branch of the vagus nerve

Path

Right

Arises anterior to the subclavian artery and ascends obliquely next to the
trachea, behind the common carotid artery
It is either anterior or posterior to the inferior thyroid artery

Left

Arises left to the arch of the aorta


Winds below the aorta
Ascends along the side of the trachea

Then both

Pass in a groove between the trachea and oesophagus


Enters the larynx behind the articulation between the thyroid cartilage and
cricoid
Distributed to larynx muscles

Branches to

Cardiac plexus
Mucous membrane and muscular coat of the oesophagus and trachea

Innervates

Intrinsic larynx muscles (excluding cricothyroid)


yomna92009@yahoo.com - MRCS Part A - My account

Reference ranges End and review

Question 7 of 129 Question stats Score: 14.3%

1
A 14.1%
From which of the following embryological structures is the ureter derived? 2
B 11.2%
C 5.9% 3

A. Uranchus D 59.8% 4

B. Wolffian duct E 9% 5

C. Vitello-intestinal duct 59.8% of users answered this 6


question correctly
D. Mesonephric duct 7

E. Cloaca

Next question

The ureter develops from the mesonephric duct.

Ureter

25-35 cm long
Muscular tube lined by transitional epithelium
Surrounded by thick muscular coat. Becomes 3 muscular layers as it
crosses the bony pelvis
Retroperitoneal structure overlying transverse processes L2-L5
Lies anterior to bifurcation of iliac vessels
Blood supply is segmental; renal artery, aortic branches, gonadal
branches, common iliac and internal iliac
Lies beneath the uterine artery

Rate question: Next question

Com m ent on this question

All contents of this site are © 2012 E-Medical Revision Ltd Terms and Conditions Privacy policy
yomna92009@yahoo.com - MRCS Part A - My account

Reference ranges End and review

Question 8 of 129 Question stats Score: 12.5%

1
A 16.5%
A 16 year old boy is hit by a car and sustains a blow to the right side of his head. 2
B 59.8%
He is initially conscious but on arrival in the emergency department is comatose. 3
C 6.4%
On examination his right pupil is fixed and dilated. The neurosurgeons plan
D 11.8% 4
immediate surgery. What type of initial approach should be made?
E 5.5% 5

59.8% of users answered this 6


A. Left parieto-temporal craniotomy
question correctly
7
B. Right parieto-temporal craniotomy
8
C. Posterior fossa craniotomy
D. Left parieto-temporal burr holes
E. None of the above

Next question

Theme from April 2012 Exam


A unilateral dilated pupil is a classic sign of transtentorial herniation. The medial
aspect of the temporal lobe (uncus) herniates across the tentorium and causes
pressure on the ipsilateral oculomotor nerve, interrupting parasympathetic input
to the eye and resulting in a dilated pupil. In addition the brainstem is
compressed. As the ipsilateral oculomotor nerve is being compressed, craniotomy
(rather than Burr Holes) should be made on the ipsilateral side.

Head injury

Patients who suffer head injuries should be managed according to ATLS


principles and extra cranial injuries should be managed alongside cranial
trauma. Inadequate cardiac output will compromise CNS perfusion
irrespective of the nature of the cranial injury.

Types of traumatic brain injury


Extradural Bleeding into the space between the dura mater and the skull. Often results
haematoma from acceleration-deceleration trauma or a blow to the side of the head. The
majority of extradural haematomas occur in the temporal region where skull
fractures cause a rupture of the middle meningeal artery.

Features
Raised intracranial pressure
Some patients may exhibit a lucid interval

Subdural Bleeding into the outermost meningeal layer. Most commonly occur around
haematoma the frontal and parietal lobes. May be either acute or chronic.

Risk factors include old age and alcoholism.

Slower onset of symptoms than a extradural haematoma.

Subarachnoid Usually occurs spontaneously in the context of a ruptured cerebral


haemorrhage aneurysm but may be seen in association with other injuries when a patient
has sustained a traumatic brain injury

Pathophysiology

Primary brain injury may be focal (contusion/ haematoma) or diffuse


(diffuse axonal injury)
Diffuse axonal injury occurs as a result of mechanical shearing following
deceleration, causing disruption and tearing of axons
Intra-cranial haematomas can be extradural, subdural or intracerebral,
while contusions may occur adjacent to (coup) or contralateral (contre-
coup) to the side of impact
Secondary brain injury occurs when cerebral oedema, ischaemia, infection,
tonsillar or tentorial herniation exacerbates the original injury. The normal
cerebral auto regulatory processes are disrupted following trauma
rendering the brain more susceptible to blood flow changes and hypoxia
The Cushings reflex (hypertension and bradycardia) often occurs late and
is usually a pre terminal event

Management

Where there is life threatening rising ICP such as in extra dural haematoma
and whilst theatre is prepared or transfer arranged use of IV mannitol/
frusemide may be required.
Diffuse cerebral oedema may require decompressive craniotomy
Exploratory Burr Holes have little management in modern practice except
where scanning may be unavailable and to thus facilitate creation of formal
craniotomy flap
Depressed skull fractures that are open require formal surgical reduction
and debridement, closed injuries may be managed non operatively if there
is minimal displacement.
ICP monitoring is appropriate in those who have GCS 3-8 and normal CT
scan.
ICP monitoring is mandatory in those who have GCS 3-8 and Abnormal CT
scan.
Hyponatraemia is most likely to be due to syndrome of inappropriate ADH
secretion.
Minimum of cerebral perfusion pressure of 70mmHg in adults.
Minimum cerebral perfusion pressure of between 40 and 70 mmHg in
children.

Interpretation of pupillary findings in head injuries


Pupil size Light response Interpretation

Unilaterally dilated Sluggish or fixed 3rd nerve compression secondary to


tentorial herniation

Bilaterally dilated Sluggish or fixed Poor CNS perfusion


Bilateral 3rd nerve palsy

Unilaterally dilated Cross reactive (Marcus Optic nerve injury


or equal - Gunn)

Bilaterally May be difficult to Opiates


constricted assess Pontine lesions
Metabolic encephalopathy

Unilaterally Preserved Sympathetic pathway disruption


constricted

Rate question: Next question

Com m ent on this question

All contents of this site are © 2012 E-Medical Revision Ltd Terms and Conditions Privacy policy
yomna92009@yahoo.com - MRCS Part A - My account

Reference ranges End and review

Question 9 of 129 Question stats Score: 11.1%

1
A 12.4%
Which of the following relationship descriptions regarding the scalene muscles is 2
B 13.1%
incorrect? 3
C 51.6%
D 14.1% 4
A. The brachial plexus passes anterior to the middle scalene muscle E 8.8% 5
B. The phrenic nerve lies anterior to the anterior scalene muscle 6
51.6% of users answered this
C. The subclavian artery passes posterior to the middle scalene question correctly
7
D. The subclavian vein lies anterior to the anterior scalene muscle at
8
the level of the first rib
9
E. The anterior scalene inserts into the first rib

Next question

The subclavian artery passes anterior to the middle scalene.

Scalene muscles

The 3 paired muscles are:

Scalenus anterior: Elevate 1st rib and laterally flex the neck to same side
Scalenus medius: Same action as scalenus anterior
Scalenus posterior: Elevate 2nd rib and tilt neck to opposite side

Innervation Spinal nerves C4-6

Origin Transverse processes C2 to C7

Insertion First and second ribs

Important The brachial plexus and subclavian artery pass between


relations the anterior and middle scalenes through a space called
the scalene hiatus/fissure.
The subclavian vein and phrenic nerve pass anteriorly to
the anterior scalene as it crosses over the first rib.

Image sourced from Wikipedia


Image sourced from Wikipedia

Thoracic outlet syndrome


The scalenes are at risk of adhering to the fascia surrounding the brachial plexus
or shortening causing compression of the brachial plexus when it passes between
the clavicle and 1st rib causing thoracic outlet syndrome.

Rate question: Next question

Com m ent on this question

All contents of this site are © 2012 E-Medical Revision Ltd Terms and Conditions Privacy policy
yomna92009@yahoo.com - MRCS Part A - My account

Reference ranges End and review

Question 10 of 129 Question stats Score: 10%

1
A 5.7%
A 56 year old man is having a long venous line inserted via the femoral vein into 2
B 13%
the right atrium for CVP measurements. The catheter is advanced through the 3
C 5.7%
IVC. At which of the following levels does this vessel enter the thorax?
D 67.9% 4
E 7.7% 5
A. L2
67.9% of users answered this 6
B. T10 question correctly
7
C. L1
8
D. T8
9
E. T6
10

Next question

Theme from 2010 Exam


Theme from September 2012 Exam
The IVC passes through the diaphragm at T8.

Inferior vena cava

Origin

L5

Path

Left and right common iliac veins merge to form the IVC.
Passes right of midline
Paired segmental lumbar veins drain into the IVC throughout its length
The right gonadal vein empties directly into the cava and the left gonadal
vein generally empties into the left renal vein.
The next major veins are the renal veins and the hepatic veins
Pierces the central tendon of diaphragm at T8
Right atrium

Image sourced from Wikipedia

Relations
Anteriorly Small bowel, first and third part of duodenum, head of pancreas, liver and bile
duct, right common iliac artery, right gonadal artery

Posteriorly Right renal artery, right psoas, right sympathetic chain, coeliac ganglion

Levels
yomna92009@yahoo.com - MRCS Part A - My account

Reference ranges End and review

Question 11 of 129 Question stats Score: 18.2%

1
A 7.2%
A 23 year old man falls and injures his hand. There are concerns that he may 2
B 13.5%
have a scaphoid fracture as there is tenderness in his anatomical snuffbox on 3
C 17%
clinical examination. Which of the following forms the posterior border of this
D 16.8% 4
structure?
E 45.5% 5

45.5% of users answered this 6


A. Basilic vein
question correctly
7
B. Radial artery
8
C. Extensor pollicis brevis
D. Abductor pollicis longus 9

E. Extensor pollicis longus 10

11

Next question

Theme from 2009 Exam


Theme from September 2012 Exam
Its boundaries are extensor pollicis longus, medially (posterior border) and
laterally (anterior border) by the tendons of abductor pollicis longus and extensor
pollicis brevis.

Anatomical snuffbox

Posterior border Tendon of extensor pollicis longus

Anterior border Tendons of extensor pollicis brevis and abductor pollicis longus

Proximal border Styloid process of the radius

Distal border Apex of snuffbox triangle

Floor Trapezium and scaphoid

Content Radial artery

Image showing the anatomical snuffbox

Image sourced from Wikipedia

Rate question: Next question

Com m ent on this question

All contents of this site are © 2012 E-Medical Revision Ltd Terms and Conditions Privacy policy
yomna92009@yahoo.com - MRCS Part A - My account

Reference ranges End and review

Question 12 of 129 Question stats Score: 16.7%

1
A 15.1%
A 62 year old man is undergoing a left hemicolectomy for carcinoma of the 2
B 17.3%
descending colon. The registrar commences mobilisation of the left colon by 3
C 35.1%
pulling downwards and medially. Blood soon appears in the left paracolic gutter.
D 15.8% 4
The most likely source of bleeding is the:
E 16.8% 5

35.1% of users answered this 6


A. Marginal artery
question correctly
7
B. Left testicular artery
8
C. Spleen
D. Left renal vein 9

E. None of the above 10

11

Next question 12

The spleen is commonly torn by traction injuries in colonic surgery. The other
structures are associated with bleeding during colonic surgery but would not
manifest themselves as blood in the paracolic gutter prior to incision of the
paracolonic peritoneal edge.

Left colon

Position

As the left colon passes inferiorly its posterior aspect becomes


extraperitoneal, and the ureter and gonadal vessels are close posterior
relations that may become involved in disease processes
At a level of L3-4 (variable) the left colon becomes the sigmoid colon and
wholly intraperitoneal once again
The sigmoid colon is a highly mobile structure and may even lie of the right
side of the abdomen
It passes towards the midline, the taenia blend and this marks the transition
between sigmoid colon and upper rectum.

Blood supply

Inferior mesenteric artery


However, the marginal artery (from the right colon) contributes and this
contribution becomes clinically significant when the IMA is divided surgically
(e.g. During AAA repair)

Rate question: Next question

Com m ent on this question

All contents of this site are © 2012 E-Medical Revision Ltd Terms and Conditions Privacy policy
yomna92009@yahoo.com - MRCS Part A - My account

Reference ranges End and review

Question 13 of 129 Question stats Score: 15.4%

1
A 39.7%
A man is undergoing excision of a sub mandibular gland. As the gland is 2
B 4.4%
mobilised, a vessel is injured lying between the gland and the mandible. Which of 3
C 7.2%
the following is this vessel most likely to be?
D 39.7% 4
E 8.9% 5
A. Lingual artery
39.7% of users answered this 6
B. Occipital artery question correctly
7
C. Superior thyroid artery
8
D. Facial artery
9
E. External jugular vein
10

Next question 11

12

13
The high salivary viscosity of submandibular gland secretions favors stone
formation.
Most stones are radio-opaque.

The facial artery lies between the gland and mandible and is often ligated during
excision of the gland. The lingual artery may be encountered but this is usually
later in the operative process as Whartons duct is mobilised.

Submandibular gland

Relations of the submandibular gland


Superficial Platysma, deep fascia and mandible
Submandibular lymph nodes
Facial vein (facial artery near mandible)
Marginal mandibular nerve
Cervical branch of the facial nerve

Deep Facial artery (inferior to the mandible)


Mylohoid muscle
Sub mandibular duct
Hyoglossus muscle
Lingual nerve
Submandibular ganglion
Hypoglossal nerve

Submandibular duct (Wharton's duct)

Opens lateral to the lingual frenulum on the anterior floor of mouth.


5 cm length
Lingual nerve wraps around Wharton's duct. As the duct passes forwards it
crosses medial to the nerve to lie above it and then crosses back, lateral to
it, to reach a position below the nerve.

Innervation

Sympathetic innervation- Superior Cervical ganglion via the Lingual nerve


Parasympathetic innervation- Submandibular ganglion

Arterial supply
Branch of the Facial artery. The facial artery passes through the gland to groove
its deep surface. It then emerges onto the face by passing between the gland and
the mandible.

Venous drainage
Anterior Facial vein (lies deep to the Marginal Mandibular nerve)

Lymphatic drainage
yomna92009@yahoo.com - MRCS Part A - My account

Reference ranges End and review

Question 1 of 113 Question stats Score: 0%

1
A 8.1%
The integrity of which muscle is assessed by the Trendelenberg test? B 12.5%
C 6.7%

A. Sartorius D 64.2%

B. Quadratus femoris E 8.5%

C. Semimembranosus 64.2% of users answered this


question correctly
D. Gluteus medius
E. Piriformis

Next question

Theme from 2011 Exam

Trendelenberg test

Injury or division of the superior gluteal nerve results in a motor deficit that
consists of weakened abduction of the thigh by gluteus medius, a disabling
gluteus medius limp and a compensatory list of the body weakened gluteal side.
The compensation results in a gravitational shift so that the body is supported on
the unaffected limb.
When a person is asked to stand on one leg the gluteus medius usually contracts
as soon as the contralateral leg leaves the floor, preventing the pelvis from
dipping towards the unsupported side. When a person with paralysis of the
superior gluteal nerve is asked to stand on one leg, the pelvis on the
unsupported side descends, indicating that the gluteus medius on the affected
side is weak or non functional ( a positive Trendelenberg test).

Rate question: Next question

Com m ent on this question

All contents of this site are © 2012 E-Medical Revision Ltd Terms and Conditions Privacy policy
yomna92009@yahoo.com - MRCS Part A - My account

Reference ranges End and review

Question 2 of 113 Question stats Score: 0%

1
A 14.1%
A 52 year old female renal patient needs a femoral catheter to allow for 2
B 42.5%
haemodialysis. Which of the structures listed below is least likely to be
C 17.9%
encountered during its insertion?
D 6%

E 19.5%
A. Great saphenous vein
42.5% of users answered this
B. Deep circumflex iliac artery question correctly
C. Superficial circumflex iliac artery
D. Femoral vein
E. Femoral branch of the genitofemoral nerve

Next question

Femoral access catheters are typically inserted in the region of the femoral
triangle. Therefore the physician may encounter the femoral, vein, nerve,
branches of the femoral artery and tributaries of the femoral vein. The deep
circumflex iliac artery arises above the inguinal ligament and is therefore less
likely to be encountered than the superficial circumflex iliac artery which arises
below the inguinal ligament.

Femoral triangle anatomy

Boundaries
Superiorly Inguinal ligament

Laterally Sartorius

Medially Adductor longus

Floor Iliopsoas, adductor longus and pectineus

Roof Fascia lata and Superficial fascia


Superficial inguinal lymph nodes (palpable below the inguinal ligament)
Great saphenous vein

Image sourced from Wikipedia


yomna92009@yahoo.com - MRCS Part A - My account

Reference ranges End and review

Question 3 of 113 Question stats Score: 0%

1
A 18.6%
A 53 year old man with a chronically infected right kidney is due to undergo a 2
B 37.3%
nephrectomy. Which of the following structures would be encountered first during 3
C 15.5%
a posterior approach to the hilum of the right kidney?
D 15.3%

E 13.4%
A. Right renal artery
37.3% of users answered this
B. Ureter question correctly
C. Right renal vein
D. Inferior vena cava
E. Right testicular vein

Next question

The ureter is the most posterior structure at the hilum of the right kidney and
would therefore be encountered first during a posterior approach.

Renal arteries

The right renal artery is longer than the left renal artery
The renal vein/artery/pelvis enter the kidney at the hilum

Relations

Right:

Anterior- IVC, right renal vein, the head of the pancreas, and the descending part
of the duodenum.

Left:

Anterior- left renal vein, the tail of the pancreas.

Branches

The renal arteries are direct branches off the aorta (upper border of L2)
In 30% there may be accessory arteries (mainly left side). Instead of
entering the kidney at the hilum, they usually pierce the upper or lower part
of the organ.
Before reaching the hilum of the kidney, each artery divides into four or five
segmental branches (renal vein anterior and ureter posterior); which then
divide within the sinus into lobar arteries supplying each pyramid and
cortex.
Each vessel gives off some small inferior suprarenal branches to the
suprarenal gland, the ureter, and the surrounding cellular tissue and
muscles.

Rate question: Next question

Com m ent on this question

All contents of this site are © 2012 E-Medical Revision Ltd Terms and Conditions Privacy policy
yomna92009@yahoo.com - MRCS Part A - My account

Reference ranges End and review

Question 4 of 113 Question stats Score: 25%

1
A 12.2%
A 28 year old man is stabbed outside a nightclub in the upper arm. The median 2
B 11.2%
nerve is transected. Which of the following muscles will demonstrate impaired 3
C 18.6%
function as a result?
D 22% 4
E 35.9%
A. Palmaris brevis
35.9% of users answered this
B. Second and third interossei question correctly
C. Adductor pollicis
D. Abductor pollicis longus
E. Abductor pollicis brevis

Next question

Palmaris brevis - Ulnar nerve


Palmar interossei- Ulnar nerve
Adductor pollicis - Ulnar nerve
Abductor pollicis longus - Posterior interosseous nerve
Abductor pollicis brevis - Median nerve

The median nerve innervates all the short muscles of the thumb except the
adductor and the deep head of the short flexor. Palmaris and the interossei are
innervated by the ulnar nerve.

Median nerve

The median nerve is formed by the union of a lateral and medial root respectively
from the lateral (C5,6,7) and medial (C8 and T1) cords of the brachial plexus; the
medial root passes anterior to the third part of the axillary artery. The nerve
descends lateral to the brachial artery, crosses to its medial side (usually passing
anterior to the artery). It passes deep to the bicipital aponeurosis and the median
cubital vein at the elbow.
It passes between the two heads of the pronator teres muscle, and runs on the
deep surface of flexor digitorum superficialis (within its fascial sheath).
Near the wrist it becomes superficial between the tendons of flexor digitorum
superficialis and flexor carpi radialis, deep to palmaris longus tendon. It passes
deep to the flexor retinaculum to enter the palm, but lies anterior to the long flexor
tendons within the carpal tunnel.

Branches
Region Branch

Upper No branches, although the nerve commonly communicates with the


arm musculocutaneous nerve

Forearm Pronator teres


Flexor carpi radialis
Palmaris longus
Flexor digitorum superficialis
Flexor pollicis longus
Flexor digitorum profundus (only the radial half)

Distal Palmar cutaneous branch


forearm

Hand Motor supply (LOAF)


(Motor)
Lateral 2 lumbricals
Opponens pollicis
Abductor pollicis brevis
Flexor pollicis brevis

Hand Over thumb and lateral 2 ½ fingers


(Sensory) On the palmar aspect this projects proximally, on the dorsal aspect only
the distal regions are innervated with the radial nerve providing the more
yomna92009@yahoo.com - MRCS Part A - My account

Reference ranges End and review

Question 5 of 113 Question stats Score: 20%

1
A 33.3%
A 22 year old man sustains a blow to the side of his head with a baseball bat 2
B 13%
during a fight. He is initially conscious. However, he subsequently loses 3
C 7.2%
consciousness and then dies. Post mortem examination shows an extradural
D 40.5% 4
haematoma. The most likely culprit vessel is a branch of which of the following?
E 6.1% 5

A. Middle cerebral artery 40.5% of users answered this


question correctly
B. Internal carotid artery
C. Anterior cerebral artery
D. Maxillary artery
E. Mandibular artery

Next question

The middle meningeal artery is the most likely source of the extradural
haematoma in this setting. It is a branch of the maxillary artery. The middle
cerebral artery does not give rise to the middle meningeal artery. Note that the
question is asking for the vessel which gives rise to the middle meningeal artery
("the likely culprit vessel is a branch of which of the following")

Middle meningeal artery

Middle meningeal artery is typically the third branch of the first part of the
maxillary artery, one of the two terminal branches of the external carotid
artery. After branching off the maxillary artery in the infratemporal fossa, it
runs through the foramen spinosum to supply the dura mater (the
outermost meninges) .
The middle meningeal artery is the largest of the three (paired) arteries
which supply the meninges, the others being the anterior meningeal artery
and the posterior meningeal artery.
The middle meningeal artery runs beneath the pterion. It is vulnerable to
injury at this point, where the skull is thin. Rupture of the artery may give
rise to an extra dural hematoma.
In the dry cranium, the middle meningeal, which runs within the dura mater
surrounding the brain, makes a deep indention in the calvarium.
The middle meningeal artery is intimately associated with the
auriculotemporal nerve which wraps around the artery making the two
easily identifiable in the dissection of human cadavers and also easily
damaged in surgery.

Rate question: Next question

Com m ent on this question

All contents of this site are © 2012 E-Medical Revision Ltd Terms and Conditions Privacy policy
yomna92009@yahoo.com - MRCS Part A - My account

Reference ranges End and review

Question 6 of 113 Question stats Score: 16.7%

1
A 36.8%
A 72 year old man with carcinoma of the lung is undergoing a left 2
B 9.9%
pneumonectomy. The left main bronchus is divided. Which of the following 3
C 35.3%
thoracic vertebrae lies posterior to this structure?
D 10.1% 4
E 8% 5
A. T3
35.3% of users answered this 6
B. T7 question correctly
C. T6
D. T10
E. T1

Next question

The left main bronchus lies at T6. Topographical anatomy of the thorax is
important as it helps surgeons to predict the likely structures to be injured in
trauma scenarios (so popular with examiners)

Lung anatomy

The right lung is composed of 3 lobes divided by the oblique and transverse
fissures. The left lung has two lobes divided by the lingula.The apex of both lungs
is approximately 4cm superior to the sterno-costal joint of the first rib. Immediately
below this is a sulcus created by the subclavian artery.

Peripheral contact points of the lung

Base: diaphragm
Costal surface: corresponds to the cavity of the chest
Mediastinal surface: Contacts the mediastinal pleura. Has the cardiac
impression. Above and behind this concavity is a triangular depression
named the hilum, where the structures which form the root of the lung enter
and leave the viscus. These structures are invested by pleura, which,
below the hilus and behind the pericardial impression, forms the pulmonary
ligament

Right lung
Above the hilum is the azygos vein; Superior to this is the groove for the superior
vena cava and right innominate vein; behind this, and nearer the apex, is a furrow
for the innominate artery. Behind the hilus and the attachment of the pulmonary
ligament is a vertical groove for the oesophagus; In front and to the right of the
lower part of the oesophageal groove is a deep concavity for the extrapericardiac
portion of the inferior vena cava.

The root of the right lung lies behind the superior vena cava and the right atrium,
and below the azygos vein.

The right main bronchus is shorter, wider and more vertical than the left main
bronchus and therefore the route taken by most foreign bodies.

Left lung
Above the hilum is the furrow produced by the aortic arch, and then superiorly the
groove accommodating the left subclavian artery; Behind the hilum and
pulmonary ligament is a vertical groove produced by the descending aorta, and in
front of this, near the base of the lung, is the lower part of the oesophagus.

The root of the left lung passes under the aortic arch and in front of the
descending aorta.

Inferior borders of both lungs


yomna92009@yahoo.com - MRCS Part A - My account

Reference ranges End and review

Question 7 of 113 Question stats Score: 14.3%

1
A 11.6%
Which of the following regions of the male urethra is entirely surrounded by Bucks 2
B 10.7%
fascia? 3
C 28.8%
D 40.4% 4
A. Preprostatic part E 8.5% 5
B. Prostatic part 6
40.4% of users answered this
C. Membranous part question correctly
7
D. Spongiose part
E. None of the above

Next question

Theme from 2010 Exam


Bucks fascia is a layer of deep fascia that covers the penis it is continuous with
the external spermatic fascia and the penile suspensory ligament. The
membranous part of the urethra may partially pass through Bucks fascia as it
passes into the penis. However, the spongiose part of the urethra is contained
wholly within Bucks fascia.

Image of penile cross section


Bucks fascia corresponds to the layer of deep fascia

Image sourced from Wikipedia

Urethral anatomy

Female urethra
The female urethra is shorter and more acutely angulated than the male urethra.
It is an extra-peritoneal structure and embedded in the endopelvic fascia. The
neck of the bladder is subjected to transmitted intra-abdominal pressure and
therefore deficiency in this area may result in stress urinary incontinence.
Between the layers of the urogenital diaphragm the female urethra is surrounded
by the external urethral sphincter, this is innervated by the pudendal nerve. It
ultimately lies anterior to the vaginal orifice.

Male urethra
In males the urethra is much longer and is divided into four parts.

Pre-prostatic Extremely short and lies between the bladder and prostate gland.It has a
urethra stellate lumen and is between 1 and 1.5cm long.Innervated by sympathetic
noradrenergic fibres, as this region is composed of striated muscles
bundles they may contract and prevent retrograde ejaculation.

Prostatic This segment is wider than the membranous urethra and contains several
urethra openings for the transmission of semen (at the midpoint of the urethral
crest).

Membranous Narrowest part of the urethra and surrounded by external sphincter. It


urethra traverses the perineal membrane 2.5cm postero-inferior to the symphysis
pubis.

Penile Travels through the corpus songiosum on the underside of the penis. It is
urethra the longest urethral segment.It is dilated at its origin as the infrabulbar fossa
and again in the gland penis as the navicular fossa. The bulbo-urethral
glands open into the spongiose section of the urethra 2.5cm below the
perineal membrane.

The urothelium is transitional in nature near to the bladder and becomes


squamous more distally.

Rate question: Next question

Com m ent on this question

All contents of this site are © 2012 E-Medical Revision Ltd Terms and Conditions Privacy policy
yomna92009@yahoo.com - MRCS Part A - My account

Reference ranges End and review

Question 8 of 113 Question stats Score: 12.5%

1
A 11.3%
Which of the following statements relating to the knee joint is false? 2
B 30.7%
C 8.7% 3

A. It is the largest synovial joint in the body D 34.3% 4

B. When the knee is fully extended all ligaments of the knee joint are E 15% 5
taut
34.3% of users answered this 6
C. Rupture of the anterior cruciate ligament may result in question correctly
7
haemarthrosis
8
D. The posterior aspect of the patella is extrasynovial
E. The joint is innervated by the femoral, sciatic and obturator nerves

Next question

The posterior aspect is intrasynovial and the knee itself comprises the largest
synovial joint in the body. It may swell considerably following trauma such as ACL
injury. Which may be extremely painful owing to rich innervation from femoral,
sciatic and ( a smaller) contribution from the obturator nerve. During full extension
all ligaments are taut and the knee is locked.

Knee joint

The knee joint is a synovial joint, the largest and most complicated. It consists of
two condylar joints between the femure and tibia and a sellar joint between the
patella and the femur. The tibiofemoral articular surfaces are incongruent,
however, this is improved by the presence of the menisci. The degree of
congruence is related to the anatomical position of the knee joint and is greatest
in full extension.

Knee joint compartments

Tibiofemoral Comprise of the patella/femur joint, lateral and medial


compartments (between femur condyles and tibia)
Synovial membrane and cruciate ligaments partially
separate the medial and lateral compartments

Patellofemoral Ligamentum patellae


Actions: provides joint stability in full extension

Fibrous capsule
The capsule of the knee joint is a complex, composite structure with contributions
from adjacent tendons.
Anterior The capsule does not pass proximal to the patella. It blends with
fibres the tendinous expansions of vastus medialis and lateralis

Posterior These fibres are vertical and run from the posterior surface of the
fibres femoral condyles to the posterior aspect of the tibial condyle

Medial Attach to the femoral and tibial condyles beyond their articular
fibres margins, blending with the tibial collateral ligament

Lateral Attach to the femur superior to popliteus, pass over its tendon to
fibres head of fibula and tibial condyle

Bursae

Anterior Subcutaneous prepatellar bursa; between patella and skin


Deep infrapatellar bursa; between tibia and patellar ligament
Subcutaneous infrapatellar bursa; between distal tibial
tuberosity and skin

Laterally Bursa between lateral head of gastrocnemius and joint


capsule
Bursa between fibular collateral ligament and tendon of
biceps femoris
yomna92009@yahoo.com - MRCS Part A - My account

Reference ranges End and review

Question 9 of 113 Question stats Score: 11.1%

1
A 18.8%
A 48 year old lady is undergoing a left sided adrenalectomy for an adrenal 2
B 44.2%
adenoma. The superior adrenal artery is injured and starts to bleed, from which of 3
C 23.4%
the following does this vessel arise?
D 8.7% 4
E 4.9% 5
A. Left renal artery
44.2% of users answered this 6
B. Inferior phrenic artery question correctly
7
C. Aorta
8
D. Splenic
9
E. None of the above

Next question

The superior adrenal artery is a branch of the inferior phrenic artery.

Adrenal gland anatomy

Anatomy

Location Superomedially to the upper pole of each kidney

Relationships of Diaphragm-Posteriorly, Kidney-Inferiorly, Vena Cava-Medially, Hepato-


the right adrenal renal pouch and bare area of the liver-Anteriorly

Relationships of Crus of the diaphragm-Postero- medially, Pancreas and splenic


the left adrenal vessels-Inferiorly, Lesser sac and stomach-Anteriorly

Superior adrenal arteries- from inferior phrenic artery, Middle adrenal


Arterial supply arteries - from aorta, Inferior adrenal arteries -from renal arteries

Venous drainage Via one central vein directly into the IVC
of the right
adrenal

Venous drainage Via one central vein into the left renal vein
of the left adrenal

Rate question: Next question

Com m ent on this question

All contents of this site are © 2012 E-Medical Revision Ltd Terms and Conditions Privacy policy
yomna92009@yahoo.com - MRCS Part A - My account

Reference ranges End and review

Question 10 of 113 Question stats Score: 10%

1
A 10.2%
Which of the following does not exit the pelvis through the greater sciatic 2
B 15.6%
foramen? 3
C 7%
D 54.6% 4
A. Superior gluteal artery E 12.5% 5
B. Internal pudendal vessels 6
54.6% of users answered this
C. Sciatic nerve question correctly
7
D. Obturator nerve
8
E. Inferior gluteal nerve
9

10
Next question

The obturator nerve exits through the obturator foramen.

Greater sciatic foramen

Contents

Nerves Sciatic Nerve


Superior and Inferior Gluteal Nerves
Internal Pudendal Nerve
Posterior Femoral Cutaneous Nerve
Nerve to Quadratus Femoris
Nerve to Obturator internus

Vessels Superior Gluteal Artery and vein


Inferior Gluteal Artery and vein
Internal Pudendal Artery and vein

Piriformis
The piriformis is a landmark for identifying structures passing out of the sciatic
notch

Above piriformis: Superior gluteal vessels


Below piriformis: Inferior gluteal vessels, sciatic nerve (10% pass through it,
<1% above it), posterior cutaneous nerve of the thigh

Greater sciatic foramen boundaries


Anterolaterally Greater sciatic notch of the ilium

Posteromedially Sacrotuberous ligament

Inferior Sacrospinous ligament and the ischial spine

Superior Anterior sacroiliac ligament

The greater sciatic foramen


yomna92009@yahoo.com - MRCS Part A - My account

Reference ranges End and review

0/3 Question 11-13 of 113 Question stats Score: 7.7%

Average score for registered users: 1

Theme: Abdominal pain 2


11 78.2%
3
12 83.5%
A. Appendicitis
13 79.3% 4
B. Threatened miscarriage
5
C. Ectopic pregnancy
6
D. Irritable bowel syndrome
7
E. Mittelschmerz
8
F. Pelvic inflammatory disease
9
G. Adnexial torsion
10
H. Endometriosis
11-13 0 / 3
I. Degenerating fibroid

Please select the most likely cause of abdominal pain for the clinical scenario
given. Each option may be used once, more than once or not at all.

11. An 18 year-old girl presents to the Emergency Department with


sudden onset sharp, tearing pelvic pain associated with a small
amount of vaginal bleeding. She also complains of shoulder tip pain.
On examination she is hypotensive, tachycardic and has marked
cervical excitation.

You answered Degenerating fibroid

The correct answer is Ectopic pregnancy

The history of tearing pain and haemodynamic compromise in a


women of child bearing years should prompt a diagnosis of ectopic
pregnancy.

12. A 25 year-old lady presents to her GP complaining of a two day


history of right upper quadrant pain, fever and a white vaginal
discharge. She has seen the GP twice in 12 weeks complaining of
pelvic pain and dyspareunia.

You answered Degenerating fibroid

The correct answer is Pelvic inflammatory disease

The most likely diagnosis is pelvic inflammatory disease. Right


upper quadrant pain occurs as part of the Fitz Hugh Curtis
syndrome in which peri hepatic inflammation occurs.

13. A 16 year old female presents to the emergency department with a


12 hour history of pelvic discomfort. She is otherwise well and her
last normal menstrual period was 2 weeks ago. On examination she
has a soft abdomen with some mild supra pubic discomfort.

You answered Degenerating fibroid

The correct answer is Mittelschmerz

Mid cycle pain is very common and is due to the small amount of
fluid released during ovulation. Inflammatory markers are usually
normal and the pain typically subsides over the next 24-48 hours.

Next question
Gynaecological causes of abdominal pain

A number of women will present with abdominal pain and subsequently be


diagnosed with a gynaecological disorder. In addition to routine diagnostic work
up of abdominal pain, all female patients should also undergo a bimanual vaginal
examination, urine pregnancy test and consideration given to abdominal and
pelvic ultrasound scanning.
When diagnostic doubt persists a laparoscopy provides a reliable method of
assessing suspected tubulo-ovarian pathology.

Differential diagnoses of abdominal pain in females


Diagnosis Features Investigation Treatment

Mittelschmerz Usually mid cycle pain. Full blood Conservative


Often sharp onset. count-
Little systemic disturbance. usually
May have recurrent normal
episodes. Ultrasound-
Usually settles over 24-48 may show
hours. small
quantity of
free fluid

Endometriosis 25% asymptomatic, in a Ultrasound- Usually managed medically,


further 25% associated may show complex disease will often
with other pelvic organ free fluid require surgery and some
pathology. Laparoscopy patients will even require
Remaining 50% may have will usually formal colonic and rectal
menstrual irregularity, show resections if these areas are
infertility, pain and deep lesions involved
dyspareurina.
Complex disease may
result in pelvic adhesional
formation with episodes of
intermittent small bowel
obstruction.
Intra-abdominal bleeding
may produce localised
peritoneal inflammation.
Recurrent episodes are
common.

Ovarian Usually sudden onset of Ultrasound Laparoscopy


torsion deep seated colicky may show
abdominal pain. free fluid
Associated with vomiting Laparoscopy
and distress. is usually
Vaginal examination may both
reveal adnexial tenderness. diagnostic
and
therapeutic

Ectopic Symptoms of pregnancy Ultrasound Laparoscopy or laparotomy is


gestation without evidence of intra showing no haemodynamically unstable.
uterine gestation. intra uterine A salphingectomy is usually
Present as an emergency pregnancy performed.
with evidence of rupture or and beta
impending rupture. HCG that is
Open tubular ruptures may elevated
have sudden onset of May show
abdominal pain and intra
circulatory collapse, in abdominal
other the symptoms may free fluid
be more prolonged and
less marked.
Small amount of vaginal
discharge is common.
There is usually adnexial
tenderness.

Pelvic Bilateral lower abdominal Full blood Usually medical management


inflammatory pain associated with count-
disease vaginal discharge. Leucocytosis
Dysuria may also be Pregnancy
present. test negative
Peri-hepatic inflammation (Although
secondary to Chlamydia infection and
(Fitz Hugh Curtis pregnancy
Syndrome) may produce may co-exist)
right upper quadrant Amylase -
discomfort. usually
Fever >38o normal or
slightly
raised
High vaginal
and urethral
swabs

Rate question: Next question

Com m ent on this question

All contents of this site are © 2012 E-Medical Revision Ltd Terms and Conditions Privacy policy
yomna92009@yahoo.com - MRCS Part A - My account

Reference ranges End and review

Question 14 of 113 Question stats Score: 7.1%

1
A 14.8%
Which statement is false about the foramina of the skull? 2
B 41%
C 15.3% 3

A. The foramen lacerum transmits the internal carotid artery D 18.4% 4

B. The foramen spinosum is at the base of the medial pterygoid plate. E 10.5% 5

C. The jugular foramen transmits the accessory nerve 41% of users answered this 6
question correctly
D. The foramen lacerum is located in the sphenoid bone 7

E. The stylomastoid foramen transmits the facial nerve 8

9
Next question
10

11-13 0 / 3

14

Foramina of the base of the skull

Foramen Location Contents

Foramen Sphenoid Otic ganglion


ovale bone V3 (Mandibular nerve:3rd branch of
trigeminal)
Accessory meningeal artery
Lesser petrosal nerve
Emissary veins

Foramen Sphenoid Middle meningeal artery


spinosum bone Meningeal branch of the Mandibular nerve

Foramen Sphenoid Maxillary nerve (V2)


rotundum bone

Foramen Sphenoid Base of the medial pterygoid plate.


lacerum bone Internal carotid artery
Nerve and artery of the pterygoid canal

Jugular Temporal Anterior: inferior petrosal sinus


foramen bone Intermediate: glossopharyngeal, vagus, and accessory nerves.
Posterior: sigmoid sinus (becoming the internal jugular vein) and
some meningeal branches from the occipital and ascending
pharyngeal arteries.

Foramen Occipital Anterior and posterior spinal arteries


magnum bone Vertebral arteries
Medulla oblongata

Stylomastoid Temporal Stylomastoid artery


foramen bone Facial nerve

Superior Sphenoid Oculomotor nerve (III)


orbital bone trochlear nerve (IV)
fissure lacrimal, frontal and nasociliary branches of ophthalmic nerve
(V1)
abducent nerve (VI)
Superior and inferior ophthalmic vein

Base of skull anatomical overview

Image sourced from Wikipedia


yomna92009@yahoo.com - MRCS Part A - My account

Reference ranges End and review

Question 15 of 113 Question stats Score: 6.7%

1
A 10.8%
An 80 year old lady with a caecal carcinoma is undergoing a right hemicolectomy 2
B 39.7%
performed through a transverse incision. The procedure is difficult and the 3
C 14.1%
incision is extended medially by dividing the rectus sheath. Brisk arterial
D 9.2% 4
haemorrhage ensues. From which of the following does the damaged vessel
E 26.2% 5
originate?

39.7% of users answered this 6


question correctly
A. Internal iliac artery 7

B. External iliac artery 8

C. Superior vesical artery 9

D. Inferior vesical artery 10

E. None of the above 11-13 0 / 3

14
Next question 15

The vessel damaged is the epigastric artery. This originates from the external iliac
artery (see below).

Epigastric artery

The inferior epigastric artery arises from the external iliac artery immediately
above the inguinal ligament. It then passes along the medial margin of the deep
inguinal ring. From here it continues superiorly to lie behind the rectus abdominis
muscle.

This is illustrated below:

Image sourced from Wikipedia

Rate question: Next question


yomna92009@yahoo.com - MRCS Part A - My account

Reference ranges End and review

Question 16 of 113 Question stats Score: 6.3%

1
A 30.2%
A 73 year old man has a large abdominal aortic aneurysm. During a laparotomy 2
B 40.9%
for planned surgical repair the surgeons find the aneurysm is far more proximally 3
C 12.6%
located and lies near the origin of the SMA. During the dissection a vessel lying
D 8.1% 4
transversely across the aorta is injured. What is this vessel most likely to be?
E 8.2% 5

30.2% of users answered this 6


A. Left renal vein
question correctly
7
B. Right renal vein
8
C. Inferior mesenteric artery
D. Ileocolic artery 9

E. Middle colic artery 10

11-13 0 / 3

Next question 14

15
Theme from April 2012 Exam 16
The left renal vein runs across the surface of the aorta and may require
deliberate ligation during juxtarenal aneurysm repair.

Abdominal aorta

Abdominal aortic topography


Origin T12

Termination L4

Posterior relations L1-L4 Vertebral bodies

Anterior relations Lesser omentum


Liver
Left renal vein
Inferior mesenteric vein
Third part of duodenum
Pancreas
Parietal peritoneum
Peritoneal cavity

Right lateral relations Right crus of the diaphragm


Cisterna chyli
Azygos vein
IVC (becomes posterior distally)

Left lateral relations 4th part of duodenum


Duodenal-jejunal flexure
Left sympathetic trunk

The abdominal aorta

Image sourced from Wikipedia


yomna92009@yahoo.com - MRCS Part A - My account

Reference ranges End and review

Question 17 of 113 Question stats Score: 5.9%

1
A 7.2%
A 18 year old man presents with an indirect inguinal hernia and undergoes 2
B 59.9%
surgery. The deep inguinal ring is exposed. Which of the following lies at the 3
C 8.8%
medial edge of this structure?
D 8.2% 4
E 15.9% 5
A. Ureter
59.9% of users answered this 6
B. Inferior epigastric artery question correctly
7
C. Internal iliac vein
8
D. Femoral artery
9
E. Lateral border of rectus abdominis
10

11-13 0 / 3
Next question
14

15
Boundaries of the deep inguinal ring:
16

Superolaterally - transversalis fascia 17


Inferomedially - inferior epigastric artery

The deep inguinal ring is closely related to the inferior epigastric artery. The
inferior epigastric artery forms part of the structure referred to as Hesselbach's
triangle.

Image sourced from Wikipedia

Inguinal canal
yomna92009@yahoo.com - MRCS Part A - My account

Reference ranges End and review

Question 18 of 113 Question stats Score: 5.6%

1
A 57%
A 73 year old man presents with a tumour at the tip of his tongue. To which of the 2
B 18.4%
following regions will the tumour initially metastasise? 3
C 6.7%
D 11% 4
A. Sub mental nodes E 7% 5
B. Ipsilateral deep cervical nodes 6
57% of users answered this
C. Tonsil question correctly
7
D. Ipsilateral superficial cervical nodes
8
E. Contralateral deep cervical nodes
9

10
Next question
11-13 0 / 3

14

15

Lymphatic drainage of the tongue 16

17

The lymphatic drainage of the anterior two thirds of the tongue shows only 18
minimal communication of lymphatics across the midline, so metastasis to
the ipsilateral nodes is usual.
The lymphatic drainage of the posterior third of the tongue have
communicating networks, as a result early bilateral nodal metastases are
more common in this area.
Lymphatics from the tip of the tongue usually pass to the sub mental nodes
and from there to the deep cervical nodes.
Lymphatics from the mid portion of the tongue usually drain to the
submandibular nodes and then to the deep cervical nodes. Mid tongue
tumours that are laterally located will usually drain to the ipsilateral deep
cervical nodes, those from more central regions may have bilateral deep
cervical nodal involvement.

Rate question: Next question

Com m ent on this question

All contents of this site are © 2012 E-Medical Revision Ltd Terms and Conditions Privacy policy
yomna92009@yahoo.com - MRCS Part A - My account

Reference ranges End and review

Question 19 of 113 Question stats Score: 10.5%

1
A 12.4%
A 78 year old man is undergoing a femoro-popliteal bypass graft. The operation 2
B 20%
is not progressing well and the surgeon is complaining of poor access. Retraction 3
C 14.6%
of which of the following structures will improve access to the femoral artery in the
D 11.1% 4
groin?
E 42% 5

42% of users answered this 6


A. Quadriceps
question correctly
7
B. Adductor longus
8
C. Adductor magnus
D. Pectineus 9

E. Sartorius 10

11-13 0 / 3

Next question 14

15
At the lower border of the femoral triangle the femoral artery passes under the 16
sartorius muscle. This can be retracted to improve access.
17

Femoral triangle anatomy 18

19
Boundaries
Superiorly Inguinal ligament

Laterally Sartorius

Medially Adductor longus

Floor Iliopsoas, adductor longus and pectineus

Roof Fascia lata and Superficial fascia


Superficial inguinal lymph nodes (palpable below the inguinal ligament)
Great saphenous vein

Image sourced from Wikipedia


yomna92009@yahoo.com - MRCS Part A - My account

Reference ranges End and review

Question 20 of 113 Question stats Score: 10%

1
A 19.6%
A builder falls off a ladder whilst laying roof tiles. He sustains a burst fracture of 2
B 37.6%
L3. The MRI scan shows complete cord transection at this level as a result of the 3
C 6.2%
injury. Which of the following clinical signs will not be present?
D 14.2% 4
E 22.3% 5
A. Flaccid paralysis of the legs
37.6% of users answered this 6
B. Extensor plantar response question correctly
7
C. Sensory loss in the legs
8
D. Incontinence
9
E. Loss of patellar tendon reflex
10

11-13 0 / 3
Next question
14

15
In lower motor neuron lesions everything is reduced
16

17

18
The main purpose of this question is to differentiate the features of an UMN lesion
and a LMN lesion. The features of a LMN lesion include: 19

20
Flaccid paralysis of muscles supplied
Atrophy of muscles supplied.
Loss of reflexes of muscles supplied.
Muscles fasciculation

For lesions below L1 LMN signs will occur. Hence in an L3 lesion, there will be
loss of the patella reflex but there will be no extensor plantar reflex.

Spinal cord

Located in a canal within the vertebral column that affords it structural


support.
Rostrally is continues to the medulla oblongata of the brain and caudally it
tapers at a level corresponding to the L1-2 interspace (in the adult), a
central structure, the filum terminale anchors the cord to the first coccygeal
vertebra.
The spinal cord is characterised by cervico-lumbar enlargements and
these, broadly speaking, are the sites which correspond to the brachial and
lumbar plexuses respectively.

There are some key points to note when considering the surgical anatomy of the
spinal cord:

* During foetal growth the spinal cord becomes shorter than the spinal canal,
hence the adult site of cord termination at the L1-2 level.

* Due to growth of the vertebral column the spine segmental levels may not
always correspond to bony landmarks as they do in the cervical spine.

* The spinal cord is incompletely divided into two symmetrical halves by a dorsal
median sulcus and ventral median fissure. Grey matter surrounds a central
canal that is continuous rostrally with the ventricular system of the CNS.

* The grey matter is sub divided cytoarchitecturally into Rexeds laminae.

* Afferent fibres entering through the dorsal roots usually terminate near their
point of entry but may travel for varying distances in Lissauers tract. In this way
yomna92009@yahoo.com - MRCS Part A - My account

Reference ranges End and review

Question 21 of 113 Question stats Score: 9.5%

1
A 63.6%
A 66 year old man with peripheral vascular disease is undergoing a below knee 2
B 12.2%
amputation. In which of the lower leg compartments does peroneus brevis lie? 3
C 6.8%
D 11.1% 4
A. Lateral compartment E 6.2% 5
B. Anterior compartment 6
63.6% of users answered this
C. Superficial posterior compartment question correctly
7
D. Deep posterior compartment
8
E. None of the above
9

10
Next question
11-13 0 / 3

14
The interosseous membrane separates the anterior and posterior compartments.
The deep and superficial compartments are separated by the deep transverse 15
fascia. The peroneus brevis is part of the lateral compartment. 16

17
Fascial compartments of the leg
18

Compartments of the thigh 19

20
Formed by 3 septae passing from the femur to the fascia lata.
21
Compartment Nerve Muscles Blood supply

Anterior Femoral Iliacus Femoral artery


compartment Tensor fasciae latae
Sartorius
Quadriceps femoris

Medial Obturator Adductor Profunda femoris artery and


compartment longus/magnus/brevis obturator artery
Gracilis
Obturator externus

Posterior Sciatic Semimembranosus Branches of Profunda


compartment Semitendinosus femoris artery
Biceps femoris

Compartments of the lower leg


Separated by the interosseous membrane (anterior and posterior compartments),
anterior fascial septum (separate anterior and lateral compartments) and
posterior fascial septum (separate lateral and posterior compartments)

Compartment Nerve Muscles Blood


supply

Anterior Deep Tibialis anterior Anterior


compartment peroneal Extensor digitorum longus tibial
nerve Extensor hallucis longus artery
Peroneus tertius

Posterior Tibial Muscles: deep and superficial Posterior


compartment compartments (separated by deep tibial
transverse fascia)
Deep: Flexor hallucis longus, Flexor
digitalis longus, Tibialis posterior,
Popliteus
Superficial: Gastrocnemius, Soleus,
Plantaris

Lateral Superficial Peroneus longus/brevis Anterior


yomna92009@yahoo.com - MRCS Part A - My account

Reference ranges End and review

Question 22 of 113 Question stats Score: 9.1%

1
A 8.6%
A 70 year old man is due to undergo an arterial bypass procedure for 2
B 15.7%
claudication and foot ulceration. The anterior tibial artery will form the site of the 3
C 56.3%
distal arterial anastomosis. Which of the following structures is not closely related
D 10.4% 4
to it?
E 9% 5

56.3% of users answered this 6


A. Interosseous membrane
question correctly
7
B. Deep peroneal nerve
8
C. Tibialis posterior
D. Extensor hallucis longus 9

E. Dorsalis pedis artery 10

11-13 0 / 3

Next question 14

15
As an artery of the anterior compartment, the anterior tibial artery is closely 16
related to tibialis anterior.
17

Anterior tibial artery 18

19

20
Begins opposite the distal border of popliteus
Terminates in front of the ankle, continuing as the dorsalis pedis artery 21
As it descends it lies on the interosseous membrane, distal part of the tibia
22
and front of the ankle joint
Passes between the tendons of flexor digitorum and flexor hallucis longus
distally
It is related to the deep peroneal nerve, it lies anterior to the middle third of
the vessel and lateral to it in the lower third

Rate question: Next question

Com m ent on this question

All contents of this site are © 2012 E-Medical Revision Ltd Terms and Conditions Privacy policy
yomna92009@yahoo.com - MRCS Part A - My account

Reference ranges End and review

Question 23 of 113 Question stats Score: 13%

1
A 17.3%
Which of the following muscles does not cause lateral rotation of the hip? 2
B 19.5%
C 8.2% 3

A. Obturator internus D 12% 4

B. Quadratus femoris E 42.9% 5

C. Gemellus inferior 42.9% of users answered this 6


question correctly
D. Piriformis 7

E. Pectineus 8

9
Next question
10

11-13 0 / 3

Mnemonic lateral hip rotators: P-GO-GO-Q (top to bottom) 14

15
Piriformis
16
Gemellus superior
Obturator internus 17

Gemellus inferior 18
Obturator externus
19
Quadratus femoris
20

21
Pectineus adducts and medially rotates the femur.
22

Hip joint 23

Head of femur articulates with acetabulum of the pelvis


Both covered by articular hyaline cartilage
The acetabulum forms at the union of the ilium, pubis, and ischium
The triradiate cartilage (Y-shaped growth plate) separates the pelvic bones
The acetabulum holds the femoral head by the acetabular labrum
Normal angle between femoral head and femoral shaft is 130o

Ligaments

Transverse ligament: joints anterior and posterior ends of the articular


cartilage
Head of femur ligament (ligamentum teres): acetabular notch to the fovea.
Contains arterial supply to head of femur in children.

Image sourced from Wikipedia


yomna92009@yahoo.com - MRCS Part A - My account

Reference ranges End and review

Question 1 of 90 Question stats Score: 100%

1
A 4.9%
Which of the following is not a content of the posterior triangle of the neck? B 20.8%
C 13.8%

A. Spinal accessory nerve D 11.3%

B. Phrenic nerve E 49.2%

C. External jugular vein 49.2% of users answered this


question correctly
D. Occipital lymph nodes
E. Internal jugular vein

Next question

The IJV does not lie in the posterior triangle. However, the terminal branches of
the external jugular vein do.

Posterior triangle of the neck

Boundaries
Apex Sternocleidomastoid and the Trapezius muscles at the Occipital bone

Anterior Posterior border of the Sternocleidomastoid

Posterior Anterior border of the Trapezius

Base Middle third of the clavicle

Image sourced from Wikipedia

Contents

Nerves Accessory nerve


Phrenic nerve
Three trunks of the brachial plexus
Branches of the cervical plexus: Supraclavicular nerve, transverse cervical
nerve, great auricular nerve, lesser occipital nerve

Vessels External jugular vein


Subclavian artery

Muscles Inferior belly of omohyoid


Scalene

Lymph Supraclavicular
nodes Occipital

Rate question: Next question


yomna92009@yahoo.com - MRCS Part A - My account

Reference ranges End and review

Question 2 of 90 Question stats Score: 50%

1
A 10.2%
Which nerve is at risk during submandibular gland excision? 2
B 10.3%
C 7.2%

A. Maxillary nerve D 61.1%

B. Buccal nerve E 11.2%

C. Zygomatic nerve 61.1% of users answered this


question correctly
D. Marginal mandibular nerve
E. Cervical nerve

Next question

The marginal mandibular nerve lies deep to platysma. It supplies the depressor
anguli oris and the depressor labii inferioris. If injured it may lead to facial
asymmetry and dribbling.

Submandibular gland

Relations of the submandibular gland


Superficial Platysma, deep fascia and mandible
Submandibular lymph nodes
Facial vein (facial artery near mandible)
Marginal mandibular nerve
Cervical branch of the facial nerve

Deep Facial artery (inferior to the mandible)


Mylohoid muscle
Sub mandibular duct
Hyoglossus muscle
Lingual nerve
Submandibular ganglion
Hypoglossal nerve

Submandibular duct (Wharton's duct)

Opens lateral to the lingual frenulum on the anterior floor of mouth.


5 cm length
Lingual nerve wraps around Wharton's duct. As the duct passes forwards it
crosses medial to the nerve to lie above it and then crosses back, lateral to
it, to reach a position below the nerve.

Innervation

Sympathetic innervation- Superior Cervical ganglion via the Lingual nerve


Parasympathetic innervation- Submandibular ganglion

Arterial supply
Branch of the Facial artery. The facial artery passes through the gland to groove
its deep surface. It then emerges onto the face by passing between the gland and
the mandible.

Venous drainage
Anterior Facial vein (lies deep to the Marginal Mandibular nerve)

Lymphatic drainage
Deep cervical and jugular chains of nodes

Rate question: Next question

Com m ent on this question


yomna92009@yahoo.com - MRCS Part A - My account

Reference ranges End and review

Question 3 of 90 Question stats Score: 66.7%

1
A 7.7%
In a patient with a carcinoma of the distal sigmoid colon, what is the most likely 2
B 4.4%
source of its blood supply? 3
C 9.6%
D 6%

A. Ileocolic artery E 72.3%

B. External iliac artery


72.3% of users answered this
C. Internal iliac artery question correctly

D. Superior mesenteric artery


E. Inferior mesenteric artery

Next question

During a high anterior resection of such tumours, the inferior mesenteric artery is
ligated. Note that the branches (mainly middle rectal branch) of the internal iliac
artery are important in maintaining vascularity of the rectal stump and hence the
integrity of the anastomoses.

Rectum

The rectum is approximately 12 cm long. It is a capacitance organ. It has both


intra and extraperitoneal components. The transition between the sigmoid colon is
marked by the disappearance of the tenia coli.The extra peritoneal rectum is
surrounded by mesorectal fat that also contains lymph nodes. This mesorectal
fatty layer is removed surgically during rectal cancer surgery (Total Mesorectal
Excision). The fascial layers that surround the rectum are important clinical
landmarks, anteriorly lies the fascia of Denonvilliers. Posteriorly lies Waldeyers
fascia.

Extra peritoneal rectum

Posterior upper third


Posterior and lateral middle third
Whole lower third

Relations
Anteriorly (Males) Rectovesical pouch
Bladder
Prostate
Seminal vesicles

Anteriorly (Females) Recto-uterine pouch (Douglas)


Cervix
Vaginal wall

Posteriorly Sacrum
Coccyx
Middle sacral artery

Laterally Levator ani


Coccygeus

Arterial supply
Superior rectal artery

Venous drainage
Superior rectal vein

Lymphatic drainage

Mesorectal lymph nodes (superior to dentate line)


Internal iliac and then para-aortic nodes
Inguinal nodes (inferior to dentate line)
yomna92009@yahoo.com - MRCS Part A - My account

Reference ranges End and review

Question 4 of 90 Question stats Score: 50%

1
A 38.7%
Which of the following openings transmits the facial nerve into the temporal bone? 2
B 6.3%
C 7.7% 3

A. Internal acoustic meatus D 39.9% 4

B. Foramen lacerum E 7.3%

C. Foramen spinosum 38.7% of users answered this


question correctly
D. Stylomastoid foramen
E. Jugular foramen

Next question

It enters the temporal bone through the internal acoustic meatus and exits
through the stylomastoid foramen.

Facial nerve

The facial nerve is the main nerve supplying the structures of the second
embryonic branchial arch. It is predominantly an efferent nerve to the muscles of
facial expression, digastric muscle and also to many glandular structures. It
contains a few afferent fibres which originate in the cells of its genicular ganglion
and are concerned with taste.

Supply - 'face, ear, taste, tear'

Face: muscles of facial expression


Ear: nerve to stapedius
Taste: supplies anterior two-thirds of tongue
Tear: parasympathetic fibres to lacrimal glands, also salivary glands

Path
Subarachnoid path

Origin: motor- pons, sensory- nervus intermedius


Pass through the petrous temporal bone into the internal auditory meatus
with the vestibulocochlear nerve. Here they combine to become the facial
nerve.

Facial canal path

The canal passes superior to the vestibule of the inner ear


At the medial aspect of the middle ear, it becomes wider and contains the
geniculate ganglion.

- 3 branches:
1. greater petrosal nerve
2. nerve to stapedius
3. chorda tympani

Stylomastoid foramen

Passes through the stylomastoid foramen (tympanic cavity anterior and


mastoid antrum posteriorly)
Posterior auricular nerve and branch to Posterior belly of Digastric and
Stylohyoid muscle

Face
Enters parotid gland and divides into 5 branches:

Temporal branch
yomna92009@yahoo.com - MRCS Part A - My account

Reference ranges End and review

Question 5 of 90 Question stats Score: 40%

1
A 20.4%
A motor cyclist is involved in a road traffic accident causing severe right shoulder 2
B 56.4%
injuries. He is found to have an adducted, medially rotated shoulder. The elbow is 3
C 7.8%
fully extended and the forearm pronated. Which is the most likely diagnosis?
D 5.5% 4
E 9.9% 5
A. C8, T1 root lesion
56.4% of users answered this
B. C5, C6 root lesion question correctly
C. Radial nerve lesion
D. Ulnar nerve lesion
E. Axillary nerve lesion

Next question

Erbs Palsy C5, C6 lesion


The features include:

Waiter's tip position


Loss of shoulder abduction (deltoid and supraspinatus paralysis)
Loss of external rotation of the shoulder (paralysis of infraspinatus and
teres major)
Loss of elbow flexion (paralysis of biceps, brachialis and
brachioradialis)
Loss of forearm supination (paralysis of Biceps)

The motorcyclist has had an Erb's palsy (C5, C6 root lesion). This is commonly
known to be associated with birth injury when a baby has a shoulder dystocia.

Brachial plexus

Origin Anterior rami of C5 to T1

Sections of the Roots, trunks, divisions, cords, branches


plexus Mnemonic:Real Teenagers Drink Cold Beer

Roots Located in the posterior triangle


Pass between scalenus anterior and medius

Trunks Located posterior to middle third of clavicle


Upper and middle trunks related superiorly to the subclavian
artery
Lower trunk passes over 1st rib posterior to the subclavian
artery

Divisions Apex of axilla

Cords Related to axillary artery

Diagram illustrating the branches of the brachial plexus


yomna92009@yahoo.com - MRCS Part A - My account

Reference ranges End and review

Question 6 of 90 Question stats Score: 33.3%

1
A 13.3%
Which of the following structures does not pass anterior to the lateral malleolus? 2
B 11%
C 12.1% 3

A. Anterior tibial artery D 45.3% 4

B. Extensor digitorum longus E 18.2% 5

C. Tibialis anterior 45.3% of users answered this 6


question correctly
D. Peroneus brevis
E. Peroneus tertius

Next question

Peroneus brevis passes posterior to the lateral malleolus.

Lateral malleolus

Structures posterior to the lateral malleolus and superficial to superior


peroneal retinaculum

Sural nerve
Short saphenous vein

Structures posterior to the lateral malleolus and deep to superior


peroneal retinaculum

Peroneus longus tendon


Peroneus brevis tendon

The calcaneofibular ligament is attached at the lateral malleolus

Rate question: Next question

Com m ent on this question

All contents of this site are © 2012 E-Medical Revision Ltd Terms and Conditions Privacy policy
yomna92009@yahoo.com - MRCS Part A - My account

Reference ranges End and review

Question 7 of 90 Question stats Score: 28.6%

1
A 12.8%
A patient is due to undergo a right hemicolectomy for a carcinoma of the caecum. 2
B 6.8%
Which of the following vessels will require high ligation to provide optimal 3
C 23.1%
oncological control?
D 51.3% 4
E 6.1% 5
A. Middle colic artery
51.3% of users answered this 6
B. Inferior mesenteric artery question correctly
7
C. Superior mesenteric artery
D. Ileo-colic artery
E. None of the above

Next question

The ileo - colic artery supplies the caecum and would require high ligation during
a right hemicolectomy. The middle colic artery should generally be preserved
when resecting a caecal lesion.
This question is essentially asking you to name the vessel supplying the caecum.
The SMA does not directly supply the caecum, it is the ileocolic artery which does
this.

Caecum

Location Proximal right colon below the ileocaecal valve


Intraperitoneal

Posterior relations Psoas


Iliacus
Femoral nerve
Genitofemoral nerve
Gonadal vessels

Anterior relations Greater omentum

Arterial supply Ileocolic artery

Lymphatic drainage Mesenteric nodes accompany the venous drainage

The caecum is the most distensible part of the colon and in complete large
bowel obstruction with a competent ileocaecal valve the most likely site of
eventual perforation.

Rate question: Next question

Com m ent on this question

All contents of this site are © 2012 E-Medical Revision Ltd Terms and Conditions Privacy policy
yomna92009@yahoo.com - MRCS Part A - My account

Reference ranges End and review

Question 8 of 90 Question stats Score: 37.5%

1
A 10%
A 72 year old man is undergoing a repair of an abdominal aortic aneurysm. The 2
B 18.4%
aorta is cross clamped both proximally and distally. The proximal clamp is applied 3
C 7%
immediately inferior to the renal arteries. Both common iliac arteries are clamped
D 12.4% 4
distally. A longitudinal aortotomy is performed. After evacuating the contents of
E 52.3% 5
the aneurysm sac a significant amount of ongoing bleeding is encountered. This
is most likely to originate from: 6
52.3% of users answered this
question correctly
7
A. The coeliac axis 8
B. Testicular artery
C. Splenic artery
D. Superior mesenteric artery
E. Lumbar arteries

Next question

The lumbar arteries are posteriorly sited and are a common cause of back
bleeding during aortic surgery. The other vessels cited all exit the aorta in the
regions that have been cross clamped.

Abdominal aortic branches

Branches Level Paired Type

Inferior phrenic T12 (Upper border) Yes Parietal

Coeliac T12 No Visceral

Superior mesenteric L1 No Visceral

Middle suprarenal L1 Yes Visceral

Renal L1-L2 Yes Visceral

Gonadal L2 Yes Visceral

Lumbar L1-L4 Yes Parietal

Inferior mesenteric L3 No Visceral

Median sacral L4 No Parietal

Common iliac L4 Yes Terminal

Rate question: Next question

Com m ent on this question

All contents of this site are © 2012 E-Medical Revision Ltd Terms and Conditions Privacy policy
yomna92009@yahoo.com - MRCS Part A - My account

Reference ranges End and review

Question 9 of 90 Question stats Score: 44.4%

1
A 7.8%
Which of the following statements relating to sartorius is untrue? 2
B 9.8%
C 10% 3

A. It is supplied by the femoral nerve D 24.1% 4

B. It forms the lateral boundary of the femoral triangle E 48.3% 5

C. The middle third forms the roof of the adductor canal 48.3% of users answered this 6
question correctly
D. It is a flexor of the hip and knee 7

E. It inserts into the medial femoral condyle 8

9
Next question

It inserts into the medial aspect of the upper part of the tibia.

Sartorius

Longest strap muscle in the body


Most superficial muscle in the anterior compartment of the thigh

Origin Anterior superior iliac spine

Insertion Medial surface of the of the body of the tibia (upper part). It inserts anterior to
gracilis and semitendinosus

Nerve Femoral nerve (L2,3)


Supply

Action Flexor of the hip and knee, slight abducts the thigh and rotates it laterally
It assists with medial rotation of the tibia on the femur. For example it
would play a pivotal role in placing the right heel onto the left knee ( and
vice versa)

Important The middle third of this muscle, and its strong underlying fascia forms the roof
relations of the adductor canal , in which lie the femoral vessels, the saphenous nerve
and the nerve to vastus medialis.

Rate question: Next question

Com m ent on this question

All contents of this site are © 2012 E-Medical Revision Ltd Terms and Conditions Privacy policy
yomna92009@yahoo.com - MRCS Part A - My account

Reference ranges End and review

Question 10 of 90 Question stats Score: 40%

1
A 5.6%
The superficial inguinal ring is traversed by which of the following nerves? 2
B 13.5%
C 68.7% 3

A. Subcostal D 5.4% 4

B. Iliohypogastric E 6.8% 5

C. Ilioinguinal 68.7% of users answered this 6


question correctly
D. Obturator 7

E. Pudendal 8

9
Next question
10

Ilioinguinal nerve entrapment may be a cause of neuropathic pain following


inguinal hernia surgery.

The ilioinguinal nerve passes through the superfical inguinal ring and is routinely
encountered when exploring the inguinal canal during hernia surgery. The
iliohypogastric nerve pierces the aponeurosis of the external oblique muscle
superior to the superficial inguinal ring.

Ilioinguinal nerve

Arises from the first lumbar ventral ramus with the iliohypogastric nerve. It passes
inferolaterally through the substance of psoas major and over the anterior
surface of quaratus lumborum. It pierces the internal oblique muscle and passes
deep to the aponeurosis of the external oblique muscle. It enters the inguinal
canal and then passes through the superficial inguinal ring to reach the skin.

Branches

To supply those muscles of the abdominal wall through which it passes.


Skin and fascia over the pubic symphysis, superomedial part of the femoral
triangle, surface of the scrotum, root and dorsum of penis or labum majus
in females.

Rate question: Next question

Com m ent on this question

All contents of this site are © 2012 E-Medical Revision Ltd Terms and Conditions Privacy policy
yomna92009@yahoo.com - MRCS Part A - My account

Reference ranges End and review

Question 11 of 90 Question stats Score: 45.5%

1
A 51.9%
A 63 year old man undergoes a radical cystectomy for carcinoma of the bladder. 2
B 37%
During the procedure there is considerable venous bleeding. What is the primary 3
C 7.4%
site of venous drainage of the urinary bladder?
D 0% 4
E 3.7%
5
A. Vesicoprostatic venous plexus
51.9% of users answered this 6
B. Internal iliac vein question correctly
7
C. External iliac vein
8
D. Gonadal vein
9
E. Common iliac vein
10

Next question 11

The urinary bladder has a rich venous plexus surrounding it, this drains
subsequently into the internal iliac vein. The vesicoprostatic plexus may be a site
of considerable venous bleeding during cystectomy.

Bladder

The empty bladder is contained within the pelvic cavity. It is usually a three sided
pyramid. The apex of the bladder points forwards towards the symphysis pubis
and the base lies immediately anterior to the rectum or vagina. Continuous with
the apex is the medial umbilical ligament, during development this was the site of
the uranchus.
The inferior aspect of the bladder is retroperitoneal and the superior aspect
covered by peritoneum. As the bladder distends it will tend to separate the
peritoneum from the fascia of tansversalis. For this reason a bladder that is
distended due to acute urinary retention may be approached with a suprapubic
catheter that avoids entry into the peritoneal cavity.
The trigone is the least mobile part of the bladder and forms the site of the
ureteric orifices and internal urethral orifice. In the empty bladder the ureteric
orifices are approximately 2-3cm apart, this distance may increase to 5cm in the
distended bladder.

Arterial supply
The superior and inferior vesical arteries provide the main blood supply to the
bladder. These are branches of the internal iliac artery.

Venous drainage
In males the bladder is drained by the vesicoprostatic venous plexus. In females
the bladder is drained by the vesicouterine venous plexus. In both sexes this
venous plexus will ultimately drain to the internal iliac veins.

Lymphatic drainage
Lymphatic drainage is predominantly to the external iliac nodes, internal iliac and
obturator nodes also form sites of bladder lymphatic drainage.

Innervation
Parasympathetic nerve fibres innervate the bladder from the pelvic splanchnic
nerves. Sympathetic nerve fibres are derived from L1 and L2 via the hypogastric
nerve plexuses. The parasympathetic nerve fibres will typically cause detrusor
muscle contraction and result in voiding. The muscle of the trigone is innervated
by the sympathetic nervous system. The external urethral sphincter is under
concious control. During bladder filling the rate of firing of nerve impulses to the
detrusor muscle is low and receptive relaxation occurs. At higher volumes and
increased intra vesical pressures the rate of neuronal firing will increase and
eventually voiding will occur.

Rate question:
yomna92009@yahoo.com - MRCS Part A - My account

Reference ranges End and review

Question 12 of 90 Question stats Score: 41.7%

1
A 37%
A 60 year old female is undergoing a Whipples procedure for adenocarcinoma of 2
B 6.5%
the pancreas. As the surgeons begin to mobilise the pancreatic head they identify 3
C 10.4%
a large vessel passing inferiorly over the anterior aspect of the pancreatic head.
D 18.5% 4
What is it likely to be?
E 27.6% 5

37% of users answered this 6


A. Superior mesenteric artery
question correctly
7
B. Coeliac axis
8
C. Inferior mesenteric artery
D. Aorta 9

E. Left gastric artery 10

11

Next question 12

Theme from January 2012 Exam


The superior mesenteric artery arises from the aorta and passes anterior to the
lower part of the pancreas. Invasion of this structure is a relative contra indication
to resectional surgery.

Pancreas

The pancreas is a retroperitoneal organ and lies posterior to the stomach. It may
be accessed surgically by dividing the peritoneal reflection that connects the
greater omentum to the transverse colon. The pancreatic head sits in the
curvature of the duodenum. It's tail lies close to the hilum of the spleen, a site of
potential injury during splenectomy.

Relations
Posterior to the pancreas
Pancreatic head Inferior vena cava
Common bile duct
Right and left renal veins
Superior mesenteric vein and artery

Pancreatic neck Superior mesenteric vein, portal vein

Pancreatic body- Left renal vein


Crus of diaphragm
Psoas muscle
Adrenal gland
Kidney
Aorta

Pancreatic tail Left kidney

Anterior to the pancreas


Pancreatic head 1st part of the duodenum
Pylorus
Gastroduodenal artery
SMA and SMV(uncinate process)

Pancreatic body Stomach


Duodenojejunal flexure

Pancreatic tail Splenic hilum

Superior to the pancreas


Coeliac trunk and its branches common hepatic artery and splenic artery

Grooves of the head of the pancreas


2nd and 3rd part of the duodenum

Arterial supply

Head: pancreaticoduodenal artery


yomna92009@yahoo.com - MRCS Part A - My account

Reference ranges End and review

Question 13 of 90 Question stats Score: 38.5%

1
A 10%
An 18 year old man is undergoing an orchidectomy via a scrotal approach. The 2
B 64.6%
surgeons mobilise the spermatic cord. From which of the following is the 3
C 12.4%
outermost layer of this structure derived?
D 4.9% 4
E 8.2% 5
A. Internal oblique aponeurosis
64.6% of users answered this 6
B. External oblique aponeurosis question correctly
7
C. Transversalis fascia
8
D. Rectus sheath
9
E. Campers fascia
10

Next question 11

12
The outermost covering of the spermatic cord is derived from the external oblique 13
aponeurosis.This layer is added as the cord passes through the superficial
inguinal ring.

Scrotal and testicular anatomy

Spermatic cord
Formed by the vas deferens and is covered by the following structures:
Layer Origin

Internal spermatic fascia Transversalis fascia

Cremasteric fascia From the fascial coverings of internal oblique

External spermatic fascia External oblique aponeurosis

Contents of the cord


Vas deferens Transmits sperm and accessory gland secretions

Testicular artery Branch of abdominal aorta supplies testis and


epididymis

Artery of vas deferens Arises from inferior vesical artery

Cremasteric artery Arises from inferior epigastic artery

Pampiniform plexus Venous plexus, drains into right or left testicular vein

Sympathetic nerve fibres Lie on arteries, the parasympathetic fibres lie on the
vas

Genital branch of the genitofemoral Supplies cremaster


nerve

Lymphatic vessels Drain to lumbar and para-aortic nodes

Scrotum

Composed of skin and closely attached dartos fascia.


Arterial supply from the anterior and posterior scrotal arteries
Lymphatic drainage to the inguinal lymph nodes
Parietal layer of the tunica vaginalis is the innermost layer

Testes

The testes are surrounded by the tunica vaginalis (closed peritoneal sac).
The parietal layer of the tunica vaginalis adjacent to the internal spermatic
fascia.
The testicular arteries arise from the aorta immediately inferiorly to the
renal arteries.
The pampiniform plexus drains into the testicular veins, the left drains into
the left renal vein and the right into the inferior vena cava.
Lymphatic drainage is to the para-aortic nodes.
yomna92009@yahoo.com - MRCS Part A - My account

Reference ranges End and review

Question 14 of 90 Question stats Score: 35.7%

1
A 9.8%
Which of the following is not a carpal bone? 2
B 8.5%
C 6.4% 3

A. Trapezium D 70.7% 4

B. Triquetrum E 4.6% 5

C. Trapezoid 70.7% of users answered this 6


question correctly
D. Trapezius 7

E. Lunate 8

9
Next question
10

11

Mnemonic for the Carpal Bones 12

13
Sally Likes To Play The Tiny Chrome Harmonica
14

Scared Lovers Try Positions That They Can't Handle

Trapezius is a muscle of the back.

Carpal bones

Diagrammatic image of carpal bones

Image sourced from Wikipedia

Key to image
A Scaphoid

B Lunate

C Triquetrum

D Pisiform

E Trapezium
yomna92009@yahoo.com - MRCS Part A - My account

Reference ranges End and review

Question 15 of 90 Question stats Score: 33.3%

1
A 16.2%
A 53 year old male presents with a carcinoma of the transverse colon. Which of 2
B 8.5%
the following structures should be ligated close to their origin to maximise 3
C 61.9%
clearance of the tumour?
D 8.5% 4
E 5% 5
A. Superior mesenteric artery
61.9% of users answered this 6
B. Inferior mesenteric artery question correctly
7
C. Middle colic artery
8
D. Ileo-colic artery
9
E. Superior rectal artery
10

Next question 11

12
The middle colic artery supplies the transverse colon and requires high ligation 13
during cancer resections. It is a branch of the superior mesenteric artery.
14

Transverse colon 15

The right colon undergoes a sharp turn at the level of the hepatic flexure to
become the transverse colon.
At this point it also becomes intraperitoneal.
It is connected to the inferior border of the pancreas by the transverse
mesocolon.
The greater omentum is attached to the superior aspect of the transverse
colon from which it can easily be separated. The mesentery contains the
middle colic artery and vein. The greater omentum remains attached to the
transverse colon up to the splenic flexure. At this point the colon undergoes
another sharp turn.

Relations
Superior Liver and gall-bladder, the greater curvature of the stomach, and
the lower end of the spleen

Inferior Small intestine

Anterior Greater omentum

Posterior From right to left with the descending portion of the duodenum, the
head of the pancreas, convolutions of the jejunum and ileum, spleen

Rate question: Next question

Com m ent on this question

All contents of this site are © 2012 E-Medical Revision Ltd Terms and Conditions Privacy policy
yomna92009@yahoo.com - MRCS Part A - My account

Reference ranges End and review

Question 16 of 90 Question stats Score: 31.3%

1
A 15.6%
Which of the following structures are not closely related to the adductor longus 2
B 30.2%
muscle? 3
C 18.3%
D 13.6% 4
A. Long saphenous vein E 22.4% 5
B. Tendon of iliacus 6
30.2% of users answered this
C. The profunda branch of the femoral artery question correctly
7
D. Pectineus muscle
8
E. Femoral nerve
9

10
Next question
11

12
Femoral triangle:
13
Adductor longus medially
Inguinal ligament superiorly 14

Sartorius muscle laterally 15

16

Adductor longus forms the medial border of the femoral triangle. It is closely
related to the long saphenous vein which overlies it and the profunda branch of
the femoral artery. The femoral nerve is related to it inferiorly. However, the
tendon of iliacus inserts proximally and is not in contact with adductor longus.

Adductor longus

Origin Anterior body of pubis

Insertion Middle third of linea aspera

Action Adducts and flexes the thigh, medially rotate the hip

Innervation Anterior division of obturator nerve (L2, L3, L4)

The schematic image below demonstrates the relationship of the adductor


muscles

Image sourced from Wikipedia


yomna92009@yahoo.com - MRCS Part A - My account

Reference ranges End and review

Question 17 of 90 Question stats Score: 35.3%

1
A 7.1%
Which of the following structures does not lie posterior to the right kidney? 2
B 34.1%
C 4.9% 3

A. Psoas major D 19.1% 4

B. Transversus abdominis E 34.9% 5

C. Quadratus lumborum 34.9% of users answered this 6


question correctly
D. Medial artcuate ligament 7

E. 10th rib 8

9
Next question
10

11
Theme from April 2012 Exam
The 10th rib lies more superior. The 12th rib is a closer relation posteriorly. 12

13
Renal anatomy
14

15
Each kidney is about 11cm long, 5cm wide and 3cm thick. They are located in a
deep gutter alongside the projecting verterbral bodies, on the anterior surface of 16
psoas major. In most cases the left kidney lies approximately 1.5cm higher than 17
the right. The upper pole of both kidneys approximates with the 11th rib (beware
pneumothorax during nephrectomy). On the left hand side the hilum is located at
the L1 vertebral level and the right kidney at level L1-2. The lower border of the
kidneys is usually alongside L3.

The table below shows the anatomical relations of the kidneys:

Relations
Relations Right Kidney Left Kidney

Posterior Quadratus lumborum, diaphragm, Quadratus lumborum, diaphragm,


psoas major, transversus abdominis psoas major, transversus abdominis

Anterior Hepatic flexure of colon Stomach, Pancreatic tail

Superior Liver, adrenal gland Spleen, adrenal gland

Fascial covering
Each kidney and suprarenal gland is enclosed within a common and layer of
investing fascia that is derived from the transversalis fascia into anterior and
posterior layers (Gerotas fascia).

Renal structure
Kidneys are surrounded by an outer cortex and an inner medulla which usually
contains between 6 and 10 pyramidal structures. The papilla marks the innermost
apex of these. They terminate at the renal pelvis, into the ureter.
Lying in a hollow within the kidney is the renal sinus. This contains:
1. Branches of the renal artery
2. Tributaries of the renal vein
3. Major and minor calyces's
4. Fat

Structures at the renal hilum


The renal vein lies most anteriorly, then renal artery (it is an end artery) and the
ureter lies most posterior.

Rate question: Next question

Com m ent on this question


yomna92009@yahoo.com - MRCS Part A - My account

Reference ranges End and review

Question 18 of 90 Question stats Score: 33.3%

1
A 7.3%
A 56 year old man is undergoing a radical nephrectomy via a posterior approach. 2
B 12%
Which of the following structures is most likely to be encountered during the 3
C 9.4%
operative approach?
D 64% 4
E 7.3% 5
A. 8th rib
64% of users answered this 6
B. 10th rib question correctly
7
C. 6th rib
8
D. 12th rib
9
E. 9th rib
10

Next question 11

12
The 11th and 12th ribs lie posterior to the kidneys and may be encountered 13
during a posterior approach. A pneumothorax is a recognised complication of this
14
type of surgery.
15

Renal anatomy 16

17
Each kidney is about 11cm long, 5cm wide and 3cm thick. They are located in a
18
deep gutter alongside the projecting verterbral bodies, on the anterior surface of
psoas major. In most cases the left kidney lies approximately 1.5cm higher than
the right. The upper pole of both kidneys approximates with the 11th rib (beware
pneumothorax during nephrectomy). On the left hand side the hilum is located at
the L1 vertebral level and the right kidney at level L1-2. The lower border of the
kidneys is usually alongside L3.

The table below shows the anatomical relations of the kidneys:

Relations
Relations Right Kidney Left Kidney

Posterior Quadratus lumborum, diaphragm, Quadratus lumborum, diaphragm,


psoas major, transversus abdominis psoas major, transversus abdominis

Anterior Hepatic flexure of colon Stomach, Pancreatic tail

Superior Liver, adrenal gland Spleen, adrenal gland

Fascial covering
Each kidney and suprarenal gland is enclosed within a common and layer of
investing fascia that is derived from the transversalis fascia into anterior and
posterior layers (Gerotas fascia).

Renal structure
Kidneys are surrounded by an outer cortex and an inner medulla which usually
contains between 6 and 10 pyramidal structures. The papilla marks the innermost
apex of these. They terminate at the renal pelvis, into the ureter.
Lying in a hollow within the kidney is the renal sinus. This contains:
1. Branches of the renal artery
2. Tributaries of the renal vein
3. Major and minor calyces's
4. Fat

Structures at the renal hilum


The renal vein lies most anteriorly, then renal artery (it is an end artery) and the
ureter lies most posterior.

Rate question: Next question

Com m ent on this question


yomna92009@yahoo.com - MRCS Part A - My account

Reference ranges End and review

Question 19 of 90 Question stats Score: 31.6%

1
A 5.6%
A 73 year old lady presents with a femoral hernia. Which of the following 2
B 57.3%
structures forms the lateral wall of the femoral canal? 3
C 9.6%
D 12.9% 4
A. Pubic tubercle E 14.6% 5
B. Femoral vein 6
57.3% of users answered this
C. Femoral artery question correctly
7
D. Conjoint tendon
8
E. Femoral nerve
9

10
Next question
11

The canal exists to allow for the physiological expansion of the femoral vein, which 12
lies lateral to it. 13

14
Femoral canal
15

The femoral canal lies at the medial aspect of the femoral sheath. The femoral 16
sheath is a fascial tunnel containing both the femoral artery laterally and femoral 17
vein medially. The canal lies medial to the vein.
18

Borders of the femoral canal 19

Laterally Femoral vein

Medially Lacunar ligament

Anteriorly Inguinal ligament

Posteriorly Pectineal ligament

Image showing dissection of femoral canal


yomna92009@yahoo.com - MRCS Part A - My account

Reference ranges End and review

Question 20 of 90 Question stats Score: 35%

1
A 11%
Which of the following is not a muscle of the rotator cuff? 2
B 8.2%
C 6.4% 3

A. Subscapularis D 5.6% 4

B. Teres minor E 68.7% 5

C. Supraspinatus 68.7% of users answered this 6


question correctly
D. Infraspinatus 7

E. Deltoid 8

9
Next question
10

11
Deltoid may abduct the shoulder and is not a rotator cuff muscle.
12

Muscles of the rotator cuff 13

14
Muscle Innervation
15
Supraspinatus muscle Suprascapular nerve
16
Infraspinatus muscle Suprascapular nerve
17
Teres minor muscle Axillary nerve
18
Subscapularis muscle Superior and inferior subscapular nerves
19

20

Rate question: Next question

Com m ent on this question

All contents of this site are © 2012 E-Medical Revision Ltd Terms and Conditions Privacy policy
yomna92009@yahoo.com - MRCS Part A - My account

Reference ranges End and review

Question 1 of 70 Question stats Score: 0%

1
A 71.8%
Which of the following muscles is not within the posterior compartment of the B 6%
lower leg?
C 5.7%
D 7.7%

A. Peroneus brevis E 8.8%

B. Flexor digitalis longus


71.8% of users answered this
C. Soleus question correctly

D. Popliteus
E. Flexor hallucis longus

Next question

Theme from 2007 Exam

Peroneus brevis lies in the lateral compartment.

Fascial compartments of the leg

Compartments of the thigh

Formed by 3 septae passing from the femur to the fascia lata.


Compartment Nerve Muscles Blood supply

Anterior Femoral Iliacus Femoral artery


compartment Tensor fasciae latae
Sartorius
Quadriceps femoris

Medial Obturator Adductor Profunda femoris artery and


compartment longus/magnus/brevis obturator artery
Gracilis
Obturator externus

Posterior Sciatic Semimembranosus Branches of Profunda


compartment Semitendinosus femoris artery
Biceps femoris

Compartments of the lower leg


Separated by the interosseous membrane (anterior and posterior compartments),
anterior fascial septum (separate anterior and lateral compartments) and
posterior fascial septum (separate lateral and posterior compartments)

Compartment Nerve Muscles Blood


supply

Anterior Deep Tibialis anterior Anterior


compartment peroneal Extensor digitorum longus tibial
nerve Extensor hallucis longus artery
Peroneus tertius

Posterior Tibial Muscles: deep and superficial Posterior


compartment compartments (separated by deep tibial
transverse fascia)
Deep: Flexor hallucis longus, Flexor
digitalis longus, Tibialis posterior,
Popliteus
Superficial: Gastrocnemius, Soleus,
Plantaris

Lateral Superficial Peroneus longus/brevis Anterior


yomna92009@yahoo.com - MRCS Part A - My account

Reference ranges End and review

Question 2 of 70 Question stats Score: 0%

1
A 9.6%
How many unpaired branches leave the abdominal aorta to supply the abdominal 2
B 8.9%
viscera?
C 49.1%
D 21.9%

A. One E 10.5%

B. Two
49.1% of users answered this
C. Three question correctly

D. Four
E. Five

Next question

There are three unpaired branches to the abdominal viscera. These include the
coeliac axis, the SMA and IMA. Branches to the adrenals, renal arteries and
gonadal vessels are paired. The fourth unpaired branch of the abdominal aorta,
the median sacral artery, does not directly supply the abdominal viscera.

Abdominal aortic branches

Branches Level Paired Type

Inferior phrenic T12 (Upper border) Yes Parietal

Coeliac T12 No Visceral

Superior mesenteric L1 No Visceral

Middle suprarenal L1 Yes Visceral

Renal L1-L2 Yes Visceral

Gonadal L2 Yes Visceral

Lumbar L1-L4 Yes Parietal

Inferior mesenteric L3 No Visceral

Median sacral L4 No Parietal

Common iliac L4 Yes Terminal

Rate question: Next question

Com m ent on this question

All contents of this site are © 2012 E-Medical Revision Ltd Terms and Conditions Privacy policy
yomna92009@yahoo.com - MRCS Part A - My account

Reference ranges End and review

Question 3 of 70 Question stats Score: 0%

1
A 30.2%
A 34 year old man with a submandibular gland stone is undergoing excision of the 2
B 26.7%
submandibular gland. The incision is sited transversely approximately 4cm below 3
C 19.9%
the mandible. After incising the skin, platysma and deep fascia which of the
D 15.6%
following structures is most likely to be encountered.
E 7.6%

A. Facial artery 26.7% of users answered this


question correctly
B. Facial vein
C. Lingual nerve
D. Hypoglossal nerve
E. Glossopharyngeal nerve

Next question

When approaching the submandibular gland the facial vein and submandibular
lymph nodes are the most superficially encountered structures. Each sub
mandibular gland has a superficial and deep part, separated by the mylohoid
muscle. The facial artery grooves the deep aspect of the superficial aspect of the
gland. It then emerges onto the surface of the face by passing between the gland
and the mandible. The facial vein is encountered first in this surgical approach
because the incision is made 4cm below the mandible (to avoid injury to the
marginal mandibular nerve).

Submandibular gland

Relations of the submandibular gland


Superficial Platysma, deep fascia and mandible
Submandibular lymph nodes
Facial vein (facial artery near mandible)
Marginal mandibular nerve
Cervical branch of the facial nerve

Deep Facial artery (inferior to the mandible)


Mylohoid muscle
Sub mandibular duct
Hyoglossus muscle
Lingual nerve
Submandibular ganglion
Hypoglossal nerve

Submandibular duct (Wharton's duct)

Opens lateral to the lingual frenulum on the anterior floor of mouth.


5 cm length
Lingual nerve wraps around Wharton's duct. As the duct passes forwards it
crosses medial to the nerve to lie above it and then crosses back, lateral to
it, to reach a position below the nerve.

Innervation

Sympathetic innervation- Superior Cervical ganglion via the Lingual nerve


Parasympathetic innervation- Submandibular ganglion

Arterial supply
Branch of the Facial artery. The facial artery passes through the gland to groove
its deep surface. It then emerges onto the face by passing between the gland and
the mandible.

Venous drainage
Anterior Facial vein (lies deep to the Marginal Mandibular nerve)
yomna92009@yahoo.com - MRCS Part A - My account

Reference ranges End and review

Question 4 of 70 Question stats Score: 0%

1
A 24.9%
You are working as an anatomy demonstrator and the medical students decide to 2
B 14.2%
test your knowledge on the circle of willis. Which of the following comments is 3
C 27.6%
false?
D 14.9% 4
E 18.4%
A. Does not include the middle cerebral artery
27.6% of users answered this
B. Asymmetry of the circle of willis is a risk factor for the development question correctly
of intracranial aneurysms
C. Majority of blood passing through the vessels mix together
D. Includes the anterior communicating arteries
E. The circle surrounds the stalk of the pituitary gland

Next question

There is minimum mixing of blood passing through the vessels.

Circle of Willis

May also be called the circulus arteriosus

Inferior surface of brain


Supplied by the internal carotid arteries and the vertebral arteries
If artery is occluded, collaterals may be able to compensate
Components include:

1. Anterior communicating arteries


2. Anterior cerebral arteries
3. Internal carotid arteries
4. Posterior communicating arteries
5. Posterior cerebral arteries and the termination of the basilar artery

Supply: Corpus striatum, internal capsule, diencephalon, midbrain


yomna92009@yahoo.com - MRCS Part A - My account

Reference ranges End and review

1/3 Question 5-7 of 70 Question stats Score: 14.3%

Average score for registered users: 1

Theme: Surgical nerve lesions 2


5 76.6%
3
6 89.2%
A. Recurrent laryngeal nerve
7 37.3% 4
B. Accessory nerve 5-7 1/3
C. Hypoglossal nerve
D. Vagus nerve
E. Common peroneal nerve
F. Tibial nerve
G. Long saphenous nerve
H. Phrenic nerve
I. Thoracodorsal nerve

Please choose which of the listed nerves is at greatest risk for the procedures
given. Each option may be used once, more than once or not at all.

5. A 64 year old man has a suspected lymphoma and lymph node


biopsy from the posterolateral aspect of the right neck is planned.

You answered Phrenic nerve

The correct answer is Accessory nerve

The accessory nerve has a superficial course and is easily injured. It


lies under platysma and may be divided during the early part of the
procedure.

6. A 43 year old women is due to undergo an axillary node clearance


following a positive sentinel node biopsy.

Thoracodorsal nerve

The long thoracic nerve is also at risk. This nerve traverses the level
2 axillary nerves to supply latissimus dorsi and may be divided or
damaged with diathermy.

7. A 53 year old man is to undergo an abdomino-perineal excision of


the colon and rectum for a tumour 2cm from the anal verge.

You answered Thoracodorsal nerve

The correct answer is Common peroneal nerve

This operation is commonly performed in the Lloyd Davies position


and if the patient is not positioned correctly a nerve lesion may occur
as a result of pressure neuropraxia.

Next question

Nerve lesions during surgery

A variety of different procedures carry the risk of iatrogenic nerve injury. These
are important not only from the patients perspective but also from a medicolegal
standpoint.
yomna92009@yahoo.com - MRCS Part A - My account

Reference ranges End and review

Question 8 of 70 Question stats Score: 12.5%

1
A 42%
A 45 year old man presents with bilateral inguinal hernias. The surgical team 2
B 21.5%
decide to repair these laparoscopically through an extraperitoneal approach. 3
C 12.3%
Through an infraumbilical incision the surgeons displace the inferior aspect of the
D 15.1% 4
rectus abdominis muscle anteriorly and place a prosthetic mesh into the area to
E 9.1% 5-7 1/3
repair the hernias. Which structure will lie posterior to the mesh?
8
42% of users answered this
question correctly
A. Peritoneum
B. Internal oblique aponeurosis
C. External oblique aponeurosis
D. Posterior aspect of the rectus sheath
E. Bucks fascia

Next question

During a TEP repair of inguinal hernia the only structure to lie posterior to the
mesh is peritoneum. The question is really only asking which structure lies
posterior to the rectus abdominis muscle. Since this region is below the arcuate
line the transversalis fascia and peritoneum lie posterior to it. Bucks fascia lies in
the penis.

Rectus abdominis muscle

The rectus sheath is formed by the aponeuroses of the lateral abdominal wall
muscles. The rectus sheath has a composition that varies according to
anatomical level.

1. Above the costal margin the anterior sheath is composed of external oblique
aponeurosis, the costal cartilages are posterior to it.
2. From the costal margin to the arcuate line, the anterior rectus sheath is
composed of external oblique aponeurosis and the anterior part of the internal
oblique aponeurosis. The posterior part of the internal oblique aponeurosis and
transversus abdominis form the posterior rectus sheath.
3. Below the arcuate line the aponeuroses of all the abdominal muscles lie in
anterior aspect of the rectus sheath. Posteriorly lies the transversalis fascia and
peritoneum.

The arcuate line is the point at which the inferior epigastric vessels enter the
rectus sheath.

Rate question: Next question

Com m ent on this question

All contents of this site are © 2012 E-Medical Revision Ltd Terms and Conditions Privacy policy
yomna92009@yahoo.com - MRCS Part A - My account

Reference ranges End and review

Question 9 of 70 Question stats Score: 11.1%

1
A 16.2%
A 20 year old man undergoes an open appendicectomy performed via a lanz 2
B 50.8%
incision. This surgeon places the incision on a level of the anterior superior iliac 3
C 6.3%
spine in an attempt to improve cosmesis. During the procedure the appendix is
D 21.2% 4
found to be retrocaecal and the incision is extended laterally. Which of the
E 5.5% 5-7 1/3
following nerves is at greatest risk of injury?
8
50.8% of users answered this
question correctly 9
A. Genitofemoral
B. Ilioinguinal
C. Obturator
D. Lateral femoral cutaneous
E. Femoral

Next question

Theme from April 2012 Exam

Ilioinguinal nerve

Arises from the first lumbar ventral ramus with the iliohypogastric nerve. It passes
inferolaterally through the substance of psoas major and over the anterior
surface of quaratus lumborum. It pierces the internal oblique muscle and passes
deep to the aponeurosis of the external oblique muscle. It enters the inguinal
canal and then passes through the superficial inguinal ring to reach the skin.

Branches

To supply those muscles of the abdominal wall through which it passes.


Skin and fascia over the pubic symphysis, superomedial part of the femoral
triangle, surface of the scrotum, root and dorsum of penis or labum majus
in females.

Rate question: Next question

Com m ent on this question

All contents of this site are © 2012 E-Medical Revision Ltd Terms and Conditions Privacy policy
yomna92009@yahoo.com - MRCS Part A - My account

Reference ranges End and review

Question 10 of 70 Question stats Score: 10%

1
A 7.6%
The femoral nerve is transected by a rather careless surgeon during a botched 2
B 7.7%
femoro-popliteal bypass operation. Which of the following actions will be 3
C 13.3%
impaired?
D 66.9% 4
E 4.5% 5-7 1/3
A. Extension of the great toe 8
66.9% of users answered this
B. Adduction of the thigh question correctly 9
C. Flexion of the knee joint
10
D. Extension of the knee joint
E. Eversion of the foot

Next question

The femoral nerve supplies the quadriceps muscle which is responsible for
extension at the knee joint.

Femoral nerve

Root values L2, 3, 4

Innervates Pectineus
Sartorius
Quadriceps femoris
Vastus lateralis/medialis/intermedius

Branches Medial cutaneous nerve of thigh


Saphenous nerve
Intermediate cutaneous nerve of thigh

Path
Penetrates psoas major and exits the pelvis by passing under the inguinal
ligament to enter the femoral triangle, lateral to the femoral artery and vein.

Image sourced from Wikipedia


yomna92009@yahoo.com - MRCS Part A - My account

Reference ranges End and review

Question 11 of 70 Question stats Score: 9.1%

1
A 21.5%
Which of the following is not a direct branch of the facial nerve? 2
B 11.3%
C 43.3% 3

A. Greater petrosal nerve D 14% 4

B. Nerve to stapedius E 9.9% 5-7 1/3

C. Auriculotemporal 8
43.3% of users answered this
question correctly 9
D. Chorda tympani
E. Buccal 10

11
Next question

The auriculotemporal nerve is a direct branch of the mandibular nerve.


Other branches of the mandibular nerve include:
Lingual
Inferior Alveolar
N. To the Mylohyoid
Mental

Facial nerve

The facial nerve is the main nerve supplying the structures of the second
embryonic branchial arch. It is predominantly an efferent nerve to the muscles of
facial expression, digastric muscle and also to many glandular structures. It
contains a few afferent fibres which originate in the cells of its genicular ganglion
and are concerned with taste.

Supply - 'face, ear, taste, tear'

Face: muscles of facial expression


Ear: nerve to stapedius
Taste: supplies anterior two-thirds of tongue
Tear: parasympathetic fibres to lacrimal glands, also salivary glands

Path
Subarachnoid path

Origin: motor- pons, sensory- nervus intermedius


Pass through the petrous temporal bone into the internal auditory meatus
with the vestibulocochlear nerve. Here they combine to become the facial
nerve.

Facial canal path

The canal passes superior to the vestibule of the inner ear


At the medial aspect of the middle ear, it becomes wider and contains the
geniculate ganglion.

- 3 branches:
1. greater petrosal nerve
2. nerve to stapedius
3. chorda tympani

Stylomastoid foramen

Passes through the stylomastoid foramen (tympanic cavity anterior and


mastoid antrum posteriorly)
Posterior auricular nerve and branch to Posterior belly of Digastric and
Stylohyoid muscle
yomna92009@yahoo.com - MRCS Part A - My account

Reference ranges End and review

Question 12 of 70 Question stats Score: 8.3%

1
A 8.3%
A 32 year old man is stabbed in the neck and the inferior trunk of his brachial 2
B 8.9%
plexus is injured. Which of the modalities listed below is least likely to be affected? 3
C 60.6%
D 11.2% 4
A. Initiating abduction of the shoulder E 11% 5-7 1/3

B. Abduction of the fingers 8


8.3% of users answered this
C. Flexion of the little finger question correctly 9

D. Sensation on the palmar aspect of the little finger 10

E. Gripping a screwdriver 11

12
Next question

Inferior trunk of brachial plexus.

C8 and T1 roots
Contributes to ulnar nerve and part of median nerve

Theme from September 2012 Exam


The inferior trunk of the brachial plexus is rarely injured. Nerve roots C8 and T1
are the main contributors to this trunk. Therefore an injury to this site will most
consistently affect the ulnar nerve. The inferior trunk also contributes to the
median nerve by way of the posterior division and therefore some impairment of
grip is almost inevitable.

Brachial plexus

Origin Anterior rami of C5 to T1

Sections of the Roots, trunks, divisions, cords, branches


plexus Mnemonic:Real Teenagers Drink Cold Beer

Roots Located in the posterior triangle


Pass between scalenus anterior and medius

Trunks Located posterior to middle third of clavicle


Upper and middle trunks related superiorly to the subclavian
artery
Lower trunk passes over 1st rib posterior to the subclavian
artery

Divisions Apex of axilla

Cords Related to axillary artery

Diagram illustrating the branches of the brachial plexus


yomna92009@yahoo.com - MRCS Part A - My account

Reference ranges End and review

Question 13 of 70 Question stats Score: 7.7%

1
A 36.7%
During embryological development, which of the following represent the correct 2
B 26.6%
origin of the pancreas? 3
C 16.9%
D 12.7% 4
A. Ventral and dorsal endodermal outgrowths of the duodenum E 7.1% 5-7 1/3

B. Ventral and dorsal outgrowths of mesenchymal tissue from the 8


36.7% of users answered this
posterior abdominal wall question correctly 9
C. Ventral and dorsal outgrowths of the vitellointestinal duct
10
D. Ventral and dorsal biliary tract diverticulae
11
E. Buds from the inferior aspect of the caudate lobe
12

13
Next question

The pancreas develops from a ventral and dorsal endodermal outgrowth of the
duodenum. The ventral arises close to, or in common with the hepatic
diverticulum, and the larger, dorsal outgrowth arises slightly cranial to the ventral
extending into the mesoduodenum and mesogastrium. When the buds eventually
fuse the duct of the ventral rudiment becomes the main pancreatic duct.

Pancreas

The pancreas is a retroperitoneal organ and lies posterior to the stomach. It may
be accessed surgically by dividing the peritoneal reflection that connects the
greater omentum to the transverse colon. The pancreatic head sits in the
curvature of the duodenum. It's tail lies close to the hilum of the spleen, a site of
potential injury during splenectomy.

Relations
Posterior to the pancreas
Pancreatic head Inferior vena cava
Common bile duct
Right and left renal veins
Superior mesenteric vein and artery

Pancreatic neck Superior mesenteric vein, portal vein

Pancreatic body- Left renal vein


Crus of diaphragm
Psoas muscle
Adrenal gland
Kidney
Aorta

Pancreatic tail Left kidney

Anterior to the pancreas


Pancreatic head 1st part of the duodenum
Pylorus
Gastroduodenal artery
SMA and SMV(uncinate process)

Pancreatic body Stomach


Duodenojejunal flexure

Pancreatic tail Splenic hilum

Superior to the pancreas


Coeliac trunk and its branches common hepatic artery and splenic artery

Grooves of the head of the pancreas


2nd and 3rd part of the duodenum

Arterial supply

Head: pancreaticoduodenal artery


yomna92009@yahoo.com - MRCS Part A - My account

Reference ranges End and review

Question 14 of 70 Question stats Score: 7.1%

1
A 13.9%
Which of the following is not a content of the anterior triangle of the neck? 2
B 9.8%
C 12% 3

A. Ansa cervicalis D 45.3% 4

B. Vagus nerve E 18.9% 5-7 1/3

C. Anterior jugular vein 8


45.3% of users answered this
question correctly 9
D. Transverse cervical nerve
E. Hypoglossal nerve 10

11
Next question 12

13

14

Anterior triangle of the neck

Boundaries

Anterior border of the Sternocleidomastoid


Lower border of mandible
Anterior midline

Sub triangles (divided by Digastric above and Omohyoid)

Muscular triangle: Neck strap muscles


Carotid triangle: Carotid sheath
Submandibular Triangle (digastric)

Contents of the anterior triangle


Digastric triangle Submandibular gland
Submandibular nodes
Facial vessels
Hypoglossal nerve

Muscular triangle Strap muscles


External jugular vein

Carotid triangle Carotid sheath (Common carotid, vagus and internal jugular vein)
Ansa cervicalis

Nerve supply to digastric muscle

Anterior: Mylohyoid nerve


Posterior: Facial nerve

Image sourced from Wikipedia

Rate question: Next question

Com m ent on this question


yomna92009@yahoo.com - MRCS Part A - My account

Reference ranges End and review

Question 15 of 70 Question stats Score: 6.7%

1
A 18.2%
A 23 year old man presents with delayed diagnosis of appendicitis. The appendix 2
B 11.1%
is retrocaecal and has perforated causing a psoas abscess. Into which structure 3
C 53%
does the psoas major muscle insert?
D 11.7% 4
E 5.9% 5-7 1/3
A. Greater trochanter of the femur 8
53% of users answered this
B. Linea aspera of the femur question correctly 9
C. Lesser trochanter of the femur
10
D. Iliac crest
11
E. None of the above
12

13
Next question
14

Theme based on 2011 exam 15


The psoas major inserts into the lesser trochanter.

Psoas Muscle

Origin
The deep part originates from the transverse processes of the five lumbar
vertebrae, the superficial part originates from T12 and the first 4 lumbar
vertebrae.

Insertion
Lesser trochanter of the femur.

Innervation
Anterior rami of L1 to L3.

Action
Flexion and external rotation of the hip. Bilateral contraction can raise the trunk
from the supine position.

Rate question: Next question

Com m ent on this question

All contents of this site are © 2012 E-Medical Revision Ltd Terms and Conditions Privacy policy
yomna92009@yahoo.com - MRCS Part A - My account

Reference ranges End and review

Question 16 of 70 Question stats Score: 12.5%

1
A 13.3%
A 63 year old man is due to undergo a splenectomy. Which splenic structure lies 2
B 12.5%
most posteriorly? 3
C 12.2%
D 7.7% 4
A. Gastrosplenic ligament E 54.3% 5-7 1/3

B. Splenic vein 8
54.3% of users answered this
C. Splenic artery question correctly 9

D. Splenic notch 10

E. Lienorenal ligament 11

12
Next question
13

14
Theme from 2011 Exam
15
The lienorenal ligament lies most posteriorly. The antero-lateral connection is via 16
the phrenicocolic ligament. Anteriorly the gastro splenic ligament. These
structures condense around the vessels at the splenic hilum.

Splenic anatomy

The spleen is the largest lymphoid organ in the body. It is an intraperitoneal


organ, the peritoneal attachments condense at the hilum where the vessels enter
the spleen. Its blood supply is from the splenic artery (derived from the coeliac
axis) and the splenic vein (which is joined by the IMV and unites with the SMV).

Embryology: derived from mesenchymal tissue


Shape: clenched fist
Position: below 9th-12th ribs
Weight: 75-150g

Relations

Superiorly- diaphragm
Anteriorly- gastric impression
Posteriorly- kidney
Inferiorly- colon
Hilum: tail of pancreas and splenic vessels
Forms apex of lesser sac (containing short gastric vessels)

Rate question: Next question

Com m ent on this question

All contents of this site are © 2012 E-Medical Revision Ltd Terms and Conditions Privacy policy
yomna92009@yahoo.com - MRCS Part A - My account

Reference ranges End and review

Question 17 of 70 Question stats Score: 11.8%

1
A 6.1%
Which of the following statements about the spleen is false? 2
B 58.7%
C 10.8% 3

A. The hilum contains the splenic vessels. D 13.5% 4

B. The spleen is derived from endodermal tissue. E 10.8% 5-7 1/3

C. The white pulp has immune function. 8


58.7% of users answered this
question correctly 9
D. The colon lies inferiorly.
E. Weighs 150g. 10

11
Next question 12

13

1,3,5,7,9,11 (odd numbers up to 11) 14

15
The spleen is: 1 inch thick, 3 inches wide, 5 inches long, weighs 7oz (200g),
16
lies between the 9th and 11th ribs
17

Most of the gut is derived endodermally except for the spleen which is from
mesenchymal tissue.

Spleen

Embryology: derived from mesenchymal tissue


Shape: orange segment
Position: below 9th-12th ribs
Weight: 75-150g

Relations

Superiorly- diaphragm
Anteriorly- gastric impression
Posteriorly- kidney
Inferiorly- colon
Hilum: tail of pancreas and splenic vessels (splenic artery divides here,
branches pass to the white pulp transporting plasma)
Forms apex of lesser sac (containing short gastric vessels)

Contents
- White pulp: immune function. Contains central trabecular artery. The germinal
centres are supplied by arterioles called penicilliary radicles.
- Red pulp: filters abnormal red blood cells

Function

Filtration of abnormal blood cells and foreign bodies such as bacteria.


Immunity: IgM. Production of properdin, and tuftsin which help target fungi
and bacteria for phagocytosis.
Haematopoiesis: up to 5th month gestation or in haematological disorders.
Pooling: storage of 40% platelets.
Iron reutilisation
Storage red blood cells-animals, not humans.
Storage monocytes

Disorders of the spleen


Massive splenomegaly

Myelofibrosis
Chronic myeloid leukaemia
Visceral leishmaniasis (kala-azar)
Malaria
Gaucher's syndrome

Other causes (as above plus)

Portal hypertension e.g. secondary to cirrhosis


Lymphoproliferative disease e.g. CLL, Hodgkin's
Haemolytic anaemia
Infection: hepatitis, glandular fever
Infective endocarditis
Sickle-cell*, thalassaemia
Rheumatoid arthritis (Felty's syndrome)

*the majority of adults patients with sickle-cell will have an atrophied spleen due to
repeated infarction

Rate question: Next question

Com m ent on this question

All contents of this site are © 2012 E-Medical Revision Ltd Terms and Conditions Privacy policy
yomna92009@yahoo.com - MRCS Part A - My account

Reference ranges End and review

Question 18 of 70 Question stats Score: 11.1%

1
A 6.4%
As it exits the axilla the radial nerve lies on which of the following muscles? 2
B 15.2%
C 52.6% 3

A. Supraspinatus D 13.3% 4

B. Infraspinatus E 12.5% 5-7 1/3

C. Teres major 8
52.6% of users answered this
question correctly 9
D. Deltoid
E. Pectoralis major 10

11
Next question 12

13
The radial nerve passes through the triangular space to leave the axilla. The
14
superior border of this is bounded by the teres major muscle to which the radial
nerve is closely related. 15

16
Radial nerve
17

18
Continuation of posterior cord of the brachial plexus (root values C5 to T1)

Path

In the axilla: lies posterior to the axillary artery on subscapularis, latissimus


dorsi and teres major.
Enters the arm between the brachial artery and the long head of triceps
(medial to humerus).
Spirals around the posterior surface of the humerus in the groove for the
radial nerve.
At the distal third of the lateral border of the humerus it then pierces the
intermuscular septum and descends in front of the lateral epicondyle.
At the lateral epicondyle it lies deeply between brachialis and
brachioradialis where it then divides into a superficial and deep terminal
branch.
Deep branch crosses the supinator to become the posterior interosseous
nerve.

In the image below the relationships of the radial nerve can be appreciated

Image sourced from Wikipedia

Regions innervated

Motor (main Triceps


nerve) Anconeus
yomna92009@yahoo.com - MRCS Part A - My account

Reference ranges End and review

Question 19 of 70 Question stats Score: 10.5%

1
A 5.2%
Into which of the following veins does the middle thyroid vein drain? 2
B 20.7%
C 49.2% 3

A. Vertebral D 17% 4

B. External jugular E 7.9% 5-7 1/3

C. Internal jugular 8
49.2% of users answered this
question correctly 9
D. Subclavian
E. Anterior jugular 10

11
Next question 12

13
It drains to the internal jugular vein. Which is one of the reasons why it bleeds so
14
copiously if a ligature slips.
15

Thyroid gland 16

17

Right and left lobes connected by isthmus 18


Surrounded by sheath from pretracheal layer of deep fascia
19
Apex: Lamina of thyroid cartilage
Base: 4th-5th tracheal ring
Pyramidal lobe: from isthmus
May be attached to foramen caecum at the base of the tongue

Relations

Anteromedially Sternothyroid
Superior belly of omohyoid
Sternohyoid
Anterior aspect of sternocleidomastoid

Posterolaterally Carotid sheath

Medially Larynx
Trachea
Pharynx
Oesophagus
Cricothyroid muscle

External laryngeal nerve (near superior thyroid artery)


Recurrent laryngeal nerve (near inferior thyroid artery)

Posterior Parathyroid glands


Anastomosis of superior and inferior thyroid arteries

Isthmus Anteriorly: Sternothyroids, sternohyoids, anterior jugular veins


Posteriorly: 2nd, 3rd, 4th tracheal rings (attached via Ligament of
Berry)

Blood Supply

Arterial Superior thyroid artery (1st branch of external carotid)


Inferior thyroid artery (from thyrocervical trunk)
Thyroidea ima (in 10% of population -from brachiocephalic artery or aorta)

Venous Superior and middle thyroid veins - into the IJV


Inferior thyroid vein - into the brachiocephalic veins

Rate question: Next question


yomna92009@yahoo.com - MRCS Part A - My account

Reference ranges End and review

Question 20 of 70 Question stats Score: 10%

1
A 9.2%
Which of the following structures is not at the level of the transpyloric plane? 2
B 7%
C 6.3% 3

A. Hilum left kidney D 64.3% 4

B. Superior mesenteric artery E 13.1% 5-7 1/3

C. Fundus of the gallbladder 8


64.3% of users answered this
question correctly 9
D. Cardioesophageal junction
E. Root of transverse mesocolon 10

11
Next question 12

13
A knowledge of this anatomic level is commonly tested.
14

Levels 15

16
Transpyloric plane
17
Level of the body of L1
18

19
Pylorus stomach
20
Left kidney hilum (L1- left one!)
Right hilum of the kidney (1.5cm lower than the left)
Fundus of the gallbladder
Neck of pancreas
Duodenojejunal flexure
Superior mesenteric artery
Portal vein
Left and right colic flexure
Root of the transverse mesocolon
2nd part of the duodenum
Upper part of conus medullaris
Spleen

Can be identified by asking the supine patient to sit up without using their arms.
The plane is located where the lateral border of the rectus muscle crosses the
costal margin.

Anatomical planes
Subcostal plane Lowest margin of 10th costal cartilage

Intercristal plane Level of body L4 (highest point of iliac crest)

Intertubercular plane Level of body L5

Common level landmarks


Inferior mesenteric artery L3

Bifurcation of aorta into common iliac arteries L4

Formation of IVC L5 (union of common iliac veins)

Diaphragm apertures Vena cava T8


Oesophagus T10
Aortic hiatus T12

Rate question: Next question

Com m ent on this question


yomna92009@yahoo.com - MRCS Part A - My account

Reference ranges End and review

Question 1 of 50 Question stats Score: 0%

1
A 5.2%
A 62 year old man presents with arm weakness. On examination he has a B 7.9%
weakness of elbow extension and loss of sensation on the dorsal aspect of the
C 6.7%
first digit. What is the site of the most likely underlying defect?
D 71.7%

E 8.5%
A. Axillary nerve
71.7% of users answered this
B. Median nerve question correctly
C. Ulnar nerve
D. Radial nerve
E. Musculocutaneous nerve

Next question

Theme from April 2012 Exam


The long head of the triceps muscle may be innervated by the axillary nerve and
therefore complete loss of triceps muscles function may not be present even with
proximally sited nerve lesions.

Radial nerve

Continuation of posterior cord of the brachial plexus (root values C5 to T1)

Path

In the axilla: lies posterior to the axillary artery on subscapularis, latissimus


dorsi and teres major.
Enters the arm between the brachial artery and the long head of triceps
(medial to humerus).
Spirals around the posterior surface of the humerus in the groove for the
radial nerve.
At the distal third of the lateral border of the humerus it then pierces the
intermuscular septum and descends in front of the lateral epicondyle.
At the lateral epicondyle it lies deeply between brachialis and
brachioradialis where it then divides into a superficial and deep terminal
branch.
Deep branch crosses the supinator to become the posterior interosseous
nerve.

In the image below the relationships of the radial nerve can be appreciated

Image sourced from Wikipedia


yomna92009@yahoo.com - MRCS Part A - My account

Reference ranges End and review

Question 2 of 50 Question stats Score: 0%

1
A 19.8%
From which of the following structures does the long head of the triceps muscle 2
B 13.5%
arise?
C 51.3%
D 8.4%

A. Coracoid process E 7%

B. Acromion
51.3% of users answered this
C. Infraglenoid tubercle question correctly

D. Coraco-acromial ligament
E. Coraco-humeral ligament

Next question

The long head arises from the infraglenoid tubercle. The fleshy lateral and medial
heads are attached to the posterior aspect of the humerus between the insertion
of the teres minor and the olecranon fossa.

Triceps

Origin Long head- infraglenoid tubercle of the scapula.


Lateral head- dorsal surface of the humerus, lateral and proximal to the
groove of the radial nerve
Medial head- posterior surface of the humerus on the inferomedial side
of the radial groove and both of the intermuscular septae

Insertion Olecranon process of the ulna. Here the olecranon bursa is between
the triceps tendon and olecranon.
Some fibres insert to the deep fascia of the forearm, posterior capsule
of the elbow (preventing the capsule from being trapped between
olecranon and olecranon fossa during extension)

Innervation Radial nerve

Blood Profunda brachii artery


supply

Action Elbow extension. The long head can adduct the humerus and and extend it
from a flexed position

Relations The radial nerve and profunda brachii vessels lie between the lateral and
medial heads

Rate question: Next question

Com m ent on this question

All contents of this site are © 2012 E-Medical Revision Ltd Terms and Conditions Privacy policy
yomna92009@yahoo.com - MRCS Part A - My account

Reference ranges End and review

Question 3 of 50 Question stats Score: 0%

1
A 68.5%
A 45 year old man is undergoing a left hemicolectomy. As the surgeons mobilise 2
B 6.8%
the left colon they identify a tubular structure lying at the inferior aspect of psoas 3
C 8%
major. What is it most likely to be?
D 8.9%

E 7.7%
A. Left ureter
68.5% of users answered this
B. Left common iliac vein question correctly
C. Left common iliac artery
D. Left external iliac artery
E. Left external iliac vein

Next question

The left ureter lies posterior to the left colon. The sigmoid colon and upper rectum
may be more closely related to the iliac vessels. These are not typically found
above L4.

Ureter

25-35 cm long
Muscular tube lined by transitional epithelium
Surrounded by thick muscular coat. Becomes 3 muscular layers as it
crosses the bony pelvis
Retroperitoneal structure overlying transverse processes L2-L5
Lies anterior to bifurcation of iliac vessels
Blood supply is segmental; renal artery, aortic branches, gonadal
branches, common iliac and internal iliac
Lies beneath the uterine artery

Rate question: Next question

Com m ent on this question

All contents of this site are © 2012 E-Medical Revision Ltd Terms and Conditions Privacy policy
yomna92009@yahoo.com - MRCS Part A - My account

Reference ranges End and review

Question 4 of 50 Question stats Score: 0%

1
A 35.8%
A 42 year old woman is due to undergo a left nephroureterectomy for a 2
B 15.8%
transitional cell carcinoma involving the ureter. Which of the following structures is 3
C 13.5%
not related to the left ureter?
D 19.6% 4
E 15.2%
A. Round ligament of the uterus
35.8% of users answered this
B. Internal iliac artery question correctly
C. Ovarian artery
D. Peritoneum
E. Sigmoid mesocolon

Next question

The ureter is not related to the round ligament of the uterus, it is related to the
broad ligament and is within 1.5cm of the supravaginal part of the cervix.

Ureter

25-35 cm long
Muscular tube lined by transitional epithelium
Surrounded by thick muscular coat. Becomes 3 muscular layers as it
crosses the bony pelvis
Retroperitoneal structure overlying transverse processes L2-L5
Lies anterior to bifurcation of iliac vessels
Blood supply is segmental; renal artery, aortic branches, gonadal
branches, common iliac and internal iliac
Lies beneath the uterine artery

Rate question: Next question

Com m ent on this question

All contents of this site are © 2012 E-Medical Revision Ltd Terms and Conditions Privacy policy
yomna92009@yahoo.com - MRCS Part A - My account

Reference ranges End and review

Question 5 of 50 Question stats Score: 0%

1
A 11.1%
Which of the following can be a branch of the brachiocephalic artery? 2
B 8.7%
C 51.9% 3

A. Vertebral artery D 13.1% 4

B. Subscapular artery E 15.2% 5

C. Thyroidea ima artery 51.9% of users answered this


question correctly
D. Left Subclavian artery
E. None of the above

Next question

Other occasional branches include the thymic and bronchial branch.

Brachiocephalic artery

The brachiocephalic artery is the largest branch of the aortic arch. From its aortic
origin it ascends superiorly, it initially lies anterior to the trachea and then on its
right hand side. It branches into the common carotid and right subclavian arteries
at the level of the sternoclavicular joint.

Path
Origin- apex of the midline of the aortic arch
Passes superiorly and posteriorly to the right
Divides into the right subclavian and right common carotid artery

Relations

Anterior Sternohyoid
Sternothyroid
Thymic remnants
Left brachiocephalic vein
Right inferior thyroid veins

Posterior Trachea
Right pleura

Right lateral Right brachiocephalic vein


Superior part of SVC

Left lateral Thymic remnants


Origin of left common carotid
Inferior thyroid veins
Trachea (higher level)

Branches
Normally none but may have the thyroidea ima artery
yomna92009@yahoo.com - MRCS Part A - My account

Reference ranges End and review

Question 6 of 50 Question stats Score: 0%

1
A 12.6%
A 28 year old man is undergoing an appendicectomy. The external oblique 2
B 44.3%
aponeurosis is incised and the underlying muscle split in the line of its fibres. At 3
C 13.1%
the medial edge of the wound is a tough fibrous structure. Entry to this structure
D 14.6% 4
will most likely encounter which of the following?
E 15.4% 5

44.3% of users answered this 6


A. Internal oblique
question correctly
B. Rectus abdominis
C. Transversus abdominis
D. Linea alba
E. Peritoneum

Next question

This structure will be the rectus sheath and when entered the rectus abdominis
muscle will be encountered.

Abdominal incisions

Theme in January 2012 exam

Midline Commonest approach to the abdomen


incision Structures divided: linea alba, transversalis fascia, extraperitoneal
fat, peritoneum (avoid falciform ligament above the umbilicus)
Bladder can be accessed via an extraperitoneal approach through
the space of Retzius

Paramedian Parallel to the midline (about 3-4cm)


incision Structures divided/retracted: anterior rectus sheath, rectus
(retracted), posterior rectus sheath, transversalis fascia,
extraperitoneal fat, peritoneum
Incision is closed in layers

Battle Similar location to paramedian but rectus displaced medially (and


thus denervated)
Now seldom used

Kocher's Incision under right subcostal margin e.g. Cholecystectomy (open)

Lanz Incision in right iliac fossa e.g. Appendicectomy

Gridiron Oblique incision centered over McBurneys point- usually appendicectomy


(less cosmetically acceptable than Lanz

Gable Rooftop incision

Pfannenstiel's Transverse supra pubic, primarily used to access pelvic organs

McEvedy's Groin incision e.g. Emergency repair strangulated femoral hernia

Rutherford Extraperitoneal approach to left or right lower quadrants. Gives excellent


Morrison access to iliac vessels and is the approach of choice for first time renal
transplantation.
yomna92009@yahoo.com - MRCS Part A - My account

Reference ranges End and review

Question 7 of 50 Question stats Score: 0%

1
A 10%
A 35 year old man presents to the surgical clinic with a suspected direct inguinal 2
B 20.5%
hernia. These will pass through Hesselbach's triangle. Which of the following 3
C 55.3%
forms the medial edge of this structure?
D 8.7% 4
E 5.5% 5
A. External oblique aponeurosis
55.3% of users answered this 6
B. Inferior epigastric artery question correctly
7
C. Rectus abdominis muscle
D. Inferior epigastric vein
E. Obturator nerve

Next question

Direct inguinal hernias pass through Hesselbachs triangle (although this is


minimal clinical significance!). Its medial boundary is the rectus muscle.

Hesselbach's triangle

Direct hernias pass through Hesselbachs triangle.

Superolaterally Epigastric vessels

Medially Lateral edge of rectus muscle

Inferiorly Inguinal ligament

The boundaries of Hesselbachs triangle are commonly tested and illustrated


below

Image sourced from Wikipedia

Rate question: Next question


yomna92009@yahoo.com - MRCS Part A - My account

Reference ranges End and review

Question 8 of 50 Question stats Score: 0%

1
A 10.5%
A 58 year old lady presents with a mass in the upper outer quadrant of the right 2
B 45.4%
breast. Which of the following statements relating to the breast is untrue? 3
C 15.3%
D 15.2% 4
A. The internal mammary artery provides the majority of its arterial E 13.6% 5
supply
45.4% of users answered this 6
B. Nipple retraction may occur as a result of tumour infiltration of the question correctly
clavipectoral fascia 7

C. The internal mammary artery is a branch of the subclavian artery 8

D. Up to 70% of lymphatic drainage is to the ipsilateral axillary nodes


E. None of the above

Next question

Nipple retraction is a feature of breast malignancy. However, it is typically caused


by tumour infiltration of Coopers Ligaments that run through the breast and
surround the lobules. The clavipectoral fascia encases the axillary contents. The
lymphatic drainage of the breast is to the axilla and also to the internal mammary
chain. The breast is well vascularised and the internal mammary artery is a
branch of the subclavian artery.

Breast

The breast itself lies on a layer of pectoral fascia and the following muscles:
1. Pectoralis major
2. Serratus anterior
3. External oblique

Image showing the topography of the female breast

Image sourced from Wikipedia

Breast anatomy
Nerve supply Branches of intercostal nerves from T4-T6.

Arterial supply Internal mammary (thoracic) artery


External mammary artery (laterally)
yomna92009@yahoo.com - MRCS Part A - My account

Reference ranges End and review

Question 9 of 50 Question stats Score: 11.1%

1
A 32.3%
Where are accessory spleens not found? 2
B 7.1%
C 9.2% 3

A. Gonads D 7% 4

B. Tail of pancreas E 44.4% 5

C. Greater omentum 44.4% of users answered this 6


question correctly
D. Splenorenal ligament 7

E. Ureter 8

9
Next question

Accessory spleens

- 10% population
- 1 cm size
- locations: hilum of the spleen, tail of the pancreas, along the splenic vessels, in
the gastrosplenic ligament, the splenorenal ligament, the walls of the stomach or
intestines, the greater omentum, the mesentery, the gonads

Spleen

Embryology: derived from mesenchymal tissue


Shape: orange segment
Position: below 9th-12th ribs
Weight: 75-150g

Relations

Superiorly- diaphragm
Anteriorly- gastric impression
Posteriorly- kidney
Inferiorly- colon
Hilum: tail of pancreas and splenic vessels (splenic artery divides here,
branches pass to the white pulp transporting plasma)
Forms apex of lesser sac (containing short gastric vessels)

Contents
- White pulp: immune function. Contains central trabecular artery. The germinal
centres are supplied by arterioles called penicilliary radicles.
- Red pulp: filters abnormal red blood cells

Function

Filtration of abnormal blood cells and foreign bodies such as bacteria.


Immunity: IgM. Production of properdin, and tuftsin which help target fungi
and bacteria for phagocytosis.
Haematopoiesis: up to 5th month gestation or in haematological disorders.
Pooling: storage of 40% platelets.
Iron reutilisation
Storage red blood cells-animals, not humans.
Storage monocytes

Disorders of the spleen


Massive splenomegaly

Myelofibrosis
Chronic myeloid leukaemia
yomna92009@yahoo.com - MRCS Part A - My account

Reference ranges End and review

0/3 Question 10-12 of 50 Question stats Score: 8.3%

Average score for registered users: 1

Theme: Nerve injury 2


10 60.1%
3
11 39.4%
A. Median nerve
12 76.1% 4
B. Ulnar nerve
5
C. Radial nerve
6
D. Anterior interosseous nerve
7
E. Posterior interosseous nerve
8
F. Axillary nerve
9
G. Musculocutaneous nerve
10-12 0 / 3

Please select the nerve at risk of injury in each scenario. Each option may be
used once, more than once or not at all.

10. A 43 year old typist presents with pain at the dorsal aspect of the
upper part of her forearm. She also complains of weakness when
extending her fingers. On examination triceps and supinator are
both functioning normally. There is weakness of most of the
extensor muscles. However, there is no sensory deficit.

You answered Musculocutaneous nerve

The correct answer is Posterior interosseous nerve

The radial nerve may become entrapped in the "arcade of Frohse"


which is a superficial part of the supinator muscle which overlies the
posterior interosseous nerve. This nerve is entirely muscular and
articular in its distribution. It passes postero-inferiorly and gives
branches to extensor carpi radialis brevis and supinator. It enters
supinator and curves around the lateral and posterior surfaces of
the radius. On emerging from the supinator the posterior
interosseous nerve lies between the superficial extensor muscles
and the lowermost fibres of supinator. It then gives branches to the
extensors.

11. A 28 year teacher reports difficulty with writing. There is no sensory


loss. She is known to have an aberrant Gantzer muscle.

You answered Musculocutaneous nerve

The correct answer is Anterior interosseous nerve

Anterior interosseous lesions occur due to fracture, or rarely due to


compression. The Gantzer muscle is an aberrant accessory of the
flexor pollicis longus and is a risk factor for anterior interosseous
nerve compression. Remember loss of pincer grip and normal
sensation indicates an interosseous nerve lesion.

12. A 35 year tennis player attends reporting tingling down his arm. He
says that his 'funny bone' was hit very hard by a tennis ball. There
is weakness of abduction and adduction of his extended fingers.

You answered Musculocutaneous nerve

The correct answer is Ulnar nerve

The ulnar nerve arises from the medial cord of the brachial plexus
(C8, T1 and contribution from C7). The nerve descends between
the axillary artery and vein, posterior to the cutaneous nerve of the
forearm and then lies anterior to triceps on the medial side of the
brachial artery. In the distal half of the arm it passes through the
medial intermuscular septum, and continues between this structure
and the medial head of triceps to enter the forearm between the
medial epicondyle of the humerus and the olecranon. It may be
injured at this site in this scenario.

Next question

Brachial plexus

Origin Anterior rami of C5 to T1

Sections of the Roots, trunks, divisions, cords, branches


plexus Mnemonic:Real Teenagers Drink Cold Beer

Roots Located in the posterior triangle


Pass between scalenus anterior and medius

Trunks Located posterior to middle third of clavicle


Upper and middle trunks related superiorly to the subclavian
artery
Lower trunk passes over 1st rib posterior to the subclavian
artery

Divisions Apex of axilla

Cords Related to axillary artery

Diagram illustrating the branches of the brachial plexus

Image sourced from Wikipedia

Cutaneous sensation of the upper limb


yomna92009@yahoo.com - MRCS Part A - My account

Reference ranges End and review

Question 13 of 50 Question stats Score: 7.7%

1
A 11.8%
A 72 year old man is undergoing a left pneumonectomy for carcinoma of the 2
B 59%
bronchus. As the surgeons approach the root of the lung, which structure will lie 3
C 6.9%
most anteriorly (in the anatomical plane)?
D 11.4% 4
E 10.9% 5
A. Vagus nerve
59% of users answered this 6
B. Phrenic nerve question correctly
7
C. Bronchus
8
D. Pulmonary vein
9
E. Pulmonary artery
10-12 0 / 3

13
Next question

The phrenic nerve is the most anteriorly located structure in the lung root. The
vagus nerve lies most posteriorly.

Lung anatomy

The right lung is composed of 3 lobes divided by the oblique and transverse
fissures. The left lung has two lobes divided by the lingula.The apex of both lungs
is approximately 4cm superior to the sterno-costal joint of the first rib. Immediately
below this is a sulcus created by the subclavian artery.

Peripheral contact points of the lung

Base: diaphragm
Costal surface: corresponds to the cavity of the chest
Mediastinal surface: Contacts the mediastinal pleura. Has the cardiac
impression. Above and behind this concavity is a triangular depression
named the hilum, where the structures which form the root of the lung enter
and leave the viscus. These structures are invested by pleura, which,
below the hilus and behind the pericardial impression, forms the pulmonary
ligament

Right lung
Above the hilum is the azygos vein; Superior to this is the groove for the superior
vena cava and right innominate vein; behind this, and nearer the apex, is a furrow
for the innominate artery. Behind the hilus and the attachment of the pulmonary
ligament is a vertical groove for the oesophagus; In front and to the right of the
lower part of the oesophageal groove is a deep concavity for the extrapericardiac
portion of the inferior vena cava.

The root of the right lung lies behind the superior vena cava and the right atrium,
and below the azygos vein.

The right main bronchus is shorter, wider and more vertical than the left main
bronchus and therefore the route taken by most foreign bodies.

Left lung
Above the hilum is the furrow produced by the aortic arch, and then superiorly the
groove accommodating the left subclavian artery; Behind the hilum and
pulmonary ligament is a vertical groove produced by the descending aorta, and in
front of this, near the base of the lung, is the lower part of the oesophagus.

The root of the left lung passes under the aortic arch and in front of the
descending aorta.

Inferior borders of both lungs


yomna92009@yahoo.com - MRCS Part A - My account

Reference ranges End and review

Question 14 of 50 Question stats Score: 7.1%

1
A 17.4%
Which of the following structures does not lie anterior to the rectum? 2
B 56.1%
C 12.4% 3

A. Denonvilliers' fascia D 5.3% 4

B. Middle sacral artery E 8.8% 5

C. Pouch of Douglas 56.1% of users answered this 6


question correctly
D. Rectovesical pouch 7

E. Seminal vesicles 8

9
Next question
10-12 0 / 3

13
With the exception of the middloe sacral artery all of the other structures lie
14
anterior to the rectum. They may all be palpated during digital rectal examination.

Rectum

The rectum is approximately 12 cm long. It is a capacitance organ. It has both


intra and extraperitoneal components. The transition between the sigmoid colon is
marked by the disappearance of the tenia coli.The extra peritoneal rectum is
surrounded by mesorectal fat that also contains lymph nodes. This mesorectal
fatty layer is removed surgically during rectal cancer surgery (Total Mesorectal
Excision). The fascial layers that surround the rectum are important clinical
landmarks, anteriorly lies the fascia of Denonvilliers. Posteriorly lies Waldeyers
fascia.

Extra peritoneal rectum

Posterior upper third


Posterior and lateral middle third
Whole lower third

Relations
Anteriorly (Males) Rectovesical pouch
Bladder
Prostate
Seminal vesicles

Anteriorly (Females) Recto-uterine pouch (Douglas)


Cervix
Vaginal wall

Posteriorly Sacrum
Coccyx
Middle sacral artery

Laterally Levator ani


Coccygeus

Arterial supply
Superior rectal artery

Venous drainage
Superior rectal vein

Lymphatic drainage

Mesorectal lymph nodes (superior to dentate line)


Internal iliac and then para-aortic nodes
Inguinal nodes (inferior to dentate line)

Rate question: Next question


yomna92009@yahoo.com - MRCS Part A - My account

Reference ranges End and review

Question 15 of 50 Question stats Score: 6.7%

1
A 11.7%
A 25 year old man has an inguinal hernia, which of the following structures must 2
B 67.9%
be divided (at open surgery) to gain access to the inguinal canal? 3
C 7.5%
D 6.3% 4
A. Transversalis fascia E 6.7% 5
B. External oblique aponeurosis 6
67.9% of users answered this
C. Conjoint tendon question correctly
7
D. Rectus abdominis
8
E. Inferior epigastric artery
9

10-12 0 / 3
Next question
13

14
This question is asking what structure forms the anterior wall of the inguinal canal.
The anterior wall is formed by the external oblique aponeurosis. Once this is 15
divided the canal is entered, the cord can be mobilised and a hernia repair
performed. The transversalis fascia and conjoint tendons form the posterior wall
and would not routinely be divided to gain access to the inguinal canal itself.

External oblique muscle

External oblique forms the outermost muscle of the three muscles comprising the
anterolateral aspect of the abdominal wall. Its aponeurosis comprises the anterior
wall of the inguinal canal.

Origin Outer surfaces of the lowest eight ribs

Insertion Anterior two thirds of the outer lip of the iliac crest.
The remainder becomes the aponeurosis that fuses with the linea alba
in the midline.

Nerve Ventral rami of the lower six thoracic nerves


supply

Actions Contains the abdominal viscera, may contract to raise intra abdominal pressure.
Moves trunk to one side.

Rate question: Next question

Com m ent on this question

All contents of this site are © 2012 E-Medical Revision Ltd Terms and Conditions Privacy policy
yomna92009@yahoo.com - MRCS Part A - My account

Reference ranges End and review

Question 16 of 50 Question stats Score: 6.3%

1
A 7.7%
In relation to the middle cranial fossa, which of the following statements relating to 2
B 10%
the foramina are incorrect? 3
C 13.4%
D 57.1% 4
A. The foramen rotundum transmits the maxillary nerve E 11.8% 5
B. The foramen lacerum transmits the internal carotid artery 6
57.1% of users answered this
C. The foramen spinosum lies posterolateral to the foramen ovale question correctly
7
D. The foramen ovale transmits the middle meningeal artery
8
E. The foramen rotundum lies anteromedial to the foramen ovale
9

10-12 0 / 3
Next question
13

14
Theme addressed in 2010 and 2011 exam
The foramen spinosum transmits the middle meningeal artery. The foramen ovale 15
transmits the mandibular nerve. As the foramina weaken the bone a fracture at 16
this site is not uncommon.

Foramina of the base of the skull

Foramen Location Contents

Foramen Sphenoid Otic ganglion


ovale bone V3 (Mandibular nerve:3rd branch of
trigeminal)
Accessory meningeal artery
Lesser petrosal nerve
Emissary veins

Foramen Sphenoid Middle meningeal artery


spinosum bone Meningeal branch of the Mandibular nerve

Foramen Sphenoid Maxillary nerve (V2)


rotundum bone

Foramen Sphenoid Base of the medial pterygoid plate.


lacerum bone Internal carotid artery
Nerve and artery of the pterygoid canal

Jugular Temporal Anterior: inferior petrosal sinus


foramen bone Intermediate: glossopharyngeal, vagus, and accessory nerves.
Posterior: sigmoid sinus (becoming the internal jugular vein) and
some meningeal branches from the occipital and ascending
pharyngeal arteries.

Foramen Occipital Anterior and posterior spinal arteries


magnum bone Vertebral arteries
Medulla oblongata

Stylomastoid Temporal Stylomastoid artery


foramen bone Facial nerve

Superior Sphenoid Oculomotor nerve (III)


orbital bone trochlear nerve (IV)
fissure lacrimal, frontal and nasociliary branches of ophthalmic nerve
(V1)
abducent nerve (VI)
Superior and inferior ophthalmic vein

Base of skull anatomical overview


yomna92009@yahoo.com - MRCS Part A - My account

Reference ranges End and review

Question 17 of 50 Question stats Score: 11.8%

1
A 17.7%
During an operation for varicose veins the surgeons are mobilising the long 2
B 18.2%
saphenous vein. Near its point of entry to the femoral vein an artery is injured and 3
C 21.6%
bleeding is encountered. From where is the bleeding most likely to originate?
D 11% 4
E 31.5% 5
A. Femoral artery
31.5% of users answered this 6
B. Profunda femoris artery question correctly
7
C. Superficial circumflex iliac artery
8
D. Superficial epigastric artery
9
E. Deep external pudendal artery
10-12 0 / 3

13
Next question
14

Theme from 2011 Exam 15


The deep external pudendal artery is a branch of the SFA and it runs medially 16
under the long saphenous vein near its point of union with the femoral vein. The
17
superficial external pudendal artery lies superior to the SFJ. Neither vessel is
functionally important and if injured they are best ligated.

Femoral triangle anatomy

Boundaries
Superiorly Inguinal ligament

Laterally Sartorius

Medially Adductor longus

Floor Iliopsoas, adductor longus and pectineus

Roof Fascia lata and Superficial fascia


Superficial inguinal lymph nodes (palpable below the inguinal ligament)
Great saphenous vein

Image sourced from Wikipedia


yomna92009@yahoo.com - MRCS Part A - My account

Reference ranges End and review

Question 18 of 50 Question stats Score: 16.7%

1
A 10.3%
What is embryological origin of the pulmonary artery? 2
B 17.7%
C 23.4% 3

A. First pharyngeal arch D 14.1% 4

B. Second pharyngeal arch E 34.5% 5

C. Fourth pharyngeal arch 34.5% of users answered this 6


question correctly
D. Fifth pharyngeal arch 7

E. Sixth pharyngeal arch 8

9
Next question
10-12 0 / 3

13
Theme from September 2011 Exam
14
Theme from September 2012 Exam
The proximal part of the sixth right pharyngeal arch gives origin to the right 15
pulmonary artery. The distal part gives origin to the left pulmonary artery and the 16
ductus arteriosus.
17

Pharyngeal arches 18

These develop during the fourth week of embryonic growth from a series of
mesodermal outpouchings of the developing pharynx.
They develop and fuse in the ventral midline. Pharyngeal pouches form on the
endodermal side between the arches.
There are 6 pharyngeal arches, the fifth does not contribute any useful structures
and often fuses with the sixth arch.

Pharyngeal arches
Pharyngeal Muscular Skeletal Endocrine Artery Nerve
arch contributions contributions

First Muscles of Maxilla n/a Maxillary Mandibular


mastication Meckels External
Anterior belly of cartilage carotid
digastric Incus
Mylohyoid Malleus
Tensor tympanic
Tensor veli
palatini

Second Buccinator Stapes n/a Inferior Facial


Platysma Styloid branch of
Muscles of facial process superior
expression Lesser corn thyroid
Stylohyoid and upper artery
Posterior belly of body of hyoid Stapedial
digastric artery
Stapedius

Third Stylopharyngeus Greater horn Thymus Common Glossopharyngeal


and lower Inferior and
part of hyoid parathyroids internal
carotid

Fourth Cricothyroid Thyroid and Superior Right- Vagus


All intrinsic epiglottic parathyroids subclavian
muscles of the cartilages artery,
soft palate Left-aortic
arch

Sixth All intrinsic Cricoid, n/a Right - Vagus and


muscles of the arytenoid and Pulmonary recurrent
larynx (except corniculate artery, laryngeal nerve
cricothyroid) cartilages Left-
Pulmonary
artery and
ductus
arteriosus
yomna92009@yahoo.com - MRCS Part A - My account

Reference ranges End and review

Question 19 of 50 Question stats Score: 15.8%

1
A 10.3%
A 53 year old lady presents with pain and discomfort in her hand. She works as a 2
B 47.9%
typist and notices that the pain is worst when she is working. She also suffers 3
C 12.8%
symptoms at night. Her little finger is less affected by the pain. Which of the
D 13.1% 4
nerves listed below is most likely to be affected?
E 15.8% 5

47.9% of users answered this 6


A. Radial
question correctly
7
B. Median
8
C. Ulnar
D. Anterior interosseous nerve 9

10-12 0 / 3
E. Posterior interosseous nerve
13

Next question 14

15

16
Motor supply: LOAF
17

L ateral 2 lumbricals 18
O pponens pollicis
19
A bductor pollicisbrevis
F lexor pollicis brevis

Theme from April 2012 Exam


The most likely diagnosis here is carpal tunnel syndrome, the median nerve is
compressed in the wrist and symptoms usually affect the fingers and wrist either
at night or when the hand is being used (e.g. as a typist).

Median nerve

The median nerve is formed by the union of a lateral and medial root respectively
from the lateral (C5,6,7) and medial (C8 and T1) cords of the brachial plexus; the
medial root passes anterior to the third part of the axillary artery. The nerve
descends lateral to the brachial artery, crosses to its medial side (usually passing
anterior to the artery). It passes deep to the bicipital aponeurosis and the median
cubital vein at the elbow.
It passes between the two heads of the pronator teres muscle, and runs on the
deep surface of flexor digitorum superficialis (within its fascial sheath).
Near the wrist it becomes superficial between the tendons of flexor digitorum
superficialis and flexor carpi radialis, deep to palmaris longus tendon. It passes
deep to the flexor retinaculum to enter the palm, but lies anterior to the long flexor
tendons within the carpal tunnel.

Branches
Region Branch

Upper No branches, although the nerve commonly communicates with the


arm musculocutaneous nerve

Forearm Pronator teres


Flexor carpi radialis
Palmaris longus
Flexor digitorum superficialis
Flexor pollicis longus
Flexor digitorum profundus (only the radial half)

Distal Palmar cutaneous branch


forearm

Hand Motor supply (LOAF)


(Motor)
Lateral 2 lumbricals
Opponens pollicis
Abductor pollicis brevis
Flexor pollicis brevis
yomna92009@yahoo.com - MRCS Part A - My account

Reference ranges End and review

Question 20 of 50 Question stats Score: 15%

1
A 51.9%
Which of the following muscles lies medial to the long thoracic nerve? 2
B 13.4%
C 15.7% 3

A. Serratus anterior D 11.6% 4

B. Latissimus dorsi E 7.4% 5

C. Pectoralis major 51.9% of users answered this 6


question correctly
D. Pectoralis minor 7

E. None of the above 8

9
Next question
10-12 0 / 3

13
Theme from 2009 Exam
14

Long thoracic nerve 15

16

17
Derived from ventral rami of C5, C6, and C7 (close to their emergence from
intervertebral foramina) 18
It runs downward and passes either anterior or posterior to the middle
19
scalene muscle
20
It reaches upper tip of serratus anterior muscle and descends on outer
surface of this muscle, giving branches into it
Winging of Scapula occurs in long thoracic nerve injury (most common) or
from spinal accessory nerve injury (which denervates the trapezius) or a
dorsal scapular nerve injury

Rate question: Next question

Com m ent on this question

All contents of this site are © 2012 E-Medical Revision Ltd Terms and Conditions Privacy policy
yomna92009@yahoo.com - MRCS Part A - My account

Reference ranges End and review

Question 21 of 50 Question stats Score: 14.3%

1
A 26.8%
The thesbian veins contribute to the venous drainage of the heart. Into which of 2
B 47.5%
the following structures do they primarily drain? 3
C 9.8%
D 7.9% 4
A. Great cardiac vein E 8% 5
B. Atrium 6
47.5% of users answered this
C. Superior vena cava question correctly
7
D. Oblique vein
8
E. Small cardiac vein
9

10-12 0 / 3
Next question
13

14
The thesbian veins are numerous small veins running over the surface of the
heart they drain into the heart itself. Usually this is to the atria. 15

16
Heart anatomy
17

18
The walls of each cardiac chamber comprise:
19
Epicardium
20
Myocardium
Endocardium 21

Cardiac muscle is attached to the cardiac fibrous skeleton.

Relations
The heart and roots of the great vessels within the pericardial sac are related
anteriorly to the sternum, medial ends of the 3rd to 5th ribs on the left and their
associated costal cartilages. The heart and pericardial sac are situated obliquely
two thirds to the left and one third to the right of the median plane.

The pulmonary valve lies at the level of the left third costal cartilage.
The mitral valve lies at the level of the fourth costal cartilage.

Coronary sinus
This lies in the posterior part of the coronary groove and receives blood from the
cardiac veins. The great cardiac vein lies at its left and the middle and small
cardiac veins lie on its right. The smallest cardiac veins drain into the atria
directly.

Aortic sinus
Right coronary artery arises from the right aortic sinus, the left is derived from the
left aortic sinus and no vessel emerges from the posterior sinus.

Right and left ventricles

Structure Left Ventricle

A-V Valve Mitral (double leaflet)

Walls Twice as thick as right

Trabeculae carnae Much thicker and more numerous

Right coronary artery


The RCA supplies:

Right atrium
Diaphragmatic part of the left ventricle
Usually the posterior third of the interventricular septum
The sino atrial node (60% cases)
yomna92009@yahoo.com - MRCS Part A - My account

Reference ranges End and review

Question 22 of 50 Question stats Score: 13.6%

1
A 7.1%
Which of the following is not contained within the deep posterior compartment of 2
B 7.3%
the lower leg? 3
C 11.8%
D 61.6% 4
A. Tibialis posterior muscle E 12% 5
B. Posterior tibial artery 6
61.6% of users answered this
C. Tibial nerve question correctly
7
D. Sural nerve
8
E. Flexor hallucis longus
9

10-12 0 / 3
Next question
13

14
The deep posterior compartment lies anterior to soleus. The sural nerve is
superficially sited and therefore not contained within it. 15

16
Lower limb- Muscular compartments
17

18
Anterior compartment
Muscle Nerve Action 19

Tibialis anterior Deep peroneal Dorsiflexes ankle joint, inverts foot 20


nerve
21
Extensor digitorum Deep peroneal Extends lateral four toes, dorsiflexes ankle
longus nerve joint 22

Peroneus tertius Deep peroneal Dorsiflexes ankle, everts foot


nerve

Extensor hallucis Deep peroneal Dorsiflexes ankle joint, extends big toe
longus nerve

Peroneal compartment
Muscle Nerve Action

Peroneus longus Superficial peroneal nerve Everts foot, assists in plantar flexion

Peroneus brevis Superficial peroneal nerve Plantar flexes the ankle joint

Superficial posterior compartment


Nerve Action

Gastrocnemius Tibial nerve Plantar flexes the foot, may also flex the knee

Soleus Tibial nerve Plantar flexor

Deep posterior compartment


Muscle Nerve Action

Flexor digitorum longus Tibial Flexes the lateral four toes

Flexor hallucis longus Tibial Flexes the great toe

Tibialis posterior Tibial Plantar flexor, inverts the foot

Rate question: Next question

Com m ent on this question

All contents of this site are © 2012 E-Medical Revision Ltd Terms and Conditions Privacy policy
yomna92009@yahoo.com - MRCS Part A - My account

Reference ranges End and review

Question 23 of 50 Question stats Score: 17.4%

1
A 17.7%
When performing minor surgery in the scalp, which of the following regions is 2
B 7.5%
considered a danger area as regards spread of infection into the CNS? 3
C 19.2%
D 13.9% 4
A. Aponeurosis epicranialis E 41.8% 5
B. Skin 6
41.8% of users answered this
C. Pericranium question correctly
7
D. Connective tissue
8
E. Loose areolar tissue
9

10-12 0 / 3
Next question
13

14
This area is most dangerous as infections can spread easily. The emissary veins
that drain this area may allow sepsis to spread to the cranial cavity. 15

16
Head injury
17

18
Patients who suffer head injuries should be managed according to ATLS
19
principles and extra cranial injuries should be managed alongside cranial
20
trauma. Inadequate cardiac output will compromise CNS perfusion
irrespective of the nature of the cranial injury. 21

22

Types of traumatic brain injury 23

Extradural Bleeding into the space between the dura mater and the skull. Often results
haematoma from acceleration-deceleration trauma or a blow to the side of the head. The
majority of extradural haematomas occur in the temporal region where skull
fractures cause a rupture of the middle meningeal artery.

Features

Raised intracranial pressure


Some patients may exhibit a lucid interval

Subdural Bleeding into the outermost meningeal layer. Most commonly occur around
haematoma the frontal and parietal lobes. May be either acute or chronic.

Risk factors include old age and alcoholism.

Slower onset of symptoms than a extradural haematoma.

Subarachnoid Usually occurs spontaneously in the context of a ruptured cerebral


haemorrhage aneurysm but may be seen in association with other injuries when a patient
has sustained a traumatic brain injury

Pathophysiology

Primary brain injury may be focal (contusion/ haematoma) or diffuse


(diffuse axonal injury)
Diffuse axonal injury occurs as a result of mechanical shearing following
deceleration, causing disruption and tearing of axons
Intra-cranial haematomas can be extradural, subdural or intracerebral,
while contusions may occur adjacent to (coup) or contralateral (contre-
coup) to the side of impact
Secondary brain injury occurs when cerebral oedema, ischaemia, infection,
tonsillar or tentorial herniation exacerbates the original injury. The normal
cerebral auto regulatory processes are disrupted following trauma
rendering the brain more susceptible to blood flow changes and hypoxia
The Cushings reflex (hypertension and bradycardia) often occurs late and
is usually a pre terminal event

Management
yomna92009@yahoo.com - MRCS Part A - My account

Reference ranges End and review

Question 24 of 50 Question stats Score: 16.7%

1
A 55.1%
Which of the following structures are at risk of direct injury following a fracture 2
B 12.4%
dislocation of the femoral condyles? 3
C 8.9%
D 7.7% 4
A. Popliteal artery E 15.8% 5
B. Sciatic nerve 6
55.1% of users answered this
C. Plantaris muscle question correctly
7
D. Tibial artery
8
E. Tibial nerve
9

10-12 0 / 3
Next question
13

14
The heads of gastrocnemius will contract to pull the fracture segment posteriorly.
The popliteal artery lies against the bone and may be damaged or compressed. 15

16
Popliteal fossa
17

18
Boundaries of the popliteal fossa
Laterally Biceps femoris above, lateral head of gastrocnemius and plantaris below 19

Medially Semimembranosus and semitendinosus above, medial head of gastrocnemius 20


below
21
Floor Popliteal surface of the femur, posterior ligament of knee joint and popliteus
muscle 22

Roof Superficial and deep fascia 23

24
Image showing the popliteal fossa

Image sourced from Wikipedia

Contents

Popliteal artery and vein


Small saphenous vein
Common peroneal nerve
Tibial nerve
Posterior cutaneous nerve of the thigh
Genicular branch of the obturator nerve
Lymph nodes
yomna92009@yahoo.com - MRCS Part A - My account

Reference ranges End and review

Question 25 of 50 Question stats Score: 16%

1
A 19.7%
A 25 year old man is being catheterised, prior to a surgical procedure. As the 2
B 28.2%
catheter enters the prostatic urethra which of the following changes will occur? 3
C 9.8%
D 34.3% 4
A. Resistance will increase significantly E 8% 5
B. Resistance will increase slightly 6
34.3% of users answered this
C. It will lie horizontally question correctly
7
D. Resistance will decrease
8
E. It will deviate laterally
9

10-12 0 / 3
Next question
13

14
Theme from September 2011 Exam
15
The prostatic urethra is much wider than the membranous urethra and therefore 16
resistance will decrease. The prostatic urethra is inclined vertically.
17

Prostate gland 18

19
The prostate gland is approximately the shape and size of a walnut and is located
20
inferior to the bladder. It is separated from the rectum by Denonvilliers fascia and
21
its blood supply is derived from the internal iliac vessels. The internal sphincter
lies at the apex of the gland and may be damaged during prostatic surgery, 22
affected individuals may complain of retrograde ejaculation. 23

24
Summary of prostate gland
Arterial supply Inferior vesical artery (from internal iliac) 25

Venous Prostatic venous plexus (to paravertebral veins)


drainage

Lymphatic Internal iliac nodes


drainage

Innervation Inferior hypogastric plexus

Dimensions Transverse diameter (4cm)


AP diameter (2cm)
Height (3cm)

Lobes Posterior lobe: posterior to urethra


Median lobe: posterior to urethra, in between ejaculatory ducts
Lateral lobes x 2
Isthmus

Zones Peripheral zone: subcapsular portion of posterior prostate. Most


prostate cancers are here
Central zone
Transition zone
Stroma

Relations
Anterior Pubic symphysis
Prostatic venous plexus

Posterior Denonvilliers fascia


Rectum
Ejaculatory ducts

Lateral Venous plexus (lies on prostate)


Levator ani (immediately below the puboprostatic ligaments)
yomna92009@yahoo.com - MRCS Part A - My account

Reference ranges End and review

Question 1 of 25 Question stats Score: 0%

1
A 72.7%
A 24 year female is admitted to A&E with tingling of her hand after a fall. She is B 6.6%
found to have a fracture of the medial epicondyle. What is the most likely nerve
C 11.2%
lesion?
D 4.4%

E 5.2%
A. Ulnar nerve
72.7% of users answered this
B. Radial nerve question correctly
C. Median nerve
D. Axillary nerve
E. Cutaneous nerve

Next question

The radial nerve is located near the lateral epicondyle.

Ulnar nerve

Origin

C8, T1

Supplies (no muscles in the upper arm)

Flexor carpi ulnaris


Flexor digitorum profundus
Flexor digiti minimi
Abductor digiti minimi
Opponens digiti minimi
Adductor pollicis
Interossei muscle
Third and fourth lumbricals
Palmaris brevis

Path

Posteromedial aspect of ulna to flexor compartment of forearm, then along


the ulnar. Passes beneath the flexor carpi ulnaris muscle, then superficially
through the flexor retinaculum into the palm of the hand.
yomna92009@yahoo.com - MRCS Part A - My account

Reference ranges End and review

Question 2 of 25 Question stats Score: 50%

1
A 15.9%
During a gangland gunfight a man is shot in the chest. The bullet passes through 2
B 9.5%
the posterior mediastinum (from left to right). Which of the following structures is
C 16.1%
least likely to be injured
D 12.8%

E 45.7%
A. Thoracic duct
45.7% of users answered this
B. Oesophagus question correctly
C. Vagus nerve
D. Descending thoracic aorta
E. Arch of the azygos vein

Next question

The arch of the azygos vein lies in the middle mediastinum.

Mediastinum

Region between the pulmonary cavities.


It is covered by the mediastinal pleura. It does not contain the lungs.
It extends from the thoracic inlet superiorly to the diaphragm inferiorly.

Mediastinal regions

Superior mediastinum
Inferior mediastinum
Posterior mediastinum
Anterior mediastinum

Region Contents

Superior mediastinum Superior vena cava


Brachiocephalic veins
Arch of aorta
Thoracic duct
Trachea
Oesophagus
Thymus

Vagus nerve
Left recurrent laryngeal nerve
Phrenic nerve

Anterior mediastinum Thymic remnants


Lymph nodes
Fat

Middle mediastinum Pericardium


Heart
Aortic root
Arch of azygos vein
Main bronchi

Posterior mediastinum Oesophagus


Thoracic aorta
Azygos vein
Thoracic duct
Vagus nerve
Sympathetic nerve trunks
Splanchnic nerves

Rate question: Next question


yomna92009@yahoo.com - MRCS Part A - My account

Reference ranges End and review

Question 3 of 25 Question stats Score: 66.7%

1
A 17.2%
The space between the vocal cords is referred to as which of the following? 2
B 13.6%
C 17.5% 3

A. Piriform recess D 18.7%

B. Rima vestibuli E 32.8%

C. Vestibule 32.8% of users answered this


question correctly
D. Glottis
E. Rima glottidis

Next question

The rima glottidis is the narrowest part of the laryngeal cavity.

Larynx

The larynx lies in the anterior part of the neck at the levels of C3 to C6 vertebral
bodies. The laryngeal skeleton consists of a number of cartilagenous segments.
Three of these are paired; arytenoid, corniculate and cuneiform. Three are
single; thyroid, cricoid and epiglottic. The cricoid cartilage forms a complete ring
(the only one to do so).
The laryngeal cavity extends from the laryngeal inlet to the level of the inferior
border of the cricoid cartilage.

Divisions of the laryngeal cavity


Laryngeal vestibule Superior to the vestibular folds

Laryngeal ventricle Lies between vestibular folds and superior to the vocal cords

Infraglottic cavity Extends from vocal cords to inferior border of the cricoid cartilage

The vocal folds (true vocal cords) control sound production. The apex of each
fold projects medially into the laryngeal cavity. Each vocal fold includes:

Vocal ligament
Vocalis muscle (most medial part of thyroarytenoid muscle)

The glottis is composed of the vocal folds, processes and rima glottidis. The rima
glottidis is the narrowest potential site within the larynx, as the vocal cords may be
completely opposed, forming a complete barrier.

Muscles of the larynx


Muscle Origin Insertion Innervation Action

Posterior Posterior aspect Muscular process of Recurrent Abducts vocal fold


cricoarytenoid of lamina of arytenoid Laryngeal
cricoid

Lateral Arch of cricoid Muscular process of Recurrent Adducts vocal fold


cricoarytenoid arytenoid laryngeal

Thyroarytenoid Posterior aspect Muscular process of Recurrent Relaxes vocal fold


of thyroid cartilage arytenoid laryngeal

Transverse Arytenoid cartilage Contralateral Recurrent Closure of


and oblique arytenoid laryngeal intercartilagenous
arytenoids part of the rima
glottidis

Vocalis Depression Vocal ligament and Recurrent Relaxes posterior


between lamina vocal process of laryngeal vocal ligament,
of thyroid cartilage arytenoid cartilage tenses anterior
part

Cricothyroid Anterolateral part Inferior margin and External Tenses vocal fold
of cricoid horn of thyroid laryngeal
cartilage

Blood supply
yomna92009@yahoo.com - MRCS Part A - My account

Reference ranges End and review

Question 4 of 25 Question stats Score: 50%

1
A 18.2%
A 78 year old man develops a carcinoma of the scrotum. To which of the following 2
B 4.7%
lymph node groups may the tumour initially metastasise? 3
C 65.8%
D 5.9% 4
A. Para aortic E 5.4%

B. Obturator
65.8% of users answered this
C. Inguinal question correctly

D. Meso rectal
E. None of the above

Next question

The scrotum is drained by the inguinal nodes.

Scrotal and testicular anatomy

Spermatic cord
Formed by the vas deferens and is covered by the following structures:
Layer Origin

Internal spermatic fascia Transversalis fascia

Cremasteric fascia From the fascial coverings of internal oblique

External spermatic fascia External oblique aponeurosis

Contents of the cord


Vas deferens Transmits sperm and accessory gland secretions

Testicular artery Branch of abdominal aorta supplies testis and


epididymis

Artery of vas deferens Arises from inferior vesical artery

Cremasteric artery Arises from inferior epigastic artery

Pampiniform plexus Venous plexus, drains into right or left testicular vein

Sympathetic nerve fibres Lie on arteries, the parasympathetic fibres lie on the
vas

Genital branch of the genitofemoral Supplies cremaster


nerve

Lymphatic vessels Drain to lumbar and para-aortic nodes

Scrotum

Composed of skin and closely attached dartos fascia.


Arterial supply from the anterior and posterior scrotal arteries
Lymphatic drainage to the inguinal lymph nodes
Parietal layer of the tunica vaginalis is the innermost layer

Testes

The testes are surrounded by the tunica vaginalis (closed peritoneal sac).
The parietal layer of the tunica vaginalis adjacent to the internal spermatic
fascia.
The testicular arteries arise from the aorta immediately inferiorly to the
renal arteries.
The pampiniform plexus drains into the testicular veins, the left drains into
the left renal vein and the right into the inferior vena cava.
Lymphatic drainage is to the para-aortic nodes.

Rate question: Next question


yomna92009@yahoo.com - MRCS Part A - My account

Reference ranges End and review

Question 5 of 25 Question stats Score: 40%

1
A 14.3%
A 63 year old man is undergoing an upper GI endoscopy for dysphagia. At 23 cm 2
B 58.2%
a malignant looking stricture is encountered. The endoscopist attempts a balloon 3
C 12.6%
dilatation.Unfortunately the tumour splits through the oesophageal wall. Into which
D 6.8% 4
region will the oesophageal contents now drain?
E 8% 5

A. Superior mediastinum 58.2% of users answered this


question correctly
B. Posterior mediastinum
C. Middle mediastinum
D. Anterior mediastinum
E. Peritoneal cavity

Next question

At this position the oesophagus is still likely to be intrathoracic and located in the
posterior mediastinum.

Mediastinum

Region between the pulmonary cavities.


It is covered by the mediastinal pleura. It does not contain the lungs.
It extends from the thoracic inlet superiorly to the diaphragm inferiorly.

Mediastinal regions

Superior mediastinum
Inferior mediastinum
Posterior mediastinum
Anterior mediastinum

Region Contents

Superior mediastinum Superior vena cava


Brachiocephalic veins
Arch of aorta
Thoracic duct
Trachea
Oesophagus
Thymus
Vagus nerve
Left recurrent laryngeal nerve
Phrenic nerve

Anterior mediastinum Thymic remnants


Lymph nodes
Fat

Middle mediastinum Pericardium


Heart
Aortic root
Arch of azygos vein
Main bronchi

Posterior mediastinum Oesophagus


Thoracic aorta
Azygos vein
Thoracic duct
Vagus nerve
Sympathetic nerve trunks
Splanchnic nerves
yomna92009@yahoo.com - MRCS Part A - My account

Reference ranges End and review

Question 6 of 25 Question stats Score: 33.3%

1
A 14.8%
During a tricuspid valve repair the right atrium is opened following establishment 2
B 9.9%
of cardiopulmonary bypass. Which of the following structures do not lie within the 3
C 8.5%
right atrium?
D 37.9% 4
E 28.8% 5
A. Crista terminalis
37.9% of users answered this 6
B. Tricuspid valve question correctly
C. Fossa ovalis
D. Trabeculae carnae
E. Musculi pectinati

Next question

Structures within the right atrium:

Musculi pectinati
Crista terminalis
Opening of the coronary sinus
Fossa ovalis

The trabeculae carnae are located in the right ventricle.

Heart anatomy

The walls of each cardiac chamber comprise:

Epicardium
Myocardium
Endocardium

Cardiac muscle is attached to the cardiac fibrous skeleton.

Relations
The heart and roots of the great vessels within the pericardial sac are related
anteriorly to the sternum, medial ends of the 3rd to 5th ribs on the left and their
associated costal cartilages. The heart and pericardial sac are situated obliquely
two thirds to the left and one third to the right of the median plane.

The pulmonary valve lies at the level of the left third costal cartilage.
The mitral valve lies at the level of the fourth costal cartilage.

Coronary sinus
This lies in the posterior part of the coronary groove and receives blood from the
cardiac veins. The great cardiac vein lies at its left and the middle and small
cardiac veins lie on its right. The smallest cardiac veins drain into the atria
directly.

Aortic sinus
Right coronary artery arises from the right aortic sinus, the left is derived from the
left aortic sinus and no vessel emerges from the posterior sinus.

Right and left ventricles

Structure Left Ventricle

A-V Valve Mitral (double leaflet)

Walls Twice as thick as right


yomna92009@yahoo.com - MRCS Part A - My account

Reference ranges End and review

Question 7 of 25 Question stats Score: 28.6%

1
A 8.7%
Which of the following is a recognised tributary of the retromandibular vein? 2
B 14.5%
C 15.5% 3

A. Internal jugular vein D 48.1% 4

B. External jugular vein E 13.3% 5

C. Anterior temporal diploic vein 48.1% of users answered this 6


question correctly
D. Maxillary vein 7

E. Inferior opthalmic vein

Next question

The retromandibular vein is formed from the union of the maxillary and superficial
temporal veins.

Retromandibular vein

Formed by a union of the maxillary vein and superficial temporal vein


It descends through the parotid gland and bifurcates within it
The anterior division passes forwards to join the facial vein, the posterior
division is one of the tributaries of the external jugular vein

Rate question: Next question

Com m ent on this question

All contents of this site are © 2012 E-Medical Revision Ltd Terms and Conditions Privacy policy
yomna92009@yahoo.com - MRCS Part A - My account

Reference ranges End and review

Question 8 of 25 Question stats Score: 25%

1
A 59.6%
An 22 year old soldier is shot in the abdomen and amongst his various injuries is 2
B 9.3%
a major disruption to the abdominal aorta. There is torrential haemorrhage and 3
C 12.2%
the surgeons decide to control the aorta by placement of a vascular clamp
D 7.2% 4
immediately inferior to the diaphragm. Which of the following vessels may be
E 11.7% 5
injured in this maneouvre?

59.6% of users answered this 6


question correctly
A. Inferior phrenic arteries 7

B. Superior phrenic arteries 8

C. Splenic artery
D. Renal arteries
E. Superior mesenteric artery

Next question

As the first branches of the abdominal aorta the inferior phrenic arteries are at
greatest risk. The superior phrenic arteries lie in the thorax. The potential space
at the level of the diaphragmatic hiatus is a potentially useful site for aortic
occlusion. However, leaving the clamp applied for more than about 10 -15
minutes usually leads to poor outcomes.

Abdominal aorta

Abdominal aortic topography


Origin T12

Termination L4

Posterior relations L1-L4 Vertebral bodies

Anterior relations Lesser omentum


Liver
Left renal vein
Inferior mesenteric vein
Third part of duodenum
Pancreas
Parietal peritoneum
Peritoneal cavity

Right lateral relations Right crus of the diaphragm


Cisterna chyli
Azygos vein
IVC (becomes posterior distally)

Left lateral relations 4th part of duodenum


Duodenal-jejunal flexure
Left sympathetic trunk

The abdominal aorta


yomna92009@yahoo.com - MRCS Part A - My account

Reference ranges End and review

Question 9 of 25 Question stats Score: 33.3%

1
A 9.2%
Which of the following statements relating to the gallbladder is untrue? 2
B 4.9%
C 6.6% 3

A. The fundus is usually intra peritoneal D 7.7% 4

B. Arterial supply is from the cystic artery E 71.6% 5

C. The cystic artery is usually located in Calots triangle 71.6% of users answered this 6
question correctly
D. Calots triangle may rarely contain an aberrant hepatic artery 7

E. Cholecystokinin causes relaxation of the gallbladder 8

9
Next question

CCK causes gallbladder contraction.

Gallbladder

Fibromuscular sac with capacity of 50ml


Columnar epithelium

Relations of the gallbladder


Anterior Liver

Posterior Covered by peritoneum


Transverse colon
1st part of the duodenum

Laterally Right lobe of liver

Medially Quadrate lobe of liver

Arterial supply
Cystic artery (branch of Right hepatic artery)

Venous drainage
Cystic vein

Nerve supply
Sympathetic- mid thoracic spinal cord, Parasympathetic- anterior vagal trunk

Common bile duct

Origin Confluence of cystic and common hepatic ducts

Relations at Medially - Hepatic artery


origin Posteriorly- Portal vein

Relations Duodenum - anteriorly


distally Pancreas - medially and laterally
Right renal vein - posteriorly

Arterial supply Branches of hepatic artery and retroduodenal branches of


gastroduodenal artery

Calot's triangle

Medially Common hepatic duct

Inferiorly Cystic duct

Superiorly Inferior edge of liver

Contents Cystic artery


yomna92009@yahoo.com - MRCS Part A - My account

Reference ranges End and review

Question 10 of 25 Question stats Score: 30%

1
A 8.8%
A 22 year old fit and well male undergoes an emergency appendicectomy. He is 2
B 72.7%
given suxamethonium. An inflamed appendix is removed and the patient is 3
C 6.5%
returned to recovery. One hour post operatively the patient develops a
D 4.7% 4
tachycardia of 120 bpm and a temperature of 40 ºC. He has generalised muscular
E 7.2% 5
rigidity. What is the most likely diagnosis?

72.7% of users answered this 6


question correctly
A. Acute dystonic reaction 7

B. Malignant hyperthermia 8

C. Pelvic abscess 9

D. Epilepsy 10

E. Serotonin syndrome

Next question

Anaesthetic agents, such as suxamethonium, can cause malignant hyperthermia


in patients with a genetic defect. Acute dystonic reaction normally is associated
with antipsychotics (haloperidol) and metoclopramide. These lead to marked
extrapyramidal effects. Serotonin syndrome is associated with the
antidepressants selective serotonin reuptake inhibitors (SSRIs) and selective
serotonin/norepinephrine reuptake inhibitors (SSNRIs). This causes a syndrome
of agitation, tachycardia, hallucinations and hyper-reflexia.

Malignant hyperthermia

Overview

Condition often seen following administration of anaesthetic agents


Characterised by hyperpyrexia and muscle rigidity
Cause by excessive release of Ca2+ from the sarcoplasmic reticulum of
skeletal muscle
Associated with defects in a gene on chromosome 19 encoding the
ryanodine receptor, which controls Ca2+ release from the sarcoplasmic
reticulum
Neuroleptic malignant syndrome may have a similar aetiology

Causative agents

Halothane
Suxamethonium
Other drugs: antipsychotics (neuroleptic malignant syndrome)

Investigations

CK raised
Contracture tests with halothane and caffeine

Management

Dantrolene - prevents Ca2+ release from the sarcoplasmic reticulum

Rate question: Next question

Com m ent on this question


yomna92009@yahoo.com - MRCS Part A - My account

Reference ranges End and review

Question 11 of 25 Question stats Score: 27.3%

1
A 44.6%
Which of the following nerves is the primary source of innervation to the anterior 2
B 13%
scrotal skin? 3
C 28.4%
D 7.2% 4
A. Genital branch of the genitofemoral nerve E 6.7% 5
B. Pudendal nerve 6
28.4% of users answered this
C. Ilioinguinal nerve question correctly
7
D. Femoral branch of the genitofemoral nerve
8
E. Obturator nerve
9

10
Next question
11

Theme from April 2012 Exam


The pudendal nerve may innervate the posterior skin of the scrotum. The anterior
innervation of the scrotum is primarily provided by the ilioinguinal nerve. The
genital branch of the genitofemoral nerve provides a smaller contribution.

Scrotal sensation

The scrotum is innervated by the ilioinguinal nerve and the pudendal nerve. The
ilioinguinal nerve arises from L1 and pierces the internal oblique muscle. It
eventually passes through the superficial inguinal ring to innervate the anterior
skin of the scrotum.

The pudendal nerve is the principal nerve of the perineum. It arises in the pelvis
from 3 nerve roots. It passes through both greater and lesser sciatic foramina to
enter the perineal region. The perineal branches pass anteromedially and divide
into posterior scrotal branches. The posterior scrotal branches pass superficially
to supply the skin and fascia of the perineum. It cross communicates with the
inferior rectal nerve.

Rate question: Next question

Com m ent on this question

All contents of this site are © 2012 E-Medical Revision Ltd Terms and Conditions Privacy policy
yomna92009@yahoo.com - MRCS Part A - My account

Reference ranges End and review

Question 12 of 25 Question stats Score: 25%

1
A 11%
The transversalis fascia contributes to which of the following? 2
B 54.7%
C 15% 3

A. Pectineal ligament D 10% 4

B. Deep inguinal ring E 9.3% 5

C. Cremaster muscle and fascia 54.7% of users answered this 6


question correctly
D. Inguinal ligament 7

E. External spermatic fascia 8

9
Next question
10

11

The internal spermatic fascia (derived from transversalis fascia) invests: 12


Ducuts deferens
Testicular vessels

The principal outpouching of the transversalis fascia is the internal spermatic


fascia. The mouth of the outpouching is the deep inguinal ring.

Abdominal wall

The 2 main muscles of the abdominal wall are the rectus abdominis (anterior)
and the quadratus lumborum (posterior).
The remaining abdominal wall consists of 3 muscular layers. Each muscle passes
from the lateral aspect of the quadratus lumborum posteriorly to the lateral margin
of the rectus sheath anteriorly. Each layer is muscular posterolaterally and
aponeurotic anteriorly.

Image sourced from Wikipedia

Muscles of abdominal wall

External Lies most superficially


oblique Originates from 5th to 12th ribs
Inserts into the anterior half of the outer aspect of the iliac crest, linea
alba and pubic tubercle
More medially and superiorly to the arcuate line, the aponeurotic layer
overlaps the rectus abdominis muscle
The lower border forms the inguinal ligament
The triangular expansion of the medial end of the inguinal ligament is
the lacunar ligament.

Internal Arises from the thoracolumbar fascia, the anterior 2/3 of the iliac crest
oblique and the lateral 2/3 of the inguinal ligament
The muscle sweeps upwards to insert into the cartilages of the lower
3 ribs
The lower fibres form an aponeurosis that runs from the tenth costal
cartilage to the body of the pubis
At its lowermost aspect it joins the fibres of the aponeurosis of
transversus abdominis to form the conjoint tendon.

Transversus Innermost muscle


abdominis Arises from the inner aspect of the costal cartilages of the lower 6 ribs
, from the anterior 2/3 of the iliac crest and lateral 1/3 of the inguinal
ligament
Its fibres run horizontally around the abdominal wall ending in an
aponeurosis. The upper part runs posterior to the rectus abdominis.
Lower down the fibres run anteriorly only.
yomna92009@yahoo.com - MRCS Part A - My account

Reference ranges End and review

Question 13 of 25 Question stats Score: 23.1%

1
A 10.7%
A 63 year old man is undergoing a right pneumonectomy for carcinoma of the 2
B 11.4%
bronchus. As the surgeons approach the root of the lung, which structure will lie 3
C 55.6%
most posteriorly (in the anatomical plane)?
D 12.6% 4
E 9.8% 5
A. Phrenic nerve
55.6% of users answered this 6
B. Main bronchus question correctly
7
C. Vagus nerve
8
D. Pulmonary vein
9
E. Pulmonary artery
10

Next question 11

12
The vagus nerve is the most posteriorly located structure at the lung root. The 13
phrenic nerve lies most anteriorly.

Lung anatomy

The right lung is composed of 3 lobes divided by the oblique and transverse
fissures. The left lung has two lobes divided by the lingula.The apex of both lungs
is approximately 4cm superior to the sterno-costal joint of the first rib. Immediately
below this is a sulcus created by the subclavian artery.

Peripheral contact points of the lung

Base: diaphragm
Costal surface: corresponds to the cavity of the chest
Mediastinal surface: Contacts the mediastinal pleura. Has the cardiac
impression. Above and behind this concavity is a triangular depression
named the hilum, where the structures which form the root of the lung enter
and leave the viscus. These structures are invested by pleura, which,
below the hilus and behind the pericardial impression, forms the pulmonary
ligament

Right lung
Above the hilum is the azygos vein; Superior to this is the groove for the superior
vena cava and right innominate vein; behind this, and nearer the apex, is a furrow
for the innominate artery. Behind the hilus and the attachment of the pulmonary
ligament is a vertical groove for the oesophagus; In front and to the right of the
lower part of the oesophageal groove is a deep concavity for the extrapericardiac
portion of the inferior vena cava.

The root of the right lung lies behind the superior vena cava and the right atrium,
and below the azygos vein.

The right main bronchus is shorter, wider and more vertical than the left main
bronchus and therefore the route taken by most foreign bodies.

Left lung
Above the hilum is the furrow produced by the aortic arch, and then superiorly the
groove accommodating the left subclavian artery; Behind the hilum and
pulmonary ligament is a vertical groove produced by the descending aorta, and in
front of this, near the base of the lung, is the lower part of the oesophagus.

The root of the left lung passes under the aortic arch and in front of the
descending aorta.

Inferior borders of both lungs


yomna92009@yahoo.com - MRCS Part A - My account

Reference ranges End and review

Question 14 of 25 Question stats Score: 21.4%

1
A 35.7%
A 43 year old lady is undergoing an axillary node clearance for breast cancer. 2
B 9.9%
The nodal disease is bulky. During clearance of the level 3 nodes there is 3
C 24.1%
suddenly brisk haemorrhage. The most likely vessel responsible is:
D 12.5% 4
E 17.8% 5
A. Thoracoacromial artery
35.7% of users answered this 6
B. Cephalic vein question correctly
7
C. Thoracodorsal trunk
8
D. Internal mammary artery
9
E. Posterior circumflex humeral artery
10

Next question 11

12
The thoracoacromial artery pierces the pectoralis major and gives off branches 13
within this space. The level 3 axillary nodes lie between pectoralis major and
14
minor.Although the thoracodorsal trunk may be injured during an axillary
dissection it does not lie within the level 3 nodes.

Thoracoacromial artery

The thoracoacromial artery (acromiothoracic artery; thoracic axis) is a short trunk,


which arises from the forepart of the axillary artery, its origin being generally
overlapped by the upper edge of the Pectoralis minor.

Projecting forward to the upper border of the Pectoralis minor, it pierces the
coracoclavicular fascia and divides into four branches: pectoral, acromial,
clavicular, and deltoid.

Branch Description

Pectoral Descends between the two Pectoral muscles, and is distributed to them and to
branch the breast, anastomosing with the intercostal branches of the internal thoracic
artery and with the lateral thoracic.

Acromial Runs laterally over the coracoid process and under the Deltoid, to which it gives
branch branches; it then pierces that muscle and ends on the acromion in an arterial
network formed by branches from the suprascapular, thoracoacromial, and
posterior humeral circumflex arteries.

Clavicular Runs upwards and medially to the sternoclavicular joint, supplying this
branch articulation, and the Subclavius

Deltoid Arising with the acromial, it crosses over the Pectoralis minor and passes in the
branch same groove as the cephalic vein, between the Pectoralis major and Deltoid,
and gives branches to both muscles.

Rate question: Next question

Com m ent on this question

All contents of this site are © 2012 E-Medical Revision Ltd Terms and Conditions Privacy policy
yomna92009@yahoo.com - MRCS Part A - My account

Reference ranges End and review

Question 15 of 25 Question stats Score: 20%

1
A 12.4%
A 73 year old lady with long standing atrial fibrillation develops a cold and 2
B 11.2%
pulseless white arm. A brachial embolus is suspected and a brachial embolectomy 3
C 9.7%
is performed. Which of the following structures is at greatest risk of injury during
D 59.4% 4
this procedure?
E 7.4% 5

59.4% of users answered this 6


A. Radial nerve
question correctly
7
B. Cephalic vein
8
C. Ulnar nerve
D. Median nerve 9

E. None of the above 10

11

Next question 12

13
The median nerve lies close to the brachial artery in the antecubital fossa. This is
14
the usual site of surgical access to the brachial artery for an embolectomy
procedure. The median nerve may be damaged during clumsy application of 15
vascular clamps to the artery.

Brachial artery

The brachial artery begins at the lower border of teres major as a continuation of
the axillary artery. It terminates in the cubital fossa at the level of the neck of the
radius by dividing into the radial and ulnar arteries.

Relations
Posterior relations include the long head of triceps with the radial nerve and
profunda vessels intervening. Anteriorly it is overlapped by the medial border of
biceps.
It is crossed by the median nerve in the middle of the arm.
In the cubital fossa it is separated from the median cubital vein by the bicipital
aponeurosis.
The basilic vein is in contact at the most proximal aspect of the cubital fossa and
lies medially.

Rate question: Next question

Com m ent on this question

All contents of this site are © 2012 E-Medical Revision Ltd Terms and Conditions Privacy policy
yomna92009@yahoo.com - MRCS Part A - My account

Reference ranges End and review

Question 16 of 25 Question stats Score: 18.8%

1
A 12.3%
Which one of the following cells secretes the majority of tumour necrosis factor in 2
B 41.1%
humans? 3
C 22.4%
D 12.5% 4
A. Neutrophils E 11.6% 5
B. Macrophages 6
41.1% of users answered this
C. Natural killer cells question correctly
7
D. Killer-T cells
8
E. Helper-T cells
9

10
Next question
11

12

13
Tumour necrosis factor
14

15
Tumour necrosis factor (TNF) is a pro-inflammatory cytokine with multiple roles in
the immune system 16

TNF is secreted mainly by macrophages and has a number of effects on the


immune system, acting mainly in a paracrine fashion:

activates macrophages and neutrophils


acts as costimulator for T cell activation
key mediator of bodies response to Gram negative septicaemia
similar properties to IL-1
anti-tumour effect (e.g. phospholipase activation)

TNF-alpha binds to both the p55 and p75 receptor. These receptors can induce
apoptosis. It also cause activation of NFkB

Endothelial effects include increase expression of selectins and increased


production of platelet activating factor, IL-1 and prostaglandins

TNF promotes the proliferation of fibroblasts and their production of protease and
collagenase. It is thought fragments of receptors act as binding points in serum

Systemic effects include pyrexia, increased acute phase proteins and disordered
metabolism leading to cachexia

TNF is important in the pathogenesis of rheumatoid arthritis - TNF blockers (e.g.


infliximab, etanercept) are now licensed for treatment of severe rheumatoid

Rate question: Next question

Com m ent on this question

All contents of this site are © 2012 E-Medical Revision Ltd Terms and Conditions Privacy policy
yomna92009@yahoo.com - MRCS Part A - My account

Reference ranges End and review

Question 17 of 25 Question stats Score: 23.5%

1
A 8.3%
A 73 year old lady is admitted with right iliac fossa pain. A plain abdominal x-ray is 2
B 7.4%
taken and the caecal diameter measured. Which of the following caecal diameters 3
C 15.6%
are pathological?
D 21.1% 4
E 47.6% 5
A. 4cm
47.6% of users answered this 6
B. 5cm question correctly
7
C. 6cm
8
D. 7cm
9
E. 10cm
10

Next question 11

12
8 cm is still within normal limits. However, caecal diameters of 9 and 10 are 13
pathological and should prompt further investigation.
14

Right colon 15

16
Ileocaecal valve 17

Entry point of the terminal ileum to the caecum


An important colonoscopic landmark
The ileocaecal valve is not always competent and this may allow partial
decompression of an obstructed colon

Appendix

At the base of the caecum the taenia coalesce to mark the base of the
appendix
This is a reliable way of locating the appendix surgically and is a constant
landmark
The appendix has a small mesentery (the mesoappendix) and in this runs
the appendiceal artery, a branch of the ileocolic artery.

The posterior aspect of the right colon is extra peritoneal and the anterior aspect
intraperitoneal.

Relations

Posterior

Iliacus, Iliolumbar ligament, Quadratus lumborum, Transverse abdominis,


Diaphragm at the tip of the last rib; Lateral cutaneous, ilioinguinal, and
iliohypogastric nerves; the iliac branches of the iliolumbar vessels, the fourth
lumbar artery, gonadal vessels, ureter and the right kidney.

Superior

Right kidney which is embedded in the perinephric fat

Medial
Mesentery which contains the ileocolic artery that supplies the right colon and
terminal ileum. A further branch , the right colic artery, also contributes to supply
the hepatic flexure and proximal transverse colon. Medially these pass through
the mesentery to join the SMA. This occurs near to the head of the pancreas and
care has to be taken when ligating the ileocolic artery near to its origin in cancer
cases for fear of impinging on the SMA.

- Anterior
Coils of small intestine, the right edge of the greater omentum, and the anterior
abdominal wall.

Nerve supply

Parasympathetic fibres of the vagus nerve (CN X)

Arterial supply

Ileocolic artery and right colic artery, both branches of the SMA. While the
ileocolic artery is almost always present, the right colic can be absent in 5-
15% of individuals.

Rate question: Next question

Com m ent on this question

All contents of this site are © 2012 E-Medical Revision Ltd Terms and Conditions Privacy policy
yomna92009@yahoo.com - MRCS Part A - My account

Reference ranges End and review

Question 18 of 25 Question stats Score: 22.2%

1
A 40.8%
Which of the following fingers is not a point of attachment for the palmar 2
B 13.6%
interossei? 3
C 8.1%
D 10.6% 4
A. Middle finger E 27% 5
B. Little finger 6
40.8% of users answered this
C. Ring finger question correctly
7
D. Index finger
8
E. None of the above
9

10
Next question
11

The middle finger has no attachment of the palmar interosseous. 12

13

14

15

16

17

18

Image sourced from Wikipedia

Hand

Anatomy of the hand

Bones 8 Carpal bones


5 Metacarpals
14 phalanges

Intrinsic 7 Interossei - Supplied by ulnar nerve


Muscles
3 palmar-adduct fingers
4 dorsal- abduct fingers

Intrinsic Lumbricals
muscles
Flex MCPJ and extend the IPJ.
Origin deep flexor tendon and insertion dorsal extensor hood
mechanism.
Innervation: 1st and 2nd- median nerve, 3rd and 4th- deep
branch of the ulnar nerve.

Thenar Abductor pollicis brevis


eminence Opponens pollicis
Flexor pollicis brevis

Hypothenar Opponens digiti minimi


eminence Flexor digiti minimi brevis
Abductor digiti minimi
yomna92009@yahoo.com - MRCS Part A - My account

Reference ranges End and review

Question 19 of 25 Question stats Score: 21.1%

1
A 34.5%
A 6 year old sustains a supracondylar fracture of the distal humerus. There are 2
B 9.3%
concerns that the radial nerve may have been injured. What is the relationship of 3
C 39.1%
the radial nerve to the humerus at this point?
D 9.8% 4
E 7.4% 5
A. Anterolateral
34.5% of users answered this 6
B. Anteromedial question correctly
7
C. Posterolateral
8
D. Posteromedial
9
E. Immediately anterior
10

Next question 11

12
The radial nerve lies anterolateral to the humerus in the supracondylar area. 13

14
Radial nerve
15

Continuation of posterior cord of the brachial plexus (root values C5 to T1) 16

17
Path
18
In the axilla: lies posterior to the axillary artery on subscapularis, latissimus 19
dorsi and teres major.
Enters the arm between the brachial artery and the long head of triceps
(medial to humerus).
Spirals around the posterior surface of the humerus in the groove for the
radial nerve.
At the distal third of the lateral border of the humerus it then pierces the
intermuscular septum and descends in front of the lateral epicondyle.
At the lateral epicondyle it lies deeply between brachialis and
brachioradialis where it then divides into a superficial and deep terminal
branch.
Deep branch crosses the supinator to become the posterior interosseous
nerve.

In the image below the relationships of the radial nerve can be appreciated

Image sourced from Wikipedia

Regions innervated

Motor (main Triceps


nerve) Anconeus
yomna92009@yahoo.com - MRCS Part A - My account

Reference ranges End and review

Question 20 of 25 Question stats Score: 20%

1
A 9.7%
Which of the following muscles is penetrated by the parotid duct? 2
B 57.7%
C 5.6% 3

A. Medial pterygoid D 6.6% 4

B. Buccinator E 20.4% 5

C. Levator anguli oris 57.7% of users answered this 6


question correctly
D. Temporalis 7

E. Masseter 8

9
Next question
10

11
The duct crosses the masseter muscle and buccal fat pad and then penetrates
the buccinator muscle to enter the oral cavity opposite the second upper molar 12
tooth. 13

14
Parotid gland
15

Anatomy of the parotid gland 16

Location Overlying the mandibular ramus; anterior and inferior to the ear. 17

Salivary duct Crosses the masseter, pierces the buccinator and drains adjacent to 18
the 2nd upper molar tooth (Stensen's duct).
19
Structures Facial nerve (Mnemonic: The Zebra Buggered My Cat; Temporal 20
passing through Zygomatic, Buccal, Mandibular, Cervical)
the gland External carotid artery
Retromandibular vein
Auriculotemporal nerve

Relations Anterior: masseter, medial pterygoid, superficial temporal and


maxillary artery, facial nerve, stylomandibular ligament
Posterior: posterior belly digastric muscle,
sternocleidomastoid, stylohyoid, internal carotid artery, mastoid
process, styloid process

Arterial supply Branches of external carotid artery

Venous drainage Retromandibular vein

Lymphatic Deep cervical nodes

drainage

Nerve innervation Parasympathetic-Secretomotor


Sympathetic-Superior cervical ganglion
Sensory- Greater auricular nerve

Parasympathetic stimulation produces a water rich, serous saliva. Sympathetic


stimulation leads to the production of a low volume, enzyme-rich saliva.

Rate question: Next question

Com m ent on this question

All contents of this site are © 2012 E-Medical Revision Ltd Terms and Conditions Privacy policy
yomna92009@yahoo.com - MRCS Part A - My account

Reference ranges End and review

Question 21 of 25 Question stats Score: 19%

1
A 6.6%
The following are true of the ulnar nerve except: 2
B 8%
C 67.3% 3

A. It innervates the palmar interossei D 6.8% 4

B. Derived from the medial cord of the brachial plexus E 11.4% 5

C. Supplies the muscles of the thenar eminence 67.3% of users answered this 6
question correctly
D. Supplies the medial half of flexor digitorum profundus 7

E. Passes superficial to the flexor retinaculum 8

9
Next question
10

11
These are supplied by the median nerve and atrophy of these is a feature of
carpal tunnel syndrome 12

13
Ulnar nerve
14

15
Origin
16
C8, T1
17

18
Supplies (no muscles in the upper arm)
19
Flexor carpi ulnaris 20
Flexor digitorum profundus
21
Flexor digiti minimi
Abductor digiti minimi
Opponens digiti minimi
Adductor pollicis
Interossei muscle
Third and fourth lumbricals
Palmaris brevis

Path

Posteromedial aspect of ulna to flexor compartment of forearm, then along


the ulnar. Passes beneath the flexor carpi ulnaris muscle, then superficially
through the flexor retinaculum into the palm of the hand.
yomna92009@yahoo.com - MRCS Part A - My account

Reference ranges End and review

Question 22 of 25 Question stats Score: 22.7%

1
A 17.6%
Which of the following options in relation to the liver is true? 2
B 12.7%
C 9% 3

A. Ligamentum venosum is an anterior relation of the liver D 23.4% 4

B. The portal triad comprises the hepatic artery, hepatic vein and E 37.3% 5
tributary of the bile duct
37.3% of users answered this 6
C. The liver is completely covered by peritoneum question correctly
7
D. There are no nerves within the porta hepatis
8
E. The caudate lobe is superior to the porta hepatis
9

10
Next question
11

12
'VC goes with VC'
13

The ligamentun Venosum and Caudate is on same side as Vena Cava 14


[posterior]. 15

16
Ligamentum venosum is posterior to the liver. The portal triad contains the portal 17
vein rather than the hepatic vein. There is the 'bare area of the liver' created by a
18
void due to the coronary ligament layers being widely separated. There are
sympathetic and parasympathetic nerves in the porta hepatis. 19

20
Liver
21

22
Structure of the liver

Right lobe Supplied by right hepatic artery


Contains Couinard segments V to VIII (-/+Sg I)

Left lobe Supplied by the left hepatic artery


Contains Couinard segments II to IV (+/- Sg1)

Quadrate Part of the right lobe anatomically, functionally is part of the left
lobe Couinard segment IV
Porta hepatis lies behind

On the right lies the gallbladder fossa


On the left lies the fossa for the umbilical vein

Caudate lobe Supplied by both right and left hepatic arteries


Couinard segment I
Lies behind the plane of the porta hepatis
Anterior and lateral to the inferior vena cava
Bile from the caudate lobe drains into both right and left hepatic
ducts

Detailed knowledge of Couinard segments is not required for MRCS Part A

Between the liver lobules are portal canals which contain the portal triad:
Hepatic Artery, Portal Vein, tributary of Bile Duct.

Relations of the liver


Anterior Postero inferiorly

Diaphragm Oesophagus

Xiphoid process Stomach

Duodenum
yomna92009@yahoo.com - MRCS Part A - My account

Reference ranges End and review

Question 23 of 25 Question stats Score: 21.7%

1
A 10%
The following statements regarding the rectus abdominis muscle are true except: 2
B 15%
C 7.8% 3

A. It runs from the symphysis pubis to the xiphoid process D 54.6% 4

B. Its nerve supply is from the ventral rami of the lower 6 thoracic E 12.6% 5
nerves
54.6% of users answered this 6
C. It has collateral supply from both superior and inferior epigastric question correctly
7
vessels
8
D. It lies in a muscular aponeurosis throughout its length
9
E. It has a number of tendinous intersections that penetrate through
the anterior layer of the muscle 10

11
Next question
12

13
The aponeurosis is deficient below the arcuate line.
Rectus abdominis: 14
Arises from 5th, 6th, 7th costal cartilages. 15
Inserts into the pubis.
16
The muscle lies in the rectal sheath, which also contains the superior & inferior
epigastric artery and vein. 17

Action: flexion of thoracic and lumbar spine. 18


Nerve supply: anterior primary rami of T7-12
19

20
Abdominal wall
21
The 2 main muscles of the abdominal wall are the rectus abdominis (anterior) 22
and the quadratus lumborum (posterior).
23
The remaining abdominal wall consists of 3 muscular layers. Each muscle passes
from the lateral aspect of the quadratus lumborum posteriorly to the lateral margin 24 -

of the rectus sheath anteriorly. Each layer is muscular posterolaterally and


aponeurotic anteriorly.

Image sourced from Wikipedia

Muscles of abdominal wall

External Lies most superficially


oblique Originates from 5th to 12th ribs
Inserts into the anterior half of the outer aspect of the iliac crest, linea
alba and pubic tubercle
More medially and superiorly to the arcuate line, the aponeurotic layer
overlaps the rectus abdominis muscle
The lower border forms the inguinal ligament
The triangular expansion of the medial end of the inguinal ligament is
the lacunar ligament.

Internal Arises from the thoracolumbar fascia, the anterior 2/3 of the iliac crest
oblique and the lateral 2/3 of the inguinal ligament
The muscle sweeps upwards to insert into the cartilages of the lower
3 ribs
The lower fibres form an aponeurosis that runs from the tenth costal
cartilage to the body of the pubis
At its lowermost aspect it joins the fibres of the aponeurosis of
yomna92009@yahoo.com - MRCS Part A - My account

Reference ranges End and review

Question 24 of 25 Question stats Score: 20.8%

1
A 27.5%
Which of the following statements relating to sternocleidomastoid is untrue? 2
B 48%
C 8.4% 3

A. The common carotid is related posteromedially. D 10.2% 4

B. The external jugular vein lies posteromedially. E 5.9% 5

C. It is supplied by the accessory nerve. 48% of users answered this 6


question correctly
D. It inserts into the lateral aspect of the mastoid process. 7

E. It marks the anterior border of the posterior triangle. 8

9
Next question
10

11
The external jugular vein lies lateral (i.e. superficial) to the sternocleidomastoid.
12

Sternocleidomastoid 13

14
Anatomy
15
Origin Rounded tendon attached to upper manubrium sterni and muscular head
attached to medial third of the clavicle 16

Insertion Mastoid process of the temporal bone and lateral area of the superior nuchal 17
line of the occipital bone
18
Innervation Spinal part of accessory nerve and anterior rami of C2 and C3 (proprioception)
19
Action Both: extend the head at atlanto-occipital joint and flex the cervical 20
vertebral column. Accessory muscles of inspiration.
Single: lateral flexion of neck, rotates head so face looks upward to the 21
opposite side
22

23
Sternocleidomastoid divides the anterior and posterior triangles of the neck.
24

Rate question: Next question

Com m ent on this question

All contents of this site are © 2012 E-Medical Revision Ltd Terms and Conditions Privacy policy
yomna92009@yahoo.com - MRCS Part A - My account

Reference ranges End and review

Question 25 of 25 Question stats Score: 24%

1
A 15.4%
During liver mobilisation for a cadaveric liver transplant the hepatic ligaments will 2
B 19.9%
require mobilisation. Which of the following statements relating to these structures 3
C 11.1%
is untrue?
D 16.8% 4
E 36.8% 5
A. Lesser omentum arises from the porta hepatis and passes the
36.8% of users answered this 6
lesser curvature of the stomach
question correctly
7
B. The falciform ligament divides into the left triangular ligament and
coronary ligament 8
C. The liver has an area devoid of peritoneum 9
D. The coronary ligament is attached to the liver 10
E. The right triangular ligament is an early branch of the left triangular 11
ligament
12

13
The right triangular ligament is a continuation of the coronary ligament. 14

15
Liver
16

Structure of the liver 17

18
Right lobe Supplied by right hepatic artery
Contains Couinard segments V to VIII (-/+Sg I) 19

20
Left lobe Supplied by the left hepatic artery
21
Contains Couinard segments II to IV (+/- Sg1)
22

Quadrate Part of the right lobe anatomically, functionally is part of the left 23
lobe Couinard segment IV
24
Porta hepatis lies behind
On the right lies the gallbladder fossa 25
On the left lies the fossa for the umbilical vein

Caudate lobe Supplied by both right and left hepatic arteries


Couinard segment I
Lies behind the plane of the porta hepatis
Anterior and lateral to the inferior vena cava
Bile from the caudate lobe drains into both right and left hepatic

ducts

Detailed knowledge of Couinard segments is not required for MRCS Part A

Between the liver lobules are portal canals which contain the portal triad:
Hepatic Artery, Portal Vein, tributary of Bile Duct.

Relations of the liver


Anterior Postero inferiorly

Diaphragm Oesophagus

Xiphoid process Stomach

Duodenum

Hepatic flexure of colon

Right kidney

Gallbladder

Inferior vena cava

You might also like